Anda di halaman 1dari 401

Answers

1>The correct answer is A. Fascial straps (retinacula) and fascial coverings of


muscles or muscle groups characteristically attach to nearby bones by blending
with the covering periosteum. No deep attachments are usually made by fascia.
Cancellous bone (choice B) is spongy bone, which is usually found in marrow, and is
not the site for fascial attachment.
Fascia do not usually attach to cartilage (choice C). Fascia attaches to bony shafts,
or diaphyses (choice D), superficially via the periosteum.
Fascia do not penetrate the bone to reach the marrow (choice E).

2>The correct answer is D. The lesion is a malignant melanoma. Melanomas can


develop either de novo or in an existing mole. Sunlight exposure is a significant risk
factor and fair-skinned persons are at increased risk of developing melanoma. The
most significant factor for long term prognosis is the depth of the lesion, since the
superficial dermis lies about 1 mm under the skin surface, and penetration to this
depth is associated with a much higher incidence of metastasis than is seen with a
more superficial location.
The circumference of the lesion (choice A) is much less important than depth , since
one form of melanoma (superficial spreading) can still have good prognosis despite
large size, if it has not extended to the depth of the superficial dermal lymphatic
bed.
The darkness (choice B) or degree of variation in color (choice C) do not have
prognostic significance once melanoma is diagnosed.
Irregularity, or fuzziness at the border (choice E) of a mole-like lesion is a good clue
to potential malignancy, but does not affect prognosis once a melanoma is
diagnosed.
________________________________________

3>The correct answer is E. Beta-adrenergic blockade may blunt or prevent


the premonitory signs and symptoms (e.g., tachycardia, blood pressure
changes) of acute episodes of hypoglycemia. Non-selective beta-blockers,
such as propranolol, may even potentiate insulin-induced hypoglycemia.
Even though this effect is less likely with cardioselective agents, the
use of either cardioselective or non-selective beta-blockers in diabetics
is not recommended due to their "masking" effect of the normal warning
signs and symptoms of hypoglycemia. None of the drugs listed in the othe

r choices will blunt the premonitory signs and symptoms of hypoglycemia.


Captopril (choice A) is an angiotensin-converting enzyme (ACE) inhibito
r that can be safely used for the treatment of hypertension in diabetic
patients.
Diltiazem (choice B) is a calcium channel blocker that is also considered
to be safe and effective for the treatment of hypertension in diabetic
patients.
Both methyldopa (choice C), a centrally acting alpha-adrenergic agonist
, and prazosin (choice D), an alpha1-adrenergic antagonist, can be safely
used to treat hypertension in diabetic patients. However, due to the side
effect profile of these medications, they are generally used only in
diabetic patients who are unresponsive to ACE inhibitors and calcium
channel blockers.
________________________________________

4>The correct answer is A. Acute pyelonephritis is an infectious disease


involving the kidney parenchyma and the renal pelvis. Gram-negative
bacteria, such as Escherichia coli, Proteus, Klebsiella, and Enterobacter,
are the most common causative organisms in acute pyelonephritis.
Laboratory evaluation will often reveal leukocytosis with a left shift,
and urinalysis typically shows pyuria, varying degrees of hematuria
, and white cell casts. Since bacteremia is present, the patient should
be hospitalized and empirically started on IV ampicillin and gentamicin
. This regimen may be need to be changed, however, once the sensitivity
results are available.
Erythromycin (choice B) and tetracycline (choice E) are both bacteriostatic
antibiotics and would not be recommended in a patient with a severe
infection, such as acute pyelonephritis with bacteremia.
Vancomycin (choice C) is primarily used in the treatment of severe
gram-positive infections.

Phenazopyridine (choice D) is a urinary analgesic, and nitrofurantoin


(choice D) is a urinary tract anti-infective. Although nitrofurantoin is
indicated for the treatment of "mild" cases of pyelonephritis, as well
as cystitis, this patient's condition is severe and should be treated with
appropriate antibiotics.
________________________________________

5>The correct answer is E. The patient is suffering from hyperacute


rejection due to the preformed anti-B ABO blood group antibody found
in all type A positive individuals. Hyperacute rejection occurs within
minutes to a few hours of the time of transplantation, and is due to
the destruction of the transplanted tissue by preformed antibodies
reacting with antigens found on the transplanted tissue that activate
complement and destroy the target tissue. Preformed antibodies can
also be due to presensitization to a previous graft, blood transfusion,
or pregnancy.
Acute rejection due to antibody-mediated immunity (choice A) is incorrect
because this patient suffered from hyperacute rejection (immediate)
occurring within minutes to hours, rather than days.
Acute rejection due to cell-mediated immunity (choice B) will not occur
until several days or a week following transplantation. Acute rejection
is due to type II and type IV reactions.
Chronic rejection, due to the presence of cell-mediated immunity to minor
HLA antigens (choice C), occurs in allograft transplantation months to
even years after the transplant. Chronic rejection is generally caused
by both humoral and cell-mediated immunity.
An accelerated acute rejection, occurring in 3-5 days, can be caused by
tissue infiltration and destruction by presensitized T lymphocytes and
macrophages (choice D) and/or antibody-dependent, cell-mediated cytotoxicity
(ADCC). Note that this is not a hyperacute reaction.

________________________________________

6>The correct answer is E. Arrow E points to a smooth muscle cell in


the media of the arteriole. Alpha1 agonists stimulate alpha1 receptors
present on the smooth muscle, which leads to an increase in intracellular
calcium via phosphatidylinositol hydrolysis. This increase in calcium is
necessary for smooth muscle contraction.
Arrow A indicates an endothelial cell located in the intima of the arteriole.
Nitric oxide, also known as endothelial cell relaxing factor (EDRF),
is produced from arginine by endothelial cells. A muscarinic agonist
can lead to the evolution of NO, producing vasodilatation.
Arrow B indicates a polymorphonuclear leukocyte in the bloodstream.
Arrow C indicates the basal lamina underlying the endothelium.
Arrow D indicates the arteriolar adventitia.
________________________________________

7>The correct answer is B. Hyperlipidemia has been subclassified based on


the lipid and lipoprotein profiles. Type 2a, which this patient has, can
be seen in a hereditary form, known as familial hypercholesterolemia, and
also in secondary, acquired forms related to nephritic syndrome and
hyperthyroidism. The root problem appears to be a deficiency of LDL
receptors, which leads to a specific elevation of cholesterol in the
form of increased LDL. Heterozygotes for the hereditary form generally
develop cardiovascular disease from 30 to 50 years of age. Homozygotes
may have cardiovascular disease in childhood.
Type 1 (choice A) is characterized by isolated elevation of chylomicrons.
Type 2b (choice C) is characterized by elevations of both cholesterol and
triglycerides in the form of LDL and VLDL.
Type 3 (choice D) is characterized by elevations of triglycerides
and cholesterol in the form of chylomicron remnants and IDL.

Type 5 (choice E) is characterized by elevations of triglycerides


and cholesterol in the form of VLDL and chylomicrons.

________________________________________

8>The correct answer is D. Wernicke-Korsakoff syndrome refers to the


constellation of neurologic symptoms caused by thiamine deficiency.
Among these, a severe memory deficit, which the patient may attempt to cover by
making up bizarre explanations (confabulation), is prominent. Anatomical damage
to the mamillary bodies and periventricular structures has been postulated as the
cause. In the U.S., severe thiamine deficiency is seen most commonly in chronic
alcoholics. Thiamine deficiency can also damage peripheral nerves ("dry" beriberi)
and the heart ("wet" beriberi).
Folic acid deficiency (choice A) produces megaloblastic anemia without
neurologic symptoms.
Niacin deficiency (choice B) produces pellagra, characterized by depigmenting
dermatitis, chronic diarrhea, and anemia.
Riboflavin deficiency (choice C) produces ariboflavinosis, characterized
by glossitis, corneal opacities, dermatitis, and erythroid hyperplasia.
Vitamin B12 deficiency (choice E) produces megaloblastic anemia accompanied
by degeneration of the posterolateral spinal cord.
________________________________________

9>The correct answer is A. Fibrinogen is cleaved by thrombin twice as it is activated


to form fibrin. The initial cleavage causes it to polymerize and the second causes it
to branch. Thrombin also activates Factor XIII to XIIIa, which crosslinks the fibrin
strands and strengthens the clot.
HMWK (choice B) is a cofactor in the intrinsic pathway that converts Factor XI to XIa.
Plasminogen (choice C) is a central proenzyme in clot lysis. When plasminogen is
converted to plasmin, it digests fibrin threads, as well as a number of protein factors
including Factors V, VIII, XII, and prothrombin.
Thrombin (choice D) is an enzyme derived from prothrombin. It converts fibrinogen
to fibrin, and activates factor XIII.
vWF (choice E) is a tissue-bound protein which is exposed with vascular trauma and
helps in the process of platelet adhesion.

________________________________________

10>The correct answer is B. One of the most common side effects of any
antineoplastic therapy is weight loss secondary to decreased appetite and/or
nausea and vomiting. Furthermore, weight loss due to decreased food intake tends
to occur more frequently in elderly patients receiving antineoplastic therapy. One
medication that has consistently helped to increase appetite in such patients is
megestrol acetate. This agent is a progestational hormone with antineoplastic
properties used in the treatment of advanced carcinoma of the breast and
endometrium. Megestrol, when given in relatively high doses, can substantially
increase the appetite in most individuals, even those with advanced cancer.
Amitriptyline (choice A) is a tricyclic antidepressant used in the treatment of
depression. There is nothing mentioned in the case study to suggest that the
patient is clinically depressed; hence, this agent would provide no benefit.
Methotrexate (choice C) is an antimetabolite and folic acid antagonist commonly
used in various neoplastic disorders and in the treatment of rheumatoid arthritis.
Since nausea, vomiting, and ulcerative stomatitis are common side effects of this
medication, its usage in this patient would not be recommended.
Neostigmine (choice D) is a carbamylating acetylcholinesterase inhibitor that would
not increase appetite.
Prochlorperazine (choice E) is a phenothiazine derivative used primarily to control
severe nausea and vomiting. This patient is not experiencing nausea. Furthermore,
this agent does not possess appetite-stimulating properties.
________________________________________

11>The correct answer is A. Anthrax forms extremely stable spores and has, in fact,
been encountered in very old (i.e., deep) dirt in Israeli and Arabian excavation sites.
The causative organism, Bacillus anthracis, is found in many animal species, and
humans can acquire the organism either through contact with the animals or from
locally contaminated soil. The pustule described in the question stem is called a
"malignant pustule" and may be accompanied by lymphadenopathy. Most cases
remain localized, but death can occur as the result of complications such as
bacteremia, meningitis, and pneumonia.
Borrelia burgdorferi (choice B) causes Lyme disease. In the first stage of this
disease, there is a localized expanding erythematous rash rather than an eschar.
Furthermore, Lyme disease occurs in the U.S., Europe, and Asia, corresponding to
the distribution of Ixodid ticks, which spread the infection.

Francisella tularensis (choice C) causes tularemia. The organism can persist for
weeks to months but does not form spores that could survive for hundreds or
thousands of years. Also, most human cases occur in the endemic areas of the U.S.
Spirillium minus (choice D) is one of the causes of rat-bite fever. There is no
indication the man was bitten by a rat.
Yersinia pestis (choice E) causes bubonic plague. This organism does not form stable
spores that could survive for extended periods of time.

12>The correct answer is C. This is a classic presentation of a patient with carpal


tunnel syndrome, which typically affects females between the ages of 40 and 60
who chronically perform repetitive tasks that involve movement of the structures
that pass through the carpal tunnel. One important structure that passes through
the carpal tunnel is the median nerve. Patients often note a tingling, a loss of
sensation, or diminished sensation in the digits. There is also often a loss of
coordination and strength in the thumb, because the median nerve also sends fibers
to the abductor pollicis brevis, flexor pollicis brevis, and the opponens pollicis. A
final function of the median nerve distal to the carpal tunnel is control of the first
and second lumbricals which function to flex digits two and three at the
metacarpophalangeal joints and extend interphalangeal joints of the same digits.
Abduction of the fifth digit (choice A) is a function controlled by the ulnar nerve,
which does not pass through the carpal tunnel.
Adduction of the thumb (choice B) is a function of the adductor pollicis, which is the
only short thumb muscle that is not innervated by the median nerve, but rather by
the deep branch of the ulnar nerve.
Sensation of the lateral half of the dorsum of the hand (choice D) is mediated by the
radial nerve, which also does not pass through the carpal tunnel.
Sensation over the lateral aspect of the palm (choice E) is mediated by the median
nerve, however the branch innervating the palm (palmar cutaneous branch of the
median nerve) passes superficial to the carpal tunnel.
Sensation over the medial aspect of the dorsum of the hand (choice F) is mediated
by the ulnar nerve.
Sensation over the medial aspect of the palm (choice G) is mediated by the ulnar
nerve.

13>The correct answer is D. Duchenne muscular dystrophy (DMD) is a lethal, Xlinked recessive disease affecting approximately 1 in 3300 live male births. The
disease becomes symptomatic in early childhood. Inability to walk occurs by the
end of the first decade, and death usually occurs by the second decade. Nearly all
patients show the complete absence of the protein dystrophin, which is abundant in
skeletal and cardiac muscle. The dystrophin gene (or DMD gene), located at Xp21, is

approximately 2300 Kb in size, making it one of the largest known genes of any
species. It contains 24 regions of 109 amino acids that are similar but not identical
repeats of each other. In women, the similarity of these sequences can lead to the
misalignment of homologous material at meiotic synapsis. In association with a
recombination event, this misalignment gives rise to frameshift mutations, leading
to an untranslatable mRNA. This series of events occurs at an extremely high rate of
about 1 in 10,000. In fact, one third of DMD cases in each generation arise from this
mechanism.
Diseases such as fragile X syndrome and Huntington disease are caused by the
expansion of a trinucleotide repeat (choice A).
Infidelity (choice B) is not a plausible explanation for the child because the trait is
an X-linked recessive condition. A male child must receive the DMD trait on the
maternal X chromosome. The father of the child contributes only a Y chromosome,
and the history stated that the mother had no family history of the disorder.
DMD in a patient with no family history is not generally due to a point mutation in
the dystrophin gene (choice C).
Rare cases of DMD in females have been caused by an X chromosome-autosome
translocation (choice E) with the breakpoint on the X chromosome within the DMD
gene. Because of the translocation, only the cells in which the normal X
chromosome is inactivated survive in the female zygote. This gives rise to a female
who is heterozygous for the DMD gene but phenotypically expresses the disease.

14>The correct answer is B. The great saphenous vein is one of the two major
superficial veins of the lower limb. It is found in the superficial fascia, where it is
accessible for procedures such as saphenous cutdown and saphenous venous graft.
As the vein ascends along the lower limb, it passes anterior to the medial malleolus
at the ankle and posterior to the medial side of the knee. The vein then passes
through the saphenous hiatus of the fascia lata to empty into the femoral vein
slightly below the inguinal ligament.
No major vein is found anterior to the lateral malleolus (choice A).
Veins on the dorsum of the foot (choice C) are small tributaries of the great and
small saphenous veins.
The small saphenous vein passes posterior to the lateral malleolus (choice D).
There are no major superficial veins that pass posterior to the medial malleolus
(choice E). In this region, under the deep fascia, will be found the posterior tibial
artery and vein, along with the tibial nerve.

15>The correct answer is D. The child is suffering from minimal change or nil
disease (lipoid nephrosis), which has a peak incidence at 2-3 years of age. Minimal
change disease can be associated with food allergies, medications, or hematologic

malignancies, or it can occur idiopathically. The pathology does not appear to


involve complement, immunoglobulins, or immune complex deposition. Rather, an
altered cell-mediated immunologic response with abnormal secretion of
lymphokines by T cells is thought to reduce the production of anions in the
glomerular basement membrane, thereby increasing the glomerular permeability to
plasma albumin through a reduction of electrostatic repulsion. The loss of anionic
charges is also thought to favor foot process fusion. Some authors have noted that
other conditions associated with T-cell abnormalities, such as Hodgkin's disease and
T-cell lymphoma, are sometimes associated with minimal change disease.
Consumption of complement factors (choice A) is observed in many conditions in
which complement activation occurs, for example, membranoproliferative
glomerulonephritis.
IgG directed against renal and pulmonary basement membranes (choice B) is found
in Goodpasture's syndrome, a cause of rapidly progressive glomerulonephritis and
hemoptysis.
Immune complex deposition (choice C) is associated with type III hypersensitivity
reactions, including postinfectious glomerulonephritis, lupus nephritis, HenochSchnlein purpura, cryoglobulinemia, and bacterial endocarditis.
Mesangial IgA deposition (choice E) is associated with Berger's disease, or IgA
nephropathy, a cause of glomerulonephritis.

16>The correct answer is E. Wiskott-Aldrich syndrome is an X-linked condition


characterized by eczema, thrombocytopenia, and repeated infections. Affected
children may present with bleeding and often succumb to complications of bleeding,
infection, or lymphoreticular malignancy. The platelets are small, have a shortened
half-life, and appear to be deficient in surface sialophorin (CD43). Splenectomy can
correct the thrombocytopenia, but not the immune defect. Serum IgM is usually
decreased, while IgE is frequently increased. Mutations in the Wiskott-Aldrich serum
protein (WASP) gene on the short arm of the X chromosome are responsible for this
disease.
Acquired hypogammaglobulinemia (choice A) is a disease of adults characterized by
normal numbers of B cells but low immunoglobulin production.
Ataxia telangiectasia (choice B) is an autosomal recessive disease characterized by
progressive cerebellar dysfunction, telangiectasias, and a variable
immunodeficiency.
DiGeorge syndrome (choice C) is a developmental malformation leading to thymic
aplasia and, sometimes, hypoparathyroidism.
Selective IgA deficiency (choice D) is a relatively common condition characterized
by low levels of IgA.

________________________________________

17>The correct answer is D. The key symptom is hyperventilation. Hyperventilation


results in hypocapnia, alkalosis, increased cerebrovascular resistance, and
decreased cerebral blood flow. Carbon dioxide plays an important role in the control
of cerebral blood flow. An increase in arterial PCO2 dilates blood vessels in the brain
and a decrease in PCO2 causes vasoconstriction. The anxious, hyperventilating
woman is "blowing off" carbon dioxide, which lowers her arterial PCO2. This
decrease in PCO2 has caused the cerebrovascular resistance (choice E) to increase,
thereby decreasing cerebral blood flow. The decrease in cerebral blood flow has
caused the woman to feel faint and to have blurred vision. Other symptoms
commonly associated with the hyperventilation of anxiety states are feelings of
tightness in the chest and a sense of suffocation.
Hyperventilation increases the arterial oxygen content (choice A) and PO2 (choice
B) in a normal person.
A decrease in arterial PCO2 causes the arterial pH (choice C) to increase, i.e., the
patient becomes alkalotic.
________________________________________

18>The correct answer is E. Sheehan's syndrome is hypopituitarism due to ischemic


damage to the pituitary resulting from excessive hemorrhage during parturition. The
pituitary is enlarged during pregnancy; it is more metabolically active, and more
susceptible to hypoxemia. Furthermore, the blood vessels in the pituitary may be
more susceptible to vasospasm because of the high estrogen. In about 30% of
women who hemorrhage excessively during parturition, some degree of
hypopituitarism eventually becomes manifest. The symptoms depend on how much
of the pituitary is damaged and what cell types are destroyed. The patient
described above exhibited persistent amenorrhea after delivery of her infant. This is
due to destruction of pituitary gonadotrophs and diminished secretion of
gonadotropins (LH). There also appears to have been significant destruction of
lactotrophs since TRH injection failed to induce an increase in prolactin. Had the
women attempted to breast-feed her infant, a failure to lactate mostly likely would
have occurred. This case is also characterized by secondary hypothyroidism. The
low TSH and failure to respond to TRH injection is confirmatory. Corticotrophs
appear to have been spared since plasma ACTH is normal. It is not clear whether
somatotrophs were damaged. Further testing would be needed to see if GH reserve
is diminished.

Hashimoto's thyroiditis (choice A) is an autoimmune disorder that produces primary


hypothyroidism. Because of diminished negative feedback effects of T4, serum TSH
is usually increased (not decreased).
Isolated gonadotropin deficiency (choice B) produces amenorrhea and is associated
with low serum LH and estradiol. Hypogonadotropic hypogonadism can occur in
female athletes that over-train, in anorexia nervosa, in obesity, or with other
emotional or physical stresses. However, other pituitary hormones are unaffected
(by definition isolated gonadotropin deficiency only involves a decrease in
gonadotropins).
Primary amenorrhea (choice C), by definition, means failure of menstrual cycles to
ever begin. Since this woman has delivered a baby, primary amenorrhea is highly
unlikely.
A prolactinoma (choice D) is a functional pituitary tumor that secretes excessive
prolactin. This can cause amenorrhea by suppressing the GnRH-pituitary-gonad
axis. The patient described above has decreased prolactin secretion.

________________________________________

19>The correct answer is C. The child has subacute sclerosing panencephalitis


(SSPE), which is fortunately a very rare, late complication of measles infection at an
early age. The existence of this complication is part of the rationale to immunize
children to measles at an early age. It is thought that very young children's immune
and neurologic systems may permit the virus to become established in the brain.
The exact mechanism of injury is poorly understood, but the brain shows
encephalitis involving both gray and white matter. 40% of cases die within 1 year,
and it is thought that the disease is probably always eventually fatal.
None of the other diseases progresses to SSPE.
________________________________________

20>The correct answer is B. The diagnosis of chronic bronchitis requires the


presence of chronic productive cough over at least 3 months of the year for 2
successive years. The symptoms in this patient (green productive sputum, fever)
suggest that he has an acute infection superimposed on chronic bronchitis (history
of cigarette smoking, history of excessive mucus production over many years).
Histologically, the mucus-producing glands in the bronchi would show hyperplasia
and hypertrophy and extend to a greater depth in the bronchial wall, resulting in a
higher Reid index (ratio of thickness of mucus gland to thickness of bronchial wall).

Although the patient is a smoker and is at increased risk for bronchogenic


carcinoma (choice A), this is unlikely to be the cause of his symptoms. Hemoptysis
and weight loss might also be present if he had cancer.
Cystic fibrosis (choice C) presents earlier in life and may be associated with severe
production of mucus, especially if bronchiectasis supervenes. The age of the patient
and the relative late onset of disease preclude this diagnosis.
This patient may also have emphysema (choice D), as chronic bronchitis and
emphysema are often coexistent. However, although pure emphysema might cause
dyspnea, it would not be associated with a fever or a productive cough.
Pulmonary tuberculosis (choice E) would typically present with a history of
hemoptysis rather than abundant green sputum. Weight loss, night anorexia,
malaise, and weakness may also be present.
________________________________________

21>The correct answer is E. The thyroglossal duct develops as an evagination of the


floor of the pharynx in the region where the tongue develops. The adult foramen
cecum of the tongue marks the site of this evagination. The distal end of this duct
normally forms the thyroid gland; the proximal part of the duct normally
degenerates. Failure of a part of the duct to degenerate may lead to a thyroglossal
duct cyst or median cervical cyst, as seen in this patient.
The first pharyngeal cleft (choice A) forms the external ear canal. This cleft normally
remains patent.
The first pharyngeal pouch (choice B) forms the middle ear cavity and the auditory
tube. This pouch normally remains patent.
The second pharyngeal cleft (choice C) normally does not remain patent. It is
typically covered over by the overgrowth of the second pharyngeal arch. If part of
this pouch does remain patent, it may form a lateral cervical cyst, which is seen on
the lateral side of the neck along the posterior border of the sternocleidomastoid
muscle.
The second pharyngeal pouch (choice D) forms the tonsillar fossa of the pharynx.
The pharyngeal mucosa in this area arises from the endoderm of the pouch.
Ingrowth of mesoderm cells results in the formation of the palatine tonsil.
________________________________________

22>The correct answer is D. The disease is hairy cell leukemia, which, unlike most
forms of leukemia, is characterized by pancytopenia rather than increased numbers

of circulating cells. This disease tends to affect older men and the characteristic
cells (which are not always obviously "hairy") are positive for tartrate-resistant acid
phosphatase (TRAP).
Associate Birbeck granules (choice A) with Langerhans cell histiocytosis
(histiocytosis X).
Associate elevated leukocyte alkaline phosphatase (choice B) with myeloid
metaplasia.
Associate Philadelphia chromosome (choice C) with chronic myeloid leukemia (and
occasionally with acute lymphocytic leukemia).
Associate production of Bence-Jones proteins (choice E) with multiple myeloma.
________________________________________

23>The correct answer is A. Most of the venous drainage of the skin of the lower
extremity is to the long saphenous vein, the accompanying lymphatics of which
drain into the superficial group of the inguinal lymph nodes. However, the skin
drained by the short saphenous vein, including that of the lateral aspect of the
dorsum of the foot, is an exception to this rule. Lymphatic fluid from this area drains
into lymphatics accompanying the short saphenous vein, then drains into lymph
nodes behind the knee in the popliteal fossa.
The lateral side of the thigh (choice B), the medial side of the leg below the knee
(choice C), the medial side of the sole of the foot (choice D), and the medial side of
the thigh (choice E) all drain to the superficial inguinal nodes.
________________________________________

24>The correct answer is C. Questions like these are particularly amenable to the
use of test-taking strategies because even if you don't know 100% of the answer,
you will be able to eliminate some answer choices based on the knowledge you do
have. For example, you probably were aware that the peritoneum does not lie above
the transversus abdominis muscle, enabling you to eliminate choice A and to
increase your chances of answering correctly by 20%. Note that this is a question
surgeons love to ask students, so keep this information in mind in the OR!
________________________________________

25>The correct answer is A. The Halstead-Reitan Battery is a group of tests that


reflects the basic and higher level cognitive and neuro-sensory functioning of the
entire brain, and can be used in a serial fashion with little learning effect being

present. Since there is recovery of function for up to 2 years post CNS-trauma, test
results can demonstrate that the present loss the patient is experiencing is not
permanent.
The Stanford Binet Intelligence Test (choice B), used in the adult, mainly reflects
verbal skills and consequently would miss large portions of this man's situation.
The Vineland Adaptive Behavior Scale (choice C) assesses developmental and social
functioning, not cognitive and neuro-sensory abilities.
The Wechsler Adult Intelligence Scale (choice D) confines its results to intelligence
assessment and does not assess more basic issues like aphasia and neuro-sensory
skills.
The Wide Range Achievement Test (choice E) assesses academic achievement only.
________________________________________

26>The correct answer is B. The reaction was precipitated by a stressful event that
would cause anyone to experience a severe and intense emotional response. The
symptoms occurred within 3 months of the event and lasted for less than 6 months
after the trauma, all corresponding to the diagnosis of adjustment disorder with
mixed anxiety and depressed mood.
Acute stress disorder (choice A) and post-traumatic stress disorder (choice E) both
require that the person reexperience the traumatic event in wakeful or dream states
and the presence of dissociative symptoms.
This is not a major depressive disorder (choice C) because her symptoms are not of
psychotic proportion and there is a clear precipitating stimulus.
Panic disorder without agoraphobia (choice D) is incorrect because the panic
episodes are uncued and not the response to an environmental stressor.
________________________________________

27>The correct answer is D. The patient has acute prostatitis, which is


characterized by fever, chills, and dysuria, with a swollen, extremely tender prostate
on rectal exam. The urine Gram's stain and culture will generally be positive. The
treatment regimen for this bacterial infection is typically a 21-day course of
ampicillin, a fluoroquinolone, or sulfamethoxazole/trimethoprim (SMX-TMP). G-6-PD
deficiency is an X-linked recessive disorder affecting 10% to 15% of American black
males. The medications most commonly associated with the induction of hemolytic
anemia in deficient patients are sulfonamides, nitrofurantoin, dapsone, primaquine,

and quinine. The sulfamethoxazole in the SMX-TMP combination is a sulfonamide,


and can produce hemolytic anemia in patients with G-6-PD deficiency.
Ampicillin (choice A) is a broad-spectrum penicillin antibiotic commonly used in the
treatment of infections in the genitourinary, respiratory, or GI tracts, as well as in
the skin and soft tissues.
Cefaclor (choice B) is a second-generation cephalosporin indicated for a variety of
bacterial infections, including those of the respiratory and GI tracts.
Ciprofloxacin (choice C) is a fluoroquinolone commonly used in the treatment of
serious infections caused by gram-negative organisms.
Tetracycline (choice E) is most commonly used in the treatment of acne vulgaris and
gonococcal infections.
________________________________________

28>The correct answer is E. Testicular feminization is a disorder of the androgen


receptor. Phenotypically, the patient appears female, but has a blindly ending
vagina and lacks a uterus or other female internal reproductive organs. The patient
has an XY genotype. Since the gene for testes determining factor (TDF) is on the Y
chromosome, TDF will cause the indifferent gonad to develop into a testis
containing Sertoli cells. Sertoli cells at this stage will secrete MIF, a substance that
suppresses the paramesonephric ducts, preventing the formation of female internal
reproductive organs.
This patient would not have a streak ovary (choice A), a finding in Turner's
syndrome that is associated with a 45,XO genotype. In fact, the patient would have
testes, since the genetic complement contained a Y chromosome. The testes in
individuals with testicular feminization syndrome are often undescended and are
usually removed surgically.
This patient would not possess a uterus (choice B) or an oviduct (choice C). Both of
these structures are derived from the paramesonephric duct, which is suppressed
by MIF.
Depressed levels of testosterone (choice D) would not occur in this patient. In fact,
individuals with testicular feminization syndrome have normal or even slightly
elevated levels of testosterone. The development of female external genitalia is the
result of defective androgen receptors, not depressed levels of testosterone.
________________________________________

29>The correct answer is A. The right lymphatic duct drains the right arm, the right
side of the chest, and the right side of the head. The thoracic duct drains the rest of
the body. Both the right lymphatic duct and the thoracic duct dump into the large
venous channels at the base of the neck. Occlusion of this drainage can produce
intractable edema in sites feeding these ducts.
The left side and right leg would be affected, rather than the right side and left leg
(choice B).
The entire left side (rather than just the left arm, choice C) and right leg drain into
the thoracic duct.
The right arm and the right half of the head (choices D and E) drain to the right
lymphatic duct.
________________________________________

30>The correct answer is A. This is a classic presentation of a pulmonary abscess.


Chronic courses with less severe symptoms (with intermittent improvement
following short courses of antibiotics) are also sometimes seen, particularly if the
diagnosis was not suspected. Chest x-ray typically shows pneumonic opacification
in which a cavity, often with a fluid level, is visible. Pulmonic abscesses can be
caused by anaerobes (most common, particularly if aspiration initiated the abscess),
gram-negative aerobic bacilli, and Staphylococci. Therapy is based on the
organisms isolated, and should be continued for at least 4 to 6 weeks. In cases that
fail to resolve, the possibility of coexisting carcinoma should be considered.
Choice B is the x-ray appearance of pleural effusion.
Choice C is the x-ray appearance of lobar pneumonia.
Choice D is the x-ray appearance of bronchopneumonia.
Choice E is the x-ray appearance of bronchiectasis.
________________________________________

31>The correct answer is C. The corpus luteum secretes estrogens, progesterone,


and relaxin. hCG, secreted by the syncytiotrophoblast lining the placental villi,
maintains the corpus luteum during the first trimester of pregnancy.
Antidiuretic hormone (vasopressin; choice A) does not play a significant role during
pregnancy.
FSH (choice B) acts on granulosa cells to promote the conversion of
androstenedione to estradiol.

LH (choice D) acts on theca cells to promote androstenedione secretion.


Progesterone (choice E) is important for maintaining the pregnancy, however, it
does not act to maintain the corpus luteum.
________________________________________

32>The correct answer is C. The boy probably has Klinefelter's syndrome (47, XXY),
which has the typical presentation described in the question. The condition arises as
a result of failure of separation (nondisjunction) of the sex chromosomes, and can
be related to either paternal nondisjunction (slightly more common) or maternal
nondisjunction.
Deletions (choice A) are a common form of genetic disease and contribute to many
genetic recessive diseases.
Examples of nondisjunction of autosomes (choice B) include trisomies such as most
cases of Down's syndrome (trisomy 21), Edwards' syndrome (trisomy 18), and Patau
syndrome (trisomy 13).
There are two types of translocations: non-Robertsonian (choice D) and
Robertsonian (choice E). Non-Robertsonian (reciprocal) translocations result when
two non-homologous chromosomes exchange genetic material. Robertsonian
translocations are a special type of translocation that involve exchange of genetic
material from the long arms of one acrocentric chromosome to the long arms of
another acrocentric chromosome, with fusion of the centromeres. Four percent of
cases of Down's syndrome are caused by this mechanism.
________________________________________

33>The correct answer is E. A femoral hernia exits the abdominal cavity through
the femoral ring to enter the femoral canal. The femoral ring is bounded medially by
the lacunar ligament. Compression of the herniated intestine against the sharp edge
of the lacunar ligament may cause strangulation of the intestine. The other borders
of the inguinal ring are the inguinal ligament anteriorly, the pubis posteriorly, and
the femoral vein laterally.
The femoral artery (choice A) is within the femoral sheath along with the femoral
hernia, but is lateral to the femoral vein, which separates the artery from the hernia.
The femoral nerve (choice B) is not inside of the femoral sheath, but is lateral to the
sheath.
The femoral vein (choice C) is lateral to the femoral canal within the femoral sheath.

The inferior epigastric artery (choice D) is not related to a femoral hernia. This
artery is on the anterior abdominal wall and passes immediately medial to the deep
inguinal ring. Indirect inguinal hernias pass lateral to this artery and direct inguinal
hernias pass medial to this artery.
________________________________________

34>The correct answer is B. The border modification consists of cilia on the surface
of pseudostratified columnar epithelium. Cilia are shorter than stereocilia and
usually appear bent or wavy in sections. Kartagener's syndrome is one type of
immotile cilia syndrome in which the dynein arms of microtubules are missing or
defective. Thus, cilia cannot move properly and all functions associated with them
are affected (mucous sweeping or ciliary elevator functioning, sperm motility,
embryonic cell movement, etc.). The result is infertility, situs inversus,
bronchiectasis and/or sinusitis.
Implantation (choice A) is not affected in patients with Kartagener's syndrome. The
fertilized ovum can still reach the endometrium and implantation can occur. Also,
this is not a section of the uterine tube; the uterine tube has a simple columnar
epithelium with peg cells (secretory) and ciliated cells.
The border modification consists of cilia, not microvilli and the specimen is not a
section of the intestine, therefore malabsorption (choice C) is incorrect. Both the
small and large intestine have a simple columnar epithelium with microvilli.
Microvilli are upright and irregular and resemble a "flat top" haircut across the top of
cells.
Disaccharidases (choice D) are present in the cell membrane of microvilli.
Deficiencies in digestion occur with the loss of microvilli.

________________________________________

35>The correct answer is D. While not of major medical concern, urine color
changes as a result of medications can be very distressing to patients. These
changes are consequently worth learning so that you can warn patients when you
prescribe the medicine. Rifampin is a safe drug that is used both for prophylaxis and
for active tuberculosis therapy. In addition to discoloring urine, rifampin and its
metabolites can discolor feces, saliva, sweat, and tears (and apparently can stain
soft contact lenses).
Ethambutol (choice A) is an antituberculosis agent that is added if isoniazidresistance is suspected and does not alter urine color.

Isoniazid (choice B) remains the principle antituberculosis agent and does not alter
urine color.
Pyridoxine (choice C) is used principally in prophylaxis against tuberculosis and does
not alter urine color.
Streptomycin (choice E), which must be given intramuscularly, is sometimes used
as an adjunctive agent very early in antituberculosis therapy and has sometimes
been lifesaving in critically ill patients. It does not cause a urine color change.
________________________________________

36>The correct answer is B. This question illustrates an important strategy: knowing


what you're looking for before you consider the answer choices. If you thought
about the answer before considering the choices, this question was very
straightforward and simple. If, on the other hand, you considered each answer
choice in turn, you no doubt got pretty confused and wasted a lot of precious test
time.
The first thing to remember is that nephrotic syndrome is defined as proteinuria
(over 3.5 gm/day) with concurrent hypoalbuminemia and hyperlipidemia. The loss of
protein in the urine results in a decreased oncotic pressure in the vascular space
(decreased pc). This decrease in capillary oncotic pressure promotes movement of
fluid into the interstitium and the development of edema. This is also the cause of
edema in patients with liver disease.
Decreased interstitial oncotic pressure (pi; choice D) would actually promote the
movement of fluid into the vasculature; it would not lead to edema. The same thing
would occur with decreased capillary hydrostatic forces (Pc; choice A).
While decreased interstitial hydrostatic pressure (Pi; choice C) would lead to edema,
it is not the mechanism of action in nephrotic syndrome.
While increased capillary hydrostatic pressure (choice E) does lead to edema, it is
not the mechanism at work in nephrotic syndrome. It is, however, the mechanism of
edema in the setting of congestive heart failure (increased capillary hydrostatic
pressure due to inefficient pumping of the heart, leading to pooling) and in
glomerulonephritis (increased intravascular volume due to inefficient excretion by
the kidney).
Increased capillary oncotic pressure (choice F) would not lead to edema.
Increased interstitial hydrostatic pressure (choice G) would not lead to edema.
Increased interstitial oncotic pressure (choice H) would cause edema, but not in the
setting of nephrotic syndrome. Instead, this is the mechanism of edema (typically
localized) in the setting of burns and inflammation (increased capillary permeability
allows protein to leak into interstitium and increase oncotic pressure).
________________________________________

37>The correct answer is D. Looking at the coding segment of the normal b-gene of
hemoglobin, one should read the information codon by codon, as follows:
AAG

UAU

CAC

UAA

GCU

CGC

The normal b-globin gene has a stop codon (UAA) at the 4th position, therefore the
last 2 codons (GCU and CGC) are not translated and do not code for amino acid
residues found in the protein. Comparing this information to the coding segment of
the mutated b-gene of hemoglobin Cranston, one would notice the following:
AAG

AGU

AUC

ACU

AAG

CUC

GCU

UUC

...UAU

...etc

UAA

The insertion of two base pairs (AG) results in a frameshift mutation that eliminates
the stop codon at position 4, thereby causing the addition of amino acids normally
not translated in the hemoglobin b-chain of the child. Since the chain is now too
long, this destabilizes the tetrameric conformation of hemoglobin.
A frameshift mutation resulting in deletion of several amino acids (choice A) is
wrong, since such a mutation would have inserted a stop codon (UAA, UGA or UAG)
before position 4.
A mutation in the stop codon (choice B) would have resulted in a longer-than-normal
b-globin gene, but the information given does not indicate any changes in the stop
codon at position 4. Interestingly, a chain elongation by mutation in the stop codon
exists and is known as hemoglobin Constant Spring, affecting the a-chain of
hemoglobin.
A point mutation (choice C) is the result of a single base pair change, which is not
the case here. A point mutation resulting in the insertion of a new stop codon is
called a nonsense mutation, and it would result in a shorter-than-normal protein.
A two base pair deletion (choice E) is not evidenced by the coding sequence given
in the stem. It would also cause a frameshift in the reading frame with the possible
result of a longer or shorter protein, most likely with abnormal function due to the
change in primary structure (amino acid sequence).
Strategy note: Since the hemoglobin Cranston b-chain is clearly longer than the
normal b-chain, choices A, C, and E, which would produce a shorter chain, can be
eliminated immediately.
________________________________________

38>The correct answer is B. The drawing depicts the histology of the small
intestinal villus and crypts. The arrow points to a Paneth cell. Paneth cells are
unique to the small intestine. They are found at the base of the crypts of Lieberkhn

in the small intestinal villi, and have many secretory granules that contain enzymes
with bactericidal activity.
The enteroendocrine cells of the stomach produce low-molecular weight
polypeptides (choice A).
There are no lipase-secreting cells in the small intestine (choice C). Lipases are
secreted by cells in the salivary glands, stomach, and exocrine pancreas. So, even if
you did not know that the arrow pointed to Paneth cells, you should have eliminated
this choice based on the fact that the small intestine does not secrete lipase.
The goblet cells of the small intestinal villi are responsible for producing protective
glycoproteins called mucins (choice D). After secretion, they are hydrated and
become a mucous layer that protects the intestinal epithelium. There are many
goblet cells depicted in this figure, but they are not located at the bases of the
crypts.
Pepsinogen (choice E) is secreted by the chief cells of the stomach.
________________________________________

39>The correct answer is D. Many people who request physician-assisted suicide


have one of two conditions present: either a poorly controlled painful condition or
severe depression. If the painful condition is adequately treated or the depression is
brought under good medical control, the request for physician assistance in
terminating the situation is typically withdrawn. It is important to note that bringing
these conditions under control requires the intervention of caregivers who are
specifically trained in the management of these two conditions; primary care
physicians usually are not adequately trained to address these difficult
presentations.
While patients who are diagnosed as bipolar disorder (choice A), borderline
personality disorder (choice B), and schizophrenic disorder (choice E) often make
suicide attempts (and frequently complete those attempts), they do not generally
ask their physician for assistance in the suicide.
Persons with factitious disorder (choice C) are seeking primary gain, often for
dependency needs, and are seeking to enter the "sick role" not the "dead role.
________________________________________

40>The correct answer is B. First, eliminate all answers in which Drug X does not
produce an increase in blood pressure (BP). Choice A should be eliminated because
acetylcholine stimulates the noninnervated muscarinic (M3) receptors that are
located on endothelial cells of the vasculature. Stimulation of these receptors
releases endothelial-derived relaxing factor (EDRF; nitric oxide), which produces a
relaxation of the neighboring smooth muscle cells, leading to a decrease in BP.

Choice C should be eliminated because isoproterenol (a nonspecific beta agonist)


decreases BP by stimulating beta-2 receptors in the vasculature.
Epinephrine, norepinephrine, and phenylephrine all increase BP, so the remaining
answers must be eliminated by examining the effects of Drug Y on Drug X. Start
with choice B: Epinephrine is an agonist at alpha-1, alpha-2, beta-1, and beta-2
receptors; phentolamine is an antagonist at alpha-1 and alpha-2 receptors.
Therefore, after the administration of phentolamine, epinephrine can stimulate only
beta receptors, which would produce a decrease in BP. Epinephrine is now acting
like isoproterenol. This is called epinephrine reversal (the name stems from the fact
that epinephrine originally increases BP and then produces the opposite effect after
phentolamine administration). Therefore, choice B is correct.
Choice D: Norepinephrine is an agonist at alpha-1, alpha-2, and beta-1 receptors;
propranolol is a nonselective beta antagonist. After administration of propranolol,
norepinephrine can stimulate only alpha receptors, which will still cause
vasoconstriction (primarily via alpha-1 stimulation in the vasculature) and therefore
increase BP.
Choice E: Phenylephrine is an alpha-1 agonist; hexamethonium is a nicotinic
ganglionic blocker. Hexamethonium administration would be predicted to eliminate
the baroreceptor response after the second phenylephrine administration by
blocking the peripheral ganglia. However, phenylephrine will still reach the alpha-1
receptors on the vasculature to produce an increase in blood pressure.
________________________________________

41>The correct answer is A. Bilirubin is a degradative product of hemoglobin


metabolism. Bilirubin (pigment) stones are specifically associated with excessive
bilirubin production in hemolytic anemias, including sickle cell anemia. Bilirubin
stones can also be seen in hepatic cirrhosis and liver fluke infestation.
Calcium oxalate stones (choice B) and cystine stones (choice E) are found in the
kidney, rather than the gallbladder.
Pure cholesterol stones (choice C) are less common than mixed gallstones, but have
the same risk factors, including obesity and multiple pregnancies.
Mixed stones (choice D) are the common "garden variety" gallstones, found
especially in obese, middle aged patients, with a female predominance.
________________________________________

42The correct answer is C. The probable organism is Giardia lamblia, which


characteristically infects the small intestine. When seen in smears from duodenal
aspirates, this flagellated organism has a characteristic "face-like" appearance.
However, in biopsy specimens, the organism is often caught at an angle, and the
characteristic appearance and location is as described in the question stem. Giardia

is a common contaminant of water supplies (even in the United States), and


patients who ingest the cysts may be asymptomatic or may occasionally develop
prolonged diarrhea or intestinal malabsorption. Some patients with severe disease
have low serum IgA or low overall immunoglobulin levels.
Entamoeba histolytica (choice A) usually infects the large intestine and/or liver.
Escherichia coli (choice B) is a bacterial cause of diarrhea.
Naegleria fowleri (choice D) causes meningoencephalitis.
Trichomonas vaginalis (choice E) causes vaginitis.
________________________________________

43The correct answer is A. The spinal cord arises from the caudal end of the neural
tube. During development, an alar and a basal plate is formed, separated by a
longitudinal groove called the sulcus limitans (choice E). The alar plate forms the
dorsal (posterior) part of the spinal cord and becomes the sensory or afferent
portion of the cord. The basal plate (choice B) is the ventral (anterior) part of the
cord, and becomes the motor, or efferent, portion of the spinal cord, and therefore
would contain anterior horn cells.
The neural crest (choice C) develops into multipolar ganglion cells of autonomic
ganglia, pseudounipolar cells of spinal and cranial nerve ganglia, leptomeningeal
cells, Schwann cells, melanocytes, chromaffin cells of the adrenal medulla, and
odontoblasts.
The brain forms from the rostral end of the neural tube (choice D).
________________________________________

44>The correct answer is C. While the third-week embryo is a primitive trilaminar


plate, in the fourth week, the heart begins to form and begins beating almost
immediately. Hematopoiesis occurs in the yolk sac, and a primitive circulatory
system connects the capillary plexuses of the yolk sac and chorion to the embryo.
Partitioning of the atrium also begins in the fourth week. During the fifth week,
cardiac septa form and the atrioventricular (AV) cushions fuse. By the sixth week,
the heart is close to fully formed. This early sequence for the heart explains why it is
so difficult to try to prevent congenital malformations of the cardiovascular system
from occurring, since the mother of a 6-week-old fetus is only about 8 weeks from
her last menstrual period, and may have assumed that she just "missed a period" (a
very common phenomenon) for reasons other than pregnancy.
________________________________________

45>The correct answer is C. Laryngeal cartilages (e.g., the thyroid, cricoid,


arytenoid cartilages) are derived from neural crest.
The endothelial cells (choice A) in the simple squamous epithelium that lines the
pulmonary capillaries are derived from visceral mesoderm.
The epithelial lining of primary bronchi (choice B) is derived from endoderm.
Tracheal glands (choice D) and epithelium both derive from endoderm.
Type I pneumocytes (choice E) are derived from endoderm.
________________________________________

46The correct answer is C. The muscularis of the upper third of the esophagus
(choice E) is composed entirely of striated muscle. The middle third (choice C)
contains both striated and smooth muscle. The lower third (choice B) and lower
esophageal sphincter (choice A) contain only smooth muscle. There is no such thing
as the upper esophageal sphincter (choice D).
________________________________________

47>The correct answer is D. The cells of the anterior pituitary can be classified as
chromophils (love dyes) or chromophobes (do not stain with dyes). The chromophils
can be further divided into acidophils (stain with acidic dyes) and basophils (stain
with basic dyes). The acidophils include the somatotropes, which secrete growth
hormone, and the mammotropes, which secrete prolactin. The basophils include the
corticotropes, which secrete ACTH (choice A), the gonadotropes, which secrete FSH
and LH (choices B and C), and the thyrotropes, which secrete TSH (choice E).
________________________________________

48>The correct answer is E. The disease is myotonic dystrophy, which is an


autosomal dominant disease; the affected gene has been localized to chromosome
19. Myotonic dystrophy is relatively common and is best thought of as a systemic
disease, since it causes cataracts, testicular atrophy, heart disease, dementia, and
baldness in addition to muscular weakness.
A mutation on the X chromosome (choice A) causes Duchenne muscular dystrophy.

None of the muscle diseases are known to be related to defects on the Y


chromosome (choice B).
Facioscapulohumeral dystrophy is associated with a defective gene on chromosome
4 (choice C).
Infantile hypotonia has been related to defective genes on chromosome 5 (choice
D).
________________________________________

49>The correct answer is E. Urinary tract infections are the most common bacterial
infections encountered during pregnancy, and Escherichia coli is the most
commonly isolated organism. 70% of cases in the U.S. are caused by P pili-positive
strains.
Bundle-forming pili (choice A) are found in enteroaggregative E. coli (EAEC).
GVVPQ fimbriae (choice B) are found in EAEC.
Heat labile toxins (choice C) are pathogenic factors in enterotoxic strains (ETEC).
Heat stable toxins (choice D) are pathogenic factors in ETEC or EAEC.
Type 1 pili (choice F) are a major pathogenic factor in ETEC.
________________________________________

50>The correct answer is E. The baby probably has congenital hypertrophic pyloric
stenosis, which usually presents at several weeks of age. Partial surgical incision
through the pylorus (pyloromyotomy) is usually curative.
Antacids (choice A) are beneficial in esophageal reflux and peptic ulcer disease.
Barium enema (choice B) can reverse intussusception in a child, but would not be
therapeutic in this case.
Gastric resection (choice C) is not indicated, since the much less invasive procedure
of pyloromyotomy is actually more effective.
Oral antibiotics (choice D) are not indicated, since this is not an infectious process.

Answers

________________________________________

________________________________________

1>The correct answer is B. The most probable etiology of bacterial endocarditis


involving the tricuspid valve is illicit intravenous drug use, which can introduce skin
organisms into the venous system that then attack the tricuspid valve.
Staphylococcus aureus accounts for between 60% and 90% of cases of endocarditis
in intravenous drug users.
The endocarditis associated with congenital heart disease (choice A) typically
involves either damaged valves or atrial or ventricular septal defects. The tricuspid
valve is not particularly vulnerable.
Rheumatic fever (choice C) most commonly damages the mitral and aortic valves,
and tricuspid damage is usually less severe and seen only when the mitral and
aortic valves are heavily involved. Consequently, secondary bacterial endocarditis
involving only the tricuspid valve in a patient with a history of rheumatic fever
would be unusual.
Rheumatoid arthritis (choice D) is not associated with bacterial endocarditis.
Systemic lupus erythematosus (choice E) can produce small, aseptic vegetations on
valves, but is not associated with bacterial endocarditis.
________________________________________

2>The correct answer is B. C3b is the most critical molecule in both the classical
and alternative complement pathways. C3, the most abundant protein of all the
complement proteins, is cleaved into C3a and C3b. C3b attaches to bacterial
surfaces for opsonization by phagocytes. C3a binds to mast cells and basophils,
activating them and producing histamine release.
C1q (choice A) is a complement component in the classical pathway. It binds to the
constant heavy domain of an IgG molecule that has reacted with the bacterial
surface epitope. C1q is not involved in opsonization as it simply functions as an
enzyme in the early complement cascade.
C5 (choice C) is a protein, that once split into C5b, initiates the assembly of the
membrane attack complex. This complex consists of C5b, C6, C7, C8, and
polymerization of C9, and is responsible for lysis of the bacteria. C5a is a strong
chemotactic molecule.
This fragment is the result of C5 being split by the C5 convertases of both
pathways. This C5a (choice D) is a strong chemotactic factor for neutrophils and
results in stimulating the inflammatory response.

Factor B (choice E) is an activator protein of the alternative pathway. It combines


with C3b to form C3bBb. This C3bBb is the C3 convertase of the alternative
pathway.

________________________________________

3>The correct answer is E. The findings described are those of tabes dorsalis, a
form of tertiary syphilis caused by Treponema pallidum. Tabes dorsalis, and other
forms of tertiary syphilis, are now uncommon in this country, possibly because the
common use of antibiotics may "treat" many unsuspected cases of syphilis.
Haemophilus influenzae (choice A) and Neisseria meningitidis (choice D) can cause
meningitis.
Neisseria gonorrhoeae (choice C) causes gonorrhea, which usually does not involve
the CNS.
Herpes simplex I (choice B) can cause an encephalitis that typically involves the
frontal and temporal lobes.

________________________________________

4>The correct answer is E. Urinary tract infections are the most common bacterial
infections encountered during pregnancy, and Escherichia coli is the most
commonly isolated organism. 70% of cases in the U.S. are caused by P pili-positive
strains.
Bundle-forming pili (choice A) are found in enteroaggregative E. coli (EAEC).
GVVPQ fimbriae (choice B) are found in EAEC.
Heat labile toxins (choice C) are pathogenic factors in enterotoxic strains (ETEC).
Heat stable toxins (choice D) are pathogenic factors in ETEC or EAEC.
Type 1 pili (choice F) are a major pathogenic factor in ETEC.

________________________________________

5>The correct answer is D. This patient has developed hemolytic-uremic syndrome


(HUS), a complication of the Shiga toxin or Shiga-like toxin: exotoxins released by
Shigella species and the enterohemorrhagic E.coli. HUS in children usually develops
after a gastrointestinal or flu-like illness, and is characterized by bleeding, oliguria,
hematuria and microangiopathic hemolytic anemia. Presumably the Shiga toxin is
toxic to the microvasculature, producing microthrombi that consume platelets and
RBCs, and may fragment the red cell membrane.
The incorrect choices are all bacteria which may produce an enterocolitis, but do not
elicit HUS.
A long-term consequence of Campylobacter (choice A) infection is a reactive
arthritis or full-blown Reiter's syndrome.
Clostridial enterocolitis is produced by Clostridium difficile (choice B), a normal
inhabitant of the gut that produces pseudomembranous colitis when other gut flora
are suppressed by treatment with antibiotics.
In the United States, Salmonella infections (choice C) are almost all non-typhoid
inflammatory diarrhea, producing a simple enterocolitis that may proceed to sepsis
in some cases. Typhoid fever (produced by Salmonella typhi and S. paratyphi)
produces a protracted illness that progresses over several weeks and includes rash
and very high fevers, but not HUS.
Vibrio (choice E) infections produce copious amounts of watery diarrhea, and the
major risk of cholera and other Vibrio enteritides is shock due to hypovolemia or
electrolyte loss.

________________________________________

6>The correct answer is E. Take rashes involving the palms and soles (otherwise
unusual sites) very seriously: only a small number of infections can cause this
pattern, including Rocky Mountain spotted fever, meningococcemia, and secondary
syphilis. Rocky Mountain spotted fever is caused by the rickettsia R. rickettsii, and is
found throughout the United States, particularly in south central and eastern
portions (not the Rocky Mountains). 3-12 days after a tick bite, patients develop
malaise, frontal headache, and fever. Several days later, the rash described in the
question stem develops. Other manifestations can include hepatosplenomegaly,
thrombocytopenia, and (potentially fatal) disseminated intravascular coagulation.
Chickenpox (choice A), or varicella, is characterized by maculopapules that evolve
into vesicles over hours to days, then eventually form crusts. Typical lesions first
appear on the trunk and face and rapidly spread to involve other areas.
The maculopapular rash of German measles (choice B), or rubella, usually begins on
the face, then spreads down the body.

Although the maculopapular rash of measles (choice C), or rubeola, can include the
palms and soles, it typically begins along the hairline in frontal and temporal
regions, then spreads down the trunk to the limbs.
Mumps (choice D) is characterized by fever, malaise, parotitis, and orchitis, but not
a rash.

________________________________________

7>The correct answer is E. The patient is suffering from hyperacute rejection due to
the preformed anti-B ABO blood group antibody found in all type A positive
individuals. Hyperacute rejection occurs within minutes to a few hours of the time of
transplantation, and is due to the destruction of the transplanted tissue by
preformed antibodies reacting with antigens found on the transplanted tissue that
activate complement and destroy the target tissue. Preformed antibodies can also
be due to presensitization to a previous graft, blood transfusion, or pregnancy.
Acute rejection due to antibody-mediated immunity (choice A) is incorrect because
this patient suffered from hyperacute rejection (immediate) occurring within
minutes to hours, rather than days.
Acute rejection due to cell-mediated immunity (choice B) will not occur until several
days or a week following transplantation. Acute rejection is due to type II and type
IV reactions.
Chronic rejection, due to the presence of cell-mediated immunity to minor HLA
antigens (choice C), occurs in allograft transplantation months to even years after
the transplant. Chronic rejection is generally caused by both humoral and cellmediated immunity.
An accelerated acute rejection, occurring in 3-5 days, can be caused by tissue
infiltration and destruction by presensitized T lymphocytes and macrophages
(choice D) and/or antibody-dependent, cell-mediated cytotoxicity (ADCC). Note that
this is not a hyperacute reaction.

________________________________________

8>The correct answer is B. The presentation is typical for pelvic inflammatory


disease (PID). Chlamydia trachomata (serotypes D-K) is the most common bacterial
cause of sexually transmitted disease (STD) in this country and is the most likely
agent on the list to produce the symptoms described. It is an ATP-defective
organism that must therefore live intracellularly in the human host and can be
visualized inside epithelial cells with iodine, Giemsa, or fluorescent-antibody stains.
The remainder of the answer choices refer to other agents that could be found in

the female genital tract, either by sexual transmission or by contamination with


fecal flora, but they are not the best choices.
A naked, icosahedral double-stranded circular DNA virus (choice A) refers to human
papilloma virus, which is the most common cause of STDs in the U.S., but presents
with anogenital warts.
Intraneutrophilic gram-negative diplococci (choice C) refers to Neisseria
gonorrhoeae, which would be expected to present with dysuria and neutrophilic
exudate.
Intranuclear "owl's eye" inclusion bodies (choice D) refers to cytomegalovirus, a
common STD in the United States, but not a common agent of PID. Most cases in
average adults are manifested by mononucleosis-like symptoms.
Lactose-fermenting gram-negative bacilli (choice E) would be consistent with
Escherichia coli. Although this organism is the most common cause of urinary tract
infections in women in the United States, it would not be expected to cause PID.
Pear-shaped flagellated protozoa (choice F) refers to the protozoan parasite
Trichomonas vaginalis, the only protozoan STD in the world. Infection would be
characterized by a malodorous, cheesy exudate, and there would be more erythema
of the external genitalia than of the cervix.
Pleomorphic, gram-negative rods (choice G) are consistent with Hemophilus ducreyi,
which causes chancroid, and presents in a different manner.
Spirochetes on dark-field microscopy (choice H) refers to Treponema pallidum, the
causative agent of syphilis, which would produce rash and/or chancre, depending on
the stage of the infection.

________________________________________

9>The correct answer is B. The principle host defense in mycobacterial infections


(such as this patient's tuberculosis) is cell-mediated immunity, which causes
formation of granulomas. Unfortunately, in tuberculosis and in many other
infectious diseases characterized by granuloma formation, the organisms may
persist intracellularly for years in the granulomas, only to be a source of activation
of the infection up to decades later.
While antibody-mediated phagocytosis (choice A) is a major host defense against
many bacteria, it is not the principle defense against Mycobacteria.
IgA-mediated hypersensitivity (choice C) is not involved in the body's defense
against Mycobacteria.
IgE-mediated hypersensitivity (choice D) is not involved in the body's defense
against Mycobacteria. It is important in allergic reactions.

Neutrophil ingestion of bacteria (choice E) is a major host defense against bacteria,


but is not the principle defense against Mycobacteria.

________________________________________

10>The correct answer is A. This patient has the classic symptoms of cat scratch
disease caused by the bacillus Bartonella henselae. The disease is self-limited with
the onset of symptoms occurring 3-10 days following an inoculating scratch. The
organism can be isolated from kittens, typically less than 1 year of age, or from
fleas. A history of a new kitten in the house and the papule at the site of a scratch
with regional painful adenopathy defines the classic scenario.
Chickens (choice B) can harbor Salmonella spp. producing a gastroenteritis or
enterocolitis. Chicken guano is also a favorable environment for the fungus
Histoplasma capsulatum. The mycelial phase thrives in the rich soil. The human
disease is a granulomatous infection involving the lungs and mimicking
tuberculosis.
Dogs or puppies (choice C) carry Capnocytophaga canimorsus as part of the normal
flora of the oral cavity. Infections from licking or biting range from a self-limited
cellulitis to fatal septicemia. Patients at risk for more severe infections are those
with asplenia, alcoholism, or hematologic malignancies. This organism is also
associated with cat bites, but the patient develops cellulitis and fulminant
septicemia, especially in asplenic patients. Pasteurella multocida is another
pathogen that colonizes the nasopharynx and gastrointestinal tract of cats and
dogs. Cats have the highest rate of colonization (50-90%), followed by dogs (50%),
swine (50%), and rats (14%). P. multocida most commonly causes a localized soft
tissue infection or cellulitis after an animal bite, but systemic symptoms may be
present in about 40% of the cases. These symptoms include osteomyelitis, septic
arthritis, or tenosynovitis.
Horses (choice D) and horse manure have been associated with a pulmonary
opportunistic infection with cavitation caused by Rhodococcus equi that resembles
tuberculosis in immunocompromised patients. Burkholderia mallei (the cause of
glanders) is characterized by non-caseating granulomatous abscesses of skin,
lymphadenopathy, and pronounced involvement of the lungs.
Parrots (choice E) are associated with psittacosis caused by Chlamydia psittaci.
Psittacosis is associated with a dry, hacking cough productive of scant sputum, an
interstitial infiltrate in the lungs, severe headache, and myalgias. A pale macular
rash is also seen.

________________________________________

11>The correct answer is A. Bacillary angiomatosis is a disease that occurs


primarily in AIDS patients, and is indicative of a defect in cell-mediated immunity. It
is caused by either Bartonella henselae or Bartonella quintana. The domestic cat is
the reservoir for these organisms and they are usually transmitted to humans via a
cat scratch or cat bite. Patients with this illness usually have multiple skin lesions
and extracutaneous manifestations involving liver and bone. Diagnosis is usually
based on characteristic histopathologic findings including plump "epithelioid"
endothelial cells and mitotic figures. A macrolide, such as erythromycin or
azithromycin, is the drug of choice for the infection.
Human papillomavirus (choice B) causes warts. Infection can present as a sessile
wart or as condyloma acuminatum, which are fleshy soft growths that coalesce into
large masses. When cellular immunity is depressed, as in AIDS, the condylomata
acuminatum proliferate.
Molluscum contagiosum virus (choice C) is a pox virus that is spread by close
person-to-person contact. Infection produces a firm nodule that often becomes
umbilicated, and may resolve by discharging its contents. In AIDS, the lesions do not
resolve, but enlarge and spread.
Rickettsia prowazekii (choice D) is the cause of epidemic typhus. It is spread by the
human body louse, Pediculus humanis. Its reservoirs are humans and flying
squirrels.
Treponema pallidum (choice E) is the spirochete that causes syphilis. The
characteristic primary lesion is a chancre (a painless, indurated ulcer) at the site of
inoculation.

________________________________________

12>The correct answer is E. This is the classic presentation of a psoas abscess. This
clinical entity was formerly a fairly common complication of vertebral tuberculosis,
but is now rare in clinical practice in this country. The psoas muscle is covered by a
fibrous sheath known as the psoas fascia. This sheath is open superiorly, permitting
an infection involving the soft tissues around the spine to enter the sheath, then
track down to the groin.
The adductor longus (choice A) is a muscle of the anterior thigh, and is not related
to the lumbar portion of the back.
The gluteus maximus (choice B) gluteus minimus (choice C) and piriformis (choice
D) are muscles of the buttock with no relationship to the groin.

________________________________________

13>The correct answer is C. The patient has a fever and is extremely ill. The most
important clue to the diagnosis is the presence of bulky vegetations on the tricuspid
valve, indicating that he has infective endocarditis. On the basis of the rapid clinical
course, this is likely a case of acute infective endocarditis. The diagnosis of this
condition must be confirmed by blood cultures, which are also necessary to
determine bacterial antibiotic sensitivity. S. aureus (commonly present on the skin)
is the most frequent etiologic agent of infective endocarditis in intravenous drug
abusers. It commonly affects the tricuspid valve. Because of its high virulence, S.
aureus-related endocarditis follows an acute course and may lead to death within a
few days.
The causative agents of infective endocarditis differ depending on host factors.
Fungal organisms, such as Candida albicans (choice A), may cause infective
endocarditis in severely immunosuppressed patients, such as those with AIDS.
A minority of cases of infective endocarditis are caused by a number of normal
commensals in the oral cavity, i.e., the "HACEK" group: Hemophilus (choice B),
Actinobacillus, Cardiobacterium, Eikenella, and Kingella.
S. epidermidis (choice D) and other coagulase-negative staphylococci tend to
produce endocarditis in recipients of prosthetic valves.
Viridans streptococci (choice E) are the most frequent agents causing endocarditis
in previously abnormal valves, such as those damaged by rheumatic disease, or
congenitally abnormal valves. Coagulase-negative staphylococci and viridans (ahemolytic) streptococci are less virulent than S. aureus and are thus associated with
a subacute (more prolonged) clinical course and a better prognosis

________________________________________

14>The correct answer is A. Campylobacter jejuni is a pathogen causing an invasive


enteric infection associated with ingestion of raw or undercooked food products, or
through direct contact with infected animals. In the U.S., ingestion of contaminated
poultry that has not been sufficiently cooked is the most common means of
acquiring the infection. The patients typically have bloody diarrhea, abdominal pain,
and fever. The presence of fecal leukocytes indicates an invasive infection. The
organism is a gram negative rod with a "comma-shape."
Enterotoxigenic E. coli (choice B) causes the classic traveler's diarrhea. The infection
is non-invasive and is acquired via the fecal-oral route through consumption of
unbottled water or uncooked vegetables. The major manifestation is a copious
outpouring of fluid from the GI tract presenting as explosive diarrhea. This is due to
the action of one of two types of enterotoxins on the GI tract mucosa.
Shigella sonnei (choice C) produces a syndrome very similar to C. jejuni. However,
the organism appears as a gram-negative rod on Gram's stain. It does not have a
comma shape. Transmission is from person to person via the fecal-oral route.

Infection requires a low infective dose since the organism is fairly resistant to
gastric acidity.
Staphylococcus aureus (choice D) produces food poisoning due to the ingestion of a
pre-formed enterotoxin. The organism is present in food that is high in salt content
such as potato salad, custard, milk shakes, and mayonnaise. The patient presents
with nausea, vomiting, and abdominal pain, followed by diarrhea beginning 1-6
hours after ingestion of the enterotoxin.
Vibrio cholerae (choice E) produces a secretory diarrhea due to increases in cAMP in
the intestinal cells. The organism is not invasive. The patient presents with the
sudden onset of painless, watery diarrhea that becomes voluminous, followed by
vomiting. The stool appears nonbilious, gray, and slightly cloudy with flecks of
mucus, no blood, and a sweet odor.

________________________________________

15>The correct answer is D. The lesions are characteristic of molluscum


contagiosum, which is a typically benign and self-limited condition caused by a
poxvirus. The disease can be transmitted either venereally or through non-venereal
contact. The other viruses listed do not cause similar skin lesions. Patients with
advanced HIV infection may develop a severe, generalized, and persistent eruption,
often involving the face and upper body.
Cytomegalovirus (choice A) causes congenital infections and disseminated
infections in immunosuppressed patients.
Herpesvirus 6 (choice B)causes roseola (exanthem subitum).
Parvovirus (choice C) causes aplastic crises in patients with hemolytic anemia.
Variola (choice E) is the smallpox virus.

________________________________________

16>The correct answer is D. This patient has Wegener's granulomatosis, which is


characterized by renal involvement, severe upper respiratory tract symptoms, and
pulmonary involvement. Other organ systems may also be involved. The renal
syndrome is a crescentic rapidly progressive glomerulonephritis leading to renal
failure. The upper respiratory tract findings include sinus pain and drainage, and
purulent or bloody nasal discharge with or without nasal ulcerations. Nasal septal
perforation may follow. Pulmonary involvement may be clinically silent with only

infiltrates present on x-ray, or it may present as cough and hemoptysis. c-ANCA is a


marker for Wegener's granulomatosis, present in a high percentage of patients.
Anti-centromere antibody (choice A) is associated with approximately 90% of cases
with CREST syndrome (calcinosis, Raynaud's phenomenon, esophageal motility
syndrome, sclerodactyly, and telangiectasia) which is also called limited
scleroderma.
Anti-Ro (choice B) is also called anti-SS-A and is associated with Sjgren's syndrome
(70-95%).
Anti-SS-B (choice C) is associated with Sjgren's syndrome (60-90%).
Decreased ESR (choice E) is not a marker of Wegener's. Instead, a markedly
elevated ESR is seen. Additionally, mild anemia, thrombocytosis, leukocytosis, mild
hypergammaglobulinemia (IgA), and mildly elevated rheumatoid factor are seen in
this disorder.

________________________________________

17>The correct answer is E. Parvoviruses are small single-stranded DNA viruses, of


which only serotype B19 is pathogenic for humans. This virus causes three distinct
syndromes: a childhood febrile rash known as erythema infectiosum ("Fifth
disease"); aplastic crisis in individuals with chronic hemolytic diseases (sickle cell
anemia, thalassemia, etc); and congenital infections that can present as stillbirth,
hydrops fetalis (analogous to severe Rh incompatibility), or severe anemia.
Coxsackie viruses (choice A) usually cause cold-like illness, but can cause
herpangina, myocarditis, and meningitis.
Echoviruses (choice B) can infect a variety of organ systems (GI, CNS, eyes, heart,
respiratory, skin), but are not a cause of aplastic crises.
Hepadnavirus (choice C) is the causative agent of hepatitis B.
Herpes viruses (choice D) cause a variety of acute to chronic infections including
herpes simplex types I and II, chicken pox, chronic herpes zoster, CMV infection, and
Epstein Barr virus infections.

________________________________________

18>The correct answer is A. The cavernous sinuses are located on either side of the
body of the sphenoid bone, and become a potential route of infection because they
receive blood both from the face (via the ophthalmic veins and sphenoparietal
sinus) and some of the cerebral veins. The spread of infection, especially by Mucor

sp., into the cavernous sinus, can produce either CNS infection or cavernous sinus
thrombosis, both of which are potentially fatal.
The route from the face to the brain is not arterial (choices B and C).
The superior sagittal sinus (choice D) is located in the falx cerebri, and drains
venous blood from the brain to other dural sinuses, from which it eventually drains
into the jugular vein. Zygomycosis does not reach the brain by way of the superior
sagittal sinus.
The superior vena cava (choice E) drains blood from the upper part of the body into
the heart.

________________________________________

19>The correct answer is A. Streptococci are usually initially speciated by their


hemolytic capacity on sheep blood agar. Beta-hemolytic streptococci include groups
A, B, and D. S. agalactiae is the classic group B streptococcus. The non betahemolytic streptococci consist principally of the pneumococci and the viridans
group.
Both S. agalactiae and pneumococcus have a carbohydrate capsule (choice B), an
important virulence factor and means of subtyping streptococcal species.
None of the streptococci utilize cytochrome enzymes (choice C). They derive all of
their energy from the fermentation of sugars to lactic acid.
Neither pneumococcus nor S. agalactiae can grow in bile (choice D). This ability is
specific for the enterococcus group (group D) of streptococci.
Both pneumococci and S. agalactiae are usually treated with penicillin-type
antibiotics, although group B streptococci require a penicillinase-resistant type such
as oxacillin (choice E).

________________________________________

20>The correct answer is A. Bartonella henselae is the infective agent of cat scratch
disease, which generally presents as regional lymphadenopathy with or without low
fevers and headaches. Bartonella is a gram-variable pleomorphic rickettsial
organism that is introduced to the skin in a cat bite or scratch. It produces a selflimited granulomatous response in the draining lymph nodes.
Borrelia burgdorferi (choice B) is a spirochetal organism that is transmitted by a tick
bite (Ixodes spp.), producing Lyme disease. Lyme disease progresses from a skin
rash to fevers, headache and pain over about one month. It may produce
lymphadenopathy, but is not associated with granuloma formation.

Chlamydia psittaci (choice C) infection occurs after contact with infected bird
droppings and produces an atypical pneumonia. The central nervous system may
also be involved, but lymph nodes are spared. Chlamydia trachomatis is the
chlamydial species that typically produces suppurative nodal granulomas
(lymphogranuloma venereum).
Coxiella burnetii (choice D) infection is transmitted by inhaling dusts or drinking milk
from infected mammals, especially sheep and cows. The disease in humans, Q
fever, is marked by mild nonspecific symptoms or pneumonia, and may progress to
myocarditis or hepatitis.
Rickettsia prowazekii (choice E) produces epidemic (louse-borne) typhus, which is
transmitted by body lice and produces a rash akin to Rocky Mountain spotted fever.
Although the organism may reside in the lymph nodes in dormancy, it does not elicit
granuloma formation.

________________________________________

21>The correct answer is D. Parvovirus B19 causes erythema infectiosum, or Fifth


disease. The 5-year-old boy has the classic "slapped cheek" appearance. Adults
typically do not get the facial rash, but have arthralgias and arthritis. The
symmetrical distribution of involved joints is similar to that in rheumatoid arthritis.
The onset in adults is typically 3 to 4 weeks after exposure. Parvovirus infections
may persist in immunosuppressed patients, resulting in red blood cell aplasia.
Listeriosis (choice A) is caused by the gram-positive rod Listeria monocytogenes.
Meningitis and bacteremia are common clinical manifestations. Elderly, neonates,
pregnant women, and those taking steroids have the highest risk for infection.
Lyme disease (choice B) is caused by Borrelia burgdorferi. It is the most common
vector-borne disease (Ixodes ticks) in the U.S. The incidence is highest in the
summer and fall. The white-footed mouse and the white-tailed deer are zoonotic
reservoirs. From 3 to 32 days following a tick bite, the patient develops fever,
lymphadenopathy, meningismus, and the characteristic rash (erythema migrans).
The rash enlarges and resolves over 3 to 4 weeks. Sequelae include arthritis,
carditis, and neurologic abnormalities.
Mumps (choice C) is caused by a Paramyxovirus. The virus most commonly affects
glandular tissue. Parotitis, pancreatitis, and orchitis are characteristic. Mumps
meningoencephalitis is one of the most common viral meningitides. Mumps
polyarthritis is most common in men between the ages of 20 and 30 years. Joint
symptoms begin 1 to 2 weeks after the parotitis subsides and large joints are
involved.
Reiter's syndrome (choice E) is a seronegative, asymmetric arthropathy
predominantly affecting the lower extremities. It may be triggered by a C.
trachomatis infection. In addition to the arthritis, patients may have urethritis

(which is usually due to chlamydia), conjunctivitis, mucocutaneous disease such as


balanitis, oral ulcerations, or keratoderma. Approximately 80% of patients are HLAB27 po

________________________________________

22>The correct answer is E. Embolization from infective endocarditis typically


causes multiple, small parietal lobe abscesses. This "factoid" is worth knowing
because some patients with infective endocarditis present with what clinically looks
like multiple small "strokes", and their treatable cardiac disease may be completely
unsuspected.

________________________________________

23>The correct answer is E. The most common bacteria implicated in communityacquired pneumonia is the pneumococcus, Streptococcus pneumoniae. Other
organisms frequently implicated in patients less than age 60 without comorbidity
include Mycoplasma pneumoniae, respiratory viruses, Chlamydia pneumoniae, and
Haemophilus influenzae. When community-acquired pneumonia occurs in elderly
patients or patients with comorbidity, aerobic gram-negative bacilli and
Staphylococcus aureus are added to the list.
The organisms listed in choices A, B, and C are important causes of communityacquired pneumonia, but are not the most frequent causes.
Staphylococcus aureus (choice D) is an important cause of community-acquired
pneumonia (particularly in the elderly and in patients with comorbidity), but is not
the most frequent cause.

________________________________________

24>The correct answer is A. The combination of a one week incubation period, the
Eastern seaboard clue, and the intra-erythrocyte parasites strongly suggest Babesia
microti as the infecting organism. The clinical disease is called babesiosis. The
infecting protozoan is related to Plasmodium and is transmitted by the bite of the
Ixodes dammini tick. Occasional cases have been transmitted by blood transfusions.
Most patients are asymptomatic; symptomatic cases in reasonably healthy
individuals have the features listed in the question stem. Rare severe cases, which
may be fatal, may develop in severely debilitated or asplenic individuals and can be
accompanied by severe hemolysis (up to 30% of RBCs may have the parasites) with
subsequent hemoglobinuria, hemolysis, and renal failure.

Leishmania donovania (choice B) is an intracellular tissue protozoan, not a blood


protozoan.
Plasmodium falciparum (choice C) and Plasmodium vivax (choice D) are malarial
parasites not encountered on the Eastern seaboard.
Trypanosoma cruzi (choice E) is an intracellular tissue protozoan, and does not
infect blood cells.

________________________________________

25>The correct answer is A. The patient's presentation is consistent with a simple


urinary tract infection; there is a short history of dysuria, increased urinary
frequency and the appearance of white blood cells and gram-negative rods in the
urine. Urinary tract infections are common in women after they become sexually
active. The infection is likely caused by urethral trauma during intercourse, which
leads to bacterial contamination of the bladder. Since the majority of these
infections are caused by Escherichia coli (a gram-negative rod), the most
appropriate therapy would be ampicillin for around 10 days.
Ceftriaxone (choice B) is the treatment of choice for uncomplicated infections with
N. gonorrhoeae, now that most strains are resistant to penicillin. Intravenous
ceftriaxone is a regimen reserved for the treatment of life-threatening infections.
Fluconazole (choice C) is indicated for the treatment of vaginal candidiasis. Since
there is no vaginal discharge and the patient has gram-negative rods in the urine, a
diagnosis of vaginal candidiasis can be excluded.
Gentamicin (choice D) would be an inappropriate choice since the majority of
urinary tract infections caused by gram negative rods are sensitive to ampicillin and
the potential for toxicity secondary to gentamicin is great.
Metronidazole (choice E) is an antibiotic typically used in the treatment of
Trichomoniasis, Giardiasis, and Gardnerella, as well as serious infections believed to
be caused by anaerobic bacteria. Since there is no vaginal discharge and the
patient has gram negative rods in the urine, one can conclude that these infections
are not present and the patient instead has a urinary tract infection.

________________________________________

26>The correct answer is D. The patient is exhibiting the classic signs of acute
bacterial endocarditis. In intravenous drug abusers, Staphylococcus aureus is the
most common causative agent, because it is the most common normal flora
organism on the skin of these patients, and choice D is the classic description of this
species. Catalase positivity distinguishes the genus Staphylococcus from

Streptococcus, and Staphylococcus aureus is the only coagulase positive member of


its genus.
Antibodies to p24 capsid antigen (choice A) would be likely in an HIV-positive drug
abuser, rather than a patient with endocarditis.
Trichinella spiralis (choice B) can cause splinter hemorrhages during the larval
migration period, but would not be the most likely agent given the other symptoms.
Staphylococcus epidermidis is a catalase-positive, novobiocin-sensitive, grampositive coccus (choice C). It is sometimes implicated in subacute bacterial
endocarditis, but is less common as a causative agent in drug abusers than in
normal individuals.
Enterococci are gamma-hemolytic, gram-positive cocci that grow on bile-esculin
agar (choice E). Enterococci can cause subacute bacterial endocarditis, chiefly after
urologic instrumentation in men, rather than in association with intravenous drug
abuse.
Streptococcus viridans is an alpha-hemolytic, optochin-resistant, gram-positive
coccus (choice F). It is implicated in subacute bacterial endocarditis after oral or
dental treatments but would not be the most likely agent, given the acute nature of
this patient's presentation.
Streptococcus pneumoniae is an alpha-hemolytic, optochin-sensitive, gram-positive
coccus (choice G) that produces cough and chest pain, but would be an unlikely
cause of bacterial endocarditis.

________________________________________

27>The correct answer is E. The history is characteristic for Lyme disease, caused
by the spirochete Borrelia burgdorferi. The clues are deer, Connecticut, rash, knees,
and acute arthritis followed by chronic arthritis.
Fungal arthritis (choice A) can be caused by coccidioidomycosis, blastomycosis,
sporotrichosis, and candidiasis.
N. gonorrhoeae are gram-negative cocci (choice B) that can cause septic arthritis.
Haemophilus (young children) and Salmonella (sickle cell patients) are gramnegative rods (choice C) that can cause septic arthritis.
Staphylococcus are gram-positive cocci (choice E) that can cause septic arthritis

________________________________________

28>The correct answer is D. The patient described probably has systemic lupus
erythematosus (SLE). This is a systemic disorder that often presents with fatigue,
malaise, fever, gastrointestinal symptoms, arthralgias, and myalgias. Hematologic
abnormalities include anemia of chronic disease, hemolytic anemia, leukopenia,
lymphocytopenia, and thrombocytopenia. A circulating anticoagulant may prolong
the APTT (activated partial thromboplastin time). Cutaneous manifestations include
a malar rash and a generalized maculopapular eruption, both of which are
photosensitive. Antibodies to the Smith antigen (core proteins of small
ribonucleoproteins found in the nucleus) are present in only 20-30% of patients with
SLE, but are quite specific for the disease, occurring only rarely in other
autoimmune diseases.
Anti-centromere antibody (choice A) is specific for the CREST (Calcinosis, Raynaud's
syndrome, Esophageal dysfunction, Sclerodactyly, and Telangiectasia) variant of
progressive systemic sclerosis (scleroderma).
Rheumatoid factor is actually an autoantibody directed against the Fc portion of the
IgG molecule (choice B). It is found in more than two-thirds of patients with
rheumatoid arthritis.
The majority of patients with SLE (around 95%) develop antinuclear antibodies
(ANA; choice C), so this test is quite sensitive, but not very specific for SLE. ANA
occur in other inflammatory disorders, autoimmune diseases, viral diseases, and in
a number of normal individuals. Antibodies to double-stranded DNA are more
specific for SLE, but are not included as an answer choice.
Anti-SS-A antigen (choice E) refers to antibodies to certain ribonucleoproteins, which
are fairly specific for Sjgren's syndrome.

________________________________________

29>The correct answer is E. Systemic lupus erythematosus (SLE) is a prototype


connective tissue disease. The diagnosis requires four criteria to be met from a list
of eleven possible criteria: malar rash, discoid rash, photosensitivity, oral ulcers,
arthritis, serositis, renal disorder, neurologic disorder, hematologic disorder,
immunologic disorder, and antinuclear antibody. This patient also has anti-Sm,
which is pathognomonic for SLE, but is only found in 30% of the affected patients.
Antinuclear antibodies (ANA) are present in 95-100% of cases of SLE; anti-doublestranded DNA is found in 70% of the cases.
Generalized fatigue (choice A) due to being a single working mother of two children
could well be a possibility, but the presence of the other criteria make SLE more
likely.
Goodpasture's syndrome (choice B) is characterized by linear disposition of
immunoglobulin, and often C3, along the glomerular basement membrane (GBM).

Glomerulonephritis, pulmonary hemorrhage, and occasionally idiopathic pulmonary


hemosiderosis occur.
Mixed connective tissue disease (choice C) is an overlap syndrome characterized by
a combination of clinical features similar to those of SLE, scleroderma, polymyositis,
and rheumatoid arthritis. These patients generally have a positive ANA in virtually
100% of the cases. High titer anti-ribonucleoprotein (RNP) antibodies may be
present, generating a speckled ANA pattern. Anti-RNP is not pathognomonic for
mixed connective tissue disease, since it can be found in low titers in 30% of the
patients with SLE.
Scleroderma (choice D) is characterized by thickening of the skin caused by swelling
and thickening of fibrous tissue, with eventual atrophy of the epidermis. ANA are
often associated with the disease, but the staining pattern is generally nucleolar

________________________________________

30>The correct answer is D. The patient has acute prostatitis, which is


characterized by fever, chills, and dysuria, with a swollen, extremely tender prostate
on rectal exam. The urine Gram's stain and culture will generally be positive. The
treatment regimen for this bacterial infection is typically a 21-day course of
ampicillin, a fluoroquinolone, or sulfamethoxazole/trimethoprim (SMX-TMP). G-6-PD
deficiency is an X-linked recessive disorder affecting 10% to 15% of American black
males. The medications most commonly associated with the induction of hemolytic
anemia in deficient patients are sulfonamides, nitrofurantoin, dapsone, primaquine,
and quinine. The sulfamethoxazole in the SMX-TMP combination is a sulfonamide,
and can produce hemolytic anemia in patients with G-6-PD deficiency.
Ampicillin (choice A) is a broad-spectrum penicillin antibiotic commonly used in the
treatment of infections in the genitourinary, respiratory, or GI tracts, as well as in
the skin and soft tissues.
Cefaclor (choice B) is a second-generation cephalosporin indicated for a variety of
bacterial infections, including those of the respiratory and GI tracts.
Ciprofloxacin (choice C) is a fluoroquinolone commonly used in the treatment of
serious infections caused by gram-negative organisms.
Tetracycline (choice E) is most commonly used in the treatment of acne vulgaris and
gonococcal infections.

________________________________________

31>The correct answer is B. Streptococcus bovis is a Group D streptococcus. There


is a significant association between S. bovis bacteremia and endocarditis with

carcinoma of the colon and other colonic diseases. Every patient with S. bovis
bacteremia should undergo gastrointestinal and cardiac evaluation. Up to 50% of
patients with S. bovis bacteremia are reported to have underlying colonic
malignancies. In another study, 25-50% of cases of S. bovis bacteremia were
associated with endocarditis, especially in patients with preexisting valvular lesions.
Streptococcus agalactiae (choice A) is an important cause of maternal and neonatal
bacteremia and neonatal meningitis. It is part of the normal flora of the
gastrointestinal tract and the female genital tract.
Streptococcus pneumoniae (choice C) is a leading cause of community-acquired
pneumonia, meningitis in adults, otitis media (especially in children), and sinusitis.
Spontaneous peritonitis due to S. pneumoniae is reported in children with ascites
from nephrotic syndrome. Asplenia predisposes patients to severe infections with S.
pneumoniae and other encapsulated organisms. S. pneumoniae infections are also
more frequent and unusually severe in patients with sickle cell anemia, multiple
myeloma, alcoholism, and hypogammaglobulinemia. S. pneumoniae is now the
leading cause of invasive bacterial respiratory disease in patients with AIDS.
Streptococcus pyogenes (choice D) is the most common cause of bacterial
pharyngitis. Complications include paratonsillar abscesses, otitis media, and
sinusitis. Long-term sequelae include rheumatic fever and poststreptococcal
glomerulonephritis. S. pyogenes is also responsible for many skin and soft tissue
infections. The organism also produces many toxins that produce a variety of
diseases.
Streptococcus viridans (choice E), or the viridans Streptococci, are the most
common cause of subacute bacterial endocarditis, which should be suspected in
cases of viridans streptococcal bacteremia. One species of viridans Streptococci,
Streptococcus milleri, is frequently associated with pyogenic abscesses, especially
of the liver.

________________________________________

32>The correct answer is E. Mycobacteria, such as the causative organism of this


patient's tuberculosis, are "acid fast" because they have an envelope that contains
large amounts of lipids and even true waxes (unlike envelopes of other types of
bacteria) that prevents the acid-fast stain (carbolfuchsin) from leaking out.
The other characteristics listed in the answers are commonly tested features of
specific bacteria:
Nutritional requirement for factors V and X (choice A) is a feature of Haemophilus
influenzae.
Streptokinase (choice B) is a feature of Streptococci.
Toxic shock syndrome toxin (choice C) is a feature of Staphylococcus aureus.

Visibility under dark field illumination (choice D) is a feature of the syphilis organism
Treponema pallidum.

________________________________________

33>The correct answer is B. When individuals travel from one country to another
with marked differences in climate and sanitation standards, the risk for
development of traveler's diarrhea is dramatically increased. This type of diarrhea is
most likely to develop within 2-10 days after ingesting local water and/or eating
fresh fruits and vegetables (such as a fresh salad) that could have been "washed
off" with the local water. Traveler's diarrhea is often accompanied by the
appearance of approximately 10 or more episodes of diarrhea per day, as well as
severe abdominal cramps, nausea, vomiting, and a mild fever. Depending on the
causative bacteria, blood and/or fecal leukocytes may be present. Most cases of
traveler's diarrhea are caused by enterotoxigenic Escherichia coli, Shigella species,
and Campylobacter jejuni. The most commonly used agents to treat traveler's
diarrhea are the fluoroquinolones, such as ciprofloxacin, ofloxacin, and norfloxacin.
Trimethoprim/sulfamethoxazole is most commonly used in treatment of children.
None of the other agents are indicated for the treatment of traveler's diarrhea.
Acyclovir (choice A) is an antiviral agent indicated for the treatment of herpes virus
infections.
Mebendazole (choice C) is a broad-spectrum anthelminthic indicated for the
treatment of "worm" infections, such as Enterobius vermicularis, Trichuris trichiura,
and Ascaris lumbricoides.
Quinine (choice D) is an antimalarial agent indicated for the treatment of malaria
and severe leg cramps.
Tetracycline (choice E) is an antibacterial agent with a limited antibacterial
spectrum; it is primarily used in the treatment of acne vulgaris and gonorrhea
infections.

________________________________________

34>The correct answer is D. Penicillin remains the first-line drug of choice for
pneumococcal pneumonia, except in patients with penicillin allergy and in the
relatively few areas in which pneumococcal strains with high-level penicillin
resistance exist. Alternative therapies include erythromycin and vancomycin.
The third-generation cephalosporin cefotaxime (choice A) is not usually used for
pneumococcal pneumonia.
Chloramphenicol (choice B) is not usually used for pneumococcal pneumonia.

Erythromycin (choice C) is a good alternative therapy for pneumococcal pneumonia,


but is usually used only when a penicillin allergy is present.
Vancomycin (choice E) is not the first-line therapy, but it is a good alternative in
patients allergic to penicillin or when high-level penicillin resistance (relatively
uncommon) is present.

________________________________________

35>The correct answer is E. Wiskott-Aldrich syndrome is an X-linked condition


characterized by eczema, thrombocytopenia, and repeated infections. Affected
children may present with bleeding and often succumb to complications of bleeding,
infection, or lymphoreticular malignancy. The platelets are small, have a shortened
half-life, and appear to be deficient in surface sialophorin (CD43). Splenectomy can
correct the thrombocytopenia, but not the immune defect. Serum IgM is usually
decreased, while IgE is frequently increased. Mutations in the Wiskott-Aldrich serum
protein (WASP) gene on the short arm of the X chromosome are responsible for this
disease.
Acquired hypogammaglobulinemia (choice A) is a disease of adults characterized by
normal numbers of B cells but low immunoglobulin production.
Ataxia telangiectasia (choice B) is an autosomal recessive disease characterized by
progressive cerebellar dysfunction, telangiectasias, and a variable
immunodeficiency.
DiGeorge syndrome (choice C) is a developmental malformation leading to thymic
aplasia and, sometimes, hypoparathyroidism.
Selective IgA deficiency (choice D) is a relatively common condition characterized
by low levels of IgA.

________________________________________

36>The correct answer is B. Chlamydia, Mycoplasma, and Ureaplasma are not


effectively treated by penicillins and cephalosporins, and are important causes of
post-gonococcal urethritis. Chlamydial urethritis can be diagnosed by using
fluorescent antibodies to identify inclusions in epithelial cells.
Actinomyces (choice A) is a mouth commensal that rarely causes a deeper oral
infection.
Mycobacteria (choice C) cause chronic granulomatous diseases such as tuberculosis
and leprosy.

Nocardia (choice D) can cause necrotizing pneumonia and disseminated disease.


Rickettsia (choice E) cause typhus and Rocky Mountain spotted fever.

________________________________________

37>The correct answer is D. Pseudomonas aeruginosa causes malignant otitis


externa, which is a severe necrotizing infection of the external ear canal. Infection
tends to spread to the mastoid bone, temporal bone, sigmoid sinus, base of the
skull, meninges, and brain. Patients at increased risk include the elderly, those with
diabetes, and the immunocompromised. Pseudomonas is also associated with many
other clinical syndromes, including infection following traumatic wounds to the feet
in persons wearing sneakers or rubber-soled shoes, since the organism can be
cultured from the shoes.
Hemophilus influenzae (choice A) produces a variety of clinical syndromes.
H.Influenzae is the third-most common cause of meningitis in children aged 1 month
to 18 years. It is the most common cause of acute epiglottitis, the most common
cause of purulent bacterial conjunctivitis, and the second-most common cause of
otitis media. Patients at risk include those with COPD and cystic fibrosis, alcoholics,
splenectomized patients, and young patients.
Klebsiella pneumoniae (choice B) is a gram-negative organism that produces a
necrotizing pneumonia in diabetics and alcoholics. Patients typically present with an
abrupt onset of fever, shaking chills, and purulent, foul-smelling sputum.
Mucor (choice C) is a fungal infection that is particularly severe in the diabetic or the
immunocompromised patient. In the acidotic diabetic, the fungus produces a lifethreatening, invasive rhinocerebral infection. The infection begins in the nasal
passages, extends into the paranasal sinuses, and spreads through the cribiform
plate to the frontal lobes of the brain. Patients typically complain of headache, facial
pain, and orbital swelling.
Streptococcus pyogenes (choice E) causes bacterial pharyngitis, otitis media, and
sinusitis. It is also associated with toxin-related diseases and skin infections.

________________________________________

38>The correct answer is D. Rubella, also called German measles or 3-day measles,
is a disease caused by a Togavirus, which are small, enveloped, single-stranded, (+)
linear RNA viruses. Approximately 40% of patients are asymptomatic or have mild
symptoms. In symptomatic patients, the clinical presentation typically consists of an
erythematous rash beginning on the head, which spreads downward to involve the
trunk, lasting for approximately 3 days. In addition to a transient rash, symptoms

include fever, posterior cervical lymphadenopathy, and arthralgias. The greatest


danger from rubella is to the fetus. If clinical rubella develops or seroconversion is
demonstrated, there is a high risk of congenital abnormalities or spontaneous
abortion. The risk varies from 40%-60% if infection occurs during the first 2 months
of gestation to 10% by the 4th month. Females of childbearing age should be
warned not to become pregnant within 2-3 months from the time of immunization.
Mild arthralgias and other symptoms may develop in 25% of immunized women.
Enteroviral rashes may mimic rubella and rubeola.
Infectious mononucleosis (choice A) is caused by the Epstein-Barr virus, a
herpesvirus. Classic findings include fever, exudative pharyngitis, generalized
lymphadenopathy, severe malaise (most common complaint), and
hepatosplenomegaly. A rash is not a characteristic feature unless the patient has
been treated with ampicillin.
Lyme disease (choice B) is caused by the spirochete Borrelia burgdorferi. The
disease is transmitted by the bite of the tick, Ixodes dammini. Reservoirs in nature
include the white-tailed deer and the white-footed mouse. The initial lesion is an
annular rash with central clearing and a raised red border (erythema chronicum
migrans) at the bite site. The rash is warm, but not painful or itchy. Patients also
have fever, malaise, myalgias, arthralgias, headache, generalized
lymphadenopathy, and, occasionally, neurologic findings.
Roseola (choice C) is caused by human herpesvirus 6. Other names include
exanthem subitum or sixth disease. Children have a febrile period of 3-5 days with
rapid defervescence followed by an erythematous maculopapular rash lasting 1-3
days.
Rubeola (choice E), or regular measles, is a disease caused by a paramyxovirus.
Patients present with an upper respiratory prodrome and characteristic oral lesions
(Koplik's spots) that precede the rash. The non-pruritic maculopapular rash begins
on the face and spreads to the trunk and extremities, including palms and soles.
The incubation period is 10-14 days. Patients also have a posterior cervical
lymphadenopathy. The virus is not associated with risk to a fetus.

________________________________________

39>The correct answer is E. The patient in this question has pneumococcal


pneumonia, which must be considered in any patient with chills, fever, chest pain,
and cough productive of purulent, rust-colored sputum. Streptococcus pneumoniae
is an a-hemolytic, gram-positive coccus that grows in chains. It can be easily
distinguished from other a-hemolytic streptococci because it is exquisitely sensitive
to bile and bilelike compounds, such as optochin. It is the most common cause of
community-acquired pneumonia and the most common cause of communityacquired meningitis in adults older than 30. The only recognized virulence factor of
S. pneumoniae is its carbohydrate capsule (which contains acidic polysaccharides).

Antibody to a specific capsule type is necessary to overcome infection. More than


80 capsule types have been recognized. The 23 types that most commonly cause
disease are contained in a vaccine that is recommended for high-risk groups,
including the elderly and those undergoing splenectomy. Increased susceptibility is
also found in patients with Hodgkin disease, chronic lymphocytic leukemia, and
myeloma. The vaccine should still be given to patients with these conditions, but it
is less successful.
The alternative complement pathway (choice A) is important in clearing Neisseria
infections. Individuals with deficiencies in C5 through C8 are at increased risk of
disease from Neisseria.
The fimbria of Streptococcus pyogenes (Group A a-hemolytic streptococcus) is
composed of an a-helically coiled M protein. Antibody against a specific M type
(choice B) will prevent infection. However, raising antibodies to M proteins can lead
to rheumatic fever, so Strep throat infections are routinely treated with penicillin to
prevent an antibody response.
The C carbohydrate is an antigen of a-hemolytic streptococci used to divide them
into different groups. Antibody against C carbohydrate (choices C and D) is not
protective.

________________________________________

40>The correct answer is E. This clinical scenario is typical for Wiskott-Aldrich


syndrome. This is a sex-linked recessive disorder presenting with the triad of
thrombocytopenia, eczema, and recurrent sinopulmonary infections. Serum IgM
levels are low but IgG, IgA, and IgE levels are increased. Patients have a defective
response to polysaccharide antigens, which is due to a cytoskeletal defect in T cells
that inhibits their binding to B cells.
A CD4/CD8 ratio of < 1.5:1 (choice A) is the pattern seen in AIDS due to selective
tropism of the CD4+ T-helper cell population.
Cerebellar ataxia (choice B) is part of the ataxia-telangiectasia syndrome. The
ataxia develops between age 2 and 5. The defect is associated with a DNA repair
enzyme deficiency.
Elevated platelet count and increased serum levels of IgG, IgA, and IgE (choice C) is
not associated with a particular syndrome. Polyclonal gammopathies result in an
increase in immunoglobulin of more than one class. This benign alteration is
frequently seen in viral or bacterial infections. Thrombocythemia may be associated
with a myeloproliferative syndrome or with a secondary reactive process.
Low platelet count and low serum IgG (choice D) are not associated with any
particular disorder.

________________________________________

41>The correct answer is C. Macrophages interact with a segment of the heavy


chains of the IgG molecule, the Fc domain. Thus, the macrophage receptors for IgG
are called Fc receptors. Complement receptors are also found on the macrophage
surface. Phagocytosis occurs more readily when both of these components are
present. Neutrophils and eosinophils have similar receptors: neutrophils assist
macrophages in destruction of bacteria, while eosinophils respond to parasitic
infections.
Collagenase can be demonstrated immunologically (choice A) in a number of cells
including eosinophils, mast cells, osteoclasts, and fibroblasts.
Microvilli (choice B) or pseudopodial extensions are common in macrophages, but
also may be found on other cells such as fibroblasts and epithelial cells that
phagocytize material.
Secondary lysosomes (choice D) are a feature of macrophages, but may also be
found in fibroblasts and epithelial cells.
The shape of the nucleus (choice E) has little to do with phagocytic activity.

________________________________________

42>The correct answer is C. This is a case of Reiter's syndrome. Patients typically


present with the acute onset of arthritis (usually asymmetric and additive), with
involvement of new joints occurring over a period of a few days to 2 weeks. Joints of
the lower extremities are the most commonly involved, but wrists and fingers can
also be affected. Dactylitis (sausage digit), a diffuse swelling of a solitary finger or
toe, is a distinctive feature of Reiter's arthritis and psoriatic arthritis. Tendonitis and
fasciitis are common. Spinal pain and low back pain are common. Conjunctivitis,
urethritis, diarrhea, and skin lesions are also associated with Reiter's syndrome. Up
to 75% of patients are HLA-B27 positive. Microorganisms which can trigger Reiter's
syndrome include Shigella spp., Salmonella spp., Yersinia spp., Campylobacter
jejuni, and Chlamydia trachomatis. Most patients are younger males.
Gout (choice A) usually presents as an explosive attack of acute, very painful,
monarticular inflammatory arthritis. Hyperuricemia is the cardinal feature and
prerequisite for gout. The first metatarsophalangeal joint is involved in over 50% of
first attacks.
Lyme disease (choice B), caused by Borrelia burgdorferi, presents with a red macule
or papule at the site of the tick bite. This lesion, called erythema chronicum
migrans, slowly expands to form a large annular lesion with a red border and central

clearing. The lesion is warm, but usually not painful. The patient also has severe
headache, stiff neck, chills, arthralgias, and profound malaise and fatigue.
Untreated infection is associated with development of arthritis. The large joints
(e.g., knees) are usually involved with the arthritis lasting for weeks to months.
Rheumatoid arthritis (choice D) begins insidiously with fatigue, anorexia,
generalized weakness, and vague musculoskeletal symptoms leading up to the
appearance of synovitis. Pain in the affected joints, aggravated by movement, is the
most common manifestation of established rheumatoid arthritis. Generalized
stiffness is frequent and is usually greatest after periods of inactivity. Morning
stiffness of greater than 1 hour in duration is very characteristic. Rheumatoid
arthritis is more common in females. The metacarpophalangeal and proximal
interphalangeal joints of the hands are characteristically involved.
Septic arthritis (choice E) is caused by a variety of microorganisms, including
Neisseria gonorrhoeae and Staphylococcus aureus. Hematogenous spread is the
most common route in all age groups. 90% of patients present with involvement of
a single joint, usually the knee. The usual presentation is moderate-to-severe pain,
effusion, muscle spasm, and decreased range of motion. Peripheral leukocytosis and
a left shift are common. Disseminated gonococcal infections present as fever, chills,
rash, and articular symptoms. Papules progressing to hemorrhagic pustules develop
on the trunk and extensor surfaces of the distal extremities. Migratory arthritis and
tenosynovitis of multipl
43>The correct answer is C. Rotavirus is the most common cause of
gastroenteritis in children between 3 months and 2 years of age.
It is most prevalent in the winter. Rotavirus, one of the reoviruses,
looks like a wheel (which ROTAtes) and possesses a double-shelled
icosahedral capsid with no envelope. Its genome consists of 11
segments of double-stranded RNA.
A complex double-stranded DNA genome (choice A) is found in enteric
adenoviruses, the third most common cause of gastroenteritis in infants and
children. This organism possesses an icosahedral nucleocapsid.
A partially double-stranded circular DNA genome (choice B) is characteristic of
hepatitis B. Its envelope contains surface antigen (HBsAg). Its capsid is icosahedral
and contains the genome along with DNA-dependent DNA polymerase, which also
has reverse transcriptase activity.
A single-stranded circular RNA genome (choice D) is characteristic of hepatitis D
virus. Its envelope consists of HBsAg. The virus is defective and is able to replicate
only in cells infected with hepatitis B.
A single-stranded RNA genome (choice E) is characteristic of several viruses that
cause gastroenteritis in children, including astrovirus and Norwalk virus. Astrovirus
is the second most common cause of viral gastroenteritis in young children. Three

structural proteins form its capsid. Norwalk virus is the most common cause of
gastroenteritis outbreaks in older children and adults. Its capsid consists of one
structural protein.
________________________________________

44>The correct answer is A. Flumazenil is a benzodiazepine antagonist and has


been approved to hasten the recovery from benzodiazepines used in anesthetic and
diagnostic settings and to reverse the CNS depressant effects following an overdose
with benzodiazepines. Flumazenil can only be used for benzodiazepines and is not
useful to reverse the effects of other CNS depressants such as barbiturates and
ethanol.
Glucagon (choice B) is an antidote for beta-blocker overdose.
Naloxone (choice C), an opioid receptor antagonist, is an antidote for opioid
overdose.
Nitrite (choice D), or sodium nitrite, is an antidote for cyanide poisoning.
Protamine (choice E) is an antidote for heparin overdose.

________________________________________

45>The correct answer is G. Flumazenil is an antagonist at the benzodiazepine


receptor. It has no effect on other CNS depressants, such as barbiturates or alcohol.
Acetylcysteine (choice A) is the drug of choice for treatment of overdose of
acetaminophen, the active ingredient in Tylenol.
Atropine (choice B) is a muscarinic antagonist used in cases of acetylcholinesterase
inhibitor overdose.
Bicarbonate (choice C) infusions may be given to alkalinize the urine and enhance
the excretion of acidic drugs (e.g., aspirin).
CaNa2EDTA (choice D) is used as a chelator in lead poisoning.
Deferoxamine (choice E) is an effective chelator for poisoning with iron salts.
Ethanol (choice F) is used in cases of methanol and ethylene glycol poisoning.
Administration of EtOH in cases of diazepam overdose would be completely
inappropriate because CNS depressants are additive.
Physostigmine (choice H) is used in cases of anticholinergic agent overdose.

Pralidoxime (choice I) is an acetylcholinesterase reactivating agent used in cases of


organophosphorus acetylcholinesterase inhibitor overdose.
Protamine (choice J) is administered to reverse the anticoagulant effects of heparin
overdose.

________________________________________

46>The correct answer is B. Before you started analyzing all of the answer choices
you should have reminded yourself that glucagon increases serum glucose. So an
enzyme stimulated by glucagon might be involved in either the breakdown of
glycogen to glucose (glycogenolysis) or in the creation of glucose from
noncarbohydrate precursors (gluconeogenesis). Glycogen phosphorylase catalyzes
the first step in glycogenolysis; it makes sense that it would be stimulated by
glucagon.
Acetyl-CoA carboxylase (choice A) catalyzes the first step in fatty acid synthesis, an
anabolic process that would be stimulated by insulin, not glucagon.
As its name implies, glycogen synthase (choice C) is involved in the synthesis of
glycogen. Glucagon (and epinephrine) stimulate the phosphorylation and
inactivation of glycogen synthase.
HMG-CoA reductase (choice D) is the key enzyme involved in the synthesis of
cholesterol. Since this is an anabolic process that occurs in the well-fed state, you
would expect it to be stimulated by insulin and inhibited by glucagon (which it is).
Pyruvate kinase (choice E) catalyzes the last reaction of glycolysis. You would
expect it to be inhibited by glucagon (thus decreasing the amount of glucose
consumption). Glucagon promotes the phosphorylation of pyruvate kinase, which
renders it inactive.

________________________________________

47>The correct answer is A. The cytochrome P-450 mixed-function oxidase system


is located on smooth endoplasmic reticulum in liver cells. This system is involved in
the detoxification of some drugs and other exogenous compounds (barbiturates,
carcinogenic hydrocarbons, steroids, carbon tetrachloride, alcohol, insecticides),
and its growth can be stimulated by exposure (particularly chronic exposure) to
these agents. As a consequence, cells adapted to one drug can more rapidly
metabolize the other drugs and compounds handled by the P-450 system.

________________________________________

________________________________________

48>The correct answer is A. The clinical signs suggest that this woman had
hepatitis B three years ago and the fact that she is now feeling better also suggests
that she recovered from this infection. Since she is pregnant, it is necessary to find
out if she still has the organism in her liver (chronic hepatitis B infection) by
performing a test for HBsAg. This test will be negative if she has completely
recovered from the disease, but it would be positive if she is a chronic carrier.
The clinical signs of the disease she had 3 years ago do not match those of CMV
(choice B), and CMV does not produce chronic infections.
Measurement of IgM antibody to hepatitis B core antigen (choice D) would be of no
value at this time. This antibody is positive in acute cases of hepatitis B, but would
no longer be positive in this case. Measurement of IgM anti-hepatitis B core antigen
is one of the most important tests in the hepatitis profile because the appearance of
the antibody correlates with the disappearance of HBsAg.
Examination of IgM antibody to HBsAg (choice C) would be of no value at this time.
This IgM antibody would be formed early during the recovery from acute hepatitis B,
and would class switch to IgG later on in the disease.
Quantitation of hepatitis A virus (HAV) IgM antibody (choice E) would be useless,
since hepatitis A virus (HAV) only causes acute infection.

________________________________________

49>The correct answer is A. Alpha-1 receptors activate phospholipase C via the G


protein Gq. Phospholipase C cleaves the membrane phospholipid
phosphatidylinositol 4,5-bisphosphate to produce the products, inositol triphosphate
(IP3) and diacylglycerol (DAG). IP3 releases intracellular calcium from the
endoplasmic reticulum, and would therefore generate a robust signal. DAG activates
protein kinase C.
Beta-1 adrenergic receptors (choice B) stimulate adenylate cyclase via the G protein
Gs, leading to an increase in intracellular cAMP. All beta adrenergic receptors share
a common mechanism of action.
Dopamine-1 receptors (choice C) stimulate adenylate cyclase via the G protein Gs.
This leads to an increase in intracellular cAMP.

Muscarinic acetylcholine receptors (choice D) inhibit adenylate cyclase via the G


protein Gi. This leads to an decrease in intracellular cAMP. Muscarinic receptors also
stimulate the opening of potassium channels in the heart, via the beta and gamma
subunits of Gi.
Nicotinic acetylcholine receptors (choice E) are ligand-gated ion channels. When
stimulated, they allow sodium ions to enter the cell.

________________________________________

50>The correct answer is D. The child has Hartnup's disease. This condition
clinically resembles pellagra ("diarrhea, dementia, and dermatitis"), and may be
misdiagnosed as this nutritional (niacin) deficiency. In fact, niacin therapy may
actually be helpful in controlling the symptoms. The underlying problem is a defect
in the epithelial transport of neutral amino acids, including tryptophan, which can
act as a precursor of niacin. The defective amino acid transport leads to poor
absorption of dietary amino acids as well as excess amino acid secretion in the
urine.
Alkaptonuria (choice A) is characterized by urine that turns black upon standing and
a debilitating arthritis.
Carcinoid syndrome (choice B) is seen in patients with carcinoid tumor. It is
characterized by episodes of flushing, diarrhea, hypertension, and
bronchoconstriction.
Ehlers-Danlos syndrome (choice C) is a disease characterized by abnormal collagen
formation leading to very elastic skin, joint problems, and fragility of some blood
vessels and the intestines.
Scurvy (choice E) is due to vitamin C deficiency. It is characterized by easy bruising
and gum problems.

________________________________________
Answers

________________________________________

________________________________________

________________________________________

________________________________________

1>The correct answer is B. One of the most common side effects of any
antineoplastic therapy is weight loss secondary to decreased appetite and/or
nausea and vomiting. Furthermore, weight loss due to decreased food intake tends
to occur more frequently in elderly patients receiving antineoplastic therapy. One
medication that has consistently helped to increase appetite in such patients is
megestrol acetate. This agent is a progestational hormone with antineoplastic
properties used in the treatment of advanced carcinoma of the breast and
endometrium. Megestrol, when given in relatively high doses, can substantially
increase the appetite in most individuals, even those with advanced cancer.
Amitriptyline (choice A) is a tricyclic antidepressant used in the treatment of
depression. There is nothing mentioned in the case study to suggest that the
patient is clinically depressed; hence, this agent would provide no benefit.
Methotrexate (choice C) is an antimetabolite and folic acid antagonist commonly
used in various neoplastic disorders and in the treatment of rheumatoid arthritis.
Since nausea, vomiting, and ulcerative stomatitis are common side effects of this
medication, its usage in this patient would not be recommended.
Neostigmine (choice D) is a carbamylating acetylcholinesterase inhibitor that would
not increase appetite.
Prochlorperazine (choice E) is a phenothiazine derivative used primarily to control
severe nausea and vomiting. This patient is not experiencing nausea. Furthermore,
this agent does not possess appetite-stimulating properties.

________________________________________

2>The correct answer is B. The principle host defense in mycobacterial infections


(such as this patient's tuberculosis) is cell-mediated immunity, which causes
formation of granulomas. Unfortunately, in tuberculosis and in many other
infectious diseases characterized by granuloma formation, the organisms may
persist intracellularly for years in the granulomas, only to be a source of activation
of the infection up to decades later.
While antibody-mediated phagocytosis (choice A) is a major host defense against
many bacteria, it is not the principle defense against Mycobacteria.
IgA-mediated hypersensitivity (choice C) is not involved in the body's defense
against Mycobacteria.
IgE-mediated hypersensitivity (choice D) is not involved in the body's defense
against Mycobacteria. It is important in allergic reactions.

Neutrophil ingestion of bacteria (choice E) is a major host defense against bacteria,


but is not the principle defense against Mycobacteria.

________________________________________

3>The correct answer is E. The immunoperoxidase method uses horseradish


peroxidase to produce a visible pigment when a specific antibody binds to antigenic
sites in the tissue. Binding of the antibody to the epithelial cells indicates that the
protein in question is being expressed by those cells. The positive
immunoperoxidase results obtained here suggests that the channel protein is
expressed in proximal convoluted tubular epithelium, since this is the only site in
the kidney at which the epithelial cells have a "brush border." The brush border is
made of microvilli, which enhance the proximal tubules' ability to reabsorb plasma
constituents filtered at the glomeruli.

________________________________________

4>The correct answer is D. Pregnant women with chronic hypertension "require"


antihypertensive therapy when the diastolic pressure is greater than 100 mm Hg;
however, some clinicians may decide to treat patients with diastolic blood pressures
less than 100 mm Hg. For the initiation of therapy, methyldopa is still considered to
be the agent of choice. Methyldopa is converted intraneuronally to amethylnorepinephrine, an alpha-2 adrenergic agonist, which is subsequently
released. Release of a-methylnorepinephrine in the medulla leads to a decrease in
sympathetic outflow, thus lowering blood pressure. Methyldopa has been safely
used in the treatment of hypertension during pregnancy; this agent is not
associated with the development of teratogenic or other fetal abnormalities.
Diuretics, such as bumetanide (choice A) and hydrochlorothiazide (choice C), are
often avoided since these agents can produce hypovolemia, leading to reduced
uterine blood flow. Although these agents can be used during pregnancy,
methyldopa and hydralazine are the drugs of choice for hypertension during
pregnancy.
Fosinopril (choice B) is an angiotensin-converting enzyme (ACE) inhibitor that should
not be administered to pregnant women, especially in the second or third
trimesters. These agents have been associated with severe fetal and neonatal
injury, such as hypotension, neonatal skull hypoplasia, anuria, renal failure, and
death.
Along the same lines, the use of the angiotensin II receptor antagonists, such as
valsartan (choice E), is not recommended since these agents cause fetal
complications similar to the ACE inhibitors.

________________________________________

5>The correct answer is D. The disease is Duchenne muscular dystrophy, an Xlinked recessive muscular disease usually caused by a deletion involving the
dystrophin gene. This defect produces accelerated muscle breakdown leading
initially to proximal muscle weakness, then later to generalized weakness that
typically begins before age 5. A feature of X-linked recessive diseases is that carrier
mothers pass the disease to half their sons; affected fathers can have carrier
daughters but not affected sons. Since the mother is presumably normal (because
the disease is X-linked), she must be a carrier to have an affected son, and the
grandmother must also be a carrier, therefore the mother's brother (maternal uncle)
may also have the disease.
The father's (choice A) side of the family, including the father's brother (choice B),
most likely does not carry the defective gene (since they themselves would be
affected, and furthermore since the father cannot pass the gene on to a son). It
would be extremely unlikely for a carrier female to marry an affected male (and the
question does not mention any similar symptoms in the father).
The mother (choice C) and possibly the sister (choice E) are carriers of, but not
affected by, the defective gene.

________________________________________

6>The correct answer is B. The cell pictured is an eosinophil, a member of the


granulocytic lineage of white blood cells. The crystalline core of the granule
contains a protein called the major basic protein, which appears to function in the
destruction of parasites. Major basic protein also has deleterious effects on
epithelial cells in patients with asthmatic reactions. The light component around the
dense crystalline core contains products such as histaminase, arylsulfatase, and
other enzymes.
Lactoferrin (choice A) is found in the specific granules of the neutrophil. It inhibits
the growth of bacteria by interfering with iron metabolism.
Myeloperoxidase (choice C) is found in the azurophilic (large) granule of the
neutrophil. This enzyme is also destructive to bacteria, destroying their cell walls.
Histamine (choice D) is produced by the basophil and the mast cell. The
histaminase of the eosinophil regulates the inflammatory reaction of these two cell
types.
Tartrate-resistant acid phosphatase (choice E) is a marker for hairy cell leukemia, a
neoplasm of the B lymphocyte line.

________________________________________

7>The correct answer is E. This presentation (fine, nonblotchy, truncal rash in a notvery-ill child) is characteristic of rubella, or German measles. IgM specific for rubella
can often be detected in serum within 1-2 days of developing the rash. The principal
significance of this disease is that it can cause a devastating congenital infection
characterized by ocular problems (cataracts, retinopathy, microphthalmos,
glaucoma), cardiovascular problems (patent ductus arteriosus, ventricular septal
defect, pulmonary stenosis), deafness, thrombocytopenic purpura,
hepatosplenomegaly, CNS problems, and bony lesions.
Bullous pemphigoid (choice A) produces large, tense blisters.
Dermatitis herpetiformis (choice B) causes recurrent crops of small vesicles or
papules.
Herpes simplex (choice C) is characterized by crops of vesicles on oral or genital
sites.
Measles (choice D) causes a blotchy, maculopapular erythematous rash that begins
on the face and spreads downward. Patients with measles are usually much sicker
than those with German measles, and Koplik spots maybe seen on the buccal
mucosa.

________________________________________

8>The correct answer is A. In a syndrome called cleidocranial dysostosis, absence


of part of the clavicles accompanies a broad skull, and facial and dental anomalies.
Note that you could also have answered this question by noting that of the bones
listed, only the clavicles form by intramembranous ossification.
The femurs (choice B), metatarsals (choice C), phalanges (choice D), and tibias
(choice E) are cartilaginous (formed by endochondral ossification) rather than
membranous bones.

________________________________________

9>The correct answer is A. Acute pyelonephritis is an infectious disease involving


the kidney parenchyma and the renal pelvis. Gram-negative bacteria, such as
Escherichia coli, Proteus, Klebsiella, and Enterobacter, are the most common
causative organisms in acute pyelonephritis. Laboratory evaluation will often reveal
leukocytosis with a left shift, and urinalysis typically shows pyuria, varying degrees

of hematuria, and white cell casts. Since bacteremia is present, the patient should
be hospitalized and empirically started on IV ampicillin and gentamicin. This
regimen may be need to be changed, however, once the sensitivity results are
available.
Erythromycin (choice B) and tetracycline (choice E) are both bacteriostatic
antibiotics and would not be recommended in a patient with a severe infection, such
as acute pyelonephritis with bacteremia.
Vancomycin (choice C) is primarily used in the treatment of severe gram-positive
infections.
Phenazopyridine (choice D) is a urinary analgesic, and nitrofurantoin (choice D) is a
urinary tract anti-infective. Although nitrofurantoin is indicated for the treatment of
"mild" cases of pyelonephritis, as well as cystitis, this patient's condition is severe
and should be treated with appropriate antibiotics.

________________________________________

10>The correct answer is C. The fallopian tube is the only structure in the female
genital tract with a ciliated columnar epithelium; the beating of the cilia helps move
the egg into the uterus. This fact is also sometimes clinically helpful since dilated
and deformed fallopian tubes can be microscopically distinguished from cystic
ovarian tumors by the presence of the cilia.
The cervix (choice A) and vagina (choice E) are lined by squamous epithelium.
The endometrium (choice B) is lined by columnar epithelium (although a few ciliated
cells may be present).
The covering of the ovary (choice D) is cuboidal epithelium, and cysts within the
ovary can be lined by cuboidal or non-ciliated columnar epithelium.

________________________________________

11>The correct answer is B. The description is that of gonadal dysgenesis. In the


absence of testosterone, the Wolffian ducts will regress and fail to differentiate into
normal male internal reproductive tracts. In the absence of Mllerian regression
factor, the Mllerian ducts will automatically differentiate into oviducts and a uterus.
Differentiation of the male external genitals is dependent on adequate
dihydrotestosterone (via an action of 5 a-reductase on testosterone). In the absence
of testosterone, female-type external genitalia will develop.
Selective dysgenesis of the Sertoli cells could produce the situation described in
choice A. Normal Leydig cells would secrete testosterone and produce normal male-

type internal and external tracts. However, the absence of Mllerian regression
factor, which is secreted by the Sertoli cells, would allow formation of female-type
internal structures as well.
Female-type internal reproductive tract and male-type external genitalia (choice C)
would not be likely to occur under any circumstances.
The situation described in choice D could occur with 5 a-reductase deficiency.
Normal male-type internal tracts can form because there is no requirement for
dihydrotestosterone. Mllerian regression factor will prevent differentiation of
female-type internal tracts. Since differentiation of the normal male external
genitals requires dihydrotestosterone, 5 a-reductase deficiency will lead to
feminization.
The situation described in choice E is normal, and would not occur in the individual
described who has testicular dysgenesis.

________________________________________

12>The correct answer is B. The functional residual capacity is the amount of air
left in the lungs after a normal expiration. Because this volume cannot be expired in
its entirety, it cannot be measured by spirometry. Essentially, lung volume that
contains the residual volume, which is the amount of air remaining after maximal
expiration (e.g., functional residual capacity and total lung capacity), cannot be
measured by spirometry. These volumes can be determined using helium dilution
techniques coupled with spirometry or body plethysmography.
The expiratory reserve volume (choice A) is the volume of air that can be expired
after expiration of a tidal volume.
The inspiratory reserve volume (choice C) is the volume of air that can be inspired
after inspiration of a tidal volume.
Tidal volume (choice D) is the amount of air inspired or expired with each normal
breath.
Vital capacity (choice E) is the volume of air expired after a maximal inspiration

________________________________________

13>The correct answer is E. Mastication is a complex process involving alternating


elevation, depression, forward movement, and backward movement of the lower
jaw. The backward movement step is accomplished by the posterior fibers of the
temporalis muscle.
The digastric (choice A) helps to depress the lower jaw during chewing.

The lateral pterygoid (choice B) helps to move the lower jaw forward during
chewing.
The medial pterygoid (choice C) helps to elevate the lower jaw during chewing.
The mylohyoid (choice D) helps to depress the lower jaw during chewing.

________________________________________

14>The correct answer is C. The vignette illustrates a typical presentation for a


tumor of the heart. Primary cardiac tumors are rare and usually require an intensive
work-up to pinpoint the diagnosis. 75% of primary cardiac tumors are benign and
among these, myxoma is the most common. The tumors are usually single; the
most common location is the left atrium. They may cause syncopal episodes or
even shock and death due to obstruction by a "ball valve" mechanism.
Angiosarcoma (choice A) is a malignant tumor of vascular origin that can occur as a
primary cardiac tumor. It is the most common malignant primary cardiac tumor, but
it is still very rare. Angiosarcoma usually affects the right side of the heart.
Mesothelioma (choice B) is a benign tumor of mesothelial origin that can rarely
present as a primary cardiac tumor. It is usually a small intramyocardial tumor that
presents with disturbances of the conduction system of the heart.
Rhabdomyoma (choice D) is a benign tumor of muscle origin. It can occur as a
primary cardiac tumor, typically in infants and children, in whom it may be
associated with tuberous sclerosis. It usually occurs in the ventricles.
Rhabdomyosarcoma (choice E) is a malignant neoplasm that can also occur as a
rare primary cardiac tumor. It is of muscle origin and usually affects the right heart.

________________________________________

15>The correct answer is A. The pituitary gland is located in the pituitary fossa
within the skull. The floor of this fossa is formed by the sella turcica. The lateral
walls of the fossa are formed by the cavernous sinuses. The abducens nerve passes
through the cavernous sinus along with the internal carotid artery. As the tumor
expands laterally, the first nerve that will be encountered is the abducens nerve,
producing a lateral rectus palsy.
The oculomotor nerve (choice B) lies in the lateral wall of the cavernous sinus. It is
further from the pituitary gland than is the abducens nerve.
The optic nerve (choice C) is anterosuperior to the pituitary gland. Upward
expansion of the tumor may compress the optic chiasm.

The trigeminal nerve (choice D) is found posterior to the cavernous sinus. Two of its
three divisions (ophthalmic and maxillary divisions) pass through the lateral wall of
the cavernous sinus and are further from the pituitary gland than is the abducens
nerve.
The trochlear nerve (choice E) is also in the lateral wall of the cavernous sinus, and
would be affected later if the tumor continued to expand.

________________________________________

16>The correct answer is A. Mallory bodies are eosinophilic cytoplasmic inclusions


("alcoholic hyaline") that are found in the largest numbers in alcoholic hepatitis.
They were originally considered to be pathognomic of alcohol abuse, but have since
been found (in much smaller numbers) in many other liver conditions.
Alpha1-antitrypsin deficiency (choice B) involvement of the liver is characterized by
periodic acid Schiff (PAS)-positive cytoplasmic granules in hepatocytes.
Hepatitis A (choice C) and hepatitis B (choice D) infections are definitively
established with serologic markers.
In Wilson's disease (choice E), there is excess copper deposition in the liver

________________________________________

17>The correct answer is E. In social learning, also known as modeling, behavior is


acquired by watching other persons and assimilating their actions into the
behavioral repertoire. There is no verbal or cognitive process (choice B) that is
involved, no reinforcement (as in operant conditioning; choice D), no pairing of
stimuli to get stimulus substitution (as in classical conditioning; choice A), nor any
early-life bonding or imprinting (choice C) involved in this type of process. Because
behaviors such as spousal abuse, child abuse, and elder abuse are all based on
observing and incorporating behaviors from significant others, the person displaying
the behaviors does not realize the behaviors are inappropriate and is typically very
resistant to change. The fact that the learning is nonverbal and not dependent upon
reinforcement contributes to the resistance to change.
________________________________________

18>The correct answer is B. Sucralfate is a promising drug that is not presently in


widespread use because it is incompatible with H2 antagonists such as cimetidine,
ranitidine, famotidine and nizatidine. Sucralfate is aluminum sucrose sulfate, a
sulfated disaccharide, which polymerizes and binds to ulcerated tissue. It forms a

protective coating against acid, pepsin and bile, giving the tissue a chance to heal.
Unfortunately, a low gastric pH is required for polymerization, meaning that
sucralfate is incompatible with drugs that reduce gastric acidity, such as H2
blockers and antacids. The moral of the story is that you cannot assume that two
medications that are individually helpful in a medical condition will be synergistic.
Learning the mechanisms by which the drugs work will help you spot potential
interactions and earn you points on the USMLE.

________________________________________

19>The correct answer is B. The patient's presentation is consistent with


hypothyroidism. Serum thyroid-stimulating hormone (TSH) measurement (choice B)
is most likely to confirm the empiric diagnosis. TSH levels usually rise above normal
before serum thyroxine (T4; choice C) and serum triiodothyronine (T3; choice D)
levels do, even in mild cases of hypothyroidism. Therefore, TSH measurement would
be the most accurate test to determine the presence of hypothyroidism regardless
of the severity.
A high titer of antithyroid antibodies (choice A) is characteristic of chronic
thyroiditis, which is the most common cause of hypothyroidism. However, detection
of these antibodies would not indicate if hypothyroidism was present.
T3 resin uptake (choice E) measurement is not an accurate test of thyroid function;
it is primarily used to exclude various abnormalities in the thyroid-hormone binding
proteins.

________________________________________

20>The correct answer is A. Warfarin is the oral anticoagulant the patient was most
likely taking. This drug is commonly prescribed to patients with atrial fibrillation to
prevent the formation of atrial thrombi. Warfarin increases prothrombin time (PT)
because it interferes with the synthesis of the vitamin K clotting factors of the liver
(II, VII, IX, and X) and therefore necessitates regular monitoring of the PT. Cimetidine
is an H2-blocker that inhibits hepatic enzymes, including those that metabolize
warfarin. Consequently, coadministration of warfarin and cimetidine results in
enhanced warfarin activity, producing pronounced anticoagulation and the bleeding
diathesis in the patient in question. Cimetidine has one of the worst side effect
profiles of all the H2-blockers and may also result in gynecomastia in men.
Famotidine (choice B) is an H2-blocker that does not affect liver metabolism.
Misoprostol (choice C) is a prostaglandin E1 analog used in peptic ulcer disease. It
does not affect hepatic metabolism.

Omeprazole (choice D) is a proton-pump inhibitor used to decrease acid production


in patients with peptic ulcer disease or reflux. It does not affect drug metabolism by
the liver.
Ranitidine (choice E) is another H2-blocker. It does not inhibit liver enzymes as
strongly as cimetidine does.

________________________________________

21>The correct answer is C. Bullae with the cleavage plane above the basal layer of
the epidermis suggests pemphigus vulgaris, which is caused by autoantibodies to
intercellular junctions of epidermal cells. The autoantibodies decrease the ability of
the keratinocytes to adhere to one another, permitting formation of vesicles and
bullae. Oral involvement is common, and often precedes the characteristic skin
lesions. Separation of the epidermis upon manual stroking of the skin is known as
Nikolsky's sign. This sign is present in other disorders such as Stevens-Johnson
syndrome, but we are told the woman is not taking any medications, a typical cause
of Stevens-Johnson syndrome in the adult population.
Antibodies to epidermal basement membrane proteins (choice A) are seen in
bullous pemphigoid, which is a bullous disease characterized by blisters with a
cleavage line between the epidermis and dermis.
Antibodies to glycoprotein IIb/IIIa (choice B) are seen in autoimmune
thrombocytopenic purpura.
Antibodies to intrinsic factor (choice D) are seen in pernicious anemia.
Antibodies to Type IV collagen (choice E) are seen in Goodpasture's syndrome.

________________________________________

22>The correct answer is D. The lesion is a malignant melanoma. Melanomas can


develop either de novo or in an existing mole. Sunlight exposure is a significant risk
factor and fair-skinned persons are at increased risk of developing melanoma. The
most significant factor for long term prognosis is the depth of the lesion, since the
superficial dermis lies about 1 mm under the skin surface, and penetration to this
depth is associated with a much higher incidence of metastasis than is seen with a
more superficial location.
The circumference of the lesion (choice A) is much less important than depth, since
one form of melanoma (superficial spreading) can still have good prognosis despite
large size, if it has not extended to the depth of the superficial dermal lymphatic
bed.

The darkness (choice B) or degree of variation in color (choice C) do not have


prognostic significance once melanoma is diagnosed.
Irregularity, or fuzziness at the border (choice E) of a mole-like lesion is a good clue
to potential malignancy, but does not affect prognosis once a melanoma is
diagnosed.

________________________________________

23>The correct answer is E. By the third week of development, hematopoiesis


begins in the blood islands of the yolk sac. Beginning at 1 month of age and
continuing until 7 months of age, blood elements are also formed in the liver.
Hematopoiesis occurs in the spleen and lymphatic organs between 2 and 4 months,
and in the bone marrow after 4 months.

________________________________________

24>The correct answer is E. The patient has I-cell disease, also known as
mucolipidosis II, which is due to a defective UDP-N-acetylglucosamine-1phosphotransferase, the enzyme that phosphorylates mannose on enzymes
destined for lysosomes. Proteins coded by nuclear DNA are synthesized on
cytoplasmic ribosomes, which may be either "free" or associated with the
endoplasmic reticulum to form the rough endoplastic reticulum (RER). Proteins
synthesized on the RER are transferred into the Golgi apparatus, where they
undergo further modifications that determine whether they remain part of the Golgi
apparatus, become part of the plasma membrane, or are shipped to lysosomes or
mitochondria. Proteins not marked for transport to a specific intracellular site follow
the default pathway and are exported into the extracellular compartment. The
signal for transport of the acid hydrolases (and probably other enzymes) to the
lysosomes is phosphorylation of a terminal mannose moiety on an N-linked
oligosaccharide to form mannose 6-phosphate. In I-cell disease, this terminal
mannose moiety is not phosphorylated, and the acid hydrolases follow the default
pathway and are secreted.
Deficiency of alpha-L-iduronidase results in lysosomal accumulation of dermatan
sulfate and heparan sulfate (choice A) in several conditions such as
mucopolysaccharidosis I, Hurler's disease, or Hurler's/Scheie disease.
Hexosaminidase A deficiency (Tay-Sachs disease) is one example of a condition in
which ganglioside accumulation occurs (choice B).
There are a number of diseases in which glycogen degradation (choice C) is
defective. These are collectively termed glycogen storage diseases since they result
in abnormal cellular accumulation of glycogen. In Pompe's disease, or type II

glycogen storage disease, a lysosomal glucosidase is deficient, resulting in


lysosomal glycogen accumulation.
Deficiency of sphingomyelinase (choice D), an enzyme involved in degradation of
sphingomyelin, results in Niemann-Pick disease.
Phosphorylation of tyrosine moieties (choice F) is unrelated to lysosomes or
lysosomal enzymes; however, decreased ability to phosphorylate tyrosine moieties
might be associated with diabetes or dwarfism.

________________________________________

25>The correct answer is E. 95% of a normally distributed population will fall


between plus or minus 1.96 standard deviations from the mean. Since the
population is normally distributed with regard to IQ, this means that approximately
2.5% of the population will have IQ scores 2 standard deviations or more above the
mean, and 2.5% of the population will have IQ scores 2 standard deviations or more
below the mean. 2.5% of 200 people is 5 people.

________________________________________

26>The correct answer is E. All the compounds listed can produce crystals in joint
fluid, but only monosodium urate (associated with gout) and calcium pyrophosphate
dihydrate (associated with CPPD crystal deposition disease, also called pseudogout),
and to lesser degree basic calcium phosphate (apatite-associated arthropathy),
have a high likelihood of being encountered on a step 1 USMLE exam. The crystals
described are those of monosodium urate. Be careful not to answer "uric acid" if
that is listed as an alternative choice on an exam, since the sodium salt is the
predominant species in vivo.
Basic calcium phosphate (choice A) is seen in apatite-associated arthropathy and
produces spherical clumps of nonbirefringent submicroscopic crystals.
Calcium oxalate crystals (choice B) are seen in primary oxalosis and are
bipyramidal, positively birefringent crystals.
Calcium pyrophosphate dihydrate crystals (choice C) are a feature of pseudogout
and are rod-to-rhomboidal-shaped, weakly positively birefringent crystals.
Cholesterol crystals (choice D) are seen in chronic and chylous effusions in
inflammatory and degenerative arthritis, where they form large, flat, rhomboidal
plates with notched corners.

________________________________________

27>The correct answer is B. The innervation of the tongue is complex. The


mandibular division of the trigeminal nerve (V3) carries general somatic sensation
from the anterior two-thirds of the tongue.
The maxillary division (V2, choice A) carries somatic sensation from the palate,
upper gums, and upper lip.
The facial nerve (VII, choice C) carries taste from the anterior two-thirds of the
tongue.
The glossopharyngeal nerve (IX, choice D) carries sensation and taste from the
posterior one-third of the tongue.
The vagus nerve (X, choice E) carries sensation from the lower pharynx.

________________________________________

28>The correct answer is B. Brucella abortus produces a chronic, granulomatous


disease with caseating granulomas. Most cases occur in four states (Texas,
California, Virginia, and Florida), and are associated with cattle, in which it produces
spontaneous septic abortions. Most cases of brucellosis produce mild disease or
fevers of unknown origin. However, Brucella spp. can infect the cardiovascular
system and cause a localized infection. B. abortus is the most common species to
cause endocarditis. The aortic valve is most commonly involved, followed by the
mitral valve, and then both valves. Most cases of brucellosis are associated with
occupational exposure, in persons such as veterinarians, ranchers, and those who
handle carcasses.
Bacillus anthracis (choice A) is the causative agent for anthrax. It usually produces
cutaneous disease (malignant pustule or eschar) at the site of inoculation in
handlers of animal skins. It can also produce a severe hemorrhagic pneumonia
(Woolsorter's disease) and septicemia. At-risk groups include those who handle
animal carcasses or skins.
Coccidioides immitis (choice C) is a dimorphic fungal disease producing a
granulomatous pulmonary syndrome that is more severe in dark-skinned
individuals. Disseminated disease occurs most often in Filipinos, Mexicans, and
Africans. The infective form is the arthrospore; the diagnostic form in tissue is the
spherule containing endospores. The disease is endemic in the San Joaquin River
Valley. At-risk groups include military personnel, agricultural workers, construction
workers, oil field workers, archaeology students, participants in outdoor sports, and
sightseers. Remote infections from fomites (cotton harvested in the Southwestern
U.S.) have been reported.
Erysipelothrix rhusiopathiae (choice D) is a pleomorphic, gram-negative rod that
causes a localized skin infection. It is an occupational disease of fishermen, fish

handlers, butchers, meat-processing workers, poultry workers, farmers,


veterinarians, abattoir workers, and housewives.
Trichinella spiralis (choice E) is a nematode infection caused by the ingestion of
larvae found in undercooked meat. Pork is the most common contaminated meat.
However, outbreaks in the northern parts of the U.S. have been associated with
eating undercooked infected bear meat. Symptoms include diarrhea, periorbital
edema, myositis, fever, and eosinophilia.

________________________________________

29>The correct answer is B. This patient has Tay-Sachs disease, an autosomal


recessive disorder caused by the deficiency of hexosaminidase A, which leads to the
accumulation of ganglioside GM2 in neurons, producing a degenerative neurologic
disease. Children appear normal at birth, but then begin to suffer from diminished
responsiveness, deafness, blindness, loss of neurologic function, and seizures. A
cherry-red spot on the macula may be seen by ophthalmoscopic examination. Death
usually occurs by 4 to 5 years of age. There is no therapy. The incidence is higher
among Jews of Eastern European descent. Since the parents must be heterozygotes
for the mutant hexosaminidase A allele, they would be expected to have diminished
levels of the enzyme.
A defect in the dystrophin (choice A) gene produces Duchenne muscular dystrophy,
characterized by onset of weakness in early childhood.
A severe deficiency in HGPRT (choice C) will lead to Lesch-Nyhan syndrome,
characterized by excessive uric acid production, mental retardation, spasticity, selfmutilation, and aggressive, destructive behavior.
Deficiency of phenylalanine hydroxylase (choice D) results in classic
phenylketonuria, a disease in which phenylalanine, phenylpyruvate, phenylacetate,
and phenyllactate accumulate in plasma and urine. Clinically, there is a musty body
odor and mental retardation.
Hypophosphatemic rickets is an X-linked dominant condition causing abnormal
regulation of vitamin D3 (choice E) metabolism and defects in renal tubular
phosphate transport. Symptoms include growth retardation, osteomalacia, and
rickets.

________________________________________

30>The correct answer is C. The fibrinolytic activity of streptokinase is due to its


ability to bind and cleave plasminogen, producing plasmin. Plasmin directly cleaves
fibrin, both between and within the fibrin polymers, thus breaking up thrombi and

potentially restoring blood flow to ischemic cardiac muscle. This same mechanism
of fibrinolysis is shared by urokinase and tissue-plasminogen activator (tPA).
Antithrombin III (choice A) is a coagulation inhibitor that binds to and inactivates
thrombin. Antithrombin III is anticoagulant, not fibrinolytic.
Fibrin (choice B) is not directly acted upon by streptokinase. It is indirectly cleaved
through the action of plasmin.
Protein C (choice D) is a glycoprotein that modulates coagulation by inhibiting the
procoagulant activities of factors V/Va and VIII/VIIIa. Protein C has no inherent
fibrinolytic activity.
Thrombomodulin (choice E) is an anticoagulant protein that binds to thrombin and
diminishes its capacity to activate fibrinogen, Factor V, and platelets.
Thrombomodulin has no fibrinolytic activity.
________________________________________

31>The correct answer is C. The left adrenal vein and the left gonadal vein (either
testicular or ovarian) drain into the left renal vein. The left renal vein then drains
into the inferior vena cava. In contrast, the right adrenal vein and right gonadal vein
drain directly into the inferior vena cava.
The hemiazygos vein (choice A) receives the venous drainage from the body wall on
the left side of the thorax and abdomen. No visceral organs drain directly to the
azygos or hemiazygos veins.
The inferior vena cava (choice B) receives the direct venous drainage from the right
adrenal vein, but not the left adrenal vein. Remember, the inferior vena cava is on
the right side of the abdomen.
The splenic vein (choice D) receives the venous drainage from the spleen and part
of the pancreas and stomach. The splenic vein is part of the portal venous system.
The superior mesenteric vein (choice E) receives venous drainage from much of the
intestinal tract. It is part of the portal venous system and joins with the splenic vein
to form the portal vein.
________________________________________

32>The correct answer is E. The child has neonatal respiratory distress syndrome
(hyaline membrane disease). This condition is caused by the inability of the
immature lungs to synthesize adequate amounts of surfactant. Surfactant, which
reduces surface tension, helps keep alveoli dry and aids in expansion of the lungs, is
synthesized by type II pneumocytes.
Alveolar capillary endothelial cells (choice A) are important in maintaining the
capillary structure and permitting flow of gases into and out of the blood stream.

Bronchial mucous cells (choice B) produce the usually thin (in healthy individuals)
coat of mucus that lines the bronchi.
The ciliated bronchial respiratory epithelium (choice C) is responsible for moving the
dust coated mucus layer out of the bronchi.
Type I pneumocytes (choice D) are the squamous cells that line alveoli and permit
easy gas exchange. These cells tend to be immature (and thick) in premature
infants, but do not produce surfactant.
________________________________________

33>The correct answer is A. Surgeons worry about their obese patients more than
their skinny ones, because a thick layer of relatively poorly vascularized
subcutaneous fatty tissue is both mechanically unstable (it holds stitches poorly)
and heals very slowly. These patients have a frequent rate of dehiscence (tearing
open of the incisional site) with subsequent, difficult-to-control infection (access by
antibiotics, leukocytes, and serum antibodies are all hampered by the poor blood
supply).
Aponeuroses (choice B) are strong thickenings of muscle sheath that usually suture
and heal well after surgery.
Loose connective tissue (choice C) is well vascularized and surgeons do not usually
worry much about it during the healing process.
Muscle (choice D) usually heals well after surgery.
Skin (choice E) usually heals well, unless it becomes infected.
________________________________________

34The correct answer is E. Rotavirus is the major cause of diarrhea in infants and
children under the age of 2. The replicates in the intestinal mucosa, producing a
profuse, watery, non-bloody diarrhea, often coupled with nausea and vomiting.
Transmission is by the fecal-oral route.
Coronaviruses (choice A) usually cause cold-like illnesses.
Lymphocytic choriomeningitis virus (choice B) can cause headache, malaise,
myalgia, conjunctivitis, and, occasionally, meningitis.
Norwalk agent (choice C) can also cause diarrhea, but usually affects patients older
than 2 years.
Orbivirus (choice D) is the cause of Colorado tick fever, which is the only tick-borne
viral disease in the United States.
________________________________________

35>The correct answer is B. This is a tough two-step style question. You first needed
to make the diagnosis, and then you needed to recall the localization of this
particular disease to a specific chromosome. In this case, everything in the vignette
leads you to a diagnosis of Wilms' tumor. Wilms' tumor occurs in children and
typically presents with an abdominal mass as well as with hypertension, hematuria,
nausea, and intestinal obstruction. Because the tumor is derived from mesonephric
mesoderm, it can include mesodermal derivatives such as bone, cartilage, and
muscle. The Wilms' tumor suppressor gene (WT-1) has been localized to
chromosome 11 (11p).
The remaining answer choices provide us the opportunity to discuss some other
known relationships between genes and disease. This is a topic of intense research
that is likely to become more and more emphasized on the USMLE examinations.
Chromosome 5 (choice A) is the site of the tumor suppressor gene APC, which is
involved in the pathogenesis of colon cancer and familial adenomatous polyposis.
Chromosome 13 (choice C) is the site of the tumor suppressor gene for
retinoblastoma and osteosarcoma (Rb) as well as the BRCA-2 gene for breast
cancer.
Chromosome 17 (choice D) is the site of p53 (involved in most human cancers), NF1 (neurofibromatosis type I), and BRCA-1 (breast and ovarian cancer).
Chromosome 22 (choice E) is home to the NF-2 gene, which is involved in
neurofibromatosis type II.
________________________________________

36>The correct answer is E. The condition described is spinal bifida with


myelomeningocele. A failure of the posteriorly located vertebral arches to fuse
posteriorly causes spina bifida, which can vary in severity from a completely
asymptomatic minor abnormality to protrusion of the spinal cord and roots through
an open skin defect, with a very real risk of minor trauma or infection causing
paralysis.
Failure of development of one of the paired primary ossification centers (choice A)
of the body can produce very severe scoliosis.
The bodies of the vertebrae (choice B) are the stacking ovoid structures on the
anterior aspect of the spinal canal.
The pedicles (choice C) attach the body ring that surrounds the spinal canal to the
body of the vertebrae.
The superior articular process (choice D) articulates with the inferior articular facet
of the vertebra above it.
________________________________________

37The correct answer is E. This patient is experiencing symptoms of hypocalcemia


secondary to diminished parathyroid hormone (PTH) secretion. This must always be
considered in a patient who undergoes total or subtotal thyroidectomy because the
parathyroids are nestled in the tissue surrounding the thyroid gland. Surgical
attempts to leave portions of the parathyroids intact are sometimes unsuccessful.
Other causes of decreased PTH include neck irradiation, autoimmune phenomena
(polyglandular autoimmune syndromes), dysembryogenesis (as in DiGeorge's
syndrome), or as a result of heavy metal damage (Wilson's disease, hemosiderosis,
hemochromatosis).
Bone metastases (choice A) would cause hypercalcemia, as a result of osteolysis.
Hashimoto's thyroiditis (choice B) is the most common cause of hypothyroidism and
results in decreased thyroid hormone and elevated TSH levels. Serum calcium and
PTH should be normal.
Hypervitaminosis D (choice C) would cause hypercalcemia.
Hypomagnesemia (choice D) may cause a functional hypoparathyroidism because
magnesium is needed for PTH activity in tissue. However, in such a case, actual PTH
levels would not be decreased.
________________________________________

38>The correct answer is B. Fenestrated ("windowed") endothelial cells, which


permit free flow of serum across the endothelium, are an unusual feature of the
hepatic sinusoids. They are not seen in the other organs listed.

39>The correct answer is D. Chronic heart failure results in blood stasis in the
central veins and central sinusoids of hepatic lobules, with subsequent central
hemorrhagic necrosis. Thus, the red central regions compared with the surrounding
tan-brown viable parenchyma impart the mottled appearance of a nutmeg to the
liver cut surface.
Acute left-sided heart failure (choice A) gives rise to acute pulmonary edema, with
extravasation of plasma and red blood cells into alveolar spaces.
Acute right-sided heart failure (choice B) leads to acute congestion of the liver,
which does not acquire the typical nutmeg appearance as in chronic congestion.
Alcohol toxicity (choice C) leads to a number of hepatic alterations. Fatty change
results from alteration in lipoprotein metabolism, leading to accumulation of fats as
a large single droplet within the cytoplasm of hepatocytes (microvesicular
steatosis). Alcoholic hepatitis is an acute response to alcohol abuse and is
associated with hepatocyte necrosis and formation of Mallory bodies. Long-standing

alcohol toxicity may cause a perturbation of the liver architecture leading to liver
cirrhosis.
Liver cirrhosis (choice E) is an end-stage condition that may be caused by a number
of chronic insults, such as alcohol toxicity, viral hepatitis B or C, and
hemochromatosis. It results from simultaneous degeneration and regeneration of
the liver parenchyma with formation of broad scars connecting portal spaces. The
normal liver architecture is entirely lost and replaced by regenerating nodules. The
nodular pattern is the distinguishing macroscopic feature that allows easy diagnosis,
even on gross examination.

________________________________________

40>The correct answer is A. This man has a respiratory acidosis. Overdose with
drugs that suppress ventilation (e.g., heroin, morphine, barbiturates, methaqualone,
and "sleeping pills") often causes hypercapnia. In patients with an intact renal
response, the respiratory acidosis causes a compensatory rise in plasma HCO3-,
which lessens the fall in pH. However, the renal response requires several days to
develop fully. The plasma HCO3- of 26 mEq/L (normal: 22-28 mEq/L) for this man is
typical of acute respiratory acidosis with little or no renal compensation.
Choice B reflects metabolic acidosis.
Choice C is normal.
Choice D reflects respiratory alkalosis.
Choice E reflects metabolic alkalosis.

________________________________________

41>The correct answer is D. The forced vital capacity (FVC) is the difference in
volume between the total lung capacity (TLC) and the residual volume (RV). The TLC
and RV are represented on the diagram as the points of intersection between the
abscissa and flow-volume curve: TLC = 6 L and RV = 1.5 L. Therefore, FVC = 6 - 1.5
= 4.5 Liters. Although the diagram shows absolute lung volumes, these cannot be
obtained from a forced expiration without first determining the residual volume
using other methods (for the same reason that TLC and RV cannot be determined
using a spirometer). However, it is still possible to determine FVC because this is the
difference between TLC and RV, and differences in volume can be determined from
a forced expiration diagram.

________________________________________

42>The correct answer is A. This person is blind in the right eye. The afferent limb
of the pupillary light reflex is carried by the optic nerve (CN II), and the efferent limb
is via the oculomotor nerve (CN III), which carries parasympathetic fibers from the
Edinger-Westphal nucleus. Thus, shining a light in the affected eye will not elicit any
pupillary response. On the other hand, shining the light in the left eye will result in
simultaneous constriction of both pupils (assuming an intact right CN III), since the
left optic nerve is intact.

________________________________________

43>The correct answer is C. Flow has increased 7-fold, indicating a decrease in


cross-sectional area by a factor of 7. This would be caused by an obstruction, not an
aneurysm.
Choice A is incorrect, because a coronary artery aneurysm would produce an
increase in cross-sectional area rather than a decrease.
Flow has increased 7-fold, indicating a decrease in vessel diameter, thus choices B
and D are incorrect.

________________________________________

44>The correct answer is C. The table shows that the greatest fall in blood pressure
(50 mm Hg) occurs in the arterioles, which indicates that the arterioles account for
about 50% of the total peripheral resistance. The structural characteristics of
arterioles are consistent with their function as control valves that regulate blood
flow to the capillary networks of the body. Thus, arterioles are thick-walled vessels
with the highest ratio of wall cross-sectional area to lumen cross-sectional area. This
does not mean that arterioles have thicker walls compared to arteries. It simply
means that the walls of arterioles are relatively thick compared to their overall size
(diameter). The wall-to-lumen ratio of arteries, which includes the aorta (choice A)
as well as large (choice A) and small arteries (choice B), is less than that of
arterioles but greater than that of venules and veins (choices E and F). The
capillaries (choice D) lack smooth muscle cells in their walls, which makes wall-tolumen ratio measurements much less meaningful.

________________________________________

45>The correct answer is E. The third pharyngeal pouch normally gives rise to the
inferior parathyroid glands and the thymus. Cells from these pouches migrate
caudally in the embryo to the eventual location of these organs. The superior
parathyroid glands are derived from the fourth pharyngeal pouch. The absence of
these organs is found in DiGeorge syndrome, which typically presents with
immunodeficiency and hypoparathyroidism.
The second pharyngeal arch (choice A) gives rise to several skeletal and muscular
structures. These include the stapes, styloid process, stylohyoid ligament, a portion
of the hyoid bone, the stapedius muscle, stylohyoid muscle, posterior digastric
muscle, and all of the muscles of facial expression.
The second pharyngeal cleft (choice B) is normally obliterated. A persistence of this
cleft may result in a lateral cervical cyst, sinus, or fistula.
The second pharyngeal pouch (choice C) gives rise to the tonsillar fossa and the
palatine tonsils.
The third pharyngeal arch (choice D) gives rise to most of the hyoid bone and the
stylopharyngeus muscle. The third aortic arch, which passes through the third
pharyngeal arch, gives rise to the common carotid artery and the internal carotid
artery.

________________________________________

46>The correct answer is D. The muscles in question are the gluteus medius and
gluteus minimus, which are the innervated by the superior gluteal nerve. Textbooks
often describe these muscles as abductors of the hip, but in real life, they usually
function as described in the question.
The femoral nerve (choice A) innervates muscles of the anterior thigh, allowing
extension of the leg at the knee.
The first and second sacral nerves (choice B) innervate the piriformis muscle, which
is a lateral rotator of the thigh at the hip.
The obturator nerve (choice C) innervates muscles of the medial thigh.
The tibial nerve (choice E) innervates muscles of the posterior lower leg.

47>The correct answer is E. Approximately 60% of patients in the United States


who contract Lyme disease, but are not treated with antibiotics, will develop frank
arthritis. The pattern typically consists of intermittent attacks of oligoarticular
arthritis in large joints (especially knees) lasting for weeks to months in a given
joint. Patients with persistent arthritis have a higher frequently of HLA-DR4 class II
MHC complex than patients with brief Lyme arthritis or normal controls.

HLA-B9 (choice A), and HLA-B17 (choice B) are not thought to be associated with
Lyme disease arthritis.
HLA-DR3 (choice D) is associated with a variety of disorders, but not Lyme disease
arthritis.
HLA-B27 (choice C), although associated with reactive arthritis, is not associated
with Lyme disease arthritis.

________________________________________

48>The correct answer is F. Of the processes listed, only ketone body synthesis
occurs exclusively in the mitochondria. Other mitochondrial processes include the
production of acetyl-CoA, the TCA cycle, the electron transport chain, and fatty acid
oxidation.
Processes that occur exclusively in the cytoplasm include cholesterol synthesis
(choice A; in cytosol or in ER), fatty acid synthesis (choice B), glycolysis (choice D),
and the hexose monophosphate shunt (choice E).
Note that gluconeogenesis (choice C) and the urea cycle (choice G) occur in both
the mitochondria and the cytoplasm.

________________________________________

49>The correct answer is C. DiGeorge syndrome is due to fetal malformation of


epithelial elements of the third and fourth pharyngeal pouches, leading to thymic
aplasia or severe hypoplasia and sometimes, absence of the parathyroids. Clinically,
the condition is classified as a selective T-cell deficiency, but varying degrees of
deficiency of antibody production may also be seen. Affected infants are susceptible
to fungal and viral infections, and may exhibit tetany due to severe hypocalcemia.
In autosomal recessive ataxia telangiectasia (choice A), progressive cerebellar
ataxia is accompanied by multiple telangiectasias on exposed skin and a variable,
usually moderate, immunodeficiency that may involve both antibody production and
cell-mediated immunity. Chronic or recurrent sinus and pulmonary infections result
in bronchiectasis.
In Bruton's (X-linked) agammaglobulinemia (choice B), children have normal
numbers of circulating T cells with very few circulating B cells and only tiny amounts

of circulating antibody. Affected individuals have recurrent bacterial infections


beginning late in the first year of life.
Severe combined immunodeficiency (choice D) is actually a cluster of several
diseases with variable genetics characterized by severely deficient T cell functions
and variable (often depressed) antibody production. Multiple, sometimes
simultaneous, infections with viruses, bacteria, and fungi occur.
Wiskott-Aldrich syndrome (choice E) is an X-linked disease with thrombocytopenia,
lymphopenia, and decreased T cell function. Lymphoid malignancy (e.g., acute
lymphocytic leukemia) may occur.

________________________________________

50>The correct answer is E. von Gierke's disease is a glycogen storage disease


caused by a deficiency of glucose-6-phosphatase. It typically presents with neonatal
hypoglycemia, hyperlipidemia, lactic acidosis, and ketosis. Failure to thrive is
common in early life; convulsions may occur due to profound hypoglycemia. The
glycogen accumulation in von Gierke's disease occurs primarily in the liver and
kidneys, accounting for the enlargement of these organs. Gout may develop later
because of the derangement of glucose metabolism.
Even if you do not remember all of the details of the presentation of these genetic
diseases, you should be able to narrow the choices:
Gaucher's disease (choice A) and Niemann-Pick disease (choice C) are lipid storage
diseases, and would not be expected to produce hypoglycemia.
The other diseases are glycogen storage diseases, but McArdle's (choice B) and
Pompe's (choice D) disease affect muscle rather than liver and would not be
expected to produce profound hypoglycemia, since the liver is the major source for
blood glucose.

Answers

________________________________________

1>The correct answer is C. This fact is obviously clinically relevant: in one quarter of
patients with ischemic heart disease, the "presenting" symptom is death! This
makes a very good case for aggressive screening of any patient groups at relatively
high risk for atherosclerosis.
________________________________________

2>The correct answer is C. Laryngeal cartilages (e.g., the thyroid, cricoid, arytenoid
cartilages) are derived from neural crest.
The endothelial cells (choice A) in the simple squamous epithelium that lines the
pulmonary capillaries are derived from visceral mesoderm.
The epithelial lining of primary bronchi (choice B) is derived from endoderm.
Tracheal glands (choice D) and epithelium both derive from endoderm.
Type I pneumocytes (choice E) are derived from endoderm.
________________________________________

3>The correct answer is D. The arrow is in the fasciculus cuneatus, a tract in the
white matter of the spinal cord. Therefore, it is within the central nervous system
(CNS). Myelin in the central nervous system is formed by oligodendrocytes. Each
oligodendrocyte myelinates several axons.
Astrocytes (choice A) are stellate appearing cells possessing branching processes
that associate with pia mater, neurons, and endothelial cells within the CNS. While
they may provide a secondary component of the blood-brain barrier (tight junctions
account for the primary barrier), one of their main functions is to modulate the
molecular composition of extracellular fluid in the CNS. They do not produce myelin.
Dorsal root ganglion cells (choice B) are pseudounipolar neurons that provide
sensory input to the spinal cord. Their axons may be myelinated, but they do not
form the myelin.
Microglia (choice C) are part of the mononuclear phagocyte system, being derived
from monocytes. They are phagocytic and do not produce myelin.
Schwann cells (choice E) form myelin in the peripheral nervous system. A

________________________________________

4>The correct answer is D. The lesion is a malignant melanoma. Melanomas can


develop either de novo or in an existing mole. Sunlight exposure is a significant risk
factor and fair-skinned persons are at increased risk of developing melanoma. The

most significant factor for long term prognosis is the depth of the lesion, since the
superficial dermis lies about 1 mm under the skin surface, and penetration to this
depth is associated with a much higher incidence of metastasis than is seen with a
more superficial location.
The circumference of the lesion (choice A) is much less important than depth, since
one form of melanoma (superficial spreading) can still have good prognosis despite
large size, if it has not extended to the depth of the superficial dermal lymphatic
bed.
The darkness (choice B) or degree of variation in color (choice C) do not have
prognostic significance once melanoma is diagnosed.
Irregularity, or fuzziness at the border (choice E) of a mole-like lesion is a good clue
to potential malignancy, but does not affect prognosis once a melanoma is
diagnosed.

________________________________________

5>The correct answer is C. This is a case of Reiter's syndrome. Patients typically


present with the acute onset of arthritis (usually asymmetric and additive), with
involvement of new joints occurring over a period of a few days to 2 weeks. Joints of
the lower extremities are the most commonly involved, but wrists and fingers can
also be affected. Dactylitis (sausage digit), a diffuse swelling of a solitary finger or
toe, is a distinctive feature of Reiter's arthritis and psoriatic arthritis. Tendonitis and
fasciitis are common. Spinal pain and low back pain are common. Conjunctivitis,
urethritis, diarrhea, and skin lesions are also associated with Reiter's syndrome. Up
to 75% of patients are HLA-B27 positive. Microorganisms which can trigger Reiter's
syndrome include Shigella spp., Salmonella spp., Yersinia spp., Campylobacter
jejuni, and Chlamydia trachomatis. Most patients are younger males.
Gout (choice A) usually presents as an explosive attack of acute, very painful,
monarticular inflammatory arthritis. Hyperuricemia is the cardinal feature and
prerequisite for gout. The first metatarsophalangeal joint is involved in over 50% of
first attacks.
Lyme disease (choice B), caused by Borrelia burgdorferi, presents with a red macule
or papule at the site of the tick bite. This lesion, called erythema chronicum
migrans, slowly expands to form a large annular lesion with a red border and central
clearing. The lesion is warm, but usually not painful. The patient also has severe
headache, stiff neck, chills, arthralgias, and profound malaise and fatigue.
Untreated infection is associated with development of arthritis. The large joints
(e.g., knees) are usually involved with the arthritis lasting for weeks to months.
Rheumatoid arthritis (choice D) begins insidiously with fatigue, anorexia,
generalized weakness, and vague musculoskeletal symptoms leading up to the
appearance of synovitis. Pain in the affected joints, aggravated by movement, is the

most common manifestation of established rheumatoid arthritis. Generalized


stiffness is frequent and is usually greatest after periods of inactivity. Morning
stiffness of greater than 1 hour in duration is very characteristic. Rheumatoid
arthritis is more common in females. The metacarpophalangeal and proximal
interphalangeal joints of the hands are characteristically involved.
Septic arthritis (choice E) is caused by a variety of microorganisms, including
Neisseria gonorrhoeae and Staphylococcus aureus. Hematogenous spread is the
most common route in all age groups. 90% of patients present with involvement of
a single joint, usually the knee. The usual presentation is moderate-to-severe pain,
effusion, muscle spasm, and decreased range of motion. Peripheral leukocytosis and
a left shift are common. Disseminated gonococcal infections present as fever, chills,
rash, and articular symptoms. Papules progressing to hemorrhagic pustules develop
on the trunk and extensor surfaces of the distal extremities. Migratory arthritis and
tenosynovitis of multiple joints is common.

________________________________________

6>The correct answer is C. The cells in question are mast cells, which play an
important role in IgE-mediated allergic responses. They are a normal (minor)
constituent of dermal skin and are most definitely identified with stains, such as the
Giemsa stain, that highlight the granularity of their cytoplasm (mast cells are
closely related to blood basophils). Mast cells secrete histamine, serotonin, many
leukotrienes, and platelet aggregating factor (PAF). Their histamine secretion after
IgE stimulation helps to trigger the acute inflammatory part of the allergic response.
Bradykinin (choice A) is a nonapeptide found in the plasma. It is a potent vasodilator
and stimulates pain receptors.
Complement factor 3a (choice B) is a plasma protein that induces vascular leakage
as part of the acute inflammatory response.
Interleukin 2 (choice D) is secreted by lymphocytes and augments the immune
response.
Nitric oxide (choice E) is released by macrophages and endothelium. It causes
vasodilation and cytotoxicity.

________________________________________

7>The correct answer is D. The patient described probably has systemic lupus
erythematosus (SLE). This is a systemic disorder that often presents with fatigue,
malaise, fever, gastrointestinal symptoms, arthralgias, and myalgias. Hematologic
abnormalities include anemia of chronic disease, hemolytic anemia, leukopenia,

lymphocytopenia, and thrombocytopenia. A circulating anticoagulant may prolong


the APTT (activated partial thromboplastin time). Cutaneous manifestations include
a malar rash and a generalized maculopapular eruption, both of which are
photosensitive. Antibodies to the Smith antigen (core proteins of small
ribonucleoproteins found in the nucleus) are present in only 20-30% of patients with
SLE, but are quite specific for the disease, occurring only rarely in other
autoimmune diseases.
Anti-centromere antibody (choice A) is specific for the CREST (Calcinosis, Raynaud's
syndrome, Esophageal dysfunction, Sclerodactyly, and Telangiectasia) variant of
progressive systemic sclerosis (scleroderma).
Rheumatoid factor is actually an autoantibody directed against the Fc portion of the
IgG molecule (choice B). It is found in more than two-thirds of patients with
rheumatoid arthritis.
The majority of patients with SLE (around 95%) develop antinuclear antibodies
(ANA; choice C), so this test is quite sensitive, but not very specific for SLE. ANA
occur in other inflammatory disorders, autoimmune diseases, viral diseases, and in
a number of normal individuals. Antibodies to double-stranded DNA are more
specific for SLE, but are not included as an answer choice.
Anti-SS-A antigen (choice E) refers to antibodies to certain ribonucleoproteins, which
are fairly specific for Sjgren's syndrome.

________________________________________

8>The correct answer is C. There are two systems currently used for classifying
nerve fibers. The first system groups both sensory and motor fibers together,
describing A-a, A-b, A-g, A-d, B, and C fibers. Another system relates only to sensory
fibers, describing Ia, Ib, II, III, IV categories. Both classification schemes begin with
large, myelinated fibers, progressing to finer, unmyelinated fibers.
The C fiber (or IV fibers) is the only type of fiber that is unmyelinated. Remember
that preganglionic neurons are myelinated, but postganglionic neurons are
unmyelinated. Neurons that carry slow pain and temperature information are also
classified as C fibers. See the table below for more information.
Sensory and Motor Fibers Sensory Fibers
A-a
Ia (choice D)
spindles

Function

Alpha motor neurons, primary afferents of muscle

A-a

Ib (choice E) Golgi tendon organ afferents, touch and pressure

A-b

II

A-g

Secondary afferents of muscle spindles, touch and pressure


Gamma motor neurons

A-d (choice A)

III

Touch, pressure, pain and temperature (fast)

B (choice B)

Preganglionic autonomic, visceral afferents

C (choice C) IV

Postganglionic autonomic, pain and temperature (slow)

________________________________________

9>The correct answer is A. In a syndrome called cleidocranial dysostosis, absence


of part of the clavicles accompanies a broad skull, and facial and dental anomalies.
Note that you could also have answered this question by noting that of the bones
listed, only the clavicles form by intramembranous ossification.
The femurs (choice B), metatarsals (choice C), phalanges (choice D), and tibias
(choice E) are cartilaginous (formed by endochondral ossification) rather than
membranous bones.

________________________________________

10>The correct answer is A. Patients with nonallergic bronchospastic conditions,


such as emphysema and chronic bronchitis, are generally not prescribed betareceptor blocking agents since these agents can cause bronchoconstriction by
blocking beta-2 receptors. However, relatively low doses of selective beta-1
receptor antagonists, such as metoprolol and atenolol, are relatively well tolerated
in patients with emphysema. As a precautionary measure, emphysema patients
receiving beta-1-selective blocking agents should use a bronchodilator with beta-2stimulating activity. All the other answer choices are nonselective beta receptor
blocking agents and should not be used in the treatment of hypertension in patients
with nonallergic bronchospasm since they are likely to exacerbate the signs and
symptoms of the condition.

________________________________________

11>The correct answer is B. There is more than one way to think about this
question. One way is to determine which of each of the descriptions corresponds to
Pc, Pi, pc, and pi and then to use the Starling equation for net filtration pressure: (Pc
- Pi) - (pc - pi). Perhaps faster and more intuitive is to just envision that the filtration
pressure will be the difference between the forces pushing fluid out and the forces
pulling fluid back into the glomerulus. The pushing forces are the hydrostatic
pressure of the glomerulus (44 mm Hg) and the osmotic pressure of the tubular fluid

(0). So the total pressure forcing fluid from the glomerulus into the tubular fluid is
44 mm Hg. The forces pulling the fluid back are the hydrostatic pressure of the
Bowman's capsule (9 mm Hg) and the osmotic pressure of the plasma (28 mm Hg).
So the total pressure pushing the fluid back into the glomerulus is 9 + 28 = 37 mm
Hg. The difference between the forces favoring filtration and those opposing it are
therefore 44-37 = 7 mm Hg.

________________________________________

12>The correct answer is B. Ingestion of glucose results in secretion of a "gut


factor" into the blood that subsequently increases insulin secretion by b cells. The
most likely candidate for this action is the intestinal peptide known as gastric
inhibitory peptide (GIP), which obviously was named for its effects on the stomach.
GIP secretion is increased during ingestion of glucose and the blood level produced
is sufficient to stimulate insulin secretion. Because of this effect on insulin secretion,
GIP is sometimes referred to as glucose-dependent insulinotropic peptide.
Activation of the sympathetic innervation to the pancreas inhibits insulin secretion
via an a2-adrenergic mechanism. Hence, any sympathetic reflexes activated during
ingestion of glucose would decrease (not increase, choice A) insulin secretion.
While paracrine release of somatostatin (choice C) by D cells in the islets does
inhibit insulin secretion by b cells, there is no reason to suspect that intravenous
versus ingested glucose would have a differential effect on somatostatin release.
The same holds true for glucagon secretion by a cells. Furthermore, glucagon has a
paracrine effect to increase (not decrease, choice D) insulin secretion.

________________________________________

13>The correct answer is C. The collecting system, including the collecting ducts,
minor and major calyces, and the renal pelvis are all derived from the ureteric bud.
All of the other structures listed are derived from metanephric mesoderm.

________________________________________

14>The correct answer is E. All the compounds listed can produce crystals in joint
fluid, but only monosodium urate (associated with gout) and calcium pyrophosphate
dihydrate (associated with CPPD crystal deposition disease, also called pseudogout),
and to lesser degree basic calcium phosphate (apatite-associated arthropathy),
have a high likelihood of being encountered on a step 1 USMLE exam. The crystals
described are those of monosodium urate. Be careful not to answer "uric acid" if

that is listed as an alternative choice on an exam, since the sodium salt is the
predominant species in vivo.
Basic calcium phosphate (choice A) is seen in apatite-associated arthropathy and
produces spherical clumps of nonbirefringent submicroscopic crystals.
Calcium oxalate crystals (choice B) are seen in primary oxalosis and are
bipyramidal, positively birefringent crystals.
Calcium pyrophosphate dihydrate crystals (choice C) are a feature of pseudogout
and are rod-to-rhomboidal-shaped, weakly positively birefringent crystals.
Cholesterol crystals (choice D) are seen in chronic and chylous effusions in
inflammatory and degenerative arthritis, where they form large, flat, rhomboidal
plates with notched corners.

________________________________________

15>The correct answer is A. The posterior cord supplies the axillary and radial
nerves. Of the muscles listed, only the deltoid is supplied by one of these two
nerves, specifically the axillary nerve.
The flexor carpi ulnaris (choice B) is supplied by the ulnar nerve.
The flexor digitorum superficialis (choice C), the flexor pollicis brevis (choice D), and
the palmaris longus (choice E) are supplied by the median nerve.

________________________________________

16>The correct answer is D. The patient has nephrotic syndrome, as evidenced by


the presence of edema and massive proteinuria. The most common cause of
nephrotic syndrome in adults is membranous glomerulonephritis, which is
characterized by diffuse capillary thickening with a granular pattern on
immunofluorescence.
Berger's disease (choice A) is characterized by IgA deposition in the glomerular
mesangium. It does not usually cause nephrotic syndrome.
Diabetic nephropathy (choice B) can cause nephrotic syndrome, but there would
likely be glucose in the urine.
Membranoproliferative glomerulonephritis (choice C) is an uncommon cause of
nephrotic syndrome.
Minimal change disease (choice E) is the most important cause of nephrotic
syndrome in children.

________________________________________

17>The correct answer is C. The fallopian tube is the only structure in the female
genital tract with a ciliated columnar epithelium; the beating of the cilia helps move
the egg into the uterus. This fact is also sometimes clinically helpful since dilated
and deformed fallopian tubes can be microscopically distinguished from cystic
ovarian tumors by the presence of the cilia.
The cervix (choice A) and vagina (choice E) are lined by squamous epithelium.
The endometrium (choice B) is lined by columnar epithelium (although a few ciliated
cells may be present).
The covering of the ovary (choice D) is cuboidal epithelium, and cysts within the
ovary can be lined by cuboidal or non-ciliated columnar epithelium.

________________________________________

18>The correct answer is B. The cell pictured is an eosinophil, a member of the


granulocytic lineage of white blood cells. The crystalline core of the granule
contains a protein called the major basic protein, which appears to function in the
destruction of parasites. Major basic protein also has deleterious effects on
epithelial cells in patients with asthmatic reactions. The light component around the
dense crystalline core contains products such as histaminase, arylsulfatase, and
other enzymes.
Lactoferrin (choice A) is found in the specific granules of the neutrophil. It inhibits
the growth of bacteria by interfering with iron metabolism.
Myeloperoxidase (choice C) is found in the azurophilic (large) granule of the
neutrophil. This enzyme is also destructive to bacteria, destroying their cell walls.
Histamine (choice D) is produced by the basophil and the mast cell. The
histaminase of the eosinophil regulates the inflammatory reaction of these two cell
types.
Tartrate-resistant acid phosphatase (choice E) is a marker for hairy cell leukemia, a
neoplasm of the B lymphocyte line.

________________________________________

19>The correct answer is E. Take rashes involving the palms and soles (otherwise
unusual sites) very seriously: only a small number of infections can cause this
pattern, including Rocky Mountain spotted fever, meningococcemia, and secondary
syphilis. Rocky Mountain spotted fever is caused by the rickettsia R. rickettsii, and is
found throughout the United States, particularly in south central and eastern
portions (not the Rocky Mountains). 3-12 days after a tick bite, patients develop
malaise, frontal headache, and fever. Several days later, the rash described in the
question stem develops. Other manifestations can include hepatosplenomegaly,
thrombocytopenia, and (potentially fatal) disseminated intravascular coagulation.
Chickenpox (choice A), or varicella, is characterized by maculopapules that evolve
into vesicles over hours to days, then eventually form crusts. Typical lesions first
appear on the trunk and face and rapidly spread to involve other areas.
The maculopapular rash of German measles (choice B), or rubella, usually begins on
the face, then spreads down the body.
Although the maculopapular rash of measles (choice C), or rubeola, can include the
palms and soles, it typically begins along the hairline in frontal and temporal
regions, then spreads down the trunk to the limbs.
Mumps (choice D) is characterized by fever, malaise, parotitis, and orchitis, but not
a rash.

________________________________________

20>The correct answer is C. The vein described is the greater (long) saphenous
vein, which starts on the dorsal surface of the foot, wraps proximally to follow the
medial aspect of the leg below and past the knee, and then dives into the deep
fascia through the fossa ovalis (also called the saphenous opening) to reach the
femoral vein, which becomes the external iliac vein shortly after it is joined by the
greater saphenous vein.
The anatomic snuff box (choice A) is on the wrist. Some authors also refer to an
"anatomic snuff box of the foot," through which the saphenous vein does pass, but
this is early in its course, and not just before it joins the femoral vein.
The antecubital fossa (choice B) is at the elbow.
The inguinal canal (choice D) contains structures going to and from the testes.
The saphenous vein passes near, but not through, the popliteal fossa (choice E) to
cross the knee.
21>The correct answer is E. Cases of severe idiosyncratic hepatocellular injury have
been reported following the administration of troglitazone. The hepatic injury is
usually reversible, but rare cases of hepatic failure leading to death or liver
transplantation have been reported. The initial signs and symptoms of hepatic

dysfunction include recent onset of nausea, vomiting, abdominal pain, anorexia, and
a dark colored urine. Once these signs and symptoms begin to appear, the patient
taking troglitazone should have liver function tests performed. An AST and ALT of
greater than 3 times the normal limits and/or the appearance of jaundice are
typical.
Acarbose (choice A) is an alpha-glycosidase inhibitor used as an adjunctive
treatment measure for NIDDM; the most common side effects are abdominal
discomfort and flatulence.
Glipizide (choice B) is a sulfonylurea oral hypoglycemic agent associated with the
development of hypoglycemia and cholestatic jaundice (a rare complication).
Metformin (choice C) is a biguanide oral hypoglycemic agent associated with the
development of lactic acidosis and malabsorption of amino acids.
Repaglinide (choice D) is the non-sulfonylurea moiety of glyburide; it is commonly
associated with hypoglycemia, nausea and vomiting.
________________________________________

22>The correct answer is A. Classical conditioning involves the response toward one
stimulus being transferred to another stimulus. For example, a patient who fears
going to the doctor experiences heightened anxiety as the physician enters the
room wearing a white coat. The patient's fear then becomes associated with the
white coat itself, such that future exposure to this symbol evokes similar
apprehension in the patient.
Extinction (choice B) means that when a behavior is no longer reinforced, it will
disappear.
Habit hierarchies (choice C) are ordered statements about the probability of
occurrence of behaviors. Those behaviors that have been reinforced more strongly
will be more likely to occur and will therefore be ranked higher in the response
hierarchy.
Negative reinforcement (choice D) occurs when, in response to a behavior, an
aversive condition is removed rather than a positive reward being given. For
example, a teenager may finally take out the garbage in order to stop his mother
from nagging him. This is a method involved in operant conditioning (choice E),
which is based on the relationship between a response and the consequences
(reinforcement) that follow that response.
________________________________________

23>The correct answer is C. The pretracheal layer of the cervical fascia runs from
the investing layers in both sides of the lateral neck and splits to enclose the thyroid
gland. Superiorly, it attaches to the laryngeal cartilages; inferiorly, it fuses with the

pericardium. As a result of these connections, the thyroid gland moves with


laryngeal movements.
The carotid sheath (choice A) contains the vagus nerve, internal jugular vein,
carotid artery, and lymph nodes.
The investing layer of the deep cervical fascia (choice B) splits to enclose the
trapezius and sternocleidomastoid muscles.
The prevertebral fascia (choice D) covers muscles arising from the vertebrae.
The superficial fascia (choice E) is immediately deep to the platysma muscle
________________________________________

25>The correct answer is B. Streptococcus bovis is a Group D streptococcus. There


is a significant association between S. bovis bacteremia and endocarditis with
carcinoma of the colon and other colonic diseases. Every patient with S. bovis
bacteremia should undergo gastrointestinal and cardiac evaluation. Up to 50% of
patients with S. bovis bacteremia are reported to have underlying colonic
malignancies. In another study, 25-50% of cases of S. bovis bacteremia were
associated with endocarditis, especially in patients with preexisting valvular lesions.
Streptococcus agalactiae (choice A) is an important cause of maternal and neonatal
bacteremia and neonatal meningitis. It is part of the normal flora of the
gastrointestinal tract and the female genital tract.
Streptococcus pneumoniae (choice C) is a leading cause of community-acquired
pneumonia, meningitis in adults, otitis media (especially in children), and sinusitis.
Spontaneous peritonitis due to S. pneumoniae is reported in children with ascites
from nephrotic syndrome. Asplenia predisposes patients to severe infections with S.
pneumoniae and other encapsulated organisms. S. pneumoniae infections are also
more frequent and unusually severe in patients with sickle cell anemia, multiple
myeloma, alcoholism, and hypogammaglobulinemia. S. pneumoniae is now the
leading cause of invasive bacterial respiratory disease in patients with AIDS.
Streptococcus pyogenes (choice D) is the most common cause of bacterial
pharyngitis. Complications include paratonsillar abscesses, otitis media, and
sinusitis. Long-term sequelae include rheumatic fever and poststreptococcal
glomerulonephritis. S. pyogenes is also responsible for many skin and soft tissue
infections. The organism also produces many toxins that produce a variety of
diseases.
Streptococcus viridans (choice E), or the viridans Streptococci, are the most
common cause of subacute bacterial endocarditis, which should be suspected in
cases of viridans streptococcal bacteremia. One species of viridans Streptococci,
Streptococcus milleri, is frequently associated with pyogenic abscesses, especially
of the liver.
________________________________________

26>The correct answer is E. Rotavirus is the major cause of diarrhea in infants and
children under the age of 2. The replicates in the intestinal mucosa, producing a
profuse, watery, non-bloody diarrhea, often coupled with nausea and vomiting.
Transmission is by the fecal-oral route.
Coronaviruses (choice A) usually cause cold-like illnesses.
Lymphocytic choriomeningitis virus (choice B) can cause headache, malaise,
myalgia, conjunctivitis, and, occasionally, meningitis.
Norwalk agent (choice C) can also cause diarrhea, but usually affects patients older
than 2 years.
Orbivirus (choice D) is the cause of Colorado tick fever, which is the only tick-borne
viral disease in the United States.
________________________________________

27>The correct answer is A. While most of the liver is covered with peritoneum,
there are several reflections of the peritoneum (commonly called "ligaments") on
the posteroinferior surface of the liver. When the opposing layers of the reflections
are separated, a "bare area" on the liver is produced. The largest bare area contains
on one end the inferior vena cava and is bounded by a continuous irregular ring of
ligament that is somewhat arbitrarily divided into sections called the anterior and
posterior layers of the coronary ligaments. It is the anterior layer of the coronary
ligament that would be felt in the maneuver described in the question stem.
A fissure for the ligamentum venosum (choice B) is to the left of the coronary
ligaments on the posteroinferior surface of the liver.
The left triangular ligament (choice C) is an outpouching on the left side of the
falciform ligament.
The posterior layer of the coronary ligament (choice D) is inferior to the anterior
layer.
The right triangular ligament (choice E) is the name given for the area where the
anterior and posterior reflections of the coronary ligament merge on the inferior
part of the posteroinferior surface of the liver.

________________________________________

28>The correct answer is B. The most probable etiology of bacterial endocarditis


involving the tricuspid valve is illicit intravenous drug use, which can introduce skin
organisms into the venous system that then attack the tricuspid valve.

Staphylococcus aureus accounts for between 60% and 90% of cases of endocarditis
in intravenous drug users.
The endocarditis associated with congenital heart disease (choice A) typically
involves either damaged valves or atrial or ventricular septal defects. The tricuspid
valve is not particularly vulnerable.
Rheumatic fever (choice C) most commonly damages the mitral and aortic valves,
and tricuspid damage is usually less severe and seen only when the mitral and
aortic valves are heavily involved. Consequently, secondary bacterial endocarditis
involving only the tricuspid valve in a patient with a history of rheumatic fever
would be unusual.
Rheumatoid arthritis (choice D) is not associated with bacterial endocarditis.
Systemic lupus erythematosus (choice E) can produce small, aseptic vegetations on
valves, but is not associated with bacterial endocarditis.
________________________________________

29>The correct answer is A. Most of the venous drainage of the skin of the lower
extremity is to the long saphenous vein, the accompanying lymphatics of which
drain into the superficial group of the inguinal lymph nodes. However, the skin
drained by the short saphenous vein, including that of the lateral aspect of the
dorsum of the foot, is an exception to this rule. Lymphatic fluid from this area drains
into lymphatics accompanying the short saphenous vein, then drains into lymph
nodes behind the knee in the popliteal fossa.
The lateral side of the thigh (choice B), the medial side of the leg below the knee
(choice C), the medial side of the sole of the foot (choice D), and the medial side of
the thigh (choice E) all drain to the superficial inguinal nodes.
________________________________________

30>The correct answer is E. Testicular feminization is a disorder of the androgen


receptor. Phenotypically, the patient appears female, but has a blindly ending
vagina and lacks a uterus or other female internal reproductive organs. The patient
has an XY genotype. Since the gene for testes determining factor (TDF) is on the Y
chromosome, TDF will cause the indifferent gonad to develop into a testis
containing Sertoli cells. Sertoli cells at this stage will secrete MIF, a substance that
suppresses the paramesonephric ducts, preventing the formation of female internal
reproductive organs.
This patient would not have a streak ovary (choice A), a finding in Turner's
syndrome that is associated with a 45,XO genotype. In fact, the patient would have
testes, since the genetic complement contained a Y chromosome. The testes in
individuals with testicular feminization syndrome are often undescended and are
usually removed surgically.

This patient would not possess a uterus (choice B) or an oviduct (choice C). Both of
these structures are derived from the paramesonephric duct, which is suppressed
by MIF.
Depressed levels of testosterone (choice D) would not occur in this patient. In fact,
individuals with testicular feminization syndrome have normal or even slightly
elevated levels of testosterone. The development of female external genitalia is the
result of defective androgen receptors, not depressed levels of testosterone.
________________________________________

31>The correct answer is E. Weight loss of more than 5% of body weight is


considered a very adverse prognostic feature in cancer since it usually indicates the
presence of widespread disease. (Uncommonly, a relatively small primary lesion
that has not yet metastasized can cause cachexia.) Both tumor necrosis factor (TNF)
and interleukin 1-beta have been implicated in the production of cachexia with
weight loss, loss of appetite, and alteration in taste. Large tumor burdens may
additionally alter protein and energy balance, often with negative nitrogen balance.
Therapy, in whatever form (surgery, radiation, chemotherapy), may also contribute
to cachexia late in the course secondary to effects on the digestive system.
Clathrin (choice A) is a protein that helps to form pinocytotic vesicles.
Histamine (choice B) is released by mast cells and basophils and contributes to
allergic responses.
Interferon (choice C) is important in the body's response to viral infection.
Interleukin 1-beta, not 2 (choice D), is produced by activated monocytes and
macrophages and has been implicated in cachexia. Interleukin 2 is released by
helper T cells and augments B-cell growth as well as antibody production.
________________________________________

32>The correct answer is E. The condition described is spinal bifida with


myelomeningocele. A failure of the posteriorly located vertebral arches to fuse
posteriorly causes spina bifida, which can vary in severity from a completely
asymptomatic minor abnormality to protrusion of the spinal cord and roots through
an open skin defect, with a very real risk of minor trauma or infection causing
paralysis.
Failure of development of one of the paired primary ossification centers (choice A)
of the body can produce very severe scoliosis.
The bodies of the vertebrae (choice B) are the stacking ovoid structures on the
anterior aspect of the spinal canal.
The pedicles (choice C) attach the body ring that surrounds the spinal canal to the
body of the vertebrae.

The superior articular process (choice D) articulates with the inferior articular facet
of the vertebra above it.
________________________________________

33>The correct answer is E. Both the CSF glucose level and the small number of
lymphocytes present are within normal limits. It is normal for the CSF glucose to be
less than serum glucose, often about 2/3 of the serum value.
Bacterial meningitis (choices A and C) can profoundly lower CSF glucose levels. CSF
from bacterial meningitis cases also usually shows large numbers of neutrophils.
Viral meningitis (choices B and D) will not alter the CSF glucose level, but the CSF
often contains many lymphocytes.
________________________________________

34>The correct answer is E. Hairy cell leukemia is characterized by pancytopenia


and splenomegaly. Lymphadenopathy is unusual. The proliferating cells express the
pan B-cell markers (CD19 and CD20), the monocyte marker (CD11c), the
interleukin-2 receptor (CD25), and plasma cell-associated antigen-1 (PCA-1).
Histologically, the cells display fine, hairlike projections when viewed with phasecontrast microscopy. A "fried-egg" appearance on stained bone marrow biopsy
arises because of fixation artifacts that occur as a result of the hairlike projections
on many of the cells. Tartrate-resistant acid phosphatase is virtually diagnostic of
hairy cell leukemia.
CD4 (choice A) is a cluster designation antigen that is displayed on the surfaces of T
cells and maturing thymocytes.
GpIIb/IIIa (choice B) is a platelet membrane receptor for fibrinogen, fibronectin, and
von Willebrand's factor.
Myeloperoxidase (choice C) is a marker for late myeloblasts and mature
granulocytes. Monocytes also stain to some extent.
Diffuse nonspecific esterase (choice D) staining is used to identify cells in the
monocytic lineage. Punctate nonspecific esterase staining is observed in T cells.
________________________________________

35>The correct answer is E. The disease is necrotizing enterocolitis, which is a


common cause of gastrointestinal emergency in premature and low-birth-weight
infants. Typically, the infants are bottle-fed and develop severe abdominal distress
in the first week of life. Contributing factors include intestinal ischemia, poor
neonatal immune response, and microbial agents. Both the small and large bowel
may be affected. Necrotizing enterocolitis may be complicated by intestinal

gangrene, gastrointestinal bleeding, intestinal perforation, and sepsis. Survivors of


severe necrotizing enterocolitis may have had significant lengths of bowel surgically
removed, and later suffer from malabsorption and stricture formation. Histologically,
the appearance varies with disease stage, but typically shows varying degrees of
necrosis (transmural if perforation has occurred), inflammation, hemorrhage, and
edema. A pseudomembrane composed of coagulated fibrin, neutrophils, and cellular
debris may overlie the mucosa.
Inflammatory polyps (choice A) can be seen following reepithelialization of ulcers,
typically in ulcerative colitis, which would not be seen in the first week of life.
Multiple diverticula (choice B) are seen in diverticulosis, which does not usually
develop before middle age.
Neoplastic (precancerous) polyps (choice C), even in familial syndromes with a high
colonic cancer rate, would not be expected to cause an acute abdominal emergency
in a neonate.
A thickened collagenous band (choice D) between surface epithelial cells and the
lamina propria is seen with episodic watery diarrhea, usually in adults.
________________________________________

36>The correct answer is E. The child is likely suffering from cystic fibrosis. In this
disorder, an abnormality of chloride channels causes all exocrine secretions to be
much thicker, and more viscous than normal. Pancreatic secretion of digestive
enzymes is often severely impaired, with consequent steatorrhea and deficiency of
fat-soluble vitamins, including vitamin A.
Cystinuria (choice A) is a relatively common disorder in which a defective
transporter for dibasic amino acids (cystine, ornithine, lysine, arginine; COLA) leads
to saturation of the urine with cystine, which is not very soluble in urine, and
precipitates out to form stones.
Hypoglycemia (choice B) is not a prominent feature of children with cystic fibrosis
who are on a normal diet. Hyperglycemia may occur late in the course of the
disease.
Iron deficiency anemia (choice C) is not found with any regularity in children with
cystic fibrosis.
Sphingomyelin accumulation (choice D) is generally associated with deficiency of
sphingomyelinase, as seen in Niemann-Pick disease.
________________________________________

37>The correct answer is D. The purpose of the time out is to remove the girl from
stimuli to facilitate the extinction of unwanted behavior. To be effective, time out
must be used consistently and predictably. If the child protests during a time out, as

in this case, additional time is added to the period to extinguish the protest
behaviors. The goal is to convey the clear message that the time out will be ended
only when unacceptable behaviors are ended.
To impact behavior, the intervention must be closely associated with the behavior.
To delay in responding to the girl's protests (choice A) makes it harder for her to
realize that the additional time is a direct consequence of her behavior.
At age 3, reasoned, rational explanations (choice B) are unlikely to have any impact
on the child's behavior. The child probably lacks the cognitive capacity to grasp the
abstract rationale for why she is being disciplined.
Paddling (choice C) is a type of attention, and can actually reinforce the behavior
the parents are trying to extinguish. The child learns that protestation will bring the
parents and their attention, even if it is negative attention. In addition, as a general
rule, any response option on the USMLE exam that has someone hitting a child will
almost certainly be scored as a wrong answer!
This option encourages the child to cry and then demand a treat to become quiet
(choice E), a kind of juvenile blackmail. Avoiding bad behavior is a baseline and
need not be specially rewarded.
________________________________________

38>The correct answer is C. A pregnant patient is considered to be in preeclampsia


if she develops hypertension, proteinuria, and edema. Eclampsia includes the
addition of seizures to the triad. Approximately 7 percent of pregnant women
develop preeclampsia, typically between 20 weeks gestation to 6 weeks
postpartum. Predisposing conditions include pre-existing hypertension, diabetes
(choice A), and autoimmune diseases such as lupus (choice D). Laboratory features
can include hyperuricemia (choice B) and thrombocytopenia (choice E), but these
are not used to define the presence of eclampsia.
________________________________________

39>The correct answer is A. The patient is presenting with signs and symptoms of
acute cholecystitis, which is associated with gallstones in more than 90% of all
cases. This condition occurs when a stone becomes impacted in the cystic duct and
inflammation develops behind the obstruction. The acute attack is often
precipitated by a large fatty meal, and is characterized by the sudden appearance
of severe, steady pain localized to the epigastrium or right hypochondrium.
Laboratory findings often include elevated white blood cells (2,000 - 15,000/mL).
Total serum bilirubin values of 1-4 mg/dL may be seen in some instances, and
serum amylase may be elevated. In noncomplicated cases, treatment often includes
IV alimentation, analgesics, and antibiotics, as well as withholding of oral feedings.
Meperidine is the narcotic of choice since it is least likely to cause spasm of the
sphincter of Oddi, probably because of its antimuscarinic properties. It is therefore

preferred over morphine (choice B), oxycodone (choice D), and propoxyphene
(choice E). Furthermore, propoxyphene is a narcotic agonist with mild analgesic
properties; hence, it would most likely not be able to effectively treat this patient's
severe pain.
Naproxen (choice C) is a nonsteroidal anti-inflammatory drug (NSAID) indicated for
the treatment of mild to moderate pain; this agent would most likely not provide
sufficient pain control for this patient.
________________________________________

40>The correct answer is D. Many people who request physician-assisted suicide


have one of two conditions present: either a poorly controlled painful condition or
severe depression. If the painful condition is adequately treated or the depression is
brought under good medical control, the request for physician assistance in
terminating the situation is typically withdrawn. It is important to note that bringing
these conditions under control requires the intervention of caregivers who are
specifically trained in the management of these two conditions; primary care
physicians usually are not adequately trained to address these difficult
presentations.
While patients who are diagnosed as bipolar disorder (choice A), borderline
personality disorder (choice B), and schizophrenic disorder (choice E) often make
suicide attempts (and frequently complete those attempts), they do not generally
ask their physician for assistance in the suicide.
Persons with factitious disorder (choice C) are seeking primary gain, often for
dependency needs, and are seeking to enter the "sick role" not the "dead role."
________________________________________

41>The correct answer is E. The disease is myotonic dystrophy, which is an


autosomal dominant disease; the affected gene has been localized to chromosome
19. Myotonic dystrophy is relatively common and is best thought of as a systemic
disease, since it causes cataracts, testicular atrophy, heart disease, dementia, and
baldness in addition to muscular weakness.
A mutation on the X chromosome (choice A) causes Duchenne muscular dystrophy.
None of the muscle diseases are known to be related to defects on the Y
chromosome (choice B).
Facioscapulohumeral dystrophy is associated with a defective gene on chromosome
4 (choice C).
Infantile hypotonia has been related to defective genes on chromosome 5 (choice
D).
________________________________________

42>The correct answer is D. This patient has a family history of diabetes. Steroidinduced diabetes mellitus is a frequent consequence of long-term corticosteroid
therapy. It can be triggered by prednisone with or without a family history, but a
predisposition may increase the risk. Symptoms such as polyuria and weight loss
may be masked by the disease for which the patient is taking steroids.
Measurement of 1,25-dihydroxy vitamin D (choice A), the active vitamin D
metabolite, would not be recommended. Corticosteroids can alter calcium balance
mainly to due vitamin D deficiency secondary to impaired intestinal absorption of
calcium, but 25-hydroxy vitamin D is a better marker for assessing nutrition.
PTH, an 84 amino acid polypeptide, can be cleaved into an active N-terminal
fragment and an inactive C-terminal fragment. Measurement of C-terminal PTH
(choice B) is not recommended. Although PTH is a regulator of calcium homeostasis,
these fragment molecule measurements have not been found to correlate well with
true PTH activity on bone.
Although PTH is a calcium regulator in the body, it is not considered a major
contributor to corticosteroid-induced bone loss, so measurement of intact
parathyroid hormone (choice C) is not the best choice. It is only significant if a
person has underlying malabsorption such as inflammatory bowel disease.
Serum protein electrophoresis (choice E) is used mainly for the diagnosis of multiple
myeloma in patients with pathologic fractures or a high clinical suspicion of
myeloma.
________________________________________

43>The correct answer is E. Von Gierke's disease is a glycogen storage disease


caused by a deficiency of glucose-6-phosphatase. It typically presents with neonatal
hypoglycemia, hyperlipidemia, lactic acidosis, and ketosis. Failure to thrive is
common in early life; convulsions may occur due to profound hypoglycemia. The
glycogen accumulation in von Gierke's disease occurs primarily in the liver and
kidneys, accounting for the enlargement of these organs. Gout may develop later
because of the derangement of glucose metabolism.
Even if you don't remember all of the details of the presentation of these genetic
diseases, you should be able to narrow the choices:
Gaucher's disease (choice A) and Niemann-Pick disease (choice C) are lipid storage
diseases, and would not be expected to produce hypoglycemia.
The other diseases are glycogen storage diseases, but McArdle's (choice B) and
Pompe's (choice D) disease affect muscle rather than liver, and would not be
expected to produce profound hypoglycemia since the liver is the major source for
blood glucose.
________________________________________

44>The correct answer is A. The graph is that of a log dose response curve of
norepinephrine on alpha-1 receptors of vascular smooth muscle. Norepinephrine
alone (control curve) is expected to show an increased response as the dose is
increased. A shift to the left indicates potentiation (less norepinephrine is needed to
give the same size response), a shift to the right indicates antagonism (more
norepinephrine is needed to see a given response). Curve X shows potentiation,
curve Y shows competitive antagonism and curve Z shows non-competitive
blockade of the alpha-1 receptors found on vascular smooth muscle. The answer
must therefore include an indirect agonist, a competitive blocker, and a noncompetitive blocker of alpha-1 receptors.
Cocaine is an indirect agonist that acts by blocking monoamine reuptake, thereby
allowing norepinephrine to stay longer and at higher concentration in its synapse,
potentiating its action (curve X). (It also blocks the reuptake of dopamine and
serotonin, potentiating them in the same way.) Prazosin is a competitive alpha-1
receptor blocker used in treatment of hypertension because of its vasodilatory
effect, and would produce a right-shift of the dose-response curve (curve Y).
Phenoxybenzamine is the only non-competitive alpha-1 blocker used therapeutically
in cases of malignant hypertension and pheochromocytomas. The excessive
vasodilation produced by this agent is the result of irreversible binding to the
receptor, thereby decreasing the efficacy (decreased curve height) of
norepinephrine (curve Z).
Fluoxetine (choices B and C) is a serotonin specific reuptake blocker that would
potentiate the action of serotonin, but not norepinephrine. It is also a weak alpha-1
blocker, which explains its side-effect of orthostatic hypotension.
Propranolol (choices B and E) is a non-selective beta-blocker that would not directly
effect the norepinephrine response at alpha-1 receptors. Although norepinephrine is
also a beta-1 receptor agonist, beta-1 receptors are not present on blood vessels.
Phentolamine (choice D) is a non-selective competitive alpha-blocker that has been
largely supplanted by more selective alpha-1 blockers.
________________________________________

45>The correct answer is B. A typical menstrual cycle lasts around 26-30 days. The
luteal phase (post-ovulation) generally lasts fourteen days; the length of the
follicular phase (pre-ovulation) is far more variable, and accounts for most of the
variability observed in the length of the menstrual cycle. Just before menstruation,
sex steroid levels are low, but gonadotropin levels (especially FSH) begin rising
slightly. Basal body temperature remains high during the luteal phase of the
menstrual cycle, but falls precipitously a few days before the onset of menstruation.
Markedly increased inhibin levels (choice A) are seen in the middle of the luteal
phase, dropping to low levels just before menstruation.

LH levels peak approximately 36 hours before ovulation (choice C), then decrease
rapidly (choice D) within a few days to a low level during the mid-luteal phase,
gradually decreasing until menstruation.
The basal body temperature significantly increases (choice E) shortly after
ovulation, due to the metabolic effects of progesterone produced by the corpus
luteum.
________________________________________

46>The correct answer is D. Rib fractures are very painful (it hurts to breathe
deeply) and can compromise the normal chest morphology (the chest can't change
shape normally), both of which can impair ventilation at the peripheral level.
Ventilation is the term used for the mechanical part of breathing: the air goes in and
the air goes out. Other causes of hypoventilation of peripheral origin include phrenic
nerve paralysis, suffocation, submersion, poliomyelitis, tetanus, and the Pickwickian
syndrome.
Decreased diffusion capacity (choice A) can occur when the blood-gas barrier is
thickened (e.g., diffuse interstitial fibrosis, sarcoidosis, asbestosis, hyaline
membrane disease), when the surface area of the blood-gas barrier is reduced (e.g.,
pneumonectomy, emphysema), or when less hemoglobin is available to pick up the
oxygen (e.g., anemia, pulmonary embolism).
Decreased surface area of alveolar capillary membranes (choice B) occurs after lung
resection or in disease (e.g., emphysema).
Hypoventilation of central origin (choice C) involves the central respiratory center of
the brain, and is most commonly seen in morphine or barbiturate overdose.
Inequalities of ventilation and perfusion (choice E) are common in chronic
obstructive lung disease (both emphysema and chronic bronchitis), asthma,
bronchiectasis, pneumonia, and granulomatous diseases.
________________________________________

47>The correct answer A. Release of cholecystokinin is stimulated by the presence


of peptides, amino acids, or fatty acids in the small intestine. Cholecystokinin acts
on the pancreas to stimulate secretion of pancreatic enzymes that aid in the
digestion of these compounds.
Gastrin (choice B) secretion is stimulated by the presence of peptides or amino
acids in the lumen of the stomach, and produces an increase in gastric H+
secretion.
Motilin (choice C) is a hormone that regulates the migrating myoelectric complex, a
series of contractions that occur during fasting, clearing the stomach and small
intestine of any residual food.

Secretin (choice D) secretion is stimulated by the presence of H+ and fatty acids in


the duodenum, and causes an increase in pancreatic and biliary HCO3- release and
a decrease in gastric H+ release.
Somatostatin (choice E) secretion is stimulated by the presence of H+ in the lumen,
and results in decreased release of all gastrointestinal hormones and decreased H+
secretion in the stomach.
________________________________________

48>The correct answer is E. The majority of circulating testosterone is bound to


plasma protein (around 98%), rather than existing in free form (choice C). Of this, a
majority is bound to a specific sex (or gonadal) steroid-binding protein (choice E),
and a minority is bound to albumin (choice D). Dihydrotestosterone is produced
from testosterone in the tissues by a specific enzyme, 5-alpha-reductase, rather
than circulating in bound (choice A) or free (choice B) form.
________________________________________

49>The correct answer is E. The thyroglossal duct develops as an evagination of the


floor of the pharynx in the region where the tongue develops. The adult foramen
cecum of the tongue marks the site of this evagination. The distal end of this duct
normally forms the thyroid gland; the proximal part of the duct normally
degenerates. Failure of a part of the duct to degenerate may lead to a thyroglossal
duct cyst or median cervical cyst, as seen in this patient.
The first pharyngeal cleft (choice A) forms the external ear canal. This cleft normally
remains patent.
The first pharyngeal pouch (choice B) forms the middle ear cavity and the auditory
tube. This pouch normally remains patent.
The second pharyngeal cleft (choice C) normally does not remain patent. It is
typically covered over by the overgrowth of the second pharyngeal arch. If part of
this pouch does remain patent, it may form a lateral cervical cyst, which is seen on
the lateral side of the neck along the posterior border of the sternocleidomastoid
muscle.
The second pharyngeal pouch (choice D) forms the tonsillar fossa of the pharynx.
The pharyngeal mucosa in this area arises from the endoderm of the pouch.
Ingrowth of mesoderm cells results in the formation of the palatine tonsil.
________________________________________

50>The correct answer is A. Fascial straps (retinacula) and fascial coverings of


muscles or muscle groups characteristically attach to nearby bones by blending
with the covering periosteum. No deep attachments are usually made by fascia.

Cancellous bone (choice B) is spongy bone, which is usually found in marrow, and is
not the site for fascial attachment.
Fascia do not usually attach to cartilage (choice C).
Fascia attaches to bony shafts, or diaphyses (choice D), superficially via the
periosteum.
Fascia do not penetrate the bone to reach the marrow (choice E).

________________________________________

1>The correct answer is C. This is a classic presentation of a patient with carpal


tunnel syndrome, which typically affects females between the ages of 40 and 60
who chronically perform repetitive tasks that involve movement of the structures
that pass through the carpal tunnel. One important structure that passes through
the carpal tunnel is the median nerve. Patients often note a tingling, a loss of
sensation, or diminished sensation in the digits. There is also often a loss of
coordination and strength in the thumb, because the median nerve also sends fibers
to the abductor pollicis brevis, flexor pollicis brevis, and the opponens pollicis. A
final function of the median nerve distal to the carpal tunnel is control of the first
and second lumbricals which function to flex digits two and three at the
metacarpophalangeal joints and extend interphalangeal joints of the same digits.
Abduction of the fifth digit (choice A) is a function controlled by the ulnar nerve,
which does not pass through the carpal tunnel.
Adduction of the thumb (choice B) is a function of the adductor pollicis, which is the
only short thumb muscle that is not innervated by the median nerve, but rather by
the deep branch of the ulnar nerve.
Sensation of the lateral half of the dorsum of the hand (choice D) is mediated by the
radial nerve, which also does not pass through the carpal tunnel.
Sensation over the lateral aspect of the palm (choice E) is mediated by the median
nerve, however the branch innervating the palm (palmar cutaneous branch of the
median nerve) passes superficial to the carpal tunnel.
Sensation over the medial aspect of the dorsum of the hand (choice F) is mediated
by the ulnar nerve.
Sensation over the medial aspect of the palm (choice G) is mediated by the ulnar
nerve.
________________________________________

2> The correct answer is D. The smooth part of the right atrium (the sinus venarum)
is derived from the sinus venosus. The coronary sinus and the oblique vein of the
left atrium also derive from the sinus venosus.
The bulbus cordis (choice A) gives rise to the smooth part of the right ventricle
(conus arteriosus) and the smooth part of the left ventricle (aortic vestibule).
The primitive atrium (choice B) gives rise to the trabeculated part of the right and
left atria.
The primitive ventricle (choice C) gives rise to the trabeculated part of the right and
left ventricles.
The truncus arteriosus (choice E) gives rise to the proximal part of the aorta and the
proximal part of the pulmonary artery.

________________________________________

3>The correct answer is A. Chronic lymphocytic leukemia (CLL) is typically a disease


of the elderly, with 90% of cases occurring after the age of 50; the median age is
65. Patients will typically present with a complaint of chronic fatigue and/or
lymphadenopathy. Approximately 50% of all patients with CLL present with an
enlarged liver and/or spleen. CLL typically pursues an indolent course but can
occasionally present as a rapidly progressive disease. The hallmark of CLL is the
isolated lymphocytosis in which the white blood cell count is usually greater than
20,000/mL and between 75% and 98% of the circulating cells are small "mature"
lymphocytes. Chlorambucil is classified as a nitrogen mustard, a subcategory of the
alkylating agents. It is primarily used to treat chronic lymphocytic leukemia and
ovarian carcinoma; it can also be used to treat Hodgkin's disease and various other
lymphomas.
Cisplatin (choice B) is an alkylating agent indicated for the treatment of metastatic
testicular and ovarian tumors in combination with other agents.
Dacarbazine (choice C) is a cytotoxic agent with alkylating properties. It is used as a
single agent or in combination with other antineoplastics in the treatment of
metastatic malignant melanoma, refractory Hodgkin's disease, and various
sarcomas.
Tamoxifen (choice D) is an antiestrogen hormone used in the palliative treatment of
breast cancer in patients with estrogen-receptor-positive breast cancer.
Vinblastine (choice E) is a mitotic inhibitor antineoplastic agent indicated for the
treatment of Hodgkin's disease and non-Hodgkin's lymphomas, choriocarcinoma,
lymphosarcoma, and neuroblastoma, as well as various other types of cancer.

________________________________________

4>The correct answer is E. The mediastinum is divided into four regions. The region
above the manubriosternal junction (level of fourth thoracic vertebra) is the superior
mediastinum. The region below the manubriosternal junction is divided into the
anterior mediastinum (anterior to the pericardium), the middle mediastinum (within
the pericardium), and the posterior mediastinum (posterior to the pericardium). The
thoracic duct enters the thorax through the aortic hiatus of the diaphragm. At this
point it lies in the posterior mediastinum, the region posterior to the pericardium. As
it ascends through the thorax and passes the level of the fourth thoracic vertebra, it
enters the superior mediastinum.
The anterior mediastinum (choices A and B) contains the thymus and fatty
connective tissue.
The middle mediastinum (choices A, C, and D) contains the heart and the roots of
the great vessels.
In addition to the thoracic duct, the superior mediastinum (choices B, D, and E)
contains the ascending aorta, aortic arch, branches of the aortic arch, descending
aorta, superior vena cava, brachiocephalic veins, thymus, trachea and esophagus.
In addition to the thoracic duct, the posterior mediastinum (choices C and E)
contains the descending aorta, azygos vein, hemiazygos vein and the esophagus

________________________________________

5The correct answer is C. The boy probably has Klinefelter's syndrome (47, XXY),
which has the typical presentation described in the question. The condition arises as
a result of failure of separation (nondisjunction) of the sex chromosomes, and can
be related to either paternal nondisjunction (slightly more common) or maternal
nondisjunction.
Deletions (choice A) are a common form of genetic disease and contribute to many
genetic recessive diseases.
Examples of nondisjunction of autosomes (choice B) include trisomies such as most
cases of Down's syndrome (trisomy 21), Edwards' syndrome (trisomy 18), and Patau
syndrome (trisomy 13).
There are two types of translocations: non-Robertsonian (choice D) and
Robertsonian (choice E). Non-Robertsonian (reciprocal) translocations result when
two non-homologous chromosomes exchange genetic material. Robertsonian
translocations are a special type of translocation that involve exchange of genetic
material from the long arms of one acrocentric chromosome to the long arms of

another acrocentric chromosome, with fusion of the centromeres. Four percent of


cases of Down's syndrome are caused by this mechanism.
________________________________________

6>The correct answer is A. The probable location of lesions producing visual defects
is a favorite USMLE topic (and is also well worth knowing if you have occasion to
work up such a patient). Here is a list that may help you sort through these
problems:
Central scotoma ~ macula
Ipsilateral blindness ~ optic nerve
Bitemporal hemianopia ~ optic chiasm (choice B)
Homonymous hemianopia ~ optic tract (choice E)
Upper homonymous quadrantanopia ~ temporal optic radiations (choice D)
Lower homonymous quadrantanopia ~ parietal optic radiations (choice C)
Also, cortical lesions produce defects similar to those of the optic radiations, but
may spare the macula.
________________________________________
7>The correct answer is C. This is deja vu ("seen before"), which is the experience
of an event, person, or thing as familiar, even though it has never previously been
experienced. Severe cases often accompany an underlying neurologic problem.
Anterograde amnesia (choice A) is the inability to learn new facts. Most commonly,
it involves both verbal and nonverbal material, but it can involve one or the other.
In confabulation (choice B), a patient reports "memories" of events that did not take
place at the time in question.
In jamais vu (choice D), a patient fails to recognize familiar events that have been
encountered before.
In retrograde amnesia (choice E), a patient fails to remember facts or events that
occurred before the onset of amnesia.
________________________________________
8>The correct answer is E. Systemic lupus erythematosus (SLE) is a prototype
connective tissue disease. The diagnosis requires four criteria to be met from a list
of eleven possible criteria: malar rash, discoid rash, photosensitivity, oral ulcers,
arthritis, serositis, renal disorder, neurologic disorder, hematologic disorder,
immunologic disorder, and antinuclear antibody. This patient also has anti-Sm,
which is pathognomonic for SLE, but is only found in 30% of the affected patients.

Antinuclear antibodies (ANA) are present in 95-100% of cases of SLE; anti-doublestranded DNA is found in 70% of the cases.
Generalized fatigue (choice A) due to being a single working mother of two children
could well be a possibility, but the presence of the other criteria make SLE more
likely.
Goodpasture's syndrome (choice B) is characterized by linear disposition of
immunoglobulin, and often C3, along the glomerular basement membrane (GBM).
Glomerulonephritis, pulmonary hemorrhage, and occasionally idiopathic pulmonary
hemosiderosis occur.
Mixed connective tissue disease (choice C) is an overlap syndrome characterized by
a combination of clinical features similar to those of SLE, scleroderma, polymyositis,
and rheumatoid arthritis. These patients generally have a positive ANA in virtually
100% of the cases. High titer anti-ribonucleoprotein (RNP) antibodies may be
present, generating a speckled ANA pattern. Anti-RNP is not pathognomonic for
mixed connective tissue disease, since it can be found in low titers in 30% of the
patients with SLE.
Scleroderma (choice D) is characterized by thickening of the skin caused by swelling
and thickening of fibrous tissue, with eventual atrophy of the epidermis. ANA are
often associated with the disease, but the staining pattern is generally nucleolar.
________________________________________
9>The correct answer is D. Specificity refers to how well a test identifies persons
who do not have the disease in question. In the example, 2900 women do not have
breast cancer, and 2540 of these women have a negative test. Specificity is given
by (true negatives)/ (true negatives + false positives) = 2540/(2540+360) =
2540/2900.
Choice A, 90/100, represents the sensitivity of the test; that is, the proportion of
women with the condition who test positive.
Choice B, 90/450, gives the predictive value of a positive test. Of the 450 women
who tested positive, 90 actually had the condition.
Choice C, 2540/2550, gives the predictive value of a negative test. Of the 2550 who
tested negative, 2540 did not have cancer.
Choice E, (90+2540)/3000, refers to the efficiency of a test; that is, the proportion
of all subjects who were correctly classified by the test.
________________________________________
10>The correct answer is A. The disease is Pneumocystis pneumonia, which is
caused by an agent now believed to be a fungus rather than a true bacteria.
Pneumocystis carinii pneumonia is seen in immunocompromised patients,
particularly in those with AIDS, cancer, and in malnourished children. It can be the
AIDS-defining illness.

Congestive heart failure (choice B) predisposes the patient to pulmonary edema.


Pulmonary embolus (choice C) can cause pulmonary infarction or sudden death.
Rheumatoid arthritis (choice D), particularly in miners, can cause formation of lung
nodules similar to subcutaneous rheumatoid nodules.
Systemic lupus erythematosus (choice E) can cause pleuritis, but is not associated
with a significantly increased incidence of pneumonia.
________________________________________
11>The correct answer is C. The disease is Fragile X Syndrome, which is a familial
form of mental retardation that is roughly as common as Down's syndrome. The
phenotype has a variable expression, but can include large head circumference at
birth, perinatal complications (premature birth, asphyxia, seizures), and possibly
increased incidence of sudden infant death syndrome. Later, mental retardation,
particularly involving language, and symptoms suggestive of attention deficit
disorder and/or autism may appear. Features suggestive of connective tissue
disorder (lax skin and joints, flat feet, large ears) are common. After puberty, there
may be a long narrow face with prominent jaw and nasal bridge. Macro-orchidism is
also common after puberty. Mitral valve prolapse and aortic root dilatation, which
may appear in late adolescence or adulthood, are among the most serious
complications of this disorder.
Aortic regurgitation related to aortic root dilatation, not stenosis (choice A), can be a
problem in this population.
Common congenital cardiac malformations such as atrial septal defect (choice B) or
ventricular septal defect (choice E) are not features of Fragile X syndrome, but can
be seen with Down's syndrome.
Tricuspid atresia (choice D) is a serious congenital cardiac malformation limiting flow
into the right ventricle, but it is not part of Fragile X syndrome.
________________________________________
12>The correct answer is E. Because the right main bronchus is wider and more
vertical than the left, foreign objects are more likely to be aspirated into the right
main bronchus. The superior segmental bronchus of the lower lobar bronchus is the
only segmental bronchus that exits from the posterior wall of the lobar bronchi.
Therefore, if a patient is supine at the time of aspiration, the object is most likely to
enter the superior segmental bronchus of the lower lobe.
None of the segmental bronchi of the left lung (choices A and D) are likely to receive
the object because the object is less likely to enter the left main bronchus.
The apical segment of the right upper lobe (choice B) is not likely to receive the
foreign object because of the sharp angle that the upper lobar bronchus makes with
the right main bronchus and the sharp angle that the apical segmental bronchus
makes with the lobar bronchus.

The medial segmental bronchus of the right middle lobe (choice C) arises from the
anterior wall of the right middle lobar bronchus. Therefore, when the patient is
supine, the effect of gravity will tend to prevent the object from entering this
segmental bronchus.
________________________________________
13>The correct answer is A. The disease described is Huntington's disease, which is
now known to be related to an expanded trinucleotide tandem repeat on the short
arm of chromosome 4. The number of trinucleotide repeats frequently increases in
succeeding generations during the process of spermatogenesis.
Prader-Willi and Angelman syndromes are frequently cited examples of genetic
diseases involving genomic imprinting (choice B).
Duchenne's muscular dystrophy is an example of a disease that is frequently due to
a large deletion in a single gene (choice C).
Single amino acid substitutions (choice D) are common in recessive diseases such
as sickle cell anemia.
Translocations (choice E) occur in disorders such as chronic myelogenous leukemia
(CML; Philadelphia chromosome) and some cases of Down's syndrome.
________________________________________
14>The correct answer is A. The mass is an abdominal aortic aneurysm, typically
found in older men. Such aneurysms are almost always related to the formation of
complicated atherosclerotic plaques in the aorta. Associated coronary artery
disease is commonplace.
Bacterial infection (choice B) can cause "mycotic" aneurysms; these usually involve
smaller vessels.
The small berry aneurysms that can involve the circle of Willis are congenital
anomalies (choice C).
Cystic medial degeneration (choice D) is related to the development of dissecting
aneurysms (actually dissecting hematomas).
Tertiary syphilis (choice E) typically causes aneurysms of the root and arch of the
aorta, rather than the descending aorta.
________________________________________
15>The correct answer is C. The cell is spindle-shaped like a fibroblast; however,
the difference is that the cytoplasm contains several bundles of microfilaments.
These bundles are parallel to the long axis of the cell and are seen immediately
beneath the cell membrane and within the cytoplasm. Densities, comparable to Zlines, can be seen along some of these bundles. The microfilaments are responsible
for the contractile properties of this cell. These contractile properties, in addition to
the cell's ability to link with collagen, function in wound closure in the healing

process. Dupuytren's contracture, which is a contracture of the palmar fascia, is


caused by interaction of these cells with collagen fibrils of the fascia.
The endothelial cell (choice A) lines vessels. There are no vessels in the
photomicrograph.
The myoepithelial cell (choice B) contains microfilaments and is contractile.
However, it is closely associated with glandular epithelium (not apparent here).
The pericyte (choice D) is a multipotential connective tissue cell found near or
around blood vessels, but it does not contain microfilament bundles such as these.
There are no vessels apparent in the photomicrograph.
The smooth muscle cell (choice E) is joined by junctions to other smooth muscle
cells, arranged in bundles. Microfilaments make up most of the cytoplasm of such
cells, with the nucleus in a central location.
________________________________________
16>The correct answer is B. The structure of the cochlea is complex. The organ of
Corti contains hair cells from the cochlear branch of the vestibulocochlear nerve (CN
VIII). These cells rest on the basilar membrane (choice A), which separates the scala
tympani (choice D) from the scala media. The hairs of the hair cells are embedded
in the tectorial membrane, and movement of the basilar membrane below the cells
tends to bend the hairs, which generates action potentials by the hair cells. The
tectorial membrane that lies on the hair cells does not form a boundary between
the different scala; the membrane separating the scala media from the scala
vestibuli (choice E) is Reissner's (vestibular) membrane (choice C).
________________________________________
17>The correct answer is C. The pretracheal layer of the cervical fascia runs from
the investing layers in both sides of the lateral neck and splits to enclose the thyroid
gland. Superiorly, it attaches to the laryngeal cartilages; inferiorly, it fuses with the
pericardium. As a result of these connections, the thyroid gland moves with
laryngeal movements.
The carotid sheath (choice A) contains the vagus nerve, internal jugular vein,
carotid artery, and lymph nodes.
The investing layer of the deep cervical fascia (choice B) splits to enclose the
trapezius and sternocleidomastoid muscles.
The prevertebral fascia (choice D) covers muscles arising from the vertebrae.
The superficial fascia (choice E) is immediately deep to the platysma muscle
________________________________________
18>The correct answer is D. The best markers for identification of B cells are CD19,
CD20, and CD21. The CD21 marker is a receptor for EBV (Epstein-Barr Virus).

The CD3 marker (choice A) is present on all T cells with either a CD4 or CD8 marker.
This is the marker that is used to identify total T cell count in a blood sample. The
CD3 marker is used for signal transduction in the different T cells.
The CD4 marker (choice B) is used to identify T helper cells. These are the cells that
recognize exogenous peptides presented on MHC class II molecules by
macrophages. CD4+ T helper cells are also involved in cell-mediated delayed
hypersensitivity, production of cytokines for stimulation of antibody production by B
cells, and stimulation of macrophages.
The CD8 marker (choice C) is used to identify cytotoxic T cells. These are the cells
that recognize viral epitopes attached to the MHC class I molecules of a virally
infected cell.
The CD56 marker (choice E) is used to identify NK(natural killer) cells. These cells
are important in innate host defense, specializing in killing virally infected cells and
tumor cells by secreting granzymes and perforins.
________________________________________
19>The correct answer is A. In a syndrome called cleidocranial dysostosis, absence
of part of the clavicles accompanies a broad skull, and facial and dental anomalies.
Note that you could also have answered this question by noting that of the bones
listed, only the clavicles form by intramembranous ossification.
The femurs (choice B), metatarsals (choice C), phalanges (choice D), and tibias
(choice E) are cartilaginous (formed by endochondral ossification) rather than
membranous bones.
________________________________________
20>The correct answer is D. Metformin is a drug that is often used in conjunction
with oral hypoglycemic agents for the treatment of NIDDM. Its mechanism of action
is two-fold: (1) it decreases the production of glucose in the liver; (2) it increases the
uptake of glucose in the liver. Metformin has no effect on the secretion of pancreatic
insulin.
Acetohexamide (choice A) is an oral hypoglycemic agent that is a sulfonylurea
derivative. It stimulates secretion of insulin from the pancreas.
Chlorpropamide (choice B) is an oral hypoglycemic agent that is a sulfonylurea
derivative. It stimulates secretion of insulin from the pancreas.
Glyburide (choice C) is a sulfonylurea derivative that stimulates insulin secretion
from the pancreas.
Tolbutamide (choice E) is a sulfonylurea derivative that stimulates insulin secretion
from the pancreas.
________________________________________

21The correct answer is A. The patient is presenting with signs and symptoms of
acute cholecystitis, which is associated with gallstones in more than 90% of all
cases. This condition occurs when a stone becomes impacted in the cystic duct and
inflammation develops behind the obstruction. The acute attack is often
precipitated by a large fatty meal, and is characterized by the sudden appearance
of severe, steady pain localized to the epigastrium or right hypochondrium.
Laboratory findings often include elevated white blood cells (2,000 - 15,000/mL).
Total serum bilirubin values of 1-4 mg/dL may be seen in some instances, and
serum amylase may be elevated. In noncomplicated cases, treatment often includes
IV alimentation, analgesics, and antibiotics, as well as withholding of oral feedings.
Meperidine is the narcotic of choice since it is least likely to cause spasm of the
sphincter of Oddi, probably because of its antimuscarinic properties. It is therefore
preferred over morphine (choice B), oxycodone (choice D), and propoxyphene
(choice E). Furthermore, propoxyphene is a narcotic agonist with mild analgesic
properties; hence, it would most likely not be able to effectively treat this patient's
severe pain.
Naproxen (choice C) is a nonsteroidal anti-inflammatory drug (NSAID) indicated for
the treatment of mild to moderate pain; this agent would most likely not provide
sufficient pain control for this patient.
________________________________________
22>The correct answer is A. When the head of the humerus dislocates from the
glenohumeral joint, it exits inferiorly, where the joint capsule is the weakest.
Immediately inferior to the glenohumeral joint, the axillary nerve exits from the
axilla by passing through the quadrangular space. At this location, the downward
movement of the head of the humerus can stretch the axillary nerve. The axillary
nerve innervates the deltoid muscle after leaving the axilla.
The dorsal scapular nerve (choice B) passes along the medial border of the scapula
to innervate the rhomboid muscles. The nerve does not pass in the region of the
glenohumeral joint.
The lateral and medial pectoral nerves (choices C and D) branch from the lateral
and medial cords of the brachial plexus, respectively, and exit through the anterior
wall of the axilla to innervate the pectoralis major and minor. These nerves do not
pass in the region of the glenohumeral joint.
The suprascapular nerve (choice E) is a branch of the upper trunk of the brachial
plexus and passes over the superior border of the scapula to innervate the
supraspinatus and infraspinatus muscles. This nerve does not pass in the region of
the glenohumeral joint.
________________________________________
23>The correct answer is D. This patient most likely has subacute granulomatous
(de Quervain's) thyroiditis, which frequently develops after a viral infection.
Microscopically, it is characterized by microabscess formation within the thyroid,
eventually progressing to granulomatous inflammation with multinucleated giant

cells. Clinically, patients may experience fever, sudden painful enlargement of the
thyroid, and/or symptoms of transient hyperthyroidism. The disease usually abates
within 6 to 8 weeks.
Diffuse nontoxic goiter (choice A) by definition does not produce hyperthyroidism.
The hyperthyroidism of Graves disease (choice B) does not spontaneously remit.
Hashimoto's thyroiditis (choice C) can cause transient hyperthyroidism, but then
goes on to cause hypothyroidism.
Subacute lymphocytic thyroiditis (choice E) can cause transient hyperthyroidism,
but is characteristically painless.
________________________________________

25>
The correct answer is E. Hypersecretion of insulin by a pancreatic b cell tumor is a
major cause of fasting hypoglycemia (plasma glucose is not increased, choice D).
Symptoms are related to neuroglycopenia and could include recurrent central
nervous system dysfunction during fasting or exercise. While proinsulin only makes
up approximately 20% of plasma immunoreactive insulin in normal individuals, in
patients with an insulinoma it contributes 30-90% of the immunoreactive insulin.
Hence, plasma levels of proinsulin are increased. The increased secretion of insulin
by the tumor will also lead to an increase in C peptide secretion (not decreased,
choice B) since b cells secrete insulin and C peptide on a one-to-one molar ratio.
Given the prolonged hypoglycemia, the amount of glycosylated hemoglobin may
also be decreased, although this is not a universal finding. Certainly, an increase in
glycosylated hemoglobin would not be expected (choice A). Glucagon secretion is
increased by hypoglycemia and its plasma level in a patient with an insulinoma
would be expected to be increased compared to normal (not decreased, choice C
________________________________________
26>The correct answer is A. This question is really asking you two things. First, it
tests whether you know that lymphocytosis is associated with viral infection.
Second, it tests your knowledge of lymphocytes' histological appearance. Note that
these cells are generally small and normally constitute 25%-33% of leukocytes. Two
types of lymphocytes have been distinguished: T cells (involved in cell-mediated
immunity) and B cells (involved in humoral immunity).
Monocytes are precursors of osteoclasts and liver Kupffer cells (choice B) and also
give rise to tissue macrophages and alveolar macrophages.
Platelets contain a peripheral hyalomere and central granulomere (choice C).
Neutrophils have azurophilic granules and multilobed nuclei (choice D). They
increase in number in response to bacterial infection.
Erythrocytes remain in the circulation for about 120 days (choice E).

________________________________________
27>The correct answer is D. The lesion is a malignant melanoma. Melanomas can
develop either de novo or in an existing mole. Sunlight exposure is a significant risk
factor and fair-skinned persons are at increased risk of developing melanoma. The
most significant factor for long term prognosis is the depth of the lesion, since the
superficial dermis lies about 1 mm under the skin surface, and penetration to this
depth is associated with a much higher incidence of metastasis than is seen with a
more superficial location.
The circumference of the lesion (choice A) is much less important than depth, since
one form of melanoma (superficial spreading) can still have good prognosis despite
large size, if it has not extended to the depth of the superficial dermal lymphatic
bed.
The darkness (choice B) or degree of variation in color (choice C) do not have
prognostic significance once melanoma is diagnosed.
Irregularity, or fuzziness at the border (choice E) of a mole-like lesion is a good clue
to potential malignancy, but does not affect prognosis once a melanoma is
diagnosed.
________________________________________
28>The correct answer is A. Most of the venous drainage of the skin of the lower
extremity is to the long saphenous vein, the accompanying lymphatics of which
drain into the superficial group of the inguinal lymph nodes. However, the skin
drained by the short saphenous vein, including that of the lateral aspect of the
dorsum of the foot, is an exception to this rule. Lymphatic fluid from this area drains
into lymphatics accompanying the short saphenous vein, then drains into lymph
nodes behind the knee in the popliteal fossa.
The lateral side of the thigh (choice B), the medial side of the leg below the knee
(choice C), the medial side of the sole of the foot (choice D), and the medial side of
the thigh (choice E) all drain to the superficial inguinal nodes.
________________________________________
29>The correct answer is E. The child has "rat-bite fever." This occurs in two forms
with somewhat similar clinical manifestations. The form this child has is the
Haverhill fever form, caused by Streptobacillus moniliformis and characterized by a
short (often 1-3 days) incubation period. The Haverhill form is more common in the
United States than the Sodoku form, which is caused by Spirillium minus, has a 1-4
week incubation period, and is most prevalent in Japan. One of the problems with
diagnosing these diseases is that the victims are usually young children, and the
bite site may be inapparent by the time the disease becomes severe enough for the
child to be taken to a doctor.
Borrelia burgdorferi (choice A) causes Lyme disease, which is characterized by an
expanding erythematous rash, arthralgias, and eventual nervous system
involvement.

Pseudomonas mallei (choice B) causes glanders, which generally affects horses or


humans in close contact with equines.
Pseudomonas pseudomallei (choice C) causes melioidosis, a rare pulmonary disease
found mostly in Southeast Asia.
Spirillium minus (choice D) causes the Sodoku form of rat-bite fever.
________________________________________
30>The correct answer is A. Urticaria (hives) are pruritic wheals that form after
mast cells degranulate and trigger localized dermal edema with dilated superficial
lymphatic channels. The mast cell degranulation is sometimes, but not always,
triggered by IgE-antigen interactions.
Granular complement and IgG deposition at the dermal/epidermal junction (choice
B) is a characteristic of systemic lupus erythematosus.
Microscopic blisters (choice C) are a characteristic of dermatitis herpetiformis.
Munro abscesses (choice D) are a characteristic of psoriasis.
Solar elastosis (choice E) is found in actinic keratoses.
________________________________________
31>The correct answer is B. Direct inguinal hernias enter the inguinal canal by
tearing through the posterior wall of that structure. The typical location for this type
of hernia is through the inguinal triangle, bounded laterally by the inferior epigastric
artery, medially by the lateral border of the rectus abdominis, and inferiorly by the
inguinal ligament. Direct inguinal hernias pass medial to the inferior epigastric
artery, whereas indirect inguinal hernias pass lateral to the inferior epigastric artery
because the deep inguinal ring is lateral to the artery.
Indirect inguinal hernias are covered by all three layers of the spermatic fascia
(choice A). Direct inguinal hernias are covered by fewer than all three layers
because the direct inguinal hernia tears through one or more layers of fascia as it
emerges though the abdominal wall.
The lateral border of the rectus abdominis muscle (choice C) forms the medial
border of the inguinal triangle. All inguinal hernias pass lateral to the rectus
abdominis.
Femoral hernias pass posterior to the inguinal ligament (choice D). Inguinal hernias
emerge through the superficial inguinal ring, which is superior to the inguinal
ligament. Inguinal hernias that descend below the inguinal ligament pass anterior to
the ligament.
Indirect inguinal hernias pass through the deep inguinal ring (choice E); direct
inguinal hernias do not. Both types of inguinal hernias pass through the superficial
inguinal ring.
________________________________________

32>The correct answer is A. The patient has a severe, potentially fatal, pneumonia
with prominent systemic symptoms. Culture on BCYE is the specific clue that the
organism is Legionella pneumophila. The disease is respiratory Legionellosis, also
known as Legionnaire's disease, because the disease was first described when it
occurred in epidemic form following an American Legion convention at a
Philadelphia hotel. Patients tend to be older (40-70 years old) and may have risk
factors including cigarette use, alcoholism, diabetes, chronic illness, or
immunosuppressive therapy.
Listeria monocytogenes (choice B) causes listeriosis and is not a notable cause of
pneumonia.
Spirillium minus (choice C) is a cause of rat-bite fever and is not a notable cause of
pneumonia.
Staphylococcus aureus (choice D) can cause pneumonia, but is easily cultured on
routine media.
Streptococcus pneumoniae (choice E) can cause pneumonia, but is easily cultured
on routine media.

________________________________________

________________________________________
33The correct answer is C. Sensitivity is defined as the ability of a test to detect the
presence of a disease in those who truly have the disease. It is calculated as the
number of people with a disease who test positive (true positive) divided by the
total number of people who have the disease (true positive + false negative). In this
case, sensitivity equals the number of people with HIV antibody who test positive
(100) divided by the total number of people who have HIV antibody (120). This
yields 100/120 = 83% (not a very sensitive test).
11% (choice A) corresponds to the prevalence of the disease in the tested
population, which in this case equals the total number of people with HIV antibody
(true positive + false negative = 100 + 20 = 120) divided by the total number of
people tested (100 + 20 + 50 + 950 = 1120). This yields 120/1120 = 11%.
67% (choice B) corresponds to the positive predictive value of the test, which equals
the number of people with the disease who test positive (true positives = 100)
divided by the total number of people testing positive (all positives = 50). This
yields 100/150 = 67%.

95% (choice D) corresponds to the specificity of the test, which equals the number
of people without HIV antibody who test negative (950) divided by the total number
of people without HIV antibody (1000). This yields 950/1000 = 95%.
98% (choice E) corresponds to the negative predictive value of the test, which
equals the number of people without HIV antibody who test negative (950) divided
by the total number of people testing negative (970). This yields 950/970 = 98%. An
easy way to remember these concepts is:
Sensitivity = true positives/all diseased
Specificity = true negatives/all normal
PPV = true positives/all positives
NPV = true negatives/all negatives
Prevalence = all diseased/total population
If you prefer charts and formulas:
(Diseased)

(Normal)

People with HIV antibody People without HIV antibody


Positive test a, true pos. b, false pos.
Negative test

c, false neg d, true neg.

Sensitivity = a/(a + c)
Specificity = d/(b + d)
PPV = a/(a + b)
NPV = d/(c + d)
Prevalence = (a + c)/(a + b + c + d)
Choose your favorite approach. Just make sure you know this stuff, as it is almost
always tested and gains you easy points once you have mastered the concepts.
________________________________________
34>The correct answer is E. This is pemphigus vulgaris, in which autoantibody
directed against transmembrane cadherin adhesion molecules induces acantholysis
(breakdown of epithelial cell-cell connections) with resulting intraepidermal blister
formation. It may develop spontaneously or following triggers such as drugs (thiols,
penicillamine), physical injury (burns), cancer, pregnancy, other skin diseases, and
emotional stress. Pemphigus vulgaris is a relatively rare blistering disease; it is seen
more commonly in patients with Jewish or Mediterranean heritage. In addition to the
usually prominent oral ulcers, uncomfortable skin erosions can also occur when the
blisters rupture rapidly and are not observed. The epidermis at the edge of these
erosions is often easily disrupted by sliding pressure (Nikolsky sign).

Bullous pemphigoid (choice A) is characterized by deeper blisters, occurring at the


dermal-epidermal junction.
Dermatitis herpetiformis (choice B) is characterized by severe, intense pruritus and
groups of papules and vesicles.
Herpes simplex I (choice C) or II (choice D) can show multinucleated giant cells on
scrapings of the ulcer base.
________________________________________
35>The correct answer is A. Polycystic kidney disease leads to progressive
decrements in renal function, eventually resulting in renal failure (evidenced by the
increased BUN and creatinine). These abnormalities are caused by a reduction in
glomerular filtration rate (GFR), which produces a decrease in creatinine clearance.
The production of creatinine, a waste product of metabolism, bears a direct relation
to the muscle mass of an individual and is independent of renal function. Because
creatinine is freely filtered by the glomerulus, but not secreted or reabsorbed to a
significant extent, the renal clearance of creatinine is approximately equal to the
GFR. Therefore, creatinine clearance is commonly used to assess renal function in
the clinical setting.
The extracellular sodium concentration (choice B) is not expected to change
significantly in this patient.
Glucose (choice C) is not normally excreted, so glucose clearance is normally zero.
Thus, is not possible for glucose clearance to be decreased.
Plasma creatinine concentration (choice D) increases when GFR decreases.
Inulin clearance is used to estimate GFR. However, inulin is foreign to the body, and
inulin plasma levels (choice E) are normally zero.
________________________________________
36>The correct answer is D. The metanephric duct (also known as the ureteric bud)
is a diverticulum of the mesonephric duct. It grows to the metanephric mass of the
urogenital ridge. It induces the development of the metanephros, which will give
rise to the excretory units of the definitive kidney. The metanephros, in turn,
induces the metanephric duct to divide into the calyces and the collecting tubules.
The allantois (choice A) is an endodermal diverticulum of the yolk sac. It becomes
incorporated into the urogenital sinus and contributes to the formation of the
urinary bladder. It is not involved with the formation of the kidney.
The mesonephric duct (choice B) is a mesodermal duct into which the mesonephric
tubules drain. The mesonephric duct is the same as the pronephric duct, but at a
later stage of development. The mesonephric duct develops into the ductus
deferens in the male. The metanephric duct is a diverticulum of the mesonephric
duct.

The mesonephros (choice C) is the embryonic kidney, which functions in the embryo
before the metanephros (or definitive kidney) develops. The mesonephros develops
from the urogenital ridge.
The urogenital ridge (choice E) is a longitudinal elevation of the intermediate
mesoderm. The embryonic and adult kidneys, as well as the gonads, develop from
this mesoderm.
________________________________________
37>The correct answer is E. The child has neonatal respiratory distress syndrome
(hyaline membrane disease). This condition is caused by the inability of the
immature lungs to synthesize adequate amounts of surfactant. Surfactant, which
reduces surface tension, helps keep alveoli dry and aids in expansion of the lungs, is
synthesized by type II pneumocytes.
Alveolar capillary endothelial cells (choice A) are important in maintaining the
capillary structure and permitting flow of gases into and out of the blood stream.
Bronchial mucous cells (choice B) produce the usually thin (in healthy individuals)
coat of mucus that lines the bronchi.
The ciliated bronchial respiratory epithelium (choice C) is responsible for moving the
dust coated mucus layer out of the bronchi.
Type I pneumocytes (choice D) are the squamous cells that line alveoli and permit
easy gas exchange. These cells tend to be immature (and thick) in premature
infants, but do not produce surfactant.
________________________________________
38>The correct answer is A. Surgeons worry about their obese patients more than
their skinny ones, because a thick layer of relatively poorly vascularized
subcutaneous fatty tissue is both mechanically unstable (it holds stitches poorly)
and heals very slowly. These patients have a frequent rate of dehiscence (tearing
open of the incisional site) with subsequent, difficult-to-control infection (access by
antibiotics, leukocytes, and serum antibodies are all hampered by the poor blood
supply).
Aponeuroses (choice B) are strong thickenings of muscle sheath that usually suture
and heal well after surgery.
Loose connective tissue (choice C) is well vascularized and surgeons do not usually
worry much about it during the healing process.
Muscle (choice D) usually heals well after surgery.
Skin (choice E) usually heals well, unless it becomes infected.
________________________________________

39The correct answer is C. Renal artery stenosis can decrease the renal perfusion
pressure sufficiently to increase renin secretion significantly, which increases
angiotensin II (AII), which, in turn, increases aldosterone. Chronic hypertension due
to renal artery stenosis is the result of elevated levels of AII and aldosterone.
Aldosterone increases retention of sodium from the collecting duct, and water
follows; AII increases reabsorption of sodium from the proximal tubule, and water
follows. AII is also a vasoconstrictor, increasing peripheral vascular resistance.
Vasoconstriction of the renal vasculature decreases renal plasma flow, which would
be expected to decrease glomerular filtration rate proportionately. AII, however,
preferentially vasoconstricts the efferent arteriole in the nephron, maintaining a
reasonable glomerular filtration rate even with the reduced renal plasma flow.
Captopril (an angiotensin-converting enzyme inhibitor) inhibits the conversion of
angiotensin I to AII. The captopril-mediated decrease of AII will actually decrease
glomerular filtration in the kidney with renal artery stenosis, because decreasing
efferent constriction causes the glomerular capillary pressure to fall.
Atrial natriuretic peptide (choice A) levels would be expected to increase with
increased water and sodium retention due to renal artery stenosis.
BUN (choice B) would be expected to increase or stay the same with renal artery
stenosis, depending on the extent of AII-mediated efferent arteriole
vasoconstriction.
Metabolic alkalosis, due to the increased net acid secretion (compare with choice D)
from hyperaldosteronism, is typical in renal artery stenosis.
Potassium secretion would increase, rather than decrease (choice E), due to the
effects of excessive aldosterone secondary to renal artery stenosis.
________________________________________
40>The correct answer is E. Testicular feminization is a disorder of the androgen
receptor. Phenotypically, the patient appears female, but has a blindly ending
vagina and lacks a uterus or other female internal reproductive organs. The patient
has an XY genotype. Since the gene for testes determining factor (TDF) is on the Y
chromosome, TDF will cause the indifferent gonad to develop into a testis
containing Sertoli cells. Sertoli cells at this stage will secrete MIF, a substance that
suppresses the paramesonephric ducts, preventing the formation of female internal
reproductive organs.
This patient would not have a streak ovary (choice A), a finding in Turner's
syndrome that is associated with a 45,XO genotype. In fact, the patient would have
testes, since the genetic complement contained a Y chromosome. The testes in
individuals with testicular feminization syndrome are often undescended and are
usually removed surgically.
This patient would not possess a uterus (choice B) or an oviduct (choice C). Both of
these structures are derived from the paramesonephric duct, which is suppressed
by MIF.
________________________________________

41>The correct answer is C. Hyperventilation associated with states of anxiety can


lead to feelings of faintness, suffocation, tightness in the chest, and blurred vision.
Individuals undergoing such an attack may not be aware of overbreathing. The
anxious, hyperventilating woman is "blowing off" carbon dioxide, which lowers her
arterial PCO2. Many of the symptoms associated with anxiety attacks are probably
caused by a decrease in cerebral blood flow secondary to low arterial PCO2. Recall
that carbon dioxide is a major regulator of cerebral blood flow, i.e., carbon dioxide
dilates the brain vasculature, and conversely, the vasculature constricts when
carbon dioxide levels are low. The decrease in cerebral blood flow leads to cerebral
hypoxia, which is probably responsible for the fainting and blurred vision. An attack
may be terminated by breathing in and out of a plastic bag because this can
increase carbon dioxide levels in the blood. Inhaling a 5% carbon dioxide mixture
would also be effective.
Breathing a mixture of 10% oxygen/90% nitrogen (choice A) or 100% nitrogen
(choice B) can decrease oxygen delivery to the brain and thereby worsen the
symptoms caused by hyperventilation.
Hyperventilation results in hypocapnia (low PCO2), which causes alkalosis (high
blood pH). Bicarbonate (choice D) should not be administered to an alkalotic
patient.
The feelings of faintness and blurred vision resulting from hyperventilation are not
relieved by lying down (choice E).
________________________________________
42>The correct answer is A. The photomicrograph depicts an oocyte. The zona
pellucida at the arrow is similar to a thick basal lamina and is composed of
glycoproteins that bind to the cell membrane of the sperm head. This binding
triggers the acrosome reaction, which involves fusion of the acrosomal membrane
with the overlying sperm membrane. The acrosomal enzymes are released and
digest the zona pellucida, allowing the spermatozoon to make contact and fuse with
the ovum cell membrane.
The enzyme aromatase (choice B) is found in the granulosa cells, which surround
the oocyte and the follicular wall. Testosterone, produced by the theca cells, diffuses
through the basal lamina of the follicle and is converted to estradiol by aromatase.
Capacitation (choice C) refers to changes that occur in the spermatozoa during their
transit through the female reproductive tract. These changes occur in the oviducts
and the uterus. Sperm become motile in the epididymis.
The oocyte divides many times as it moves toward the uterus, the zona pellucida
disappearing. A portion of these cells will exhibit implantation receptors (choice D)
and become an implantation site for the cell mass in the uterine wall.

Meiosis is resumed (choice E) in the oocyte shortly before ovulation. The nucleus of
the oocyte can be seen in the center. The oocyte and the surrounding granulosa
cells make up this structure, known as an antral or secondary follicle.
________________________________________
43>The correct answer is D. This is a classic case of Cryptococcus neoformans
meningitis. Clues included the patient population (HIV positive), geographic area
(Mississippi and Missouri river beds), and diagnostic form (encapsulated yeast).
Cryptococcus is a monomorphic fungus, unlike many of the classic pathogens within
the fungal group, so the encapsulated yeast form would be found both in clinical
specimens and in the environment as the infectious form.
Broad-based, budding yeasts (choice A) would be the forms expected to be found in
clinical specimens (not environmental forms) from patients infected with
Blastomyces dermatitidis, which is far more likely to present with skin and bone
lesions than with meningitis.
Budding yeasts in a "pilot's wheel" arrangement (choice B) would be the form
expected to be found in clinical specimens (not environmental forms) from patients
infected with Paracoccidioides brasiliensis, which is limited in geographic region to
Central and South America and typically presents as a primary pulmonary disease.
Cylindrical arthroconidia (choice C) would be the transmission forms characteristic
of Coccidioides immitis, which may be a cause of fungal meningitis in
immunologically compromised individuals, but is geographically restricted to the
sub-Sonoran desert zone of the U.S. (San Joaquin valley fever).
Filamentous molds (choice E) are the transmission forms of several of the fungal
agents (eg, Coccidioides, Blastomyces, Paracoccidioides) but Cryptococcus does not
have a filamentous form.
Septate hyphae with microconidia and macroconidia (choice F) are the transmission
(environmental) forms of Histoplasma capsulatum, which is not an encapsulated
yeast in spite of its name. It is primarily a pulmonary infection acquired by exposure
to the droppings of birds or bats.
________________________________________
44>The correct answer is C. Both pyruvate kinase deficiency and glucose-6phosphate dehydrogenase deficiency are red cell enzyme deficiencies characterized
clinically by long "normal" periods interspersed with episodes of hemolytic anemia
triggered by infections and oxidant drug injury (antimalarial drugs, sulfonamides,
nitrofurans). In both of these conditions, the cell morphology between hemolytic
episodes is usually normal or close to normal.
The a (choice A) and b (choice B) thalassemias, in their major forms, are
characterized by persistent severe anemia. In the trait forms, they are
charactertized by mild anemia.

Hereditary spherocytosis (choice D) is characterized by intermittent hemolysis, but,


unlike pyruvate kinase deficiency and glucose-6-phosphate dehydrogenase
deficiency, oxidant drugs are not a specific trigger for hemolysis.
Iron deficiency anemia (choice E) is characterized by chronic anemia with
hypochromic, microcytic erythrocytes.
________________________________________
45>The correct answer is C. The fibrinolytic activity of streptokinase is due to its
ability to bind and cleave plasminogen, producing plasmin. Plasmin directly cleaves
fibrin, both between and within the fibrin polymers, thus breaking up thrombi and
potentially restoring blood flow to ischemic cardiac muscle. This same mechanism
of fibrinolysis is shared by urokinase and tissue-plasminogen activator (tPA).
Antithrombin III (choice A) is a coagulation inhibitor that binds to and inactivates
thrombin. Antithrombin III is anticoagulant, not fibrinolytic.
Fibrin (choice B) is not directly acted upon by streptokinase. It is indirectly cleaved
through the action of plasmin.
Protein C (choice D) is a glycoprotein that modulates coagulation by inhibiting the
procoagulant activities of factors V/Va and VIII/VIIIa. Protein C has no inherent
fibrinolytic activity.
Thrombomodulin (choice E) is an anticoagulant protein that binds to thrombin and
diminishes its capacity to activate fibrinogen, Factor V, and platelets.
Thrombomodulin has no fibrinolytic activity.
________________________________________
46>The correct answer is C. Enterobius vermicularis (pinworm) inhabits the large
intestine, and the gravid females migrate at night into the perianal region to lay
eggs, which can be identified by placing cellophane tape on the perianal skin and
then looking at the tape with a microscope. Enterobiasis is characterized by
extreme pruritus and is very contagious, transmitted by ingestion of the eggs of the
organism. The infection is easily treated with a single dose of mebendazole.
Ancylostoma duodenale (choice A) and Necator americanus (choice D) are
hookworms, and typically cause anemia.
Ascaris lumbricoides (choice B) is a large, intestinal roundworm that is very
common, especially in the tropics. Infection may be asymptomatic or can cause
abdominal symptoms.
Trichuris trichiura (choice E) is the whipworm, which can cause rectal prolapse with
heavy infestations.
________________________________________
47>The correct answer is D. The key symptom is hyperventilation. Hyperventilation
results in hypocapnia, alkalosis, increased cerebrovascular resistance, and
decreased cerebral blood flow. Carbon dioxide plays an important role in the control

of cerebral blood flow. An increase in arterial PCO2 dilates blood vessels in the brain
and a decrease in PCO2 causes vasoconstriction. The anxious, hyperventilating
woman is "blowing off" carbon dioxide, which lowers her arterial PCO2. This
decrease in PCO2 has caused the cerebrovascular resistance (choice E) to increase,
thereby decreasing cerebral blood flow. The decrease in cerebral blood flow has
caused the woman to feel faint and to have blurred vision. Other symptoms
commonly associated with the hyperventilation of anxiety states are feelings of
tightness in the chest and a sense of suffocation.
Hyperventilation increases the arterial oxygen content (choice A) and PO2 (choice
B) in a normal person.
A decrease in arterial PCO2 causes the arterial pH (choice C) to increase, i.e., the
patient becomes alkalotic.
________________________________________
48>The correct answer is B. Escherichia coli is a very common pathogen associated
with urinary tract infections and is a common cause of cystitis. It is part of the
normal flora of the GI tract. Patients with cystitis can develop bacteremia and
subsequent septic shock and adult respiratory distress syndrome (ARDS). The
patient described in the history has an obstructive lesion of the urinary tract that
predisposes him to the overgrowth of microorganisms such as E. coli. Pulmonary
symptoms are consistent with ARDS. E. coli is a gram-negative rod.
Gram-negative diplococci (choice A) might be Neisseria spp. or Moraxella
catarrhalis. Neisseria gonorrhea is the cause of gonorrhea, a sexually transmitted
disease that presents with urethritis, or may be asymptomatic. Patients are usually
younger, sexually active males. Moraxella spp. and Kingella kingae are gramnegative cocci that can cause a wide variety of infections. Moraxella catarrhalis is
usually implicated as a cause of otitis media and sinusitis in children, or as a cause
of purulent tracheobronchitis and pneumonia in a population of patients who are
over 50 and have underlying obstructive lung disease.
Gram-positive cocci (choice C), such as Staphylococcus aureus and S. epidermidis
and Streptococci, rarely cause cystitis. Staphylococcus saprophyticus causes urinary
tract infections, but the patients are typically young, sexually active women.
Gram-positive diplococci (choice D) would be a description of Streptococcus
pneumoniae, which is the most common cause of community-acquired pneumonia.
Gram-positive rods (choice E) would include members of the following genera:
Clostridium, Bacillus, Listeria, and the coryneform bacteria. The only significant
member of this group to produce urinary tract infections is Corynebacterium
urealyticum. (C. jeikeium). The organism creates an alkaline urine environment with
the potential for stone formation. Patients are usually immunocompromised or have
had recurrent urinary tract infections.
________________________________________

49>The correct answer is E. The nerve that was cut is the median nerve. The
median nerve innervates all of the thenar compartment muscles at the base of the
thumb: the abductor pollicis brevis, flexor pollicis brevis, and opponens pollicis.
Paralysis of the opponens pollicis will prevent the patient from opposing the thumb.
Abduction of the second digit (choice A) is performed by the first dorsal
interosseous muscle. All of the interosseous muscles are innervated by the ulnar
nerve.
Adduction of the second digit (choice B) is performed by the first palmar
interosseous muscle. All of the interosseous muscles are innervated by the ulnar
nerve.
Adduction of the thumb (choice C) is performed by the adductor pollicis muscle,
which is innervated by the ulnar nerve.
Flexion of the interphalangeal joint of the thumb (choice D) is performed by the
flexor pollicis longus muscle, which is in the proximal forearm and is innervated by
the median nerve proximal to the site of the injury. The flexor pollicis brevis muscle
causes flexion at the metacarpophalangeal joint of the thumb.
________________________________________

50>
The correct answer is A. Cyclophosphamide is metabolized to acrolein, which is
excreted in the urine. If the patient's urine is concentrated, the toxic metabolite
may cause severe bladder damage. Early symptoms of bladder toxicity include
dysuria and frequency. This can be distinguished from a urinary tract infection, since
there is no bacteriuria with cyclophosphamide-induced bladder toxicity. However,
microscopic hematuria is often present on urinalysis. In severe hemorrhagic cystitis,
large segments of the bladder mucosa may be shed which can lead to prolonged,
gross hematuria. The incidence of cyclophosphamide-induced hemorrhagic cystitis
can be decreased by ensuring that the patient maintains a high fluid intake.
Cyclophosphamide is an alkylating agent used in the treatment of breast carcinoma,
malignant lymphoma, multiple myeloma, and adenocarcinoma of the ovary, as well
as various other forms of cancer. The major toxic reactions commonly seen with this
agent include mucositis, nausea, hepatotoxicity, sterile hemorrhagic and nonhemorrhagic cystitis, leukopenia, neutropenia, and interstitial pulmonary fibrosis

Answers

________________________________________

1>The correct answer is A. A number of physiologic changes occur in a person living


at high altitude. The diminished barometric pressure at high altitude causes alveolar
hypoxia and arterial hypoxia. Pulmonary vasoconstriction occurs in response to
alveolar hypoxia; therefore, the diameter of the pulmonary vessels would be greater
in the brother living at sea level. All the other choices describe physiologic
processes that would be enhanced by living at high altitude.
Increased erythropoietin production (choice B), caused by arterial hypoxia, leads to
increases in hematocrit in people living at high altitude.
Mitochondrial density increases (choice C) in people chronically exposed to the
hypoxemia caused by living at high altitude.
At high altitudes, the ventilation rate increases, causing a respiratory alkalosis. The
kidney then compensates by increasing the excretion of HCO3- (choice D).
Increasing the rate of respiration (choice E) is a very useful adaptation to the
hypoxic conditions of high altitude. The primary stimulus is the hypoxic stimulation
of peripheral chemoreceptors.
________________________________________

2>The correct answer is A. Alopecia areata is caused by an autoimmune attack on


hair follicles. It has a wide range of clinical severity, with most cases involving a
localized patch of hair (which regrows within 1 year in half of the patients). The hair
that does regrow may be gray or depigmented. More severe cases can involve the
entire scalp (alopecia totalis) or, as in this patient, the entire body surface (alopecia
universalis). These more severe cases are less likely to resolve adequately.
Treatment of alopecia areata is often unsuccessful, but topical steroids are typically
tried.
Androgenic alopecia (choice B) is common male pattern baldness.
Chronic cutaneous lupus erythematosus (choice C) can produce localized baldness.
Lichen planopilaris (choice D) can produce localized baldness.
Trichotillomania (choice E), also called traumatic alopecia, is alopecia due to
trauma, such as hair pulling or tight braids.
________________________________________
3>The correct answer is B. Both the omental bursa and the greater omentum are
derived from the dorsal mesogastrium, which is the mesentery of the stomach
region.
The dorsal mesoduodenum (choice A) is the mesentery of the developing
duodenum, which later disappears so that the duodenum and pancreas come to lie
retroperitoneally.

The pericardioperitoneal canal (choice C) embryologically connects the thoracic and


peritoneal canals.
The pleuropericardial membranes (choice D) become the pericardium and
contribute to the diaphragm.
The ventral mesentery (choice E) forms the falciform ligament, ligamentum teres,
and lesser omentum.
________________________________________
4>The correct answer is C. Wine and cheese (and many other fermented foods)
contain tyramine, an indirect sympathomimetic that can trigger excess
catecholamine release and lead to a hypertensive crisis when ingested by patients
taking MAO inhibitors. The only such drug listed among the answer choices is
phenelzine. Other MAO inhibitors with similar effects include tranylcypromine,
isocarboxazid, and iproniazid. Whenever this particular drug class is mentioned in a
question stem, consider the possibility of interactions with foods or other
medications the patient may have taken.
Amitriptyline (choice A) is a tricyclic antidepressant. Tricyclic antidepressant drugs
(particularly amitriptyline) are known for their anticholinergic side effects. They also
produce postural hypotension (because they block alpha-adrenergic receptors) and
are sedative.
Fluoxetine and sertraline (choices B and D) are antidepressants that are selective
serotonin reuptake inhibitors (SSRIs). Fluoxetine is also useful in treatment of
obsessive-compulsive disorders.
Trazodone (choice E) is an atypical antidepressant with substantial sedative side
effects. Its most serious side effect is priapism, a medical emergency.
________________________________________
5>The correct answer is D. The cells of the anterior pituitary can be classified as
chromophils (love dyes) or chromophobes (do not stain with dyes). The chromophils
can be further divided into acidophils (stain with acidic dyes) and basophils (stain
with basic dyes). The acidophils include the somatotropes, which secrete growth
hormone, and the mammotropes, which secrete prolactin. The basophils include the
corticotropes, which secrete ACTH (choice A), the gonadotropes, which secrete FSH
and LH (choices B and C), and the thyrotropes, which secrete TSH (choice E).
________________________________________
6>The correct answer is D. Persons with this condition often perceive attacks and
danger in relatively innocuous situations. They are quick to respond with anger,
and, because personality disorders are ego-syntonic, individuals with personality
disorders do not believe themselves to be in error.
The individual with dependent personality disorder (choice A) does not confront
others but wants others to take care of him.

The individual with histrionic personality disorder (choice B) is flamboyant and


seductive, not confrontational and angry.
The individual with narcissistic personality disorder (choice C) is characterized by
feelings of entitlement because they are so "special."
And the individual with passive aggressive personality disorder (choice E) expresses
anger indirectly (e.g., always being late) rather than confronting
________________________________________
7>The correct answer is C. The loading of O2 is facilitated when the oxygen
dissociation curve shifts to the left, and the unloading of O2 is facilitated when the
oxygen dissociation curve shifts to the right. A good way to remember the
conditions that promote dissociation of O2 is to think of exercising muscle, which
has decreased pH (choice C) because of the accumulation of lactic acid, increased
PCO2 (compare with choice B) because of the increased rate of aerobic metabolism,
increased temperature (compare with choice D), and increased 2,3-DPG (2,3diphosphoglycerate; compare with choice E) because of increased glycolysis.
Carbon monoxide poisoning (choice A) left-shifts the oxygen dissociation curve,
which interferes with the unloading of O2. Carbon monoxide also strongly binds to
available sites on hemoglobin.
________________________________________
8>The correct answer is A. This patient has Huntington's disease, which has
autosomal dominant inheritance. It is characterized by severe degeneration of the
caudate nucleus along with degenerative changes in the putamen and cortex. In
addition to chorea, these patients frequently suffer from athetoid (writhing)
movements, progressive dementia, and behavioral disorders.
________________________________________
9>The correct answer is E. The rash described is that of secondary syphilis, caused
by Treponema pallidum. Involvement of palms and soles by a rash is unusual, and
secondary syphilis should come to mind. Not all patients with secondary syphilis
have a severe form of the rash, and consequentially some cases are missed.
Primary syphilis takes the form of a painless, button-like mass called chancres.
Tertiary syphilis, which is now rare, has a propensity for involving the aorta and
central nervous system and can also cause "gummas" (granulomatous-like lesions)
in many sites, notably including liver and bone.
Herpes simplex I (choice A) usually causes perioral vesicular lesions.
Herpes simplex II (choice B) usually causes genital vesicular lesions.
HIV (choice C) does not itself cause a rash, although co-infection with other
organisms can result in a rash.
Neisseria gonorrhoeae (choice D) does not typically cause a rash.
________________________________________

10>The correct answer is B. Hyperlipidemia has been subclassified based on the


lipid and lipoprotein profiles. Type 2a, which this patient has, can be seen in a
hereditary form, known as familial hypercholesterolemia, and also in secondary,
acquired forms related to nephritic syndrome and hyperthyroidism. The root
problem appears to be a deficiency of LDL receptors, which leads to a specific
elevation of cholesterol in the form of increased LDL. Heterozygotes for the
hereditary form generally develop cardiovascular disease from 30 to 50 years of
age. Homozygotes may have cardiovascular disease in childhood.
Type 1 (choice A) is characterized by isolated elevation of chylomicrons.
Type 2b (choice C) is characterized by elevations of both cholesterol and
triglycerides in the form of LDL and VLDL.
Type 3 (choice D) is characterized by elevations of triglycerides and cholesterol in
the form of chylomicron remnants and IDL.
Type 5 (choice E) is characterized by elevations of triglycerides and cholesterol in
the form of VLDL and chylomicrons.
________________________________________
11The correct answer is C. The neurohypophysis (posterior pituitary) is derived from
an evagination of diencephalic neurectoderm. This structure is responsible for
releasing oxytocin and vasopressin to the general circulation. Both hormones are
synthesized in cell bodies contained within the hypothalamus.
ACTH (choice A), prolactin (choice D), and TSH (choice E) are all synthesized and
released by the anterior pituitary, or adenohypophysis, which is derived from an
evagination of the ectoderm of Rathke's pouch, a diverticulum of the primitive
mouth. Remnants of this pouch may give rise to a craniopharyngioma in later life.
Epinephrine (choice B) is synthesized and released into the circulation by the
adrenal medulla, a neural crest derivative.
________________________________________
12>The correct answer is E. The most common bacteria implicated in communityacquired pneumonia is the pneumococcus, Streptococcus pneumoniae. Other
organisms frequently implicated in patients less than age 60 without comorbidity
include Mycoplasma pneumoniae, respiratory viruses, Chlamydia pneumoniae, and
Haemophilus influenzae. When community-acquired pneumonia occurs in elderly
patients or patients with comorbidity, aerobic gram-negative bacilli and
Staphylococcus aureus are added to the list.
The organisms listed in choices A, B, and C are important causes of communityacquired pneumonia, but are not the most frequent causes.
Staphylococcus aureus (choice D) is an important cause of community-acquired
pneumonia (particularly in the elderly and in patients with comorbidity), but is not
the most frequent cause.

________________________________________
13>The correct answer is E. Uric acid kidney stones in patients with leukemia are
secondary to increased production of uric acid from purine breakdown during
periods of active cell proliferation, especially following treatment. Vigorous
hydration and diuresis are generally instituted after the diagnosis of acute leukemia
is made. Uric acid kidney stones are also associated with inborn errors of purine
metabolism, such as gout.
Pigment gallstones (choice A) are associated with hemolytic disease. The incidence
of this type of gallstone is not increased in treated leukemias.
Cholesterol gallstones (choice B) are associated with diabetes mellitus, obesity,
pregnancy, birth control pills, and celiac disease.
Cystine kidney stones (choice C) are rare; they are found in cystinuria.
Struvite kidney stones (choice D) are associated with infection by urea-splitting
organisms, such as Proteus.
________________________________________
14>The correct answer is B. The data shown in the table indicate that the man has
developed metabolic alkalosis (increased PCO2, pH, and HCO3-), which occurs
commonly with overuse of diuretics (thiazides and loop diuretics). The overuse of a
loop diuretic increases the excretion of sodium (choice E) and potassium (choice C)
by the kidneys. The increase in potassium excretion leads to a decrease in plasma
potassium levels (choice B). The decrease in plasma potassium stimulates
aldosterone secretion, which raises plasma aldosterone levels (choice A). The
sodium depletion stimulates renin secretion (choice D), which in turn raises
angiotensin II levels in the plasma (which also stimulates aldosterone secretion
________________________________________
15>The correct answer is C. This is a case of Reiter's syndrome. Patients typically
present with the acute onset of arthritis (usually asymmetric and additive), with
involvement of new joints occurring over a period of a few days to 2 weeks. Joints of
the lower extremities are the most commonly involved, but wrists and fingers can
also be affected. Dactylitis (sausage digit), a diffuse swelling of a solitary finger or
toe, is a distinctive feature of Reiter's arthritis and psoriatic arthritis. Tendonitis and
fasciitis are common. Spinal pain and low back pain are common. Conjunctivitis,
urethritis, diarrhea, and skin lesions are also associated with Reiter's syndrome. Up
to 75% of patients are HLA-B27 positive. Microorganisms which can trigger Reiter's
syndrome include Shigella spp., Salmonella spp., Yersinia spp., Campylobacter
jejuni, and Chlamydia trachomatis. Most patients are younger males.
Gout (choice A) usually presents as an explosive attack of acute, very painful,
monarticular inflammatory arthritis. Hyperuricemia is the cardinal feature and
prerequisite for gout. The first metatarsophalangeal joint is involved in over 50% of
first attacks.

Lyme disease (choice B), caused by Borrelia burgdorferi, presents with a red macule
or papule at the site of the tick bite. This lesion, called erythema chronicum
migrans, slowly expands to form a large annular lesion with a red border and central
clearing. The lesion is warm, but usually not painful. The patient also has severe
headache, stiff neck, chills, arthralgias, and profound malaise and fatigue.
Untreated infection is associated with development of arthritis. The large joints
(e.g., knees) are usually involved with the arthritis lasting for weeks to months.
Rheumatoid arthritis (choice D) begins insidiously with fatigue, anorexia,
generalized weakness, and vague musculoskeletal symptoms leading up to the
appearance of synovitis. Pain in the affected joints, aggravated by movement, is the
most common manifestation of established rheumatoid arthritis. Generalized
stiffness is frequent and is usually greatest after periods of inactivity. Morning
stiffness of greater than 1 hour in duration is very characteristic. Rheumatoid
arthritis is more common in females. The metacarpophalangeal and proximal
interphalangeal joints of the hands are characteristically involved.
Septic arthritis (choice E) is caused by a variety of microorganisms, including
Neisseria gonorrhoeae and Staphylococcus aureus. Hematogenous spread is the
most common route in all age groups. 90% of patients present with involvement of
a single joint, usually the knee. The usual presentation is moderate-to-severe pain,
effusion, muscle spasm, and decreased range of motion. Peripheral leukocytosis and
a left shift are common. Disseminated gonococcal infections present as fever, chills,
rash, and articular symptoms. Papules progressing to hemorrhagic pustules develop
on the trunk and extensor surfaces of the distal extremities. Migratory arthritis and
tenosynovitis of multiple joints is common.
________________________________________
16>The correct answer is D. Many people who request physician-assisted suicide
have one of two conditions present: either a poorly controlled painful condition or
severe depression. If the painful condition is adequately treated or the depression is
brought under good medical control, the request for physician assistance in
terminating the situation is typically withdrawn. It is important to note that bringing
these conditions under control requires the intervention of caregivers who are
specifically trained in the management of these two conditions; primary care
physicians usually are not adequately trained to address these difficult
presentations.
While patients who are diagnosed as bipolar disorder (choice A), borderline
personality disorder (choice B), and schizophrenic disorder (choice E) often make
suicide attempts (and frequently complete those attempts), they do not generally
ask their physician for assistance in the suicide.
Persons with factitious disorder (choice C) are seeking primary gain, often for
dependency needs, and are seeking to enter the "sick role" not the "dead role."
________________________________________

17>The correct answer is E. The patient has bilateral acoustic neuromas, probably
due to neurofibromatosis type II (over 90% of patients with NF-2 develop bilateral
acoustic neuromas). This condition is a associated with the NF-2, gene, located on
22q (note all the 2's). Patients often develop meningiomas, gliomas, and
schwannomas of cranial and spinal nerves.
5q (choice A) contains the APC tumor suppressor gene, which is associated with
familial and sporadic colorectal cancers.
13q (choice B) contains the Rb tumor suppressor gene, which is associated with
retinoblastoma and osteosarcoma.
17q (choice C) contains both the NF-1 tumor suppressor gene, which is associated
with neurofibromatosis type I, and the p53 tumor suppressor gene, associated with
many human cancers.
18q (choice D) contains both the DCC gene, which is associated with colon and
gastric carcinomas and the DPC gene, associated with pancreatic cancer.

________________________________________
18>
The correct answer is D. Perforation of a peptic ulcer is potentially fatal, because of
either peritonitis with sepsis or sudden exsanguination (if the perforation damages
one of the many arteries of the stomach). Peptic ulcer disease, gastritis, and
possibly gastric carcinoma and gastric lymphoma have been strongly associated
with Helicobacter pylori colonization of the mucus layer covering the gastric
mucosa. Colonization is associated with destruction of the mucus layer, thereby
destroying its protective function.
Cryptosporidium parvum (choice A) causes diarrhea that is severe in
immunocompromised patients.
Entamoeba histolytica (choice B) produces dysentery-like symptoms or can cause
liver abscess.
Escherichia coli (choice C) causes a variety of diarrheal diseases and can infect the
bladder and soft tissues.
Mycobacterium tuberculosis (choice E) causes tuberculosis, characterized by
granuloma formation, especially in the lungs.
________________________________________
19>The correct answer is E. By the third week of development, hematopoiesis
begins in the blood islands of the yolk sac. Beginning at 1 month of age and
continuing until 7 months of age, blood elements are also formed in the liver.
Hematopoiesis occurs in the spleen and lymphatic organs between 2 and 4 months,
and in the bone marrow after 4 months.

________________________________________

15>The correct answer is C. The patient is suffering from schizophrenia. The key to
the diagnosis of psychosis is that there has been a marked decline in the level of
functioning (i.e., the man is homeless and cannot care for himself). Although
hallucinations or delusions are not mentioned in the case history, the presence of
disorganized speech, grossly disorganized behavior, and the duration of symptoms
(longer than six months) suggest a diagnosis of schizophrenia.
In schizoaffective disorder (choice A), alterations in mood are present during a
substantial portion of the illness.
Although schizoid personality disorder (choice B) produces detachment from social
relationships and is characterized by restriction of emotional expression, it is not
accompanied by a marked decline in occupational functioning.
Schizophreniform disorder (choice D) is characterized by schizophrenic-like
symptoms, but the duration of symptoms is, by definition, less than six months.
Schizotypal personality disorder (choice E) is characterized by eccentricities of
behavior, odd beliefs or magical thinking, and difficulties with social and
interpersonal relationships. Unlike schizophrenia, schizotypal personality disorder is
not characterized by a formal thought disorder.

________________________________________

________________________________________
21>The correct answer is E. Familial hypercholesterolemia, which is due to defective
function of the LDL receptor, is an area of intense research. The molecular basis of
LDL receptor abnormalities is becoming better understood, and more than 200
mutations in the gene for the LDL receptor have been identified. The gene has 5
general domains and 18 exons. Defects near exons 7 to 14 (including this case) are
in the region of homology with epidermal growth factor receptor precursor. This
region of the molecule is needed for dissociation of LDL from the receptor in the
endosome. Receptors with a defect in this area (sometimes called class II
mutations) also have trouble being initially transported to the Golgi complex
(transport-deficiency alleles) and become trapped in endoplasmic reticulum.
Decreased transcription of the LDL receptor gene (choice A) is considered a class I
mutation and involves the signal sequence domain near exon 1.

Poor internalization of LDL bound to LDL receptor (choice B) is considered a class IV


mutation. Such mutations are associated with the membrane-spanning/cytoplasmic
domain, specifically near exon 18.
Poor retention of the LDL receptor in the membrane (choice C) is considered a class
IV mutation and is associated with the membrane-spanning/cytoplasmic domain,
specifically near exons 2-6.
Reduced binding of LDL (choice D) is considered a class III mutation and involves
the LDL binding domain near exons 2-6.
________________________________________
22>The correct answer is D. Respiratory distress immediately follows amniotic fluid
embolism as the emboli consisting of squamous cells, lanugo, and mucus deposit in
the pulmonary microcirculation, producing numerous tiny pulmonary infarcts. The
dramatic respiratory distress may also reflect the action of prostaglandins and other
bioactive compounds present in high concentrations in the amniotic fluid embolus.
Hemiplegia (choice A) would reflect an ischemic injury to one hemisphere of the
cerebrum or the brainstem. A venous embolus would not produce such an insult.
Placental abruption (choice B) is partial, premature separation of the placental disc
from the endometrium. Although abruption may occur in this setting, it is not a
result of an amniotic fluid embolism.
There are numerous causes of renal failure (choice C); the most likely ones in the
peripartum interval include eclampsia, hypovolemic shock, and ascending
infections. Amniotic fluid embolism would be expected to produce severe dyspnea
well before shock and renal failure might arise.
Splinter hemorrhages (choice E) are small hemorrhages seen on toes and fingers
due to a shower of microemboli arising in the arterial circulation. Amniotic fluid
emboli arise in the veins and deposit in the lungs.
________________________________________
23>The correct answer is D. The patient has a megaloblastic anemia, which can be
due to deficiency of folate or B12. Pregnancy increases the need for folate and other
nutrients used by both baby and mother, and may "unmask" a borderline dietary
deficiency. For this reason, most obstetricians recommend vitamin supplements for
pregnant women.
Ascorbic acid (choice A) is vitamin C, and its deficiency predisposes for capillary
fragility and oral lesions.
Calcium deficiency (choice B) predisposes for osteoporosis/osteopenia.
Copper deficiency (choice C) is rare; when it occurs, it may cause a hypochromic
anemia, neutropenia, osteoporosis, or hypotonia.
Iron deficiency (choice E) causes a microcytic, hypochromic anemia, with reduced
mental and physical performance.

________________________________________
24>The correct answer is D. The lesions are characteristic of molluscum
contagiosum, which is a typically benign and self-limited condition caused by a
poxvirus. The disease can be transmitted either venereally or through non-venereal
contact. The other viruses listed do not cause similar skin lesions. Patients with
advanced HIV infection may develop a severe, generalized, and persistent eruption,
often involving the face and upper body.
Cytomegalovirus (choice A) causes congenital infections and disseminated
infections in immunosuppressed patients.
Herpesvirus 6 (choice B)causes roseola (exanthem subitum).
Parvovirus (choice C) causes aplastic crises in patients with hemolytic anemia.
Variola (choice E) is the smallpox virus.
________________________________________
25>The correct answer is D. This is a relatively simple question that requires you to
visualize the relationship among the key parts of the kidney and to identify the one
that lies most medially. If you think about it for a second, since the kidneys
ultimately drain into the ureter at their medial poles, you are looking for the
structure that is closest to the ureter. The correct answer is the renal pelvis. The
renal pelvis is the dilated upper portion of the ureter that receives the major
calyces.
In terms of the other answer choices, the order from most lateral to most medial is:
renal cortex (choice C), renal pyramid (choice E), minor calyx (choice B), major
calyx (choice A), and then the renal pelvis (choice D).
________________________________________

26>
The correct answer is A. Cyclophosphamide is metabolized to acrolein, which is
excreted in the urine. If the patient's urine is concentrated, the toxic metabolite
may cause severe bladder damage. Early symptoms of bladder toxicity include
dysuria and frequency. This can be distinguished from a urinary tract infection, since
there is no bacteriuria with cyclophosphamide-induced bladder toxicity. However,
microscopic hematuria is often present on urinalysis. In severe hemorrhagic cystitis,
large segments of the bladder mucosa may be shed which can lead to prolonged,
gross hematuria. The incidence of cyclophosphamide-induced hemorrhagic cystitis
can be decreased by ensuring that the patient maintains a high fluid intake.
Cyclophosphamide is an alkylating agent used in the treatment of breast carcinoma,
malignant lymphoma, multiple myeloma, and adenocarcinoma of the ovary, as well
as various other forms of cancer. The major toxic reactions commonly seen with this
agent include mucositis, nausea, hepatotoxicity, sterile hemorrhagic and nonhemorrhagic cystitis, leukopenia, neutropenia, and interstitial pulmonary fibrosis.

Mitomycin (choice B) is an antibiotic antineoplastic agent used in the treatment of


breast carcinoma, adenocarcinoma of the pancreas and stomach, as well as various
other forms of cancer. The major toxic reactions commonly seen with this agent
include bone marrow depression, nausea, hepatotoxicity, acute bronchospasm,
thrombocytopenia, and interstitial pneumonitis.
Paclitaxel (choice C) is an antineoplastic agent primarily used in the treatment of
ovarian and breast cancer. The major toxic reactions commonly seen with this agent
include bone marrow depression, nausea, hepatotoxicity, bronchospasm,
thrombocytopenia, and neutropenia.
Tamoxifen (choice D) is an antineoplastic hormone primarily used in the palliative
treatment of estrogen-receptor positive breast cancer patients. The major toxic
reactions commonly seen with this agent include depression, dizziness, thrombosis,
mild leukopenia or thrombocytopenia.
Vincristine (choice E) is a mitotic inhibitor antineoplastic agent used in the
treatment of breast cancer, Hodgkin's disease, non-Hodgkin's lymphoma, advanced
testicular cancer and various other types of cancer. The major toxic reactions
commonly seen with this agent include mental depression, hemorrhagic
enterocolitis, bone marrow depression, nausea, thrombocytopenia, and leukopenia
________________________________________
27>The correct answer is A. In a syndrome called cleidocranial dysostosis, absence
of part of the clavicles accompanies a broad skull, and facial and dental anomalies.
Note that you could also have answered this question by noting that of the bones
listed, only the clavicles form by intramembranous ossification.
The femurs (choice B), metatarsals (choice C), phalanges (choice D), and tibias
(choice E) are cartilaginous (formed by endochondral ossification) rather than
membranous bones.
________________________________________
28>The correct answer is D. The tumor is a neuroblastoma, which is one of the
principal forms of cancer in children. Neuroblastoma typically occurs before age 5,
with many presenting before age 2. Neuroblastoma can arise from neural crest cells
throughout the body, but the adrenal medulla is the most common site. HomerWright pseudorosettes are circles of tumor cells with central young nerve fibers
arising from the tumor cells. The oncogene associated with neuroblastoma is N-myc.
erb-B2 (choice A) is associated with breast, ovarian, and gastric carcinomas.
c-myc (choice B) is associated with Burkitt's lymphoma.
L-myc (choice C) is associated with small cell carcinoma of the lung.
ret (choice E) is associated with multiple endocrine neoplasia, types II and III.
________________________________________

29>The correct answer is B. The disease is von Hippel-Landau disease, which is


associated with a deletion involving the VHL gene on chromosome 3 (3p). Affected
individuals develop vascular tumors (hemangioblastomas) of the retina, cerebellum,
and/or medulla. Roughly half of the affected individuals later develop multiple,
bilateral renal cell carcinomas.
Berry aneurysms (choice A) are unrelated to hemangioblastomas, but are instead
associated with adult polycystic disease.
Peripheral nerve cancers (choice C) are a feature of von Recklinghausen's disease
(neurofibromatosis type I).
Choreiform movements, related to decreased GABA and acetylcholine (choice D),
are a feature of Huntington's disease.
Extremely high serum cholesterol (choice E) suggests the homozygous form of
familial hypercholesterolemia.
________________________________________
30>The correct answer is A. The space constant of an axon reflects the amount of
passive or electrotonic spread of current within an axon. The larger the space
constant, the further the current can spread, allowing action potentials to propagate
faster. This is why myelin increases the conduction velocity of action potentials
down an axon. Conversely, demyelination decreases the space constant and slows
action potential conduction.
________________________________________
31>The correct answer is A. Coarctation of the aorta occurs in two patterns. In the
infantile type, the stenosis is proximal to the insertion of the ductus arteriosus
(preductal); this pattern is associated with Turner's syndrome. In the adult form, the
stenosis is distal to the ductus arteriosus (postductal) and is associated with
notching of the ribs (secondary to continued pressure from the aorta on them),
hypertension in the upper extremities, and weak pulses in the lower extremities.
Headache, cold extremities, and lower extremity claudication with exercise are
typical if the patient is symptomatic (many adults with mild distal coarctation may
remain asymptomatic for years). Upper extremity hypertension with weak pulses in
the lower extremities, and a midsystolic (or continuous) murmur over the chest or
back may be the only obvious signs in some. Note that the chronic cough is
probably related to the man's smoking, and is not caused by the coarctation.
Eisenmenger's syndrome (choice B) is a shift from a left-to-right shunt to a right-toleft shunt secondary to pulmonary hypertension.
Tetralogy of Fallot (choice C) and transposition of great arteries (choice D) cause
cyanosis and are usually diagnosed in infancy.
Ventricular septal defect (choice E) might remain undiagnosed until adulthood, but
would not cause notching of the ribs.
________________________________________

32>The correct answer is C. The muscularis of the upper third of the esophagus
(choice E) is composed entirely of striated muscle. The middle third (choice C)
contains both striated and smooth muscle. The lower third (choice B) and lower
esophageal sphincter (choice A) contain only smooth muscle. There is no such thing
as the upper esophageal sphincter (choice D).
________________________________________
33>The correct answer is A. Bilirubin is a degradative product of hemoglobin
metabolism. Bilirubin (pigment) stones are specifically associated with excessive
bilirubin production in hemolytic anemias, including sickle cell anemia. Bilirubin
stones can also be seen in hepatic cirrhosis and liver fluke infestation.
Calcium oxalate stones (choice B) and cystine stones (choice E) are found in the
kidney, rather than the gallbladder.
Pure cholesterol stones (choice C) are less common than mixed gallstones, but have
the same risk factors, including obesity and multiple pregnancies.
Mixed stones (choice D) are the common "garden variety" gallstones, found
especially in obese, middle aged patients, with a female predominance
________________________________________

34>
The correct answer is A. Malnutrition associated with chronic alcoholism can lead to
a severe magnesium deficiency. The effect of low serum magnesium on parathyroid
hormone secretion (PTH) depends on severity and duration. An acute decrease in
serum magnesium will increase PTH secretion, while a prolonged severe deficiency
results in decreased PTH secretion. There is also evidence that the action of PTH is
decreased with chronic magnesium deficiency. Hence, this patient is suffering from
"functional" hypoparathyroidism. The low serum calcium can produce weakness,
tremors, muscle fasciculations, and seizures. A positive Trousseau's sign indicates
the presence of latent tetany. It is observed by inflating a blood pressure cuff above
systolic blood pressure for at least 2 minutes. A positive reaction consists of the
development of carpal spasm, with relaxation occurring within seconds after
deflating the cuff. In patients with magnesium deficiency, magnesium
administration will produce a prompt rise in plasma PTH with subsequent restoration
of serum calcium concentration to normal.
With functional hypoparathyroidism bone density would be decreased (not
increased, choice C).
The combination of decreased PTH secretion (not increased, choice D) and
decreased effectiveness of PTH produce hypocalcemia and hyperphosphatemia (not
hypophosphatemia, choice B).
Urinary cAMP would probably be decreased (not increased, choice E), given the low
PTH.

________________________________________
35>The correct answer is E. Enteropeptidase, formerly called enterokinase,
activates trypsinogen by limited proteolytic digestion to give trypsin. Trypsin is itself
capable of activating trypsinogen, which produces a positive feedback effect.
Trypsin also activates chymotrypsinogen (and several other proteolytic enzymes),
so deficiency of enteropeptidase results in a severe deficiency of enzymes that
digest protein.
Amylase (choice A) aids in the breakdown of starches to oligosaccharides, maltose,
and maltotriose.
Colipase (choice B), along with other lipases, functions to digest fats.
Lactase (choice C) is a brush-border disaccharidase that hydrolyzes the bond
between galactose and glucose in lactose.
Pepsin (choice D) is a proteolytic enzyme secreted in an inactive form (pepsinogen)
by the chief cells of the stomach. Pepsinogen is activated by stomach acid, and so is
not dependent on enteropeptidase. Pepsin alone will not replace the activities of
other proteolytic enzymes, partly because food does not remain in the stomach for
an extended period of time.
________________________________________
36>The correct answer is E. The child is likely suffering from cystic fibrosis. In this
disorder, an abnormality of chloride channels causes all exocrine secretions to be
much thicker, and more viscous than normal. Pancreatic secretion of digestive
enzymes is often severely impaired, with consequent steatorrhea and deficiency of
fat-soluble vitamins, including vitamin A.
Cystinuria (choice A) is a relatively common disorder in which a defective
transporter for dibasic amino acids (cystine, ornithine, lysine, arginine; COLA) leads
to saturation of the urine with cystine, which is not very soluble in urine, and
precipitates out to form stones.
Hypoglycemia (choice B) is not a prominent feature of children with cystic fibrosis
who are on a normal diet. Hyperglycemia may occur late in the course of the
disease.
Iron deficiency anemia (choice C) is not found with any regularity in children with
cystic fibrosis.
Sphingomyelin accumulation (choice D) is generally associated with deficiency of
sphingomyelinase, as seen in Niemann-Pick disease.
________________________________________
37>The correct answer is E. The ventral mesentery forms the falciform ligament,
ligamentum teres, and lesser omentum, which can be divided into the hepatogastric
and hepatoduodenal ligament.

The dorsal mesoduodenum (choice A) is the mesentery of the developing


duodenum, which later disappears so that the duodenum and pancreas lie
retroperitoneally.
Both omental bursa and the greater omentum are derived from the dorsal
mesogastrium (choice B), which is the mesentery of the stomach region.
The pericardioperitoneal canal (choice C) embryologically connects the thoracic and
peritoneal canals.
The pleuropericardial membranes (choice D) become the pericardium and
contribute to the diaphragm.
________________________________________
38>The correct answer is A. This man has a respiratory acidosis. Overdose with
drugs that suppress ventilation (e.g., heroin, morphine, barbiturates, methaqualone,
and "sleeping pills") often causes hypercapnia. In patients with an intact renal
response, the respiratory acidosis causes a compensatory rise in plasma HCO3-,
which lessens the fall in pH. However, the renal response requires several days to
develop fully. The plasma HCO3- of 26 mEq/L (normal: 22-28 mEq/L) for this man is
typical of acute respiratory acidosis with little or no renal compensation.
Choice B reflects metabolic acidosis.
Choice C is normal.
Choice D reflects respiratory alkalosis.
Choice E reflects metabolic alkalosis.
________________________________________
39>The correct answer is B. This is a case of tinea pedis, or athlete's foot, caused
by a variety of dermatophytic fungi, which are easily detected on alkali mounts of
scraped skin as colorless, branching hyphae with cross-walls.
Hyphae with rosettes of conidia (choice A) describes the
environmental/transmission form for Sporothrix schenckii, the agent of rose
gardener's disease, which is a subcutaneous mycosis.
Budding yeasts (choice C) describes the form found in clinical specimens from
patients with sporotrichosis.
Hyphae, arthroconidia, and blastoconidia (choice D) would be found in clinical
specimens from patients infected with Trichosporon beigelii (white piedra), which is
a superficial mycosis of the hair of the head.
Pigmented, septate hyphal fragments (choice E) would be found in cases of
phaeohyphomycosis, a diverse group of cyst-forming subcutaneous, pigmented
(dematiaceous) fungi, rare in the U.S.

Short, curved hyphae and round yeasts (choice F), or the "spaghetti and meatball"
presentation in clinical specimens, are characteristic of Malassezia furfur, the agent
of pityriasis versicolor.
________________________________________
40>The correct answer is C. This is a classic presentation of a patient with carpal
tunnel syndrome, which typically affects females between the ages of 40 and 60
who chronically perform repetitive tasks that involve movement of the structures
that pass through the carpal tunnel. One important structure that passes through
the carpal tunnel is the median nerve. Patients often note a tingling, a loss of
sensation, or diminished sensation in the digits. There is also often a loss of
coordination and strength in the thumb, because the median nerve also sends fibers
to the abductor pollicis brevis, flexor pollicis brevis, and the opponens pollicis. A
final function of the median nerve distal to the carpal tunnel is control of the first
and second lumbricals which function to flex digits two and three at the
metacarpophalangeal joints and extend interphalangeal joints of the same digits.
Abduction of the fifth digit (choice A) is a function controlled by the ulnar nerve,
which does not pass through the carpal tunnel.
Adduction of the thumb (choice B) is a function of the adductor pollicis, which is the
only short thumb muscle that is not innervated by the median nerve, but rather by
the deep branch of the ulnar nerve.
Sensation of the lateral half of the dorsum of the hand (choice D) is mediated by the
radial nerve, which also does not pass through the carpal tunnel.
Sensation over the lateral aspect of the palm (choice E) is mediated by the median
nerve, however the branch innervating the palm (palmar cutaneous branch of the
median nerve) passes superficial to the carpal tunnel.
Sensation over the medial aspect of the dorsum of the hand (choice F) is mediated
by the ulnar nerve.
Sensation over the medial aspect of the palm (choice G) is mediated by the ulnar
nerve.
________________________________________
41>The correct answer is C. The posterior fornix is in contact with the floor of the
rectouterine space. The rectouterine space is the lowest part of the peritoneal
cavity in the female pelvis. The patient has introduced bacteria into the peritoneal
cavity by the penetration of the sharp object, producing sepsis.
The deep perineal pouch (choice A) is the middle layer of the urogenital diaphragm
containing the sphincter urethrae muscle. The vagina passes through this region but
the posterior fornix is not related to it.
The ischioanal space (choice B) is below and lateral to the pelvic diaphragm. The
vagina does not pass through this space.

The rectovesical space (choice D) is the region in the peritoneal cavity of the male
pelvis between the urinary bladder and the rectum. This space does not exist in the
female pelvis.
The vesicouterine space (choice E) is the region within the peritoneal cavity of the
female pelvis between the urinary bladder and the uterus. This space is not related
to the posterior fornix of the vagina.
________________________________________
42>The correct answer is E. There are two zones in the lung-the conducting zone
(where there is no gas exchange) and the respiratory zone (where there is gas
exchange). Of all of the structures listed, only Type I epithelial cells are located in
the respiratory zone. Type I epithelial cells are the primary structural cell of the
alveolar wall. Type II epithelial cells, also located in the alveoli, are the cells that
produce surfactant.
Goblet cells (choice A), which are mucus-secreting cells, are present in the
conducting airways.
The main bronchi (choice B) are part of the conducting airways.
Mucous cells (choice C), which are mucus-secreting cells, are present in the
conducting airways.
Terminal bronchioles (choice D) are the most distal part of the conducting airways.
Respiratory bronchioles, which are just distal to the terminal bronchioles, are part of
the respiratory zone. These two types of bronchioles can be differentiated from each
other by whether they have alveoli budding from their walls. Respiratory bronchioles
have alveoli, terminal bronchioles do not.
________________________________________
43>The correct answer is D. The inguinal canal lies entirely above the inguinal
ligament. The deep inguinal ring is about one-half inch above the midpoint of the
inguinal ligament. The superficial inguinal ring is superolateral to the pubic tubercle.
Indirect inguinal hernias enter the inguinal canal through the deep inguinal ring.
The femoral ring is posterior to the inguinal ligament, lateral to the lacunar ligament
(choice A), and medial to the femoral vein (choice C). Femoral hernias pass through
the femoral ring.
The superficial inguinal ring is superomedial to the pubic tubercle (choice B).
Indirect and direct inguinal hernias pass through the superficial inguinal ring, but
only indirect inguinal hernias pass through the deep inguinal ring.
The inguinal canal is entirely above the inguinal ligament. The thigh is below the
inguinal ligament (choice E).
________________________________________
44>The correct answer is E. Potassium is responsible for maintenance of
intracellular tonicity, transmission of nerve pulses, contraction of muscle (striated

and smooth), and maintenance of renal function. The normal blood level of
potassium ranges from 3.5 to 5.0 mEq/L. In potassium depletion, a decrease in the
blood potassium level by 1 mEq/L equals a loss of 100-200 mEq from potassium
stores in the body. Depletion can result in the development of muscular weakness,
paralysis, and mental confusion. Since this patient is borderline hypokalemic, she
should receive a potassium-sparing diuretic, such as spironolactone, amiloride, or
triamterene. These agents are all indicated for the treatment of edematous states
as well as the prophylaxis and treatment of hypokalemia. These agents are
commonly combined with other non-potassium-sparing diuretics to prevent the
appearance of hypokalemia during therapy.
Furosemide (choice A) is a loop diuretic indicated for the treatment of edematous
states in hypertension and is commonly associated with the development of
hypokalemia.
Hydrochlorothiazide (choice B), indapamide (choice C), and metolazone (choice D)
are thiazide diuretics indicated for the treatment of edematous states in
hypertension and are also commonly associated with the development of
hypokalemia.
________________________________________
45>The correct answer is C. The boy probably has Klinefelter's syndrome (47, XXY),
which has the typical presentation described in the question. The condition arises as
a result of failure of separation (nondisjunction) of the sex chromosomes, and can
be related to either paternal nondisjunction (slightly more common) or maternal
nondisjunction.
Deletions (choice A) are a common form of genetic disease and contribute to many
genetic recessive diseases.
Examples of nondisjunction of autosomes (choice B) include trisomies such as most
cases of Down's syndrome (trisomy 21), Edwards' syndrome (trisomy 18), and Patau
syndrome (trisomy 13).
There are two types of translocations: non-Robertsonian (choice D) and
Robertsonian (choice E). Non-Robertsonian (reciprocal) translocations result when
two non-homologous chromosomes exchange genetic material. Robertsonian
translocations are a special type of translocation that involve exchange of genetic
material from the long arms of one acrocentric chromosome to the long arms of
another acrocentric chromosome, with fusion of the centromeres. Four percent of
cases of Down's syndrome are caused by this mechanism.
________________________________________
46>The correct answer is E. When the periosteum is torn during a fracture, it
supplies cells that develop into osteoblasts and are the major producers of the new
bone that reunites the two ends. Heterotopic ossification (bone formed outside the
regular bone) can occur as a complication of fracture if some of the osteoblastic
cells are misdirected into adjacent tissues.

Preexisting cancellous bone (choice A) and compact bone (choice C) are not the
major source of osteoblasts that form the new bone.
Cartilage (choice B) and marrow (choice D) do not contribute to new bone formation
after fracture.
________________________________________
47>The correct answer is D. The cell illustrated is a Reed-Sternberg cell. These cells
characteristically are neoplastic giant cells with a bilobed mirror-image nucleus that
may have large ("owl-eyes") nucleoli surrounded by a clear halo. Most commonly,
these cells are associated with Hodgkin's disease, but it is worth knowing that in
real life, a surprising variety of other lymphomas and some other diseases may
have at least occasional Reed-Sternberg look-alikes. For that reason, the diagnosis
of Hodgkin's disease often requires confirmatory immunohistochemical staining.
The Langhans cell (choice A) is a giant cell with peripherally arranged nuclei found
in granulomas.
The LE cell (choice B) is a degenerating neutrophil seen in lupus erythematosus in in
vitro preparations.
The Mott cell (choice C) is a "constipated" plasma cell filled with immunoglobulins,
producing a grape-cluster appearance to the cell. Mott cells are seen in multiple
myeloma and some parasitic infestations.
The Touton giant cell (choice E) has distributed nuclei and is found in tumors other
than Hodgkin's lymphoma.
________________________________________
48>The correct answer is D. The subclavian artery passes laterally over the upper
surface of the first rib and lies posterior to the scalenus anterior. In the case of
thoracic outlet syndrome, this artery is usually compressed between the scalenus
anterior and a cervical rib. Thoracic outlet syndrome is a broad term for a group of
disorders in which there is compression of certain neurovascular bundles. The
presence of a cervical rib adds to the compression, and repetitive motion and poor
posture are other predisposing factors. When the neurovascular bundle is
entrapped, the patient presents with neurological and/or circulatory changes in the
upper extremity on the involved side.
The axillary artery (choice A) is the continuation of the subclavian artery. This artery
begins at the outer border of the first rib and continues through the axilla to the
lower border of the teres major.
The brachial artery (choice B) is the continuation of the axillary artery in the upper
extremity and can not be compressed by a cervical rib.
The brachiocephalic artery (choice C) arises from the arch of the aorta. At the level
of the right sternoclavicular joint, it separates into the right common carotid artery
and the right subclavian artery.

The subscapular artery (choice E) is the largest branch of the axillary artery. It
supplies muscles on the lateral thoracic wall and scapular muscles.
________________________________________
49>The correct answer is C. The left adrenal vein and the left gonadal vein (either
testicular or ovarian) drain into the left renal vein. The left renal vein then drains
into the inferior vena cava. In contrast, the right adrenal vein and right gonadal vein
drain directly into the inferior vena cava.
The hemiazygos vein (choice A) receives the venous drainage from the body wall on
the left side of the thorax and abdomen. No visceral organs drain directly to the
azygos or hemiazygos veins.
The inferior vena cava (choice B) receives the direct venous drainage from the right
adrenal vein, but not the left adrenal vein. Remember, the inferior vena cava is on
the right side of the abdomen.
The splenic vein (choice D) receives the venous drainage from the spleen and part
of the pancreas and stomach. The splenic vein is part of the portal venous system.
The superior mesenteric vein (choice E) receives venous drainage from much of the
intestinal tract. It is part of the portal venous system and joins with the splenic vein
to form the portal vein.
________________________________________
50>The correct answer is C. Both pyruvate kinase deficiency and glucose-6phosphate dehydrogenase deficiency are red cell enzyme deficiencies characterized
clinically by long "normal" periods interspersed with episodes of hemolytic anemia
triggered by infections and oxidant drug injury (antimalarial drugs, sulfonamides,
nitrofurans). In both of these conditions, the cell morphology between hemolytic
episodes is usually normal or close to normal.
The a (choice A) and b (choice B) thalassemias, in their major forms, are
characterized by persistent severe anemia. In the trait forms, they are
charactertized by mild anemia.
Hereditary spherocytosis (choice D) is characterized by intermittent hemolysis, but,
unlike pyruvate kinase deficiency and glucose-6-phosphate dehydrogenase
deficiency, oxidant drugs are not a specific trigger for hemolysis.
Iron deficiency anemia (choice E) is characterized by chronic anemia with
hypochromic, microcytic erythrocytes.
________________________________________

1The correct answer is A. Fascial straps (retinacula) and fascial coverings of muscles
or muscle groups characteristically attach to nearby bones by blending with the
covering periosteum. No deep attachments are usually made by fascia.

Cancellous bone (choice B) is spongy bone, which is usually found in marrow, and is
not the site for fascial attachment.
Fascia do not usually attach to cartilage (choice C).
Fascia attaches to bony shafts, or diaphyses (choice D), superficially via the
periosteum.
Fascia do not penetrate the bone to reach the marrow (choice E).
________________________________________

2The correct answer is E. Uric acid kidney stones in patients with leukemia are
secondary to increased production of uric acid from purine breakdown during
periods of active cell proliferation, especially following treatment. Vigorous
hydration and diuresis are generally instituted after the diagnosis of acute leukemia
is made. Uric acid kidney stones are also associated with inborn errors of purine
metabolism, such as gout.
Pigment gallstones (choice A) are associated with hemolytic disease. The incidence
of this type of gallstone is not increased in treated leukemias.
Cholesterol gallstones (choice B) are associated with diabetes mellitus, obesity,
pregnancy, birth control pills, and celiac disease.
Cystine kidney stones (choice C) are rare; they are found in cystinuria.
Struvite kidney stones (choice D) are associated with infection by urea-splitting
organisms, such as Proteus.
________________________________________

3The correct answer is A. The patient is clearly presenting with signs and symptoms
of acute angle-closure glaucoma. Primary acute angle-closure glaucoma occurs
because of closure of a preexisting narrow anterior chamber angle, as is commonly
found in the elderly, hyperopes, and Asians. Patients often seek immediate medical
attention because of the intense pain and blurred vision. The blurred vision is
characteristically associated with halos around lights. The eye is often very red and
steamy, and the pupil is dilated and nonreactive to light; tonometry reveals
elevated intraocular pressure. The treatment considerations are as follows:
immediate lowering of intraocular pressure (IOP) is achieved with a single dose of
500 mg IV acetazolamide, followed by 250 mg PO qid. Osmotic diuretics such as
oral glycerol and IV urea or mannitol may also be used. Acetazolamide is an agent
that inhibits the enzyme carbonic anhydrase, leading to reduced production of
aqueous humor and a concomitant reduction in IOP.
Dorzolamide (choice B) is also a carbonic anhydrase inhibitor. This agent is
indicated for the chronic lowering of IOP in patients with open-angle glaucoma.

Epinephrine (choice C) is indicated for lowering of IOP in patients with open-angle


glaucoma in combination with miotics, beta blockers, hyperosmotics, or carbonic
anhydrase inhibitors. However, this agent is contraindicated in patients with narrowangle glaucoma.
Latanoprost (choice D) is a prostaglandin F2 analog that is believed to reduce IOP by
increasing the outflow of aqueous humor. It is indicated for lowering IOP in patients
with open-angle glaucoma and ocular hypertension who are intolerant to other
agents.
Timolol (choice E) is a beta adrenergic receptor antagonist that has peak ocular
hypotensive effects at 1-2 hours post-dosing. This agent decreases IOP with little or
no effect on pupil size or accommodation. It is indicated for chronic lowering of IOP
in patients with open-angle glaucoma.
________________________________________
4The correct answer is D. Wernicke-Korsakoff syndrome refers to the constellation of
neurologic symptoms caused by thiamine deficiency. Among these, a severe
memory deficit, which the patient may attempt to cover by making up bizarre
explanations (confabulation), is prominent. Anatomical damage to the mamillary
bodies and periventricular structures has been postulated as the cause. In the U.S.,
severe thiamine deficiency is seen most commonly in chronic alcoholics. Thiamine
deficiency can also damage peripheral nerves ("dry" beriberi) and the heart ("wet"
beriberi).
Folic acid deficiency (choice A) produces megaloblastic anemia without neurologic
symptoms.
Niacin deficiency (choice B) produces pellagra, characterized by depigmenting
dermatitis, chronic diarrhea, and anemia.
Riboflavin deficiency (choice C) produces ariboflavinosis, characterized by glossitis,
corneal opacities, dermatitis, and erythroid hyperplasia.
Vitamin B12 deficiency (choice E) produces megaloblastic anemia accompanied by
degeneration of the posterolateral spinal cord.
________________________________________

5The correct answer is C. The secretion of estrogen by the developing follicle can
best be explained using the "two cell" hypothesis. Theca cells are stimulated by LH
(theca cells express LH receptors prior to formation of the corpus luteum, choice D)
to secrete the androgens androstenedione and testosterone. The androgens then
diffuse into the granulosa cells, where they are aromatized to estrogens. Hence,
theca cells express side-chain cleavage enzyme (first step in steroidogenesis) prior
to the formation of the corpus luteum (choice E). FSH stimulates aromatase activity
in the granulosa cells (receptors for FSH and aromatase enzyme are present prior to
the formation of the corpus luteum, choices A and B). The granulosa cells
apparently have the ability to produce steroids (progesterone), but lack 17a-

hydroxylase activity and cannot synthesize estrogen themselves. Only as the follicle
approaches ovulation do LH receptors begin to be expressed by the granulosa cells.
Estrogen and FSH probably are responsible for the change. After ovulation, the scar
of the follicle undergoes luteinization. The theca cells decrease 17a-hydroxylase
activity and secrete more progesterone. The granulosa cells decrease aromatase
activity and also secrete more progesterone.
________________________________________
6The correct answer is B. Although it is far better to rely on microbiology lab results
than physical exam and history, some classic clues for an infection with Klebsiella
pneumoniae are a patient who is an elderly alcoholic or diabetic, "currant-jelly"
sputum (containing blood clots), and lobar pneumonia.
Hemophilus influenzae (choice A) is a frequent cause of community-acquired
pneumonia, but does not classically produce lobar pneumonia or currant-jelly
sputum.
Pseudomonas aeruginosa (choice C) classically produces greenish sputum, and is
associated with cystic fibrosis rather than alcoholism.
Staphylococcus aureus (choice D) often produces pulmonary abscess, but may also
produce an ordinary bronchopneumonia.
Streptococcus pneumoniae (choice E) is a classic cause of lobar pneumonia, but
does not produce currant-jelly sputum or show a particular predilection for
alcoholics.
________________________________________
7The correct answer is E. The chef had a staphylococcal abscess on his finger. S.
aureus, produces enterotoxin A, which was likely present in the cream pies. When
ingested, the toxin causes severe nausea and vomiting within a few hours (the
average incubation time is 3-6 hours). There is little diarrhea associated with this
type of food poisoning outbreak.
Bacillus cereus (choice A) is a gram-positive spore-forming rod that is associated
with food poisoning outbreaks following the ingestion of fried rice. The time of onset
and symptoms would mimic staphylococcal disease; the major differentiating
feature is the food involved. The organism survives the boiling of the rice because it
is a spore-former. It germinates as the rice cools, grows, and elaborates an
enterotoxin that is responsible for the nausea and vomiting characteristic of the
disease.
Clostridium perfringens (choice B) is a gram-positive spore-forming anaerobe that
can cause a longer incubation (18-24 hour) food poisoning, typically with marked
diarrhea. Once again, the spores allow the organism to survive the heating process
used in the preparation of the food. Both Clostridium perfringens and Clostridium
botulinum are associated with home-canned vegetable and sausages.
The symptoms of Escherichia coli (choice C) food poisoning are usually watery
diarrhea (traveler's diarrhea) with minimal nausea and vomiting, or a bloody

diarrhea caused by enteroinvasive strains of the agent. Also, E. coli would be an


unlikely cause of the primary infection in the chef.
Shigella sonnei (choice D) causes enterocolitis characterized by fever, cramps, and
diarrhea after an incubation period of one to four days. Transmission is fecal-oral,
associated with poor hygiene. A wide range of foods has been implicated.
________________________________________
8The correct answer is C. The neurohypophysis (posterior pituitary) is derived from
an evagination of diencephalic neurectoderm. This structure is responsible for
releasing oxytocin and vasopressin to the general circulation. Both hormones are
synthesized in cell bodies contained within the hypothalamus.
ACTH (choice A), prolactin (choice D), and TSH (choice E) are all synthesized and
released by the anterior pituitary, or adenohypophysis, which is derived from an
evagination of the ectoderm of Rathke's pouch, a diverticulum of the primitive
mouth. Remnants of this pouch may give rise to a craniopharyngioma in later life.
Epinephrine (choice B) is synthesized and released into the circulation by the
adrenal medulla, a neural crest derivative.
________________________________________
9The correct answer is E. The immunoperoxidase method uses horseradish
peroxidase to produce a visible pigment when a specific antibody binds to antigenic
sites in the tissue. Binding of the antibody to the epithelial cells indicates that the
protein in question is being expressed by those cells. The positive
immunoperoxidase results obtained here suggests that the channel protein is
expressed in proximal convoluted tubular epithelium, since this is the only site in
the kidney at which the epithelial cells have a "brush border." The brush border is
made of microvilli, which enhance the proximal tubules' ability to reabsorb plasma
constituents filtered at the glomeruli.
________________________________________
10The correct answer is B. The principle host defense in mycobacterial infections
(such as this patient's tuberculosis) is cell-mediated immunity, which causes
formation of granulomas. Unfortunately, in tuberculosis and in many other
infectious diseases characterized by granuloma formation, the organisms may
persist intracellularly for years in the granulomas, only to be a source of activation
of the infection up to decades later.
While antibody-mediated phagocytosis (choice A) is a major host defense against
many bacteria, it is not the principle defense against Mycobacteria.
IgA-mediated hypersensitivity (choice C) is not involved in the body's defense
against Mycobacteria.
IgE-mediated hypersensitivity (choice D) is not involved in the body's defense
against Mycobacteria. It is important in allergic reactions.

Neutrophil ingestion of bacteria (choice E) is a major host defense against bacteria,


but is not the principle defense against Mycobacteria.
________________________________________
11The correct answer is A. Because of the proximity of the axillary nerve to the
glenohumeral joint, a fracture of the surgical neck of the humerus or an inferior
dislocation of the humerus could damage the nerve. The axillary nerve innervates
the deltoid muscle. The deltoid abducts, adducts, flexes, extends, and rotates the
arm medially. The axillary nerve also innervates the teres minor, which rotates the
arm laterally.
The dorsal scapular nerve (choice B) innervates both the rhomboid major and minor.
These muscles raise the medial border of the scapula upward and laterally.
The radial nerve (choice C) innervates muscles involved in the movement of the
forearm and hand.
The suprascapular nerve (choice D) innervates the supraspinatus and infraspinatus.
The supraspinatus abducts the arm, while the infraspinatus rotates the arm laterally.
This nerve travels along the posterior aspect of the scapula and would not easily be
subjected to injury in a dislocation of the shoulder joint.
The thoracodorsal nerve (choice E) innervates the latissimus dorsi, which adducts,
extends, and rotates the arm medially.
________________________________________
12The correct answer is C. Carcinomas of the bladder and renal pelvis are usually
transitional cell (choices D and E) carcinomas. However, Schistosoma haematobium
infection (where Schistosomes lay eggs in the veins near the bladder, thereby
inducing a marked inflammatory response) is associated with squamous metaplasia
and squamous cell carcinoma of the bladder. Some authors have suggested that
medications used to kill the worms may contribute to the etiology. Adenocarcinomas
of the renal pelvis and bladder (choices A and B) are rare
________________________________________
13The correct answer is C. You should associate Ashkenazic (Eastern European) Jews
with two diseases: Tay-Sachs disease and Type I Gaucher's disease. Both of these
diseases are sphingolipidoses. Tay-Sachs disease is the more devastating of the two,
and is characterized by progressive neurologic (including visual) deterioration
beginning at about 6 months of age and leading to death by age 3. In contrast, Type
I Gaucher's disease is compatible with a normal life span and causes
hepatosplenomegaly with CNS involvement. (The infantile Type II and the juvenile
Type III forms cause more serious disease but are not seen with increased incidence
in Ashkenazic Jews.) None of the other conditions listed occur with greater
frequency in Ashkenazic Jews. In this case, also note that many perfectly normal
children cannot sit without support at 6 months of age, so the child may well be
healthy.
________________________________________

14The correct answer is E. Inequalities of ventilation and perfusion contribute to


hypoxia in many settings. In this case, blood goes to both lungs (perfusion), but air
is prevented from entering one of the lungs (ventilation). Because the right lung is
being perfused, but not ventilated, hypoxemia ensues when the deoxygenated
blood from the right lung mixes with oxygenated blood from the left lung. If the
inadequate ventilation of the lung persists long enough, the lung tissue itself can be
damaged, causing a secondary local dilation of arterioles, making the problem even
worse. Peanuts are notorious for producing this type of problem in young children
because of their size and shape, which allows them to lodge in the trachea or main
bronchus after aspiration.
Decreased diffusion capacity (choice A) can occur when the blood-gas barrier is
thickened (e.g., diffuse interstitial fibrosis, sarcoidosis, asbestosis, respiratory
distress syndrome), when the surface area of the blood-gas barrier is reduced (e.g.,
pneumonectomy, emphysema), or when less hemoglobin is available to carry
oxygen (e.g., anemia, pulmonary embolism).
Decreased PO2 in inspired air (choice B) is seen at high altitudes and when the
settings are wrong during artificial ventilation.
Hypoventilation of central origin (choice C) is seen in morphine and barbiturate
overdose.
Hypoventilation of peripheral origin (choice D) is seen in poliomyelitis and chest
trauma.
________________________________________
15The correct answer is C. The hallmark of osteomalacia is widened osteoid seams.
Although the trabeculae are normal in number and size, they do not mineralize
effectively, and the rim of uncalcified osteoid is much larger than normal.
Osteoclast activity is normal in osteomalacia. Increased osteoclast activity (choice
B) is typical of hyperparathyroidism and Paget's disease of bone.
There are increased numbers of osteoblasts in osteomalacia (compare to choice A),
which lay down increased quantities of osteoid in an effort to strengthen the bone.
Unfortunately, the increased osteoid is not mineralized and the bone remains soft
and weak.
The bone marrow space is normal in osteomalacia. Marrow fibrosis (choice D) is
more typical of hematopoietic disorders, although hyperparathyroidism may also
lead to bone marrow fibrosis.
Although poorly mineralized, the bony trabeculae are of normal abundance in
osteomalacia. Sparse trabeculae (choice E) is characteristic of osteoporosis
________________________________________
16The correct answer is A. The infectious agents listed are all important causes of
congenital disease. The triad of cutaneous hemorrhages ("blueberry muffin baby"),
deafness, and periventricular CNS calcifications suggests congenital CMV infection,

the most common cause of intrauterine fetal viral infection. Other manifestations
include microcephaly and hepatosplenomegaly.
Neonatal herpes (choice B) may be congenital, but more commonly is acquired
during vaginal delivery. The infection is characterized by vesicles on the skin and
mucous membranes, encephalitis, or disseminated disease.
Congenital rubella (choice C) can cause mental retardation, heart abnormalities,
blindness, encephalitis, and motor abnormalities.
Congenital syphilis (choice D) can cause death in utero, or a variety of problems
including abnormal teeth, bones, and central nervous system.
Toxoplasmosis (choice E) can be either acquired during delivery (mild) or congenital
(severe). Severe infections can cause stillbirth, chorioretinitis, intracerebral
calcifications, and hydro- or microcephaly.
________________________________________
17The correct answer is E. Factor XII is unusual among coagulation factors in that its
deficiency is associated with thrombosis rather than hemorrhage. The mechanism
appears to be a deficient activation of fibrinolysis, and both thrombophlebitis and
myocardial infarction have occurred in severely affected patients. The condition is
inherited in an autosomal recessive manner. Many patients with mild-to-moderate
factor XII deficiency are never detected; others are identified when a routine
preoperative clotting screen demonstrates a greatly prolonged partial
thromboplastin time. Deficiency of each of the other factors (choices A, B, C, and D)
is associated with hemorrhage.
________________________________________
18The correct answer is C. Medullary thyroid carcinoma is a tumor that arises from
the parafollicular or "C" cells of the parathyroid gland. These are the cells that
release the calcium-lowering hormone, calcitonin. This tumor either is associated
with multiple endocrine neoplasia syndrome (MEN) 2a and 2b or can be sporadic.
Familial syndrome-associated tumors tend to occur in younger patients like this one.
Pheochromocytoma is also associated with MEN 2a and 2b, accounting for the
adrenal mass on CT scan.
Follicular thyroid carcinoma (choice A) arises from the follicular cells of the thyroid.
These cells normally produce thyroid hormone, but the neoplastic cells do not, so
there is no hormonal syndrome. The clinical symptoms of follicular carcinoma
usually involve neck mass, compression, or metastasis symptoms.
Hurthle cell carcinoma (choice B) is considered by some to be a variant of follicular
thyroid carcinoma. The follicular cells undergo acidophilic metaplasia and appear to
have abundant pink cytoplasm due to mitochondrial packing in the cytoplasm.
There are no hormonal changes associated with Hurthle cells.
Parathyroid carcinoma (choice D) is a rare neoplasm involving the chief cells of the
parathyroid gland. These cells produce parathyroid hormone, which increases serum

calcium. Patients usually present with a palpable anterior neck mass and very high
serum calcium levels.
Pheochromocytoma (choice E) arises from the neuroendocrine cells of the adrenal
medulla, which are related to sympathetic ganglia. Pheochromocytoma is mainly
sporadic, but 10% to 20% are associated with MEN 2a and 2b. Patients usually
present with hypertension due to excessive catecholamine secretion.
________________________________________
19The correct answer is C. The vast majority of sweat glands in the body are
innervated by sympathetic cholinergic neurons. Sympathetic cholinergic neurons
are sympathetic postganglionic neurons that happen to release acetylcholine
instead of norepinephrine.
Bradycardia (choice A), bronchoconstriction (choice B), and increased
gastrointestinal motility (choice D) would all result from stimulating
parasympathetic cholinergic neurons.
Increased peripheral vascular resistance (choice E) would result from stimulating
sympathetic adrenergic neurons.
________________________________________
20The correct answer is B. Hyperlipidemia has been subclassified based on the lipid
and lipoprotein profiles. Type 2a, which this patient has, can be seen in a hereditary
form, known as familial hypercholesterolemia, and also in secondary, acquired
forms related to nephritic syndrome and hyperthyroidism. The root problem appears
to be a deficiency of LDL receptors, which leads to a specific elevation of cholesterol
in the form of increased LDL. Heterozygotes for the hereditary form generally
develop cardiovascular disease from 30 to 50 years of age. Homozygotes may have
cardiovascular disease in childhood.
Type 1 (choice A) is characterized by isolated elevation of chylomicrons.
Type 2b (choice C) is characterized by elevations of both cholesterol and
triglycerides in the form of LDL and VLDL.
Type 3 (choice D) is characterized by elevations of triglycerides and cholesterol in
the form of chylomicron remnants and IDL.
Type 5 (choice E) is characterized by elevations of triglycerides and cholesterol in
the form of VLDL and chylomicrons.
________________________________________
21The correct answer is C. The patient is experiencing a thyrotoxic crisis, which
occurs most commonly in patients with untreated or inadequately treated Graves'
disease. The onset is typically abrupt and may be precipitated by stressors that can
include infection, trauma, radio-iodine treatment, and childbirth. The condition, if
unrecognized, may progress to congestive cardiac failure, pulmonary edema, and
death.

De Quervain's thyroiditis (choice A), also known as subacute granulomatous


thyroiditis, can produce transient hyperthyroidism, but does not usually produce a
thyrotoxic crisis.
Both follicular (choice B) and papillary (choice E) carcinomas of the thyroid gland
are usually non-secretory and consequently do not produce hyperthyroidism.
Hashimoto's thyroiditis (choice D) is an autoimmune thyroiditis that may transiently
produce hyperthyroidism before producing hypothyroidism, but thyrotoxic crisis is
not usually a feature.
________________________________________
22The correct answer is C. Atenolol is a beta-adrenergic receptor blocking agent
used in the treatment of hypertension. Medications in this drug class lower blood
pressure by reducing both cardiac output (choice C) and decreasing renin release
from the kidney (to a lesser extent).
Blocking catecholamine release from peripheral sympathetic nerves (choice A) is
the antihypertensive effect seen with peripherally acting adrenergic neuron blockers
(e.g., guanethidine and bretylium).
Angiotensin converting enzyme (ACE) inhibitors block the conversion of angiotensin
I to angiotensin II (choice B). Diuretics decrease intravascular volume (choice D),
which ultimately leads to a reduction in blood pressure.
Increasing renin release from the kidney (choice E) would increase, not decrease,
blood pressure.
________________________________________
23The correct answer is E. Riedel thyroiditis, also called ligneous (rocklike) stroma,
is a rare form of chronic thyroiditis characterized microscopically by a marked
fibrous reaction that destroys most or all of the thyroid gland and may involve
adjacent structures. The etiology is unknown. Clinically, this disease tends to affect
middle-aged and older, mostly female patients and causes the thyroid to have a
firm "woody" texture. It may be clinically mistaken for a neck malignancy and can
cause symptoms of stridor, dyspnea, dysphasia, laryngeal nerve paralysis, or
hypothyroidism.
Eyeball protrusion (choice A) suggests the hyperthyroidism of Graves disease.
A massive, soft thyroid gland (choice B) suggests multinodular goiter.
A single large thyroid nodule (choice C) could be due to either a thyroid adenoma or
thyroid cancer.
A very tender and painful thyroid (choice D) suggests subacute granulomatous (de
Quervain) thyroiditis.
________________________________________

24The correct answer is E. The third pharyngeal pouch normally gives rise to the
inferior parathyroid glands and the thymus. Cells from these pouches migrate
caudally in the embryo to the eventual location of these organs. The superior
parathyroid glands are derived from the fourth pharyngeal pouch. The absence of
these organs is found in DiGeorge syndrome, which typically presents with
immunodeficiency and hypoparathyroidism.
The second pharyngeal arch (choice A) gives rise to several skeletal and muscular
structures. These include the stapes, styloid process, stylohyoid ligament, a portion
of the hyoid bone, the stapedius muscle, stylohyoid muscle, posterior digastric
muscle, and all of the muscles of facial expression.
The second pharyngeal cleft (choice B) is normally obliterated. A persistence of this
cleft may result in a lateral cervical cyst, sinus, or fistula.
The second pharyngeal pouch (choice C) gives rise to the tonsillar fossa and the
palatine tonsils.
The third pharyngeal arch (choice D) gives rise to most of the hyoid bone and the
stylopharyngeus muscle. The third aortic arch, which passes through the third
pharyngeal arch, gives rise to the common carotid artery and the internal carotid
artery.
________________________________________
25The correct answer is E. By the third week of development, hematopoiesis begins
in the blood islands of the yolk sac. Beginning at 1 month of age and continuing
until 7 months of age, blood elements are also formed in the liver. Hematopoiesis
occurs in the spleen and lymphatic organs between 2 and 4 months, and in the
bone marrow after 4 months.
________________________________________
26The correct answer is D. The cells of the anterior pituitary can be classified as
chromophils (love dyes) or chromophobes (do not stain with dyes). The chromophils
can be further divided into acidophils (stain with acidic dyes) and basophils (stain
with basic dyes). The acidophils include the somatotropes, which secrete growth
hormone, and the mammotropes, which secrete prolactin. The basophils include the
corticotropes, which secrete ACTH (choice A), the gonadotropes, which secrete FSH
and LH (choices B and C), and the thyrotropes, which secrete TSH (choice E).
________________________________________
27The correct answer is C. Rheumatoid arthritis can coexist with a variety of
autoimmune diseases (including those listed in the answers), but is most frequently
associated with Sjgren's syndrome. Sjgren's syndrome classically presents with
keratoconjunctivitis (dry eyes) and xerostomia (dry mouth, often resulting in dental
caries and fissures in the oral mucosa). These symptoms are due to autoimmune
involvement with subsequent scarring of the salivary and lacrimal glands. Parotid
gland enlargement is common, as is vasculitis, Raynaud's phenomenon,
hyperviscosity syndrome, and peripheral neuropathy.

The development of oral squamous cell carcinoma (choice A) is not related to the
presence of dental caries.
Polyarteritis nodosa (choice B) is a systemic necrotizing vasculitis. Patients typically
present with low-grade fever, weakness, and weight loss. Abdominal pain,
hematuria, renal failure, hypertension, and leukocytosis may occur.
Systemic lupus erythematosus (choice D) is an autoimmune disease characterized
by vasculitis, rash, renal disease, hemolytic anemia, and neurologic disturbances.
Thyrotoxicosis (choice E) is not related to the development of dental caries

________________________________________

28>The correct answer is C. The description of the agent is consistent with a


staphylococcal organism (catalase-positive, gram-positive cocci that grows on
mannitol salt agar. The organism is most likely S. aureus, as it was able to ferment
mannitol (as evidenced by the color change in the mannitol salt agar after
incubation) and was beta-hemolytic. Other characteristics of this organism are
coagulase production and excretion of DNAse from colonies. Staphylococcal
organisms are also notorious for formation of abscesses; the patient had an abscess
in the tonsillar region.
The diphtheria bacilli (Corynebacterium diphtheriae; choice A) are gram-positive,
pleomorphic rods that are arranged in palisades. They are non-hemolytic and would
not grow on mannitol salt agar. These organisms produce a whitish-gray
pseudomembrane on the pharynx or the larynx; constitutional symptoms from
toxemia are the major features of diphtheria.
Epiglottitis is the most common disease of the upper respiratory tract produced by
Haemophilus influenzae (choice B), a gram-negative, encapsulated rod. It is also a
common cause of otitis media in children and may cause bronchitis, bronchiolitis,
and pneumonia in adults.
Streptococcus salivarius (choice D) is a gram-positive coccus that is usually alphahemolytic. It is normal flora of the oral cavity and is sometimes implicated in
subacute bacterial endocarditis, but is not associated with tonsillar abscesses.
Streptococcus pyogenes (choice E) is a beta-hemolytic, gram-positive coccus that
grows in chains, as opposed to the random, grape-like clusters of the staphylococci.
These organisms are the most common cause of pharyngitis, which is usually
manifested by severe sore throat, fever, a beefy red pharynx, and a tonsillar
exudate.

________________________________________

________________________________________
29The correct answer is E. The patient has bilateral acoustic neuromas, probably
due to neurofibromatosis type II (over 90% of patients with NF-2 develop bilateral
acoustic neuromas). This condition is a associated with the NF-2, gene, located on
22q (note all the 2's). Patients often develop meningiomas, gliomas, and
schwannomas of cranial and spinal nerves.
5q (choice A) contains the APC tumor suppressor gene, which is associated with
familial and sporadic colorectal cancers.
13q (choice B) contains the Rb tumor suppressor gene, which is associated with
retinoblastoma and osteosarcoma.
17q (choice C) contains both the NF-1 tumor suppressor gene, which is associated
with neurofibromatosis type I, and the p53 tumor suppressor gene, associated with
many human cancers.
18q (choice D) contains both the DCC gene, which is associated with colon and
gastric carcinomas and the DPC gene, associated with pancreatic cancer.
________________________________________

30The correct answer is A. The patient has myocarditis due to Trypanosoma cruzi.
This infectious condition, known as Chagas disease, is endemic in vast areas of
South America and is transmitted from person to person by triatomids known as
"kissing bugs." Experts assess the number of persons with Chagas disease at about
7 million with about 35 million at risk in South America. T. cruzi is an intracellular
protozoon that localizes mainly in the heart and nerve cells of the myenteric plexus,
leading to myocarditis and dysmotility of hollow organs, such the esophagus, colon,
and ureter. Cardiac involvement manifests with ventricular dilatation and congestive
heart failure secondary to myocyte necrosis and fibrosis. Intracellular parasites can
be visualized in tissue sections. Chagas disease is a cause of acquired achalasia, in
which the distal third of the esophagus dilates because of loss of its intrinsic
innervation. A similar pathologic mechanism accounts for megacolon and
megaureter in Chagas disease.
The remaining choices refer to different infectious conditions that may also involve
the myocardium:
Chronic arthritis (choice B) is a manifestation of the chronic stage of Lyme disease,
which is caused by Borrelia burgdorferi and is transmitted to humans by deer ticks.
Skin, CNS, and heart are the main targets of this infection.

Cysts in the brain (cysticerci) (choice C) may develop as a consequence of


infestation by the tapeworm Taenia solium. Humans acquire this parasite by
ingesting the eggs from undercooked pork. Cysticercosis may also affect the heart,
skeletal muscle, and skin.
Group B coxsackievirus infections cause pleuritis (choice D) and myocarditis,
manifesting with fever, chest pain, and, if myocarditis is severe, congestive heart
failure. As in any form of viral myocarditis, the myocardium is infiltrated by
lymphocytes, but there are no morphologic markers specific for Coxsackievirus
infection.
Splenomegaly (choice E), often of massive proportions, is seen in patients with
malaria. Plasmodium organisms can also invade the myocardium, leading to
myocarditis.
________________________________________

31The correct answer is B. The history suggests an autosomal dominant disorder


(the father died, the grandfather and an aunt suffered similar conditions). Limited
information through 3 generations shows no skipping of generations (dominance),
and a affected female confirms the autosomal transmission. The symptoms of
hyperthermia, muscle spasm, autonomic and electrolyte disturbances during
general anesthesia, and the pedigree argue for a diagnosis of malignant
hyperthermia. In this disorder, a mutation in the ryanodine receptor-gated calcium
channel of the sarcoplasmic reticulum of skeletal muscle leads to increased
sensitivity to halothane and depolarizing agents. In an autosomal dominant pattern,
one defective allele is sufficient to develop the trait. The father can pass on an 11
kb, a 9 kb or a 7 kb band to his children, whereas the mother can pass on a 12 kb or
a 9 kb band. The DNA pattern of the father and grandfather can be compared, and
the likely restriction fragment involved in the inheritance ofmalignant hyperthermia
is the 7 kb band.
The 11 kb band from the father (choice A) does not carry the defective allele, since
the grandfather did not share that same banding pattern: the only similarity
between the father and his own father is the 7 kb band. However, if the statement
had been true, then his children would not have a risk of malignant hyperthermia,
because they do not carry the allele.
The 9 kb band does not appear to carry the defective allele (choice C), however, if it
did, the daughter would have a major risk of developing malignant hyperthermia
when succinylcholine was used as a neuromuscular blocking agent, because it is a
depolarizing drug.
The disease is not X-linked recessive (choice D) because an aunt had a similar
problem, but was resuscitated. If it were X-linked, only males would be affected, so
the daughter would not be affected.
Mitochondrial inheritance (choice E) would be inconsistent with the observed
transmission of the defective allele from males to their offspring.

________________________________________
32The correct answer is C. The boy probably has Klinefelter's syndrome (47, XXY),
which has the typical presentation described in the question. The condition arises as
a result of failure of separation (nondisjunction) of the sex chromosomes, and can
be related to either paternal nondisjunction (slightly more common) or maternal
nondisjunction.
Deletions (choice A) are a common form of genetic disease and contribute to many
genetic recessive diseases.
Examples of nondisjunction of autosomes (choice B) include trisomies such as most
cases of Down's syndrome (trisomy 21), Edwards' syndrome (trisomy 18), and Patau
syndrome (trisomy 13).
There are two types of translocations: non-Robertsonian (choice D) and
Robertsonian (choice E). Non-Robertsonian (reciprocal) translocations result when
two non-homologous chromosomes exchange genetic material. Robertsonian
translocations are a special type of translocation that involve exchange of genetic
material from the long arms of one acrocentric chromosome to the long arms of
another acrocentric chromosome, with fusion of the centromeres. Four percent of
cases of Down's syndrome are caused by this mechanism.
________________________________________
33The correct answer is C. The baby has Hirschsprung's disease, which is due to an
absence of ganglion cells in the wall of the colon. Neural crest cells contribute to the
formation of many adult structures. Among these are all of the postganglionic
neurons of the autonomic nervous system and the sensory neurons of the
peripheral nervous system.
Ectoderm (choice A) forms the epidermis of the skin and the parenchymal cells of
glands associated with the skin such as the sweat glands, sebaceous glands, and
mammary glands.
Endoderm (choice B) forms the epithelial lining of the gut tube and the parenchymal
cells of glands associated with the gut tube, such as the liver and pancreas.
Neural ectoderm (choice D) forms the central nervous system, the somatic motor
neurons of the peripheral nervous system, and the preganglionic neurons of the
autonomic nervous system.
Splanchnic mesoderm (choice E) forms the visceral peritoneum, visceral pleura,
visceral pericardium, and the stroma and muscle of the wall of the gut, among other
structures.
________________________________________
34The correct answer is B. The description is that of gonadal dysgenesis. In the
absence of testosterone, the Wolffian ducts will regress and fail to differentiate into
normal male internal reproductive tracts. In the absence of Mllerian regression
factor, the Mllerian ducts will automatically differentiate into oviducts and a uterus.

Differentiation of the male external genitals is dependent on adequate


dihydrotestosterone (via an action of 5 a-reductase on testosterone). In the absence
of testosterone, female-type external genitalia will develop.
Selective dysgenesis of the Sertoli cells could produce the situation described in
choice A. Normal Leydig cells would secrete testosterone and produce normal maletype internal and external tracts. However, the absence of Mllerian regression
factor, which is secreted by the Sertoli cells, would allow formation of female-type
internal structures as well.
Female-type internal reproductive tract and male-type external genitalia (choice C)
would not be likely to occur under any circumstances.
The situation described in choice D could occur with 5 a-reductase deficiency.
Normal male-type internal tracts can form because there is no requirement for
dihydrotestosterone. Mllerian regression factor will prevent differentiation of
female-type internal tracts. Since differentiation of the normal male external
genitals requires dihydrotestosterone, 5 a-reductase deficiency will lead to
feminization.
The situation described in choice E is normal, and would not occur in the individual
described who has testicular dysgenesis.
________________________________________
35The correct answer is B. This is a tough two-step style question. You first needed
to make the diagnosis, and then you needed to recall the localization of this
particular disease to a specific chromosome. In this case, everything in the vignette
leads you to a diagnosis of Wilms' tumor. Wilms' tumor occurs in children and
typically presents with an abdominal mass as well as with hypertension, hematuria,
nausea, and intestinal obstruction. Because the tumor is derived from mesonephric
mesoderm, it can include mesodermal derivatives such as bone, cartilage, and
muscle. The Wilms' tumor suppressor gene (WT-1) has been localized to
chromosome 11 (11p).
The remaining answer choices provide us the opportunity to discuss some other
known relationships between genes and disease. This is a topic of intense research
that is likely to become more and more emphasized on the USMLE examinations.
Chromosome 5 (choice A) is the site of the tumor suppressor gene APC, which is
involved in the pathogenesis of colon cancer and familial adenomatous polyposis.
Chromosome 13 (choice C) is the site of the tumor suppressor gene for
retinoblastoma and osteosarcoma (Rb) as well as the BRCA-2 gene for breast
cancer.
Chromosome 17 (choice D) is the site of p53 (involved in most human cancers), NF1 (neurofibromatosis type I), and BRCA-1 (breast and ovarian cancer).
Chromosome 22 (choice E) is home to the NF-2 gene, which is involved in
neurofibromatosis type II.

________________________________________
36The correct answer is A. This is a classic presentation of a pulmonary abscess.
Chronic courses with less severe symptoms (with intermittent improvement
following short courses of antibiotics) are also sometimes seen, particularly if the
diagnosis was not suspected. Chest x-ray typically shows pneumonic opacification
in which a cavity, often with a fluid level, is visible. Pulmonic abscesses can be
caused by anaerobes (most common, particularly if aspiration initiated the abscess),
gram-negative aerobic bacilli, and Staphylococci. Therapy is based on the
organisms isolated, and should be continued for at least 4 to 6 weeks. In cases that
fail to resolve, the possibility of coexisting carcinoma should be considered.
Choice B is the x-ray appearance of pleural effusion.
Choice C is the x-ray appearance of lobar pneumonia.
Choice D is the x-ray appearance of bronchopneumonia.
Choice E is the x-ray appearance of bronchiectasis.
________________________________________
37The correct answer is C. Bullae with the cleavage plane above the basal layer of
the epidermis suggests pemphigus vulgaris, which is caused by autoantibodies to
intercellular junctions of epidermal cells. The autoantibodies decrease the ability of
the keratinocytes to adhere to one another, permitting formation of vesicles and
bullae. Oral involvement is common, and often precedes the characteristic skin
lesions. Separation of the epidermis upon manual stroking of the skin is known as
Nikolsky's sign. This sign is present in other disorders such as Stevens-Johnson
syndrome, but we are told the woman is not taking any medications, a typical cause
of Stevens-Johnson syndrome in the adult population.
Antibodies to epidermal basement membrane proteins (choice A) are seen in
bullous pemphigoid, which is a bullous disease characterized by blisters with a
cleavage line between the epidermis and dermis.
Antibodies to glycoprotein IIb/IIIa (choice B) are seen in autoimmune
thrombocytopenic purpura.
Antibodies to intrinsic factor (choice D) are seen in pernicious anemia.
Antibodies to Type IV collagen (choice E) are seen in Goodpasture's syndrome
________________________________________
38The correct answer is D. Enlargement of the distal segments of the fingers and
toes due to proliferation of connective tissue is known as clubbing. Clubbing may be
associated with lung cancer, mesothelioma, bronchiectasis, and hepatic cirrhosis. In
these conditions, the clubbing is accompanied by formation of new subperiosteal
bone (hypertrophic osteoarthropathy).
Chronic renal failure (choice A) causes renal osteodystrophy, which may manifest as
a combination of osteitis fibrosa cystica and osteomalacia.

Multiple exostoses are sometimes associated with multiple colonic polyps and colon
cancer (choice B) in Gardner's syndrome.
A variety of endocrine adenomas (choice C) can be seen in polyostotic fibrous
dysplasia.
Profound anemia (choice E) can be seen in diseases that destroy the marrow, such
as osteopetrosis.
________________________________________
39The correct answer is B. Aspirin irreversibly acetylates platelet cyclooxygenase,
thereby inactivating this enzyme and preventing the production of thromboxane A2.
In this manner, aspirin therapy interferes with secondary aggregation. The result is
a prolonged bleeding time related to platelet dysfunction rather than to dysfunction
of the coagulation process, as would be implied by prolongation of the activated
partial thromboplastin time (choice A), prothrombin time (choice D), or
thromboplastin time (choice E). The platelet count (choice C) is not affected by
aspirin.
________________________________________
40The correct answer is E. The patient is suffering from hyperacute rejection due to
the preformed anti-B ABO blood group antibody found in all type A positive
individuals. Hyperacute rejection occurs within minutes to a few hours of the time of
transplantation, and is due to the destruction of the transplanted tissue by
preformed antibodies reacting with antigens found on the transplanted tissue that
activate complement and destroy the target tissue. Preformed antibodies can also
be due to presensitization to a previous graft, blood transfusion, or pregnancy.
Acute rejection due to antibody-mediated immunity (choice A) is incorrect because
this patient suffered from hyperacute rejection (immediate) occurring within
minutes to hours, rather than days.
Acute rejection due to cell-mediated immunity (choice B) will not occur until several
days or a week following transplantation. Acute rejection is due to type II and type
IV reactions.
Chronic rejection, due to the presence of cell-mediated immunity to minor HLA
antigens (choice C), occurs in allograft transplantation months to even years after
the transplant. Chronic rejection is generally caused by both humoral and cellmediated immunity.
An accelerated acute rejection, occurring in 3-5 days, can be caused by tissue
infiltration and destruction by presensitized T lymphocytes and macrophages
(choice D) and/or antibody-dependent, cell-mediated cytotoxicity (ADCC). Note that
this is not a hyperacute reaction.
________________________________________
41The correct answer is A. Bilirubin is a degradative product of hemoglobin
metabolism. Bilirubin (pigment) stones are specifically associated with excessive

bilirubin production in hemolytic anemias, including sickle cell anemia. Bilirubin


stones can also be seen in hepatic cirrhosis and liver fluke infestation.
Calcium oxalate stones (choice B) and cystine stones (choice E) are found in the
kidney, rather than the gallbladder.
Pure cholesterol stones (choice C) are less common than mixed gallstones, but have
the same risk factors, including obesity and multiple pregnancies.
Mixed stones (choice D) are the common "garden variety" gallstones, found
especially in obese, middle aged patients, with a female predominance.
________________________________________
42The correct answer is B. Heterotopic rests are small areas of normal tissue in
abnormal sites. These are usually clinically insignificant unless they form a
noticeable mass or are misdiagnosed in a biopsy (raising suspicion of metastatic
cancer). One exception is a gastric heterotopia, which typically occurs in the small
intestine, and can produce enough acid to cause a peptic ulcer in adjacent mucosa.
The ulcer may be a source of gastrointestinal bleeding.
Heterotopias of the CNS (choice A), parathyroid (choice D), and thyroid(choice E)
can occur, but would not be expected in the small intestine and would not be likely
to cause bleeding if present.
Pancreatic heterotopia (choice C) can occur in the small intestine, but does not
cause bleeding.
________________________________________
43The correct answer is A. Surgeons worry about their obese patients more than
their skinny ones, because a thick layer of relatively poorly vascularized
subcutaneous fatty tissue is both mechanically unstable (it holds stitches poorly)
and heals very slowly. These patients have a frequent rate of dehiscence (tearing
open of the incisional site) with subsequent, difficult-to-control infection (access by
antibiotics, leukocytes, and serum antibodies are all hampered by the poor blood
supply).
Aponeuroses (choice B) are strong thickenings of muscle sheath that usually suture
and heal well after surgery.
Loose connective tissue (choice C) is well vascularized and surgeons do not usually
worry much about it during the healing process.
Muscle (choice D) usually heals well after surgery.
Skin (choice E) usually heals well, unless it becomes infected.
________________________________________
44The correct answer is D. The key symptom is hyperventilation. Hyperventilation
results in hypocapnia, alkalosis, increased cerebrovascular resistance, and
decreased cerebral blood flow. Carbon dioxide plays an important role in the control

of cerebral blood flow. An increase in arterial PCO2 dilates blood vessels in the brain
and a decrease in PCO2 causes vasoconstriction. The anxious, hyperventilating
woman is "blowing off" carbon dioxide, which lowers her arterial PCO2. This
decrease in PCO2 has caused the cerebrovascular resistance (choice E) to increase,
thereby decreasing cerebral blood flow. The decrease in cerebral blood flow has
caused the woman to feel faint and to have blurred vision. Other symptoms
commonly associated with the hyperventilation of anxiety states are feelings of
tightness in the chest and a sense of suffocation.
Hyperventilation increases the arterial oxygen content (choice A) and PO2 (choice
B) in a normal person.
A decrease in arterial PCO2 causes the arterial pH (choice C) to increase, i.e., the
patient becomes alkalotic.
________________________________________
45The correct answer is E. This patient is experiencing symptoms of hypocalcemia
secondary to diminished parathyroid hormone (PTH) secretion. This must always be
considered in a patient who undergoes total or subtotal thyroidectomy because the
parathyroids are nestled in the tissue surrounding the thyroid gland. Surgical
attempts to leave portions of the parathyroids intact are sometimes unsuccessful.
Other causes of decreased PTH include neck irradiation, autoimmune phenomena
(polyglandular autoimmune syndromes), dysembryogenesis (as in DiGeorge's
syndrome), or as a result of heavy metal damage (Wilson's disease, hemosiderosis,
hemochromatosis).
Bone metastases (choice A) would cause hypercalcemia, as a result of osteolysis.
Hashimoto's thyroiditis (choice B) is the most common cause of hypothyroidism and
results in decreased thyroid hormone and elevated TSH levels. Serum calcium and
PTH should be normal.
Hypervitaminosis D (choice C) would cause hypercalcemia.
Hypomagnesemia (choice D) may cause a functional hypoparathyroidism because
magnesium is needed for PTH activity in tissue. However, in such a case, actual PTH
levels would not be decreased.
________________________________________
46The correct answer is D. The pudendal nerve is often anesthetized to provide
obstetric anesthesia (although epidural blocks are now more commonly used in
preparation for delivery). The pudendal nerve arises from sacral segments 2, 3, and
4. It exits the pelvis by way of the greater sciatic foramen and then courses around
the ischial spine (and sacrospinous ligament) to pass through the lesser sciatic
foramen. It then runs through Alcock's canal to supply the muscles and skin of the
perineal area.
The inferior gluteal nerve (choice A) leaves the pelvis by passing through the
greater sciatic foramen to supply the gluteus maximus muscle.

The lateral femoral cutaneous nerve (choice B) courses by the anterior superior iliac
spine and supplies the anterolateral skin of the thigh.
The obturator nerve (choice C) passes through the obturator foramen to supply
muscles and skin in the medial thigh.
The superior gluteal nerve (choice E) leaves the pelvis by passing through the
greater sciatic foramen to supply the gluteus medius and minimus muscles.
________________________________________

47>The correct answer is A. This boy has DiGeorge's syndrome, as evidenced by his
tetany (muscle spasms) due to hypocalcemia and his history of recurrent infections
and neonatal seizures. The syndrome occurs because of an embryonic failure in the
development of the third and fourth pharyngeal pouches. Patients have both
hypoplastic parathyroids (producing hypocalcemia) and thymuses (producing T-cell
deficiency and recurrent infections). Since cell-mediated immunity (which depends
on T cells) is important in defense against infections caused by intracellular
pathogens (such as viruses), patients with DiGeorge's are particularly susceptible to
viral infections, such as chickenpox (varicella). They also have trouble with fungal
pathogens (e.g., Candida) and mycobacteria.
Note that the USMLE might ask you other questions about DiGeorge's syndrome, so
you should be able to recognize other clues to the diagnosis, including: congenital
cardiac defects, esophageal atresia, bifid uvula, short philtrum, hypertelorism,
antimongoloid palpebral slant, mandibular hypoplasia, and low-set ears.
Diphtheria (choice B) is caused by Corynebacterium diphtheriae, which produces
disease by the elaboration of a very potent exotoxin. Therefore, humoral immunity
(antitoxin), which is not usually compromised in DiGeorge's patients, is essential for
defense against the organism. (Note that the C. diphtheriae exotoxin acts by
causing the ADP-ribosylation of elongation factor-2 of eukaryotic cells, thereby
inhibiting protein synthesis). The disease can be avoided by immunization with
diphtheria toxoid.
Gas gangrene (choice C) is caused by Clostridium perfringens, which produces a
potent alpha toxin that injures cell membranes. Therefore, humoral immunity would
again play a predominant role in defense against this organism. Note that the
disease occurs in wounds and would not be expected in an uninjured 4-year-old boy.
Gonorrhea (choice D) is caused by Neisseria gonorrhoeae and would not be
expected in a 4-year-old boy unless there was evidence of sexual abuse. Virulence
factors of this organism include pili, cell wall endotoxin and outer membrane, and
secretory IgA protease. Antibody responses, neutrophils, and complement are of
prime importance in defense against gonococcal infections.
Tetanus (choice E) is caused by Clostridium tetani and serves as a tricky distracter,
as you might have quickly associated the patient's muscle spasms with this answer
choice. (This is why it is important to read the question stem carefully before

prematurely jumping to the responses). C. tetani, which gains entry through deep
wounds, produces tetanus toxin (exotoxin) and can be prevented by immunization
with tetanus toxoid.

________________________________________

________________________________________
48The correct answer is E. The child is likely suffering from cystic fibrosis. In this
disorder, an abnormality of chloride channels causes all exocrine secretions to be
much thicker, and more viscous than normal. Pancreatic secretion of digestive
enzymes is often severely impaired, with consequent steatorrhea and deficiency of
fat-soluble vitamins, including vitamin A.
Cystinuria (choice A) is a relatively common disorder in which a defective
transporter for dibasic amino acids (cystine, ornithine, lysine, arginine; COLA) leads
to saturation of the urine with cystine, which is not very soluble in urine, and
precipitates out to form stones.
Hypoglycemia (choice B) is not a prominent feature of children with cystic fibrosis
who are on a normal diet. Hyperglycemia may occur late in the course of the
disease.
Iron deficiency anemia (choice C) is not found with any regularity in children with
cystic fibrosis.
Sphingomyelin accumulation (choice D) is generally associated with deficiency of
sphingomyelinase, as seen in Niemann-Pick disease.
________________________________________
49The correct answer is D. The nerve in question is the superficial peroneal nerve
(also called the musculocutaneous nerve), which innervates much of the skin of the
dorsum of the foot (except for the first web space innervated by the deep peroneal
nerve, and the medial and lateral borders of the foot innervated by the saphenous
and sural nerves, respectively). The superficial peroneal nerve is a branch of the
common peroneal nerve, and also innervates the peroneus longus and peroneus
brevis muscles.
All of the other muscles (choices A, B, C, and E) are innervated by the deep
peroneal nerve.
________________________________________
50The correct answer is A. Cholecystokinin, or CCK, is synthesized in the duodenal
and jejunal mucosa and stimulates gallbladder contraction and pancreatic enzyme
secretion. Other functions include slowing of gastric emptying, an atrophic effect on

the pancreas, and secretion of antral somatostatin, which in turn, decreases gastric
acid secretion.
Gastrin (choice B) prepares the stomach and small intestine for food processing,
including stimulating secretion of HCl, histamine, and pepsinogen, increasing gastric
blood flow, lower esophageal sphincter tone, and gastric contractions.
Gastric inhibitory peptide, or GIP (choice C), stimulates pancreatic insulin secretion
at physiologic doses and inhibits gastric acid secretion and gastric motility at
pharmacologic doses.
Secretin (choice D) stimulates secretion of bicarbonate-containing fluid from the
pancreas and biliary ducts.
Vasoactive intestinal polypeptide, or VIP (choice E), relaxes intestinal smooth
muscle and stimulates gut secretion of water and electrolytes.

Answers

________________________________________

1The correct answer is C. A small percentage of patients (5% to 10%) develop a


nonoliguric form of acute renal failure when treated with aminoglycosides such as
gentamicin. Gentamicin can accumulate in the kidney to produce a delayed form of
acute renal failure resulting in an elevation of the serum creatinine level. The
nonoliguric form of renal failure, seen in this patient, is the typical presentation for
gentamicin nephrotoxicity.
Acute glomerulonephritis (choice A) is typically associated with hypertension and
the appearance of an active urinary sediment containing casts and red blood cells.
Cephalothin (choice B) is a first-generation cephalosporin commonly used in the
treatment of severe infection of the genitourinary tract, gastrointestinal tract, and
respiratory tract, as well as skin infections. This antibiotic can produce an acute
interstitial nephritis; however, the patient's presentation is consistent with
gentamicin nephrotoxicity. Interstitial nephritis is commonly associated with the
development of acute renal failure, fever, rash, and eosinophilia.
Renal artery occlusion (choice D) is commonly caused by thrombosis or embolism.
The clinical features of acute renal artery occlusion are hematuria, flank pain, fever,
nausea, elevated LDH, elevated SGOT and acute renal failure.
Since the patient has normal vital signs and no history of hypotension, a diagnosis
of sepsis (choice E) is unlikely.
________________________________________

2The correct answer is E. The phrase "mosaic pattern" of newly formed woven bone
is a specific tip-off for Paget's disease of bone, and is not seen in other bone
conditions. The clinical and radiologic presentation are typical; an increased hat size
may also be a clue. In its early stages, Paget's disease is characterized by
osteolysis, producing patchwork areas of bone resorption with bizarre, large
osteoclasts. In the middle stage of the disease, secondary osteoblastic activity
compensates with new bone formation, producing the mosaic pattern. In late
Paget's, the bones are dense and osteosclerotic. Paget's disease is suspected to be
related to prior viral infection, but the cause remains mysterious. Complications
include myelophthisic anemia, high output cardiac failure, pain secondary to nerve
compression, deformities secondary to skeletal changes, and in about 1% of
patients, osteosarcoma or other sarcoma, typically involving the jaw, pelvis, or
femur.
An increased incidence of astrocytomas (choice A) is associated with tuberous
sclerosis.
Hodgkin's lymphoma (choice B) is usually a disease of young adults, although older
patients may have the lymphocyte-depleted form.
Meningiomas (choice C) are mostly benign tumors that affect adults, especially
females. There may be an association with breast cancer, possibly related to high
estrogen states.
Non-Hodgkin's lymphoma (choice D) is more common in AIDS and other
immunodeficiency states, although the incidence in the immunocompetent is
increasing.
________________________________________
3The correct answer is C. This question required that you know two pieces of
information: (1) renin is secreted from the juxtaglomerular (JG) cells; and (2) the
location of the JG cells in a picture that includes both the glomerulus and the
juxtaglomerular apparatus. Remember that renin is responsible for converting
angiotensinogen to angiotensin I, which is subsequently converted to angiotensin II
in the lungs by an important enzyme appropriately called angiotensin-converting
enzyme. Angiotensin II helps preserve blood pressure both by its potent
vasoconstrictor properties and by its ability to stimulate aldosterone secretion from
the adrenal cortex.
The label for choice A points to the glomerular basement membrane.
The label for choice B points to the epithelium of Bowman's capsule.
The label for choice D points to the macula densa cells, which are thought to sense
sodium concentration in the distal convoluted tubule.
The label for choice E points to the Polkissen cells. Their function is unknown.
________________________________________

4The correct answer is B. The findings suggest disseminated intravascular


coagulation (DIC), which is a feared complication of many other disorders, such as
obstetrical catastrophes, metastatic cancer, massive trauma, and bacterial sepsis.
The basic defect in DIC is a coagulopathy characterized by bleeding from mucosal
surfaces, thrombocytopenia, prolonged PT and PTT, decreased fibrinogen level, and
elevated fibrin split products. Helmet cells and schistocytes (fragmented red blood
cells) are seen on peripheral blood smear.
Autoimmune hemolysis (choice A) and hereditary spherocytosis (choice C) would be
characterized by spherocytes in the peripheral smear.
Macro-ovalocytes and hypersegmented neutrophils can be seen in megaloblastic
anemia (choice D).
Sickle cells are seen in sickle cell anemia (choice E).
________________________________________
5The correct answer is C. The most common form of atrial septal defect is located
near the foramen ovale (not to be confused with a patent foramen ovale, which is of
little or no hemodynamic significance). They result from incomplete adhesion
between the septum primum and the septum secundum during development.
Atrial septal defects less commonly result from failures of formation of the septum
primum (choice A) and septum secundum (choice B).
Malformations of the interventricular septum (choices D and E) cause ventricular
septal defects rather than atrial septal defects.
________________________________________
6The correct answer is D. Leuprolide is a GnRH analog. Given long-term in a
continuous fashion, it will inhibit FSH and LH release, thereby decreasing
testosterone production and exacting a chemical castration in men. It can be used
in the treatment of prostate cancer, polycystic ovary syndrome, uterine fibroids, and
endometriosis.
Inhibition of 5a-reductase (choice A) is the mechanism of action of finasteride. It
thereby inhibits the production of dihydrotestosterone. It is used in the treatment of
benign prostatic hyperplasia (BPH).
Flutamide is another drug used in the treatment of prostate cancer. It is a
competitive antagonist at androgen receptors (choice B).
Since LH activates interstitial cells to secrete testosterone, a synthetic analog of LH
(choice C) would not be appropriate treatment for prostatic cancer. The same goes
for a testosterone analog (choice E).
Here is a brief chart that will aid you in remembering the actions of these similar
sounding drugs:
Drug Action

Indication

Leuprolide

GnRh analog

Prostate CA

Flutamide

Competitive androgen antagonist

Finasteride

5a-reductase inhibitor

Prostate CA

BPH

Remember, "loo"prolide and "floo"tamide are both used for prostate cancer.
Finasteride is used for BPH.
________________________________________

7The correct answer is D. 21-hydroxylase deficiency is the most common form of


congenital adrenal hyperplasia. The simple virilizing variant (without salt wasting) is
most common, but with severe 21-hydroxylase deficiency, virilization and salt
wasting occur. The infant described above exhibits salt wasting and hyperkalemia
because aldosterone secretion is diminished by the enzyme deficiency. The
hypoglycemia is due to cortisol deficiency. Because cortisol secretion is diminished
in congenital adrenal hyperplasia, ACTH secretion from the anterior pituitary is
increased due to loss of negative feedback inhibition. The high levels of ACTH are
responsible for the adrenal hyperplasia and the increased secretion of the adrenal
androgens, dehydroepiandrosterone and androstenedione, which are responsible for
the virilization of the external genitalia. 17-OH progesterone is the steroid precursor
just proximal to 21-hydroxylase and is also increased because of the excessive drive
to the adrenal cortex by ACTH.
5-alpha-reductase deficiency (choice A) in male fetuses will produce normal
differentiation of the internal reproductive tracts, but the external genitalia will be
feminized. This is because testosterone needs to be converted to
dihydrotestosterone (by 5-alpha-reductase) in the external genitalia and the
prostate for normal differentiation into the male phenotype.
11-beta-hydroxylase deficiency (choice B) is another form of congenital adrenal
hyperplasia. It is characterized by salt retention due to excessive secretion by the
inner zones of the adrenal cortex of the weak mineralocorticoid,
deoxycorticosterone. Again, the excessive drive to the adrenal cortex is due to
increased ACTH resulting from diminished negative feedback suppression by
cortisol. The adrenal also secretes excessive androgens and virilization occurs in
female fetuses.
17-alpha-hydroxylase deficiency (choice C) is another from of congenital adrenal
hyperplasia that is accompanied by salt retention. The high levels of ACTH drive the
adrenal cortex to secrete increased amounts of deoxycorticosterone and
corticosterone, both of which have weak mineralocorticoid activity. Without the
ability to 17-alpha-hydroxylate progesterone or pregnenolone, steroid-secreting
cells cannot produce sex steroids. When 17-alpha-hydroxylase deficiency is present
in the adrenal cortex, it is also present in the gonads. Hence, whether it occurs in a
male or female fetus, sex steroid production will be diminished. Female fetuses will
develop normal reproductive tracts and genitalia since these structures are

programmed in utero to "automatically" become female. Male fetuses, however, will


have their reproductive tracts and genitalia feminized.
Complete androgen resistance (choice E) results in feminization of affected male
fetuses. It is characterized by an XY genotypic male with phenotypically female
external genitalia and a vagina that ends as a blind sac.
________________________________________
8The correct answer is E. Vibrio vulnificus is an extremely invasive organism,
producing a septicemia in patients after eating raw shellfish, or causing wound
infections, cellulitis, fasciitis, and myositis after exposure to seawater or after
cleaning shellfish. Patients at high risk for septicemia include those with liver
disease, congestive heart failure, diabetes mellitus, renal failure, hemochromatosis,
and immunosuppression.
Citrobacter diversus (choice A) produces neonatal meningitis and can be frequently
cultured from the umbilicus.
Enterotoxigenic E. coli (choice B) produces the classic traveler's diarrhea. The toxin
is ingested in water and salads. The incubation period is approximately 12 hours.
The diarrhea is non-inflammatory and treatment is supportive.
Providencia stuartii (choice C) is a gram-negative rod related to Proteus. It is a
common cause of nosocomial bacteremia in nursing home patients with chronic
catheterization.
Vibrio cholerae (choice D) produces a non-invasive, non-inflammatory, high-volume
secretory diarrhea that is toxin-mediated.
________________________________________
9The correct answer is D. The disease in question is leprosy, or Hansen's disease. A
key feature in the description is the fact that the organism is acid-fast. Both of the
mycobacteria, M. avium-intracellulare and M. leprae are strongly acid-fast, that is
they retain the carbol fuchsin dye in the face of acid-alcohol decolorization. M.
leprae has a predilection for the skin and cutaneous nerves, thereby producing the
symptoms of depigmentation and anesthetic cutaneous lesions. This loss of
peripheral nerve function leads to many of the disfiguring features of the disease;
because the patients do not have normal pain sensation, they sustain repeated
injuries. In addition, the organism attacks cartilage and causes granuloma formation
in the skin, leading to some of the facial disfigurement.
Bartonella henselae (choice A) is a very small, gram-negative bacterium that is
closely related to the rickettsia, although it is able to be cultured on lifeless media.
It is the cause of cat-scratch disease, a local, chronic lymphadenitis most commonly
seen in children, and bacillary angiomatosis, a disease seen particularly in AIDS
patients.
Listeria monocytogenes (choice B) is a ubiquitous microbe that causes disease in
over 100 animal species. Although it is best known as an agent of meningitis in the
newborn, it is a cause of multiple other diseases. A characteristic feature of these

infections is the development of granulomas at the site of the infection. The


organism is not acid-fast and has no particular predilection for skin or nervous
tissues.
M. avium-intracellulare (choice C) causes tuberculosis-like pulmonary disease in the
immunosuppressed.
Nocardia asteroides (choice E) primarily produces pulmonary infections in humans.
The organism is consider to be "weakly" acid-fast, meaning that if the amount of
HCl used in the decolorization step is reduced, the organisms will retain the
carbolfuchsin primary stain.
________________________________________
10The correct answer is A. The cells described are the primordial eggs, which
remain stopped in the diplotene stage of the first meiotic division from before birth
until fertilization, a period which may be 40 or more years.
Choices B and E are incorrect because the oocytes are stopped in the first, not
second meiotic division.
Choices C and D are incorrect because the cells described are oocytes and are not
in mitosis.
________________________________________
11The correct answer is D. This is impetigo, which is typically seen in preschool
children with poor hygiene, particularly in the summer in warm climates. The
characteristic lesion has a large golden crust. Most cases are caused by
Staphylococcus aureus; Streptococcus pyogenes is occasionally implicated.
Impetigo is highly infectious, and mini-epidemics can occur in daycare settings. The
initial treatment is typically with penicillins and topical preparations. Methicillinresistant strains are presently rare in this setting, but can occur.
Aphthous ulcers (choice A), commonly known as "canker sores," are painful, shallow
ulcers of the oral cavity.
Erysipelas (choice B) is a different type of skin infection, often caused by
Streptococcus pyogenes (also sometimes others including Staphylococcus), and is
characterized by large erythematous patches.
Herpes simplex I (choice C) causes tiny oral and perioral vesicles, but not large
golden crusts.
Measles (choice E) causes a blotchy erythematous rash.
________________________________________
12The correct answer is C. This little boy has a Meckel's diverticulum, an ileal
outpocketing typically located within 50-75 cm of the ileocecal valve. It is a
congenital anomaly resulting from the persistence of the vitelline
(omphalomesenteric) duct. Approximately half cause ulceration, inflammation, and
gastrointestinal bleeding due to the presence of ectopic acid-secreting gastric

epithelium. Pancreatic tissue may sometimes occur in these diverticula as well. Note
that this is the most common type of congenital gastrointestinal anomaly.
Something else to keep in mind: A favorite question attendings ask on the wards is
the rule of 2's associated with Meckel's diverticulum: it occurs in about 2% of
children, occurs within approximately 2 feet of the ileocecal valve, contains 2 types
of ectopic mucosa (gastric and pancreatic), and its symptoms usually occur by age
2.
All of the other answer choices have no relationship to Meckel's diverticulum.
________________________________________
13The correct answer is B. The patient has primary atypical pneumonia caused by
Mycoplasma pneumoniae. This organism is fastidious and difficult to culture in the
laboratory, however serodiagnosis can be most helpful. Patients typically produce
one or two heterophile antibodies during the course of the infection; one
agglutinates human O+ RBCs in the cold (the cold hemagglutinin) while the other
causes the agglutination of a strain of Streptococcus salivarius termed strain MG
(the Strep MG agglutinins).
Klebsiella pneumoniae (choice A) is readily cultured on routine laboratory media and
characteristically produces pneumonia with blood clots in the sputum (red currant
jelly sputum), which may be indicative of pulmonary abscess development.
Parainfluenza viruses (choice C) cause croup, which is characterized by a dry,
"barking" cough. It is more of a tracheitis, bronchitis, and bronchiolitis than a
pneumonitis. No heterophile antibodies are produced in these patients.
Respiratory syncytial virus (choice D) causes an atypical pneumonitis in infants. It is
usually diagnosed by the observation of syncytial masses in respiratory secretions.
Cold hemagglutinins and Strep MG agglutinins are absent.
Streptococcus pneumoniae (choice E) is the number one cause of pneumonia in
adults. It also causes septicemia and meningitis in the elderly. The patient has a
classical acute pneumonia with a productive cough, high fever with chills,
leukocytosis, tachycardia, rapid respirations and other signs of serious respiratory
disease. A vaccine, composed of the capsular carbohydrate of 23 serotypes of this
organism, is routinely given to individuals over the age of 60, as well as to
individuals with splenic abnormalities (e.g., sickle cell disease) who are at increased
risk for pneumococcal sepsis.
________________________________________
14The correct answer is F. Aneurysm of the posterior communicating artery is the
second most common aneurysm of the circle of Willis (anterior communicating
artery is most common) and can result in third cranial nerve palsy (paralysis). The
oculomotor nerve (CN III) innervates the levator palpebrae muscle. CN III paralysis
would therefore result in ptosis (drooping of the upper eyelid). CN III also innervates
all of the extraocular muscles, except for the superior oblique (CN IV) and the lateral
rectus muscles (CNVI). Thus, CN III palsy would result in unopposed action of the

superior oblique and lateral rectus muscles, causing the affected eye to look down
and out. CN III also supplies parasympathetic innervation to the sphincter muscle of
the iris (which constricts the pupil) and to the ciliary muscle. Interruption of this
pathway leads to a dilated and fixed pupil and to paralysis of accommodation.
Note that this question teaches you about another Boards-favorite pathology:
subarachnoid hemorrhage (SAH). (In this case, it was due to rupture of a posterior
communicating artery aneurysm). A classic clue to the diagnosis is a patient
presenting with "the worst headache of their life." When you are presented a case of
sudden severe headache, SAH should rank highly on your differential diagnosis list.
The anterior cerebral artery (choice A) supplies the medial surface of the cerebral
hemisphere, from the frontal pole to the parieto-occipital sulcus. Occlusion may
produce hypesthesia and paresis of the contralateral lower extremity.
The anterior choroidal artery (choice B) arises from the internal carotid artery and is
not part of the circle of Willis. It perfuses the lateral ventricular choroid plexus, the
hippocampus, parts of the globus pallidus and posterior limb of the internal capsule.
The anterior communicating artery (choice C) connects the two anterior cerebral
arteries. It is the most common site of aneurysm in the circle of Willis and may
cause aphasia, abulia (impaired initiative), and hemiparesis.
The middle cerebral artery (choice D) supplies the lateral convexity of the cerebral
hemisphere, including Broca's and Wernicke's speech areas and the face and arm
areas of the motor and sensory cortices. It also gives rise to the lateral striate
arteries, which supply the internal capsule, caudate, putamen, and globus pallidus.
The middle cerebral artery is the most common site of stroke.
The ophthalmic artery (choice E) enters the orbit with the optic nerve (CN II) and
gives rise to the central artery of the retina. Occlusion results in blindness.
The posterior inferior cerebellar artery (choice G) supplies the dorsolateral medulla
and the inferior surface of the cerebellar vermis. Occlusion may result in
Wallenberg's syndrome: cerebellar ataxia, hypotonia, loss of pain and temperature
sensation of the ipsilateral face, absence of corneal reflex ipsilaterally, contralateral
loss of pain and temperature sensation in the limbs and trunk, nystagmus,
ipsilateral Horner's syndrome, dysphagia, and dysphonia.
________________________________________
15The correct answer is C. A variety of glycogen storage diseases exist,
corresponding to defects in different enzymes in glycogen metabolism; most of
these involve the liver. McArdle's disease (Type V glycogen storage disease), due to
a defect in muscle phosphorylase, is restricted to skeletal muscle. The presentation
described in the question stem is typical. Many affected individuals also experience
myoglobinuria. Definitive diagnosis is based on demonstration of myophosphorylase
deficiency.
Hartnup's disease (choice A) is a disorder of amino acid transport.
Krabbe's disease (choice B) is a lysosomal storage disease.

Niemann-Pick disease (choice D) is a lysosomal storage disease.


Von Gierke's disease (choice E) is a glycogen storage disease with prominent
involvement of liver, intestine, and kidney.
________________________________________
16The correct answer is D. The disease is Down syndrome (trisomy 21). In addition
to mental retardation and the characteristic physical findings described in the
question stem, duodenal atresia is fairly common, as evidenced by the "double
bubble" sign on x-ray. These children are also likely to have various cardiac
anomalies; endocardial cushion defect is the most common.
Atrial septal defect (choice A) is one of the most common genetic defects in the
general population, but is less common than endocardial cushion defect in patients
with Down syndrome.
Berry aneurysms (choice B), also known as saccular aneurysms, are typically
located in the circle of Willis on the ventral surface of the brain. They occur more
frequently in patients with adult polycystic disease. Rupture can produce
subarachnoid hemorrhage.
Coarctation of the aorta (choice C) occurs more commonly in females with a 45, XO
genotype (Turner syndrome).
Tetralogy of Fallot (choice E) is the most common cause of early cyanosis, consisting
of a ventricular septal defect, right ventricular outflow tract obstruction, an
overriding aorta, and right ventricular hypertrophy.
________________________________________
17The correct answer is E. All of the laboratory data in this pregnant woman are
normal, hence no further study is necessary. In a normal pregnancy, both the
plasma volume and RBC mass are increased with a greater increase in the plasma
volume than RBC mass (2:1 ratio). This has a dilutional effect on many laboratory
tests.
Increasing plasma volume in pregnancy increases the creatinine clearance (choice
A) due to the expected elevation in the glomerular filtration rate (GFR). The
reference intervals for serum blood urea nitrogen and creatinine are lower than
normal, due to the dilutional effect of increased plasma volume and increased
clearance of both analytes in the urine caused by the rise in the GFR.
The threshold for glucose is reduced in pregnancy, so patients can have a positive
dipstick test for glucose in the presence of a normal serum glucose. Therefore, an
oral glucose tolerance test (choice B) is not indicated.
The hemoglobin (Hb) concentration in pregnancy is normally decreased because of
the dilutional effect of increased plasma volume. Since the Hb is normal (for a
pregnant woman) in this patient, a serum ferritin (choice C) to rule out iron
deficiency is unnecessary. Furthermore, iron deficiency is usually associated with a
low MCV (microcytic anemia), and her MCV is normal.

Although sickle disease is the most common genetic hemoglobinopathy among


African Americans, the patient is not anemic, so there is no reason to order a sickle
cell preparation (choice D).
________________________________________
18The correct answer is D. The endometrial phase with small glands is the
proliferative phase; the one with large glands with secretory cells is the secretory
phase. Estrogen (choice C) is necessary for both phases, but it is the addition of
progesterone (choice D), secreted by the corpus luteum after the Graafian follicle
ruptures, that triggers the switch from proliferative to secretory endometrium.
Glucocorticoids (choice B) and the mineralocorticoid aldosterone (choice A) are
secreted by the adrenal glands. They do not produce the endometrial changes
described.
Thyroxine (choice E) is secreted by the thyroid gland, and is unrelated to the
observed morphologic changes in the endometrium.
________________________________________
19The correct answer is D. The distinctive cell balls described are broken-off
papillary clusters, and are considered pathognomic for papillary carcinoma of the
thyroid. This is the most common form of thyroid carcinoma. It tends to present in
the 3rd to 5th decade and shows a modest female predominance. Despite its
propensity for local lymphatic intrusion (which may cause multifocality of tumor in
the thyroid or cervical lymph node metastases), the tumor generally has an
excellent prognosis with 90% 20-year survival.
Follicular carcinoma (choice A) is characterized by follicular cells and colloid on
aspiration, and cannot be reliably distinguished from thyroid adenoma.
Distinctive features of Hashimoto's disease (choice B) on aspiration are
lymphocytes, plasma cells, and macrophages.
The most distinctive feature of medullary carcinoma (choice C) on aspiration is the
presence of amyloid.
Thyroid adenoma (choice E) shows follicular cells and colloid on aspiration, and
cannot be reliably distinguished from follicular carcinoma.
________________________________________
20The correct answer is D. This is a straightforward question relating to the
definition of Nissl bodies. Rough endoplasmic reticulum present in neurons are
called Nissl bodies. They stain intensely with basic dyes and are found in the cell
body and proximal dendrites, but not in the axon hillock or axon.
________________________________________
21The correct answer is A. Sawtooth waves appearing in bursts are associated with
REM sleep.

Stage 1 (choice B) is associated with 4-7 Hz theta waves.


Stage 2 (choice C) is associated with 12-14 Hz sleep spindles and K-complexes.
Stage 3 (choice D) is associated with < 4 Hz, high-amplitude delta waves.
Stage 4 (choice E) is characterized by an EEG composed of about 50% delta waves.
Note that beta waves (15-18 Hz) occur during periods of more intense mental
activity while awake. Alpha waves (8-12 Hz) occur during awake, relaxed states.
REM is the stage of sleep that most resembles the awake state on the EEG.
________________________________________
22The correct answer is E. The disease is poliomyelitis. Most infections with
poliovirus cause only the influenza-like symptoms, but a small percentage progress
to paralytic poliomyelitis. The most common causes of death are aspiration and
airway obstruction as a result of bulbar paralysis and paralysis of respiratory
muscles. Arrhythmias can also be life-threatening.
Acute renal failure (choice A) is usually not seen in poliomyelitis, although the
bladder may become paralyzed.
Bowel paralysis (choice B) can be seen, but is not usually life-threatening.
Fulminant liver failure (choice C) is not a feature of poliomyelitis.
Gastrointestinal bleeding (choice D) can be seen in poliomyelitis, but is not usually
life-threatening.
________________________________________
23The correct answer is D. The ratio of cells in bone marrow developing along
myeloid lines to cells developing along erythroid lines is 3:1. An alternative way to
remember the normal marrow composition is that it typically contains about 60%
granulocytes and their precursors; 20% erythroid precursors; 10% lymphocytes,
monocytes, and their precursors; and 10% unidentified or disintegrating cells. These
numbers are worth remembering, because shifts away from normal values may be a
subtle clue to marrow abnormalities.
________________________________________
24The correct answer is E. An obese adult with glucosuria, but not ketonuria, likely
has type 2 diabetes mellitus. Type 2 diabetes is characterized by insulin resistance
resulting in hyperglycemia and increased serum osmolarity. The dehydration
associated with osmotic diuresis makes the hyperosmolarity worse. As the
osmolarity increases above 330 mOsm/L, the osmotic loss of water from neurons is
sufficient to produce coma. In nonketotic, hyperosmolar coma, blood glucose values
can range from 800 to 2,400 mg/dL and produce serum osmolarities of 330-440
mOsm/L.
Since type 2 diabetes is due to insulin resistance, plasma levels of insulin are
usually normal to increased. Because b cells secrete insulin and C-peptide in a 1:1

ratio, plasma concentration of C-peptide would also be normal to increased (not


decreased, choice A) in type 2 diabetes.
Even small amounts of insulin are sufficient to prevent ketosis. In type 2 diabetes
there is enough insulin effect to prevent significant lipolysis and subsequent
formation of excess ketone bodies. Hence, acidosis (choice B) is not typically
associated with this disorder. Since b-hydroxybutyrate is a ketone body, its
concentration in plasma is not likely to be increased (choice D).
While there is a significant autoimmune component to type 1 diabetes, type 2
diabetes is not associated with increased circulating antibodies (choice C) against b
cell proteins such as glutamic acid decarboxylase.
________________________________________
25The correct answer is B. Neuromuscular development is sufficient to allow fetal
movement in the eighth week of life. Other features of week 8 include the first
appearance of a thin skin, a head as large as the rest of the body, forward-looking
eyes, appearance of digits on the hands and feet, appearance of testes and ovaries
(but not distinguishable external genitalia), and a crown-rump length of
approximately 30 mm. By the end of the eighth week, nearly all adult structures
have at least begun to develop, and the fetus "looks like a baby".
________________________________________
26The correct answer is A. The Rb gene is an example of a tumor suppressor gene.
Tumor suppressor genes encode proteins that downregulate cell growth;
consequently, their deletion leads to the development of cells with a growth
advantage over normal cells. Even if you know nothing about the Rb protein, choice
A is still the only logical answer because it is the only example of a protein that, if
absent, would favor cell growth. The Rb protein binds to transcription factors in the
nucleus, preventing cells from progressing from the S1 to M stages of the cell cycle.
Children born with a 13q14 deletion have only one chromosome encoding Rb;
therefore only a single "hit " is required to completely knock out Rb production and
lead to the development of retinoblastoma. All of the incorrect choices are proteins
encoded by oncogenes, rather than tumor suppressor genes. Oncogenes favor
tumorigenesis through overexpression, not deletion.
Growth factors (choice B) are oncoproteins that are produced by tumors and have a
positive feedback effect. Examples of growth factors are PDGF and fibroblast growth
factor; the oncogenes encoding them are sis and hst-1, respectively.
The prototypical growth factor-binding protein (choice C) is ras, which is mutated in
a large variety of cancers. Ras normally functions as an activator of protein kinases
that regulate cell growth. Overactivity of the ras protein is highly mitogenic.
Growth factor receptors (choice D) are either expressed as mutant forms or
overexpressed in tumors, leading to upregulation of growth. An example of a growth
factor receptor oncogene is erb-B2, present in some breast cancers.

Transcription activators (choice E) are DNA-binding proteins that promote DNA


transcription. Amplification of these oncogenes causes cancer by promoting the
transcription of growth-related genes.
________________________________________
27The correct answer is A. The obturator nerve innervates the muscles of the
medial compartment of the thigh. These include the adductor longus, adductor
brevis, adductor magnus, and gracilis muscles. The adductor magnus is also
innervated by the tibial nerve.
The biceps femoris (choice B) is in the posterior compartment of the thigh. The long
head of the biceps femoris is innervated by the tibial portion of the sciatic nerve,
and the short head of the biceps femoris is innervated by the common peroneal
portion of the sciatic nerve.
The rectus femoris (choice C) and vastus medialis (choice E) are two of the four
heads of the quadriceps femoris muscle. All four heads of the quadriceps femoris
muscle are in the anterior compartment of the thigh, and are innervated by the
femoral nerve.
The sartorius muscle (choice D) is in the anterior compartment of the thigh, and is
innervated by the femoral nerve.
________________________________________

28The correct answer is A. The patient has Burkitt's lymphoma. This type of
lymphoma is a high-grade B-cell lymphoma that occurs in endemic form in Africa (it
is the most common neoplasm in children in an equatorial belt that includes Africa
and New Guinea) and sporadically in the United States and Europe. The sporadic
form is often in an abdominal site and occurs in young adults. The African form of
Burkitt's lymphoma has been strongly associated with antibodies directed against
Epstein-Barr virus; the association is weaker in sporadic cases. A characteristic
translocation, t(8;14) (q24.l3;q32.33) has been described.
Hepatitis B (choice B) is associated with hepatocellular carcinoma. t(9;22) is the
Philadelphia chromosome, which is seen in some cases of CML and AML.
Herpesvirus (choice C) does not have a strong tumor association, although a link to
cervical cancer has intermittently been proposed. CD5 is a marker seen in small
lymphocytic and mantle cell lymphomas.
HIV (choice D) is linked to Kaposi's sarcoma (and AIDS). Some patients also develop
primary lymphomas (not usually Burkitt's). CD4 is a marker for helper T cells and
some T cell lymphomas.
Human papillomavirus (choice E) is linked with common warts, genital condylomata,
and genital cancers. t(2;5) is linked to anaplastic large cell lymphoma.
________________________________________

29The correct answer is E. The patient is presenting with hypertension and signs
and symptoms of benign prostatic hyperplasia (BPH). The essential diagnostic
characteristics of BPH include a decrease in the force and caliber of the urinary
stream, nocturia, high post-void residual volume, urinary retention, and azotemia.
Terazosin is an alpha-adrenergic antagonist that selectively blocks alpha-1 receptors
in vascular smooth muscle producing relaxation. It is indicated for the treatment of
both hypertension and BPH.
Finasteride (choice A) is a specific inhibitor of 5-alpha reductase, an enzyme that
converts testosterone into the potent androgen dihydrotestosterone (DHT) in the
prostate gland. This agent is indicated only for the treatment of BPH.
Guanfacine (choice B) is a centrally acting alpha-2 agonist indicated for the
treatment of mild to moderate hypertension.
Hydralazine (choice C) is a vasodilator indicated for the treatment of hypertension
and to decrease afterload in patients with congestive heart failure.
Labetalol (choice D) is both an alpha- and beta-receptor blocking agent indicated for
the treatment of hypertension.
________________________________________
30The correct answer is E. The superior pancreaticoduodenal artery is a branch of
the gastroduodenal artery, which is a branch of the common hepatic artery, itself a
branch of the celiac trunk. The inferior pancreaticoduodenal artery is a branch of
the superior mesenteric artery. Occlusion of the celiac trunk would allow blood from
the superior mesenteric artery to reach the branches of the celiac trunk via the
connections between the superior and inferior pancreaticoduodenal arteries.
The left gastric and right gastric arteries (choice A) both receive their blood from the
celiac trunk. The left gastric artery is a direct branch of the celiac trunk. The right
gastric artery is usually a branch of the proper hepatic artery, which is a branch of
the common hepatic artery (a branch of the celiac trunk).
The left and right gastroepiploic arteries (choice B) both receive their blood supply
from the celiac trunk. The left gastroepiploic artery is a branch of the splenic artery,
which is a branch of the celiac trunk. The right gastroepiploic artery is a branch of
the gastroduodenal artery, which is a branch of the common hepatic artery (a
branch of the celiac trunk).
The proper hepatic and gastroduodenal arteries (choice C) are branches of the
common hepatic artery, which is a branch of the celiac trunk.
The right colic and middle colic arteries (choice D) are both branches of the superior
mesenteric artery.
________________________________________
31The correct answer is C. The only conclusion that can be drawn from this data is
that Drug X is less efficacious than Drug Y. Efficacy is defined as the maximum
effect that can be produced by a drug, regardless of dose. Drug X can only produce

a 50% change in resistance, whereas Drug Y can produce a 75% change in


resistance. Therefore, Drug X is less efficacious than Drug Y.
A volume of distribution (choice A) is the ratio of the amount of drug in the body to
its plasma concentration. In this experiment, we do not know the total amount of
drug used or the plasma concentration. For that matter, we do not even know if this
is an in vivo experiment. Thus, no conclusions can be drawn about volume of
distribution.
The half-life (choice B) is the time it takes for the concentration of a drug to fall 50%
from its previous measurement. There is no information given to determine half-life.
The potency (choice D) is the dose or concentration required to produce 50% of the
drug's maximal effect. We cannot determine the potency of Drug Y from this
question.
The LD50 (choice E) is the dose that causes death in 50% of a population of subject.
The experiment described above does not describe a population study, nor does it
give any indication about the toxicity of the drug.
________________________________________
32The correct answer is E. Hip fracture and prolonged bed rest are classic risk
factors for the development of pulmonary thromboemboli (PE). Common clinical
manifestations of PE are hypoxia (due to ventilation/perfusion mismatch) despite a
normal chest x-ray, tachycardia, and delirium in older patients.
Cerebral hemorrhage (choice A) might cause delirium but would not directly cause
hypoxia unless the patient was hypoventilating (e.g., because of brainstem
involvement).
Cerebral infarction (choice B) could produce delirium but would not directly cause
hypoxia unless the patient was hypoventilating.
Myocardial infarction (choice C) could account for delirium and tachycardia, but not
for hypoxia with a normal chest x-ray. Severe congestive heart failure after
myocardial infarction could cause hypoxia due to pulmonary edema, but the chest
x-ray would not be normal.
Pulmonary infarction (choice D) may cause delirium, tachycardia, and hypoxia, but
the chest x-ray may be abnormal. A chest x-ray performed within 12-36 hours after
a pulmonary infarct may reveal a peripherally located, wedge-shaped infiltrate.
________________________________________
33The correct answer is A. The woman has a urinary tract infection. Pregnancy can
predispose for urinary tract infection by compressing urinary tract structures and
partially impairing urinary flow through the urinary tract. The presence of white
blood cell casts specifically implies renal involvement, since these form when the
leukocytes are compressed together in the renal tubules. Similarly, the presence of
red blood cell casts in a bloody urine would indicate that at least some of the
bleeding was occurring in the kidney.

Infection of other sites in the urinary tract, including ureters (choice B), bladder
(choice C), and urethra (choice D), does not cause cast formation.
While spread of a urinary tract infection to a pregnant uterus (choice E) is always of
concern in a pregnant woman, the presence of casts does not specifically suggest
that this has occurred.
________________________________________
34The correct answer is D. This patient is exhibiting the Marcus-Gunn phenomenon.
When light strikes the retina, the pupillary light reflex is automatically triggered,
leading to simultaneous constriction of both pupils. In the absence of adequate light
entering the eye, for example following retinal detachment or optic neuritis,
paradoxical dilatation of the pupils occurs. The retina receives far less light than it
normally would, and the pupils dilate in order to absorb as much light as possible.
________________________________________
35The correct answer is E. The urachus is the derivative of the allantoic duct, which
passes from the urogenital sinus to the umbilical cord. Normally, this duct fuses and
is no longer patent. The adult derivative is the median umbilical ligament, which lies
in the midline along the interior surface of the anterior abdominal wall. It passes
from the upper end of the bladder to the umbilicus.
The lateral umbilical fold (choice A) is the fold of parietal peritoneum that covers the
inferior epigastric artery and vein on the interior surface of the anterior abdominal
wall.
The medial umbilical fold (choice B) is the fold of parietal peritoneum that covers
the medial umbilical ligament, the adult derivative of the umbilical artery (see
below).
The medial umbilical ligament (choice C) is the adult derivative of the distal portion
of the umbilical artery. The umbilical artery arises from the internal iliac artery. It
passes along the bladder and then the anterior abdominal wall to reach the
umbilicus. Prenatally, this artery carries fetal blood to the placenta, where it gains
oxygen and nutrients. Postnatally, the proximal part of the umbilical artery remains
patent and supplies blood to the superior surface of the bladder. Distal to the
bladder, the artery becomes fibrotic and is known as the medial umbilical ligament.
The median umbilical fold (choice D) is the fold of parietal peritoneum that covers
the median umbilical ligament, the adult derivative of the urachus.
________________________________________
36The correct answer is A. Apoptosis refers to programmed cell death, and in this
situation, appears to be a means for the elimination of unneeded osteoblasts as the
extracellular matrix is reshaped and the healing process continues. Two basic
morphologic features of apoptosis are seen in this cell. First, cytoplasmic blebbing,
in which round, elongated structures (blebs) extend from the main cell body, is
evident. Some blebs appear as though in clusters. Second, nuclear blebbing, in

which the nucleus shows the same irregular type of blebs or protrusions, is
apparent.
This cell does not have features of a macrophage (choice B) such as lysosomes, or a
heterogeneous cytoplasm. Ribosome-filled blebs are not features of macrophages.
No chromosomes are present, hence this cannot be a dividing cell (choice C).
Cytoplasmic and nuclear blebbing are not features of necrosis (choice D). In
necrosis, nuclei are washed out (karyolysis), markedly compact (pyknotic), or
fragmented (karyorrhexis).
A typical osteoblast (choice E) is seen to the left of the blebbing cell. This cell has
extensive rough endoplasmic reticulum and a Golgi apparatus with elliptical and
round granules.
________________________________________
37The correct answer is A. Features of Down's syndrome (trisomy 21) in children
include mental retardation, epicanthal folds, dysplastic ears, hypotonia, a horizontal
palmar crease (simian crease), redundant neck skin, and a short trunk. However,
most of these children eventually grow to adulthood. At that point, the aging
parents may have to deal with a physically strong and healthy mentally retarded
individual who is experiencing a deterioration in mental function. This deterioration
may be accompanied by aggressive behavior (as in the elderly with Alzheimer's
disease) arising out of the patient's diminishing ability to reason or understand his
environment.
Edwards' syndrome (choice B), or trisomy 18, causes death in infancy.
Characteristics include rocker-bottom feet, low-set ears, micrognathia, congenital
heart disease, and mental retardation.
Fragile X syndrome (choice C) is associated with enlarged testes as well as mental
retardation. The condition is unusual in that it is related to expansion of a CGG
repeat sequence located on the X chromosome.
Patau syndrome (choice D), or trisomy 13, is characterized by severe mental
retardation, microcephaly, microphthalmia, polydactyly, cleft lip and palate, renal
defects, and cardiac abnormalities. Affected infants typically die before the age of 1.
The effects of supernumerary Y chromosomes (choice E; most commonly XYY)
include increased stature, aggressive behavior, and infertility, but a supernumerary
Y chromosome is sometimes found in otherwise normal individuals.
________________________________________
38The correct answer is A. This woman is afflicted with temporal (giant cell)
arteritis. This is a panarteritis that can involve any of the branches of the aortic
arch, but which classically affects the branches of the carotid system. Temporal
arteritis commonly (40-50%) produces visual disturbances, including blindness due
to involvement of the ophthalmic artery. Biopsy of affected segments of arteries
may be diagnostic, showing granulomatous lesions with giant cells, although non-

specific inflammatory infiltrates and/or intimal fibrosis (generally with no disruption


of the internal elastic lamina) are also seen with some frequency.
Isolated deafness (choice B) is not commonly seen with temporal arteritis.
Stroke may occur in temporal arteritis, but this would not likely produce loss of all
tactile sensation (choice C).
Loss of the ability to speak (choice D) might conceivably result from a stroke
affecting the inferior frontal gyrus, or from a lesion of the brainstem centers
controlling phonation, but visual disturbances are more common than stroke in
temporal arteritis.
Paralysis (choice E) is not a common complication of temporal arteritis.
________________________________________
39The correct answer is E. The ventral pancreatic bud normally rotates around the
duodenum to fuse with the dorsal pancreatic bud. Both pancreatic buds form from
evaginations from the second part of the duodenum, hence the rotation is around
the second part of the duodenum. The normal rotation is around the right side of
the embryonic duodenum. Annular pancreas results from the ventral pancreatic bud
dividing and rotating around both the right and left sides of the second part of the
duodenum, thus encircling it.
The dorsal pancreatic bud (choices A, B, and C) does not rotate around the
duodenum and therefore is not the cause of annular pancreas.
The ventral pancreatic bud does not form from the first part of the duodenum
(choice D) and therefore does not rotate around this part of the duodenum.
________________________________________
40The correct answer is B. The patient's clinical presentation is typical of primary
atypical pneumonia. In contrast to bacterial pneumonia, primary atypical
pneumonia presents with the following features:
- Caused by M. pneumoniae; less frequently by viruses (influenza, respiratory
syncytial virus, adenovirus, rhinoviruses, rubeola and varicella virus), Chlamydia, or
Coxiella burnetii
- Characterized pathologically by interstitial, rather than intra-alveolar, inflammation
- Characterized clinically by nonspecific symptomatology and few "localizing"
symptoms
Why is M. pneumoniae, and not influenza virus (choice A) or respiratory syncytial
virus (choice D), the cause of this patient's pneumonia? First, M. pneumoniae
infections are often associated with the appearance of cold agglutinins in the serum,
detection of which is diagnostically important. Second, the patient responded
quickly to treatment with erythromycin, an antibiotic effective against M.
pneumoniae, but obviously not effective in treating viral infections.

Pneumocystis carinii (choice C) is a fungal organism causing pneumonia in severely


immunocompromised hosts, especially AIDS patients. P. carinii pneumonia (PCP) is
characterized by accumulation of a frothy exudate containing numerous organisms
within alveolar spaces. Also, P. carinii is not sensitive to erythromycin. The drug of
choice for treatment of PCP is trimethoprim-sulfamethoxazole.
Streptococcus pneumoniae (choice E) is the usual causative agent of lobar
pneumonia, characterized by consolidation of a single lobe due to intra-alveolar
acute inflammatory exudation. Lobar pneumonia is more prevalent in young,
healthy individuals, whereas primary atypical pneumonia favors old, debilitated
patients. S. pneumoniae is highly sensitive to penicillin.
________________________________________
41The correct answer is A. Cholecystokinin, or CCK, is synthesized in the duodenal
and jejunal mucosa and stimulates gall bladder contraction and pancreatic enzyme
secretion. Other functions include slowing of gastric emptying, an atrophic effect on
the pancreas, and secretion of antral somatostatin, which in turn, decreases gastric
acid secretion.
Gastric inhibitory peptide, or GIP (choice B), stimulates pancreatic insulin secretion
at physiologic doses and inhibits gastric acid secretion and gastric motility at
pharmacologic doses.
Gastrin (choice C) prepares the stomach and small intestine for food processing,
including stimulating secretion of HCl, histamine, and pepsinogen. It also increases
gastric blood flow, lower esophageal sphincter tone, and gastric contractions.
Secretin (choice D) stimulates secretion of bicarbonate-containing fluid from the
pancreas and biliary ducts.
Vasoactive intestinal polypeptide, or VIP (choice E), relaxes intestinal smooth
muscle and stimulates gut secretion of water and electrolytes.
________________________________________
42The correct answer is D. This patient expresses feelings of anxiety which are only
relieved by compulsive ritual behavior (checking the doors and windows
excessively).
Adjustment disorder with anxiety (choice A) generally occurs in response to an
identifiable stressor.
Agoraphobia without a history of panic disorder (choice B) is characterized by a fear
of all large enclosed or open spaces when alone.
In generalized anxiety disorder (choice C), excessive worry or anxiety is present
most of the time the person is awake, not just in a specific situation.
In panic disorder with agoraphobia (choice E), panic attacks occur in uncued
situations. This patient's symptoms appear consistently when she leaves for work in
the morning.

________________________________________
43The correct answer is E. Bone marrow suppression, diarrhea, and alopecia are the
most common side effects seen with cancer chemotherapy regimens. Vincristine, a
mitotic inhibitor, is a chemotherapeutic agent that is not associated with the
development of bone marrow suppression and would be the most appropriate agent
to use in this patient. Vincristine is effective in the treatment of acute lymphoblastic
leukemia and other leukemias, Hodgkin's disease, lymphosarcoma, neuroblastoma,
and various other types of cancer. Bleomycin is another antineoplastic agent that
does not cause bone marrow suppression.
Busulfan (choice A) is an alkylating agent primarily used in the palliative treatment
of chronic myelogenous leukemia; it is known to cause severe bone marrow
suppression. As a general rule, the alkylating agents typically produce severe
immunosuppressive effects.
Cisplatin (choice B) is another alkylating agent indicated for the treatment of
metastatic testicular and ovarian tumors in combination with other agents. This
agent can also cause profound bone marrow suppression.
Cyclophosphamide (choice C) is classified as a nitrogen mustard, a subcategory of
the alkylating agents. It is primarily used to treat breast, testicular, and other solid
tumors, as well as leukemia and lymphoma. This drug suppresses bone marrow.
Paclitaxel (choice D) is an antimicrotubule agent typically used in the treatment of
ovarian and breast cancer. Profound neutropenia is typically seen with this agent.
________________________________________
44The correct answer is B. Lactic acidosis, characterized by elevated blood lactate,
decreased arterial blood pH, decreased bicarbonate, and electrolyte imbalances
with an elevated anion gap (normal = 10 - 12), is a rare but serious complication of
metformin administration. The onset of lactic acidosis is usually accompanied by
several non-specific signs and symptoms including malaise, myalgias, respiratory
distress and increased somnolence. There may be associated hypothermia,
hypotension, and resistant bradyarrhythmias as the condition progresses.
Glyburide (choice A) is a sulfonylurea associated with the development of
hypoglycemia and cholestatic jaundice (a rare complication).
Miglitol (choice C) is an alpha-glucosidase inhibitor commonly associated with the
development of abdominal discomfort and flatulence.
Repaglinide (choice D) is the non-sulfonylurea moiety of glyburide; it is commonly
associated with hypoglycemia, nausea and vomiting.
Idiosyncratic hepatocellular injury has been reported during the usage of
troglitazone (choice E). The hepatic injury is usually reversible, but rare cases of
hepatic failure requiring liver transplantation or leading to death have been
reported with this agent.
________________________________________

45The correct answer is A. The dermis contains skin appendages (e.g., hair follicles)
that contain epithelial stem cells. In the process of healing a large area where the
epidermis has been lost, but the dermis is intact, re-epithelialization occurs by
ingrowth of epidermal cells from the underlying skin appendages, as well as from
the intact epidermis along the wound edges.
None of the other cell types are known to directly contribute to the regeneration of
epidermis over abraded skin.
________________________________________
46The correct answer is D. A forced expiration is the simplest test of lung function.
The individual breathes in as much air as the lungs can hold and then expels the air
as rapidly and as far as possible. The forced vital capacity (FVC) is the vital capacity
measured with a forced expiration (FVC = 3 L for patient Y). The forced expiratory
volume in one second (FEV1) is the amount of air that can be expelled from the
lungs during the first second of a forced expiration (FEV1 = 2.7 L for patient Y). The
FEV1/FVC ratio has diagnostic value for differentiating between normal, obstructive,
and restrictive patterns of a forced expiration. The FEV1/FVC ratio for the healthy
individual (X) is 4 L/5 L = 80% and the FEV1/FVC for patient Y is 2.7/3.0 = 90%.
FEV1/FVC is a function of airway resistance. Increases in airway resistance
associated with asthma (choice A), bronchospasm (choice B), emphysema (choice
C), and old age (choice E) tend to decrease the FEV1/FVC ratio below its typical
normal value of 80%. FEV1/FVC is often increased with interstitial fibrosis because
of increased radial traction of the airways, i.e., the airways are held open to a
greater extent at any given lung volume, reducing their resistance to air flow. The
increase in elastic recoil also makes it difficult to breathe deeply, which decreases
FVC. This combination of decreased FVC along with normal or slightly increased
FEV1/FVC is characteristic of fibrotic lung disease.
________________________________________
47The correct answer is D. The malignant skin tumor that forms "pearly papules" on
sun-exposed skin is basal cell carcinoma. A characteristic feature of this tumor is
palisading of the basal cell-like nuclei at the edge of clusters of cells penetrating
into the dermis. Basal cell carcinomas almost never metastasize, but can be very
locally destructive, and resection of large ones on the face may produce disfiguring
scars.
Viral cytoplasmic inclusions (choice A) are a feature of the infectious lesion
molluscum contagiosum.
Keratin pearls (choice B) are a feature of squamous cell carcinoma.
Melanin (choice C) and S-100 positivity (choice E) with immunohistochemical stains
are features of melanoma.
________________________________________
48The correct answer is A. The diseases listed in the answers are all inherited
disorders of bilirubin metabolism that are usually discussed together. Crigler-Najjar

syndrome (choices A and B) and Gilbert's syndrome (choice D) are both


unconjugated hyperbilirubinemias, while Dubin-Johnson syndrome (choice C) and
Rotor syndrome (choice E) are conjugated hyperbilirubinemias. Crigler-Najjar
syndrome (particularly the type I variant) is rare and extremely serious (with the
presentation given in the question stem) while Gilbert's syndrome is completely
benign. The type II variant of Crigler-Najjar is intermediate in severity between
Gilbert's and the Type I. Dubin-Johnson and Rotor syndrome are also relatively
benign; Dubin-Johnson is distinguished from Rotor syndrome by the presence of a
black pigment of unknown composition in the liver.
________________________________________
49The correct answer is A. The patient is presenting with signs and symptoms of
fibrositis (fibromyalgia). This disorder is most commonly seen in women between
the ages of 20 and 50, and is associated with widespread chronic musculoskeletal
pain that improves with exercise, chronic fatigue, and sometimes, severe
headaches. Examination typically reveals painful trigger points produced by
palpation of the trapezius and the lateral epicondyle of the elbow. Objective signs of
inflammation are absent and laboratory studies are normal. Patients with this
disorder are likely to respond to treatment with tricyclic antidepressants or skeletal
muscle relaxants with strong anticholinergic side effects, such as cyclobenzaprine.
One of the most effective agents in the treatment of this disorder is amitriptyline, a
tricyclic antidepressant commonly used in the treatment of depression, and as an
adjunctive pain medication.
Cefaclor (choice B) is a second generation cephalosporin. Since fibromyalgia is not
an infectious disorder, this agent would be ineffective in this patient.
Naproxen (choice C) is a non-steroidal anti-inflammatory drug indicated for the
treatment of mild-to-moderate pain. NSAIDs are generally ineffective in the
treatment of this disorder.
Oxycodone (choice D) is an opioid analgesic indicated for the treatment of
moderate to severe pain; opioids are ineffective in the treatment of fibromyalgia.
Prednisone (choice E) is a corticosteroid indicated for the treatment of a variety of
disorders caused by inflammation. Since this disease is not an inflammatory
condition, prednisone would be not be indicated for this patient.
________________________________________
50The correct answer is B. Pyridoxine, or vitamin B6, is sometimes depleted with
isoniazid (INH) use. Patients with pyridoxine deficiency may experience neurologic
symptoms, such as convulsions and fasciculations. The treatment of this disorder is
slow IV administration of 2-5 g of pyridoxine.
Niacin (choice A) deficiency, also known as pellagra, is a disease that involves
several organs, including the skin, the gastrointestinal system, and the nervous
system. A useful mnemonic for remembering the symptoms of pellagra is the "4
D's": dermatitis, dementia, diarrhea, and death. These symptoms are not consistent
with the patient history described in the question.

Riboflavin (choice C) deficiency is not typically seen alone, but rather in conjunction
with other vitamin deficiencies. Dermatitis and glossitis are the most frequent
clinical manifestations.
Thiamine (choice D) deficiency, or beri-beri, presents with dry skin and paralysis,
rather than convulsions. Severe thiamine deficiency produces Wernicke's
encephalopathy, with disorientation, ataxia, and ophthalmoplegia. This deficiency is
typically seen in alcoholic patients.
Vitamin C (choice E) deficiency, or scurvy, causes defective growth and
maintenance of gums, blood vessels, joints, and teeth. These symptoms are due to
impaired collagen hydroxylation, a process that requires vitamin C.
Answers

________________________________________

1The correct answer is B. The most specific clue in the description is the presence of
erythematous patches with central clearing, known clinically as "target lesions,"
which are associated with erythema multiforme. Both erythema multiforme and its
severe, life-threatening version, known as Stevens-Johnson syndrome, are produced
by immune complex deposition in dermal blood vessels. In approximately 50% of
patients, no specific precipitating cause is identified. In the remainder of patients,
however, a variety of causes have been implicated, including certain infections
(herpes simplex, enteroviruses, Mycoplasma pneumoniae, Chlamydia,
histoplasmosis), drugs (penicillin, sulfonamides, phenytoin, aspirin, corticosteroids,
cimetidine, allopurinol, oral contraceptives), neoplasia (leukemia, lymphoma,
multiple myeloma, internal malignancy), sarcoidosis, and foods (notably emulsifiers
in margarine).
Erythema migrans chronicum (choice A) also produces an annular erythematous
rash with central clearing, but usually affects the thigh, groin, and axilla; it is
associated with Lyme disease.
Kaposi's sarcoma (choice C) causes purple lesions with no target lesions.
Psoriasis (choice D) causes erythematous plaques with silvery scale but does not
produce target lesions.
Urticaria (choice E) causes wheals that are intensely pruritic, but does not produce
target lesions.
________________________________________
2The correct answer is C. Raloxifene is a selective estrogen receptor modulator that
helps prevent osteoporosis by lessening bone resorption and reducing bone
turnover. It lowers risk for vertebral fractures by 40% to 50%. It is a bone-preserving
alternative for women who prefer to avoid estrogen. Raloxifene does not cause

breast pain and may lessen the risk for breast cancer in menopausal women. There
is also a favorable effect on LDL and cholesterol.
Calcitonin nasal spray (choice A) is an osteoclastic bone resorption inhibitor that
modestly increases bone mineral density and reduces the incidence of vertebral
fracture. Although it is an estrogen alternative for bone preservation, its impact on
hip fracture is not known. It is also lacks the anti-breast cancer properties of
raloxifene.
Oral conjugated estrogen (choice B) and transdermal estradiol (choice E) are not the
best choices, as this patient wants to avoid estrogen because of a strong family
history of breast cancer. The route of administration of estrogen has been shown to
have similar effects on bone preservation, even though the transdermal dosage is
generally half that of the oral dosage. Breast cancer risk, however, is slightly
increased with the unopposed higher dosage oral estrogen replacement.
Tamoxifen (choice D), while indicated in the long-term care of breast cancer
patients, is not alone useful for treatment or prevention of osteoporosis. Tamoxifen
is an anti-estrogen agent that competes with estrogen for binding sites.
________________________________________
3The correct answer is B. The nerve is the axillary branch of the posterior cord (also
called the axillary nerve), which can be damaged by either fracture or dislocations
of the humeral neck. The muscle is the deltoid muscle, which is supplied by this
nerve. The deltoid arises from the lateral portions of the clavicle and the acromion
and inserts into the deltoid tuberosity of the humerus.
The deltoid arises from both the acromion (choice A) and the clavicle.
The deltoid does not arise from the coracoid process (choices C, D, and E).
________________________________________
4The correct answer is B. The patient has neurosyphilis, specifically general paresis,
a term that means "general paralysis of the insane." In this late sequela of syphilitic
infection which occurs 5 to 20 years after infection, patients develop mental
deterioration, which precedes motor system deterioration, leading eventually to
"general paralysis" with mutism and incontinence. The abnormalities may be
conveniently recalled using paresis as a mnemonic for personality, affect,
hyperactive reflexes, Argyll Robertson pupils in the eyes, defects in the sensorium,
intellectual decline and deficient speech. Specific anti-treponemal tests such as FTAABS are usually positive on both serum and cerebrospinal fluid.
The cerebrospinal (CSF) glucose (choice A) in neurosyphilis is usually normal.
Gram's stain (choice C) of CSF will not demonstrate spirochetes in neurosyphilis.
The CSF lymphocyte count (choice D) is typically elevated in neurosyphilis, but this
is a non-specific finding.
The CSF neutrophil count (choice E) is usually normal in neurosyphilis.

________________________________________

5>The correct answer is A. The man described is probably suffering from avoidant
personality disorder, characterized by feelings of inadequacy and extreme
sensitivity to criticism, leading to social inhibition and withdrawal. These individuals
often avoid interpersonal relationships entirely rather than subject themselves to
the potential risk of criticism or rejection, although they may yearn for a more
satisfying personal life.
Borderline personality disorder (choice B) is characterized by unstable interpersonal
relationships, instability of affect, impulsivity, feelings of emptiness or anger and, in
some cases, paranoid or dissociative symptoms.
Dependent personality disorder (choice C) is characterized by the need for constant
support and reassurance, with unrealistic anxieties over being forced to fend for
oneself.
Narcissistic personality disorder (choice D) is characterized by excessive grandiosity
and an exaggerated sense of self-importance, accompanied by a feeling of
entitlement and a need for attention or admiration.
Schizotypal personality disorder (choice E) is characterized by eccentricities of
behavior, odd beliefs or magical thinking, and difficulties with social and
interpersonal relationships.

________________________________________

________________________________________
6The correct answer is D. Respiratory syncytial virus is the most common cause of
bronchiolitis and pneumonia in children younger than 1 year. Outbreaks occur
seasonally in winter and early spring. Infection does not result in lasting immunity,
and reinfection can occur.
Coronavirus (choice A) causes the common cold (nasal obstruction and discharge,
sneezing, no fever or mild fever, occasional sore throat, and/or cough) and acute
pharyngitis (sore throat, with or without cervical adenopathy, ulceration, and
conjunctivitis).
Influenza type A (choice B) is the leading cause of influenza. Influenza is a systemic
illness characterized by the sudden onset of fever, headache, myalgias, malaise,

and prostration, followed by cough, nasal obstruction, and sore throat. The lower
respiratory tract may also be involved.
Parainfluenza viruses (choice C) are the leading cause of croup, or acute
laryngotracheobronchitis, in children. This infection involves both the upper and
lower respiratory tracts. Inflammation in the subglottal area leads to hoarseness,
dyspnea, a barking cough, and inspiratory stridor.
Rhinovirus (choice E) is the most common cause of the common cold.
________________________________________
7The correct answer is D. Maturation of germ cells (spermatogenesis) within the
seminiferous tubules occurs in a concentric pattern with the less mature
spermatogonia near the basal lamina and the mature forms near the tubule center.
Spermatogonia are 2N cells and mature into larger primary spermatocytes (4N)
(choice A). These mature into secondary spermatocytes (2N) (choice B), and finally
into spermatids (1N) (choice C). Spermatids undergo spermiogenesis to become
mature spermatozoa (choice E). Acrosomes form from the Golgi apparatus and a
flagellum forms from microtubules. Unneeded organelles are shed. The seminiferous
tubules of a reproductive-age male should exhibit all stages of maturation, with
mature flagellated sperm in their centers.
________________________________________

8The correct answer is C. The presentation described is classic for pulmonary


hypertension, and, more specifically, the primary idiopathic form of pulmonary
hypertension. This rare condition is suspected of being related to the collagen
vascular diseases, since up to 50% of patients have antinuclear antibodies (despite
the absence of frank presentation of other autoimmune disease). Also, a similar,
known secondary form of pulmonary hypertension is sometimes seen in patients
with a wide variety of collagen vascular diseases, including systemic lupus
erythematosus, polymyositis, dermatomyositis, systemic sclerosis, and adult and
juvenile forms of rheumatoid arthritis. A wide variety of other conditions have also
been associated with secondary pulmonary hypertension, including shunts, left
atrial hypertension, chronic hypoxia, pulmonary embolism, drug reaction, hepatic
cirrhosis, and sickle cell disease. Both primary and secondary forms of pulmonary
hypertension are associated with prominent changes in the pulmonary vasculature,
which can include muscularization of smaller arterioles, concentric hypertrophy of
the intima ("onion skinning"), and a distinctive plexiform lesion (plexogenic
pulmonary vasculopathy) in which the smallest arterioles become markedly dilated
with lumens partially occluded by endothelial (or possibly mesenchymal) cells and
sometimes, thrombus. The prognosis of untreated pulmonary hypertension is poor.
However, the use of the vasodilator hydralazine with anticoagulation can slow the
course (fatal in about 3 years in untreated patients). If the pulmonary hypertension
is secondary, therapy of the primary disease can be helpful.

Unlike cor pulmonale, atrial fibrillation with mural thrombus (choice A) formation is
uncommon in primary pulmonary hypertension.
The absence of left ventricular findings on echocardiography tends to exclude
myocardial infarction as the source of the patient's findings (choice B).
The presence of enlargement of the main pulmonary artery excludes pulmonary
artery stenosis (choice D).
The clear lung fields exclude severe pulmonary fibrosis (choice E).
________________________________________
9The correct answer is B. The ureter leaves the renal pelvis and lies on the posterior
abdominal wall as it descends to the pelvis. It crosses the pelvic brim at the level of
the bifurcation of the common iliac artery. At this point it crosses anterior to the
origin of the external iliac artery to enter the pelvis.
The origin of the common iliac artery (choice A) is at the bifurcation of the
abdominal aorta, which occurs at the level of the fourth lumbar vertebra in the
midline of the abdomen. Both ureters are lateral to the aortic bifurcation.
The origin of the internal iliac artery (choice C) is at the bifurcation of the common
iliac artery, which occurs at about the level of the first sacral vertebra. The ureter
and the internal iliac artery both enter the pelvis with the ureter on the lateral side
of the artery.
The origin of the gonadal artery (choice D) (either the testicular artery or the
ovarian artery) is from the abdominal aorta at a variable level, usually between the
renal artery and the inferior mesenteric artery. The ureters are lateral to the origin
of this artery.
The origin of the renal artery (choice E) is from the abdominal aorta at about the
level of the second lumbar vertebra. The renal arteries pass laterally to enter the
renal pelvis. At this site, the ureter is posterior to the artery.
________________________________________
10The correct answer is D. This is an example of relative polycythemia, in which
there is an increased hematocrit or RBC count without a true increase in the total
number of body RBCs. What usually happens in these cases is a significant
reduction in plasma volume due to processes such as dehydration, vomiting,
diarrhea, or diuresis.
Cyanotic heart disease (choice A), via appropriate erythropoietin secretion, can
cause secondary absolute polycythemia.
Increased erythropoietin (choice B), whether appropriately or inappropriately
secreted, can cause secondary absolute polycythemia.
Polycythemia vera (choice C) causes primary absolute polycythemia with usually
low erythropoietin levels.

Renal cell carcinoma (choice E), via inappropriate erythropoietin secretion, can
cause secondary absolute polycythemia.
________________________________________
11The correct answer is E. Hypospadias, which is congenital displacement of the
urethral opening onto the ventral surface (underside) of the penis, is due to
malformation of the urethral groove and canal. Hypospadias frequently
accompanies other genitourinary anomalies, especially cryptorchidism. Isolated
hypospadias is repaired because the abnormal opening is often constricted, leading
to urinary retention and ascending urinary tract infections. Another important
consequence of hypospadias is sterility, which occurs if the opening is too close to
the base of the penis to permit normal ejaculation. Although hypospadias can occur
in isolation, it is strongly associated with other urogenital anomalies.
Bladder exstrophy (choice A) is a completely unrelated congenital malformation in
which the abdominal wall and anterior bladder wall form incompletely, and the
bladder mucosa is exposed to the environment.
Hydrocele (choice B) is a serous accumulation in the tunica vaginalis, often
producing a readily apparent scrotal mass. Hydrocele usually arises without any
obvious cause.
Phimosis (choice C) is the condition in which the foreskin cannot be retracted over
the head of the penis. It is usually either a congenital malformation or a
consequence of scarring.
Urachal cysts (choice D) are produced by areas of persistent urachal epithelium,
which may be present anywhere between the bladder and the umbilicus.
________________________________________
12The correct answer is E. Von Willebrand factor (vWF) is a self-polymerizing
clotting protein present in the serum and the subendothelial basal lamina, which
has binding sites for collagen, platelets, and fibrin. At a site of injury, vWF forms the
bridge between the exposed collagen fibers and platelets in circulation, stimulating
platelet degranulation and initiating the cellular component of the clotting cascade.
An equally important role for vWF is binding platelets to the newly formed fibrin
strands in a blood clot.
Factor VIII (choice A) and Factor IX (choice B) are clotting proteins of the intrinsic
pathway. Factor VIII acts in concert with activated Factor IX (IXa) to cleave Factor X
to Xa. Xa is the prothrombin activator central to both the intrinsic and extrinsic
pathways.
Fibronectin (choice C) is a serum protein that acts as an opsonin for phagocytic cells
in clots. Fibronectin binds non-specifically to bacteria and other materials in the
newly forming clots, and draws the cell membrane of phagocytes into contact with
these substances.

Tissue factor (choice D) is a protein released from injured tissues that works in
concert with Factor VII to initiate the extrinsic pathway of coagulation. Like Factors
VIII and IX, tissue factor and Factor VII cleave Factor X to Xa.
________________________________________
13The correct answer is D. The patient is most likely experiencing a potentiation of
the effects of warfarin by norfloxacin, which decreases the metabolism of the
warfarin. The increased warfarin effect produces an increase in the INR. (The target
INR for patients with prosthetic heart valves is usually 1.5-4, depending on the type
of valve.) Although norfloxacin is the most likely drug among the choices given to
cause this effect in this patient, the antibiotics most commonly associated with this
type of interaction are the macrolides, such as erythromycin, metronidazole, and
the sulfonamide antibiotics.
Oral doses of penicillins, such as ampicillin (choice A), are generally not associated
with a potentiation of warfarin's effect, although large IV doses of penicillin may be.
Cephalexin (choice B) is a first-generation cephalosporin that can be used in the
treatment of acute cystitis. Although this agent is generally not associated with an
increased hypoprothrombinemic effect when given with warfarin, the
cephalosporins with a methyltetrazolethiol side chain, such as cefazolin,
cefmetazole, and cefoperazone, are known to increase warfarin's therapeutic effect.
Nitrofurantoin (choice C) is a urinary anti-infective agent that does not interact with
warfarin.
Phenazopyridine (choice E) is a urinary tract analgesic that does not interact with
warfarin, although it commonly changes the color of urine to a bright orange/red
color, which the patient may mistake as blood in the urine.
________________________________________
14The correct answer is D. Propylthiouracil works primarily by inhibiting the
peripheral conversion of T4 to T3. The thyroid extracts iodide from the plasma and,
in an oxidative process, iodinates tyrosine residues in thyroglobulin molecules.
Monoiodotyrosines and diiodotyrosines are formed and then coupled to produce
either thyroxine (tetraiodothyronine, T4) or triiodothyronine (T3). Proteolytic
cleavage of thyroglobulin molecules leads to free T3 or T4, which is then released
into the circulation; T3 is several times more potent than T4. Peripheral deiodination
of T4 at the 5' position leads to T3 formation (mainly in the liver); this step is
inhibited by propylthiouracil.
Decreasing the efficacy of TSH binding (choice A), decreasing the rate of
thyroglobulin proteolysis (choice B), increasing the amount of rT3 formation (choice
C), and inhibiting the uptake of iodide into the thyroid (choice E), would all tend to
decrease the formation of thyroid hormones in the thyroid itself.
________________________________________

15The correct answer is A. Acetylcholine is the neurotransmitter of primary


importance for the induction of REM sleep. Some of the other neurotransmitters do
function in sleep, but REM sleep can occur in their absence.
Dopamine (choice B) is a neurotransmitter with a role in voluntary movement,
mood, cognition, and regulation of prolactin release.
Epinephrine (choice C) is important in sympathetic nervous system responses. It is
also a CNS neurotransmitter.
Norepinephrine (choice D) is important in sympathetic nervous system responses. It
is also a CNS neurotransmitter involved in attention, arousal, and mood.
Serotonin (choice E) is a CNS neurotransmitter that plays an important role in mood
and sensation. In the periphery, it is involved in vascular regulation and digestive
function.
________________________________________
16The correct answer is E. This is a case of Neisseria gonorrhoeae arthritis. Patients
are continuously susceptible to reinfection because of antigenic variation and phase
variation of the pili.
N. gonorrhoeae is not an intracellular pathogen (choice A), although it may be found
intracellularly in neutrophils after it has been phagocytized.
Ceftriaxone (choice B) is the drug of choice for N. gonorrhoeae.
Gonococci are especially susceptible to complement-mediated lysis, not resistant to
it (choice C).
N. gonorrhoeae has an insignificant capsule that does not play a major role in the
pathogenesis, but its capsule is immunogenic (compare with choice D).
Streptococcus pyogenes is the best known example of a nonimmunogenic capsule,
made of hyaluronic acid, but virtually all other capsules are immunogenic.
________________________________________
17The correct answer is E. Specificity is the proportion of persons without the
disease who are correctly identified by the test as being disease-free. It is given by
TN/(FP + TN), where TN stands for true negatives (people who do not have the
disease and test negative on the test) and FP stands for false positives (people who
do not have the disease but test positive on the test). In this case, 95/(5 + 95) =
95/100 = 0.95.
________________________________________
18The correct answer is B. 2,3-diphosphoglycerate (2,3-DPG) is produced in red
cells by a variation on the glycolytic pathway, and levels diminish when glycolysis
by the red cells slows. The depletion of 2,3-DPG in stored blood causes the
hemoglobin dissociation curve to shift to the left, leading to an increase in oxygen
affinity. This increase is helpful in the picking up of oxygen by hemoglobin in the
lungs, but can be very problematic in the release of oxygen from the blood in

tissues. This is not just a theoretical point: considerable effort has been expended in
developing improved solutions for storing packed red cells and methods for
"restoring" older stored cells so that the 2,3-DPG levels are adequate. In practice, in
otherwise reasonably healthy patients, older transfused blood will quickly
regenerate 2,3-DPG when placed in the glucose-containing environment of the
serum, but even transiently decreased 2,3-DPG levels in a severely compromised
patient can be dangerous.
________________________________________
19The correct answer is C. Entamoeba are relatively common enteric pathogens
that can produce asymptomatic infection or more severe disease characterized by
mucosal ulcerations and submucosal spread causing abdominal distress and liquid
stools. Stools may show either trophozoite forms or the typical spherical cysts.
Several species of Entamoeba are seen, including Entamoeba coli and E. hartmanni.
E. histolytica cysts characteristically are spherical in shape, 10-20 mm in diameter,
and have granular cytoplasm containing 1, 2, or 4 nuclei.
Cryptosporidium parvum (choice A) infections occur in the immunocompromised
population and may cause severe diarrhea. The organism presents as minute (2-5
mm) intracellular spheres or arc-shaped merozoites under normal mucosa, and can
be difficult to appreciate by light microscopy. Cysts in the stool are too small (4-5
mm) to be confused with Entamoeba.
Dientamoeba fragilis (choice B) is an intestinal amoeba that also may produce an
infectious diarrhea. It does not have a cyst form, and only the trophozoite forms are
seen in stools.
Giardia lamblia (choice D) is a flagellate protozoan that infects the stomach and
small intestine when contaminated water is ingested. Diagnosis is usually made by
examining duodenal contents; however, the stools may contain the oval or elliptical
cysts, which are thick-walled and measure 8-14 mm in diameter. Spherical cysts are
not seen in Giardia infection.
Isospora belli (choice E) produces self-limited intestinal infections mostly in the
tropics, where fever and diarrhea may last weeks to months. The stool-borne cysts
are much larger than Entamoeba (30 x 15 mm), are asymetrical, and are typically
almond-shaped.
________________________________________
20The correct answer is A. The concrete operational stage (age 7-11 years) is
defined by the child's awareness of the conservation of volume, which
demonstrates that the child is able to reason in a logical way in terms of the
physical world. Note that the child does not develop understanding of abstract
concepts until he or she has reached the formal operational stage (choice B), at age
11 to adulthood.
The preoperational stage (choice C), ages 2 to 7 years, is associated with significant
language development. However, the child has not yet developed the ability to take
the perspective of others, and thus the child's thinking tends to remain egocentric.

The sensorimotor stage (choice D) corresponds to ages 0 to 2 years and is


characterized by the infant developing increasingly sophisticated sensorimotor skills
and behavior patterns.
________________________________________
21The correct answer is B. The reference intervals for serum alkaline phosphatase
(ALP) and serum phosphate are higher in children than in adults. Growing bone in
children requires alkaline phosphatase secreted by osteoblasts and an increase in
serum phosphate to provide a proper solubility product ([calcium] x [phosphorus])
for mineralization of bone. However, the normal serum ALP in children is no greater
than 5-times the upper limit of normal for adults, hence this patient's ALP is
abnormally elevated. The most likely scenario is that the child is a victim of abuse
and has suffered a femoral fracture that is healing. Bone repair further increases the
serum ALP owing to the increase in osteoblastic activity.
Aseptic necrosis of bone in the leg (choice A) is more likely to occur in the femoral
head (Legg-Calve-Perthes disease). This disease is more common in boys than girls
and occurs in the 3- to 10-year-old age bracket. Although aseptic necrosis is
associated with dense bone formation, it does not extend out into the soft tissue nor
would it produce skin discoloration directly over the lesion (due to trauma in this
case).
Since the elevation in serum ALP is increased for age and related to the bone lesion,
choice C is incorrect.
Since the serum ALP is not normal for the age of this patient, choices D and E are
incorrect. Furthermore, osteogenic sarcoma (choice D) does not normally occur in
the first decade and favors the metaphysis of the distal femur or proximal tibia
________________________________________
22The correct answer is C. Norepinephrine (NE) is an agonist at alpha1-, alpha2and beta1-receptors. NE exerts its vascular actions via alpha (predominantly
alpha1) receptors and its cardiac actions via beta1-receptors. Labetalol is a
nonselective antagonist at alpha- and beta-receptors, and therefore, could prevent
all actions of NE.
Atenolol (choice A) is a selective beta1 antagonist, and therefore would block only
norepinephrine's cardiac effects.
Esmolol (choice B) is a selective beta1 antagonist, and therefore would block only
norepinephrine's cardiac effects. Metaproterenol (choice D) is a selective beta2
agonist and so would not block NE's effects.
Prazosin (choice E) is a selective alpha1 antagonist and would therefore block most
of norepinephrine's actions in the vasculature, but would not antagonize other
effects.
________________________________________

23The correct answer is C. The pudendal nerve is a branch of the sacral plexus that
exits from the greater sciatic foramen and then enters the lesser sciatic foramen to
lie in the pudendal canal on the lateral wall of the ischioanal fossa. The inferior
rectal nerve is a branch of the pudendal nerve, which crosses the ischioanal fossa to
reach the anal canal where it innervates the external anal sphincter and provides
sensory innervation to the area.
The inferior gluteal nerve (choice A), a branch of the sacral plexus, exits through the
greater sciatic foramen and innervates the gluteus maximus muscle.
The pelvic splanchnic nerves (choice B) are branches of the second, third, and
fourth sacral nerves that carry preganglionic parasympathetic nerves to several
pelvic organs.
The sciatic nerve (choice D) is a branch of the lumbosacral plexus that exits through
the greater sciatic foramen, then enters the lower limb to innervate muscles of the
posterior thigh, the leg, and the foot, and provides sensory innervation to the area
as well.
The superior gluteal nerve (choice E) is a branch of the sacral plexus that exits
through the greater sciatic foramen and innervates the gluteus medius, gluteus
minimus, and tensor fascia latae muscles.
________________________________________
24The correct answer is A. The girl has Turner syndrome, which is specifically
associated with coarctation of the aorta. Even when coarctation has not been
diagnosed in early childhood and the patient appears to be doing well, surgical
correction of any significant degree of coarctation is recommended. Uncorrected
coarctation can cause death after about age 40 due to a variety of causes, including
congestive heart failure, infective aortitis (analogous to infective endocarditis), and
hypertension-induced intracranial hemorrhage or rupture of the damaged (precoarctation) aorta.
Associate dextrocardia (choice B) with Kartagener syndrome.
Associate ostium primum septal defect (choice C) with Down syndrome.
Pulmonary stenosis (choice D) and tetralogy of Fallot (choice E) are not specifically
associated with Turner syndrome.
________________________________________
25The correct answer is A. The malignant tumor is a yolk sac tumor, also known as
infantile embryonal carcinoma and endodermal sinus tumor. The biggest tip-off in
the question stem is the presence of endodermal sinuses that resemble primitive
glomeruli. The cytoplasmic globules described contain alpha-fetoprotein, indicating
yolk cell differentiation, and alpha-1-antitrypsin. Alpha-fetoprotein can also be used
as a serum marker for recurrent disease. Yolk sac tumors occur in pure form in
infants and children and may be part of mixed tumors in adults.

Estrogen receptors (choice B) are important markers in breast cancer because they
predict tumor response to hormonal manipulation.
Human chorionic gonadotropin (choice C) is found in the syncytial cells of
embryonal carcinoma.
Human papilloma virus (choice D) can be found in condylomas, cervical cancer,
penile cancer, laryngeal polyps, and warts.
Melanin (choice E) can be found in melanomas.
________________________________________
26The correct answer is D. The splenorenal ligament is the portion of the dorsal
mesentery between the posterior abdominal wall and the spleen. This mesentery
transmits the splenic artery and vein from their retroperitoneal position in the
proximal portion of their course to the peritoneal spleen.
The gastrocolic ligament (choice A) is the portion of the greater omentum between
the greater curvature of the stomach and the transverse colon. This portion of
mesentery is not related to the spleen.
The gastrosplenic ligament (choice B) is the portion of the dorsal mesogastrium
between the greater curvature of the stomach and the spleen. There are no splenic
vessels in this mesentery.
The lesser omentum (choice C) is derived from the ventral mesentery. It is the
mesentery between the lesser curvature of the stomach and the liver and between
the first portion of the duodenum and the liver. It is not related to the spleen.
________________________________________
27The correct answer is D. This is one of those questions for which having a good
idea of what you are looking for before exploring the answer choices will certainly
save you valuable time. The answers all look alike and you could have been easily
confused if you were not confident of the answer before approaching the choices.
This patient has long-standing hyperparathyroidism (elevated PTH), which
predisposes to the development of osteitis fibrosa, her bone disease. PTH acts
initially on osteocytes of bone tissue (osteocytic osteolysis) and subsequently on
osteoclasts (osteoclastic resorption) to resorb calcium from bone matrix and make it
available to the circulation. This increases plasma calcium levels. PTH also causes
decreased phosphate reabsorption in the proximal renal tubule, yielding
hypophosphatemia. Hypercalciuria is another sequela of excess PTH production,
which predisposes the patient to the formation of calcium oxalate stones.
Choices A and E correspond to neither hyper- nor hypoparathyroid states.
Choice B is the profile of hypoparathyroidism. You should have quickly eliminated
this choice since the PTH was decreased and you were looking for a profile
consistent with HYPERparathyroidism.

Choice C is the profile of secondary hyperparathyroidism. This occurs when there is


parathyroid overproduction due to a nonparathyroid cause. By far, the most
common cause is chronic renal failure. In such cases, there is decreased calcium
absorption since the kidneys are involved in the conversion of 25(OH)D3 to the
active form 1,25(OH)D3. The decreased calcium ion level stimulates the
parathyroid, leading to elevated PTH levels. Hyperphosphatemia results from
diminished renal synthesis of 1,25 dihydroxyvitamin D3, creating further calciumphosphate imbalance and enhanced PTH production.
________________________________________
28The correct answer is D. The processus vaginalis is an evagination of the parietal
peritoneum of the abdomen that descends through the inguinal canal before the
descent of the testis. Normally, the distal end of this evagination remains patent as
the tunica vaginalis and the remainder of the processus vaginalis fuses and
becomes fibrous. If a part of the processus vaginalis does not fuse (incomplete
fusion), the resulting cystic structure is a hydrocele of the spermatic cord.
A communication between the epididymis and the tunica vaginalis (choice A) will
result in the passage of sperm into the epididymis. This is a spermatocele.
Failure of the processus vaginalis to form (choice B) would result in an absence of
the tunica vaginalis.
Total failure of the processus vaginalis to fuse (choice C) results in a persistent
processus vaginalis. This results in congenital inguinal hernias.
Varicosities of the spermatic cord (choice E) or the pampiniform plexus is termed a
varicocele.
________________________________________
29The correct answer is D. Latin America and "pilot's wheel" budding yeast are
clues for paracoccidioidomycosis. This disease is caused by Paracoccidioides
brasiliensis, a dimorphic fungus that is found as a multiply-budding yeast in tissues.
Clues for questions about blastomycosis (choice A) would include spending time in
states east of the Mississippi River and fairly large yeast.
Clues for questions about coccidioidomycosis (choice B) would be spending time in
the southwestern deserts of the United States and spherules filled with endospores.
Clues for questions about histoplasmosis (choice C) would be spending time in the
Ohio, Mississippi, and Missouri River Valleys and finding tiny yeast forms in
macrophages.
Clues for questions about sporotrichosis (choice E) would be skin lesions in rose
gardeners.
________________________________________
30The correct answer is C. The patient presents with the classic symptoms of acute
hypocalcemia secondary to hypoparathyroidism, itself the result of overzealous

thyroid removal by the surgeon causing damage to the nearby parathyroid glands.
Other classic findings would have been Trousseau's sign (carpal spasm after
application of a blood pressure cuff) and the Chvostek's sign (facial muscle
contraction on tapping in front of the ear). Serum calcium is low and serum
phosphate is high, with normal alkaline phosphatase. The magnesium is normal,
ruling it out as a cause of the tetany (and also indirectly ruling out chronic
alcoholism or renal losses as causes). The calcifications seen on CT scan are a sign
of overmineralization due to an unfavorable [calcium] x [phosphate] product. Acute
treatment with calcium gluconate and, if possible, 1,25-dihydroxy vitamin D3
(cholecalciferol) is necessary.
Hypocalcemia normally triggers parathyroid hormone (PTH) release from the
parathyroid glands. PTH can increase bone resorption by stimulating osteoclastic
activity and can promote calcium reabsorption at the level of the kidney distal
tubule (to the expense of phosphate), but has no direct effect on intestinal
absorption of calcium and phosphate, which are under vitamin D control. However,
PTH is required to activate 1-hydroxylase in the kidney, the rate-limiting step in
metabolism of Vitamin D3 to its active metabolite, 1,25-dihydroxy-D3. This
metabolite helps to raise serum calcium by increasing proximal intestinal absorption
of calcium. The lack of 1,25-dihydroxy-D3 would be expected to prevent expression
of these calcium transporters. The active metabolite works in concert with PTH to
increase osteoclastic activity, promote calcium reabsorption in the kidney, oppose
the phosphate losses, and most uniquely, promote calcium and phosphate intestinal
absorption.
An increase in dietary vitamin D (choice A) would not be helpful at this time,
particularly since the lack of PTH would preclude the activation of the precursor
vitamin to the dihydroxylated metabolite. Rather oral calcium would be eventually
needed on a chronic basis.
Hepatocytes would not have a low 25-hydroxylase activity (choice B) for two
reasons: PTH has no effect on this first hydroxylation step and, more generally, it is
not a controlled enzymatic reaction.
Isolated cells from the kidney do not have high 1-hydroxylase activity (choice D)
since PTH is no longer present to activate this step.
The levels of 1,25-dihydroxy vitamin D (choice E) would be found to be abnormally
low.
________________________________________

31The correct answer is A. The patient has an acquired arteriovenous fistula,


probably caused by previous abdominal surgery. The decrease in peripheral
resistance associated with an arteriovenous fistula causes an increase in cardiac
output when the fistula is large (which usually requires involvement of a major
artery such as the aorta, subclavian artery, femoral artery, common carotid artery,
or iliac artery). The increase in cardiac output caused by the fistula is roughly equal

to the blood flow through the fistula. The increase in cardiac output is associated
with increases in both heart rate and stroke volume. The diastolic blood pressure
falls because blood can rapidly exit the arterial system through the fistula, but
mean blood pressure is maintained relatively constant because the normal longterm blood pressure regulating mechanisms (e.g., renal body fluid feedback
mechanism) still operate normally. The decrease in diastolic pressure with a normal
or slightly increased systolic pressure causes the arterial pulse pressure to increase
in arteriovenous fistula (note that pulse pressure is 80 mm Hg in this problem;
normal is ~40 mm Hg).
The pulse pressure is decreased in cardiac tamponade (choice B), heart failure
(choice C), hypovolemia (choice D), and shock (choice E).
________________________________________
32The correct answer is E. Glomerular nodules may be either the Kimmelstiel-Wilson
nodules of diabetes mellitus or may be composed of amyloid. An amyloid origin can
be confirmed by staining with Congo red, which causes the nodules to stain red with
ordinary light but exhibit a distinctive "apple-green" birefringence when viewed with
polarized light. The most common type of amyloid (type AA) is due to deposition of
altered immunoglobulin light chains and is seen in diseases with chronic antigenic
stimulation, such as tuberculosis.
Acute urinary tract infection (choice A) is an infection of too short a duration to
cause amyloidosis.
Diabetic mellitus (choice B) is associated with glomerular nodules called
Kimmelstiel-Wilson nodules, but these will not show apple-green birefringence when
viewed with polarized light.
Sarcoidosis (choice C) typically involves the lymph nodes, lungs, spleen, and to a
lesser extent, the skin and eye. It is unrelated to amyloidosis.
Systemic lupus erythematosus (choice D) can produce a wide range of renal lesions,
but amyloid nodules are not among the likely manifestations
________________________________________
33The correct answer is E. Sumatriptan is a serotonin1D agonist that is used to
abort migraine headaches. It is also effective in the treatment of cluster headaches.
Currently, no dopamine1 (D1) agonists (choice A), GABAB antagonists (choice B), or
muscarinic3 (M3) antagonists (choice C) are used clinically.
Propranolol is an example of a non-selective beta antagonist (choice D).
________________________________________
34The correct answer is E. The lesions are xanthomas, which are tumor-like dermal
collections of foamy histiocytes containing cholesterol and lipids. The lesions may
also contain multinucleated giant cells (Touton giant cells) with clustered nuclei and
foamy cytoplasm. Xanthomas may be idiopathic or may be related to

hyperlipidemia or lymphoproliferative malignancies (e.g., leukemias and


lymphomas).
Nevus cells are a type of melanocyte. Benign nevus cells (choice A) are a feature of
the common nevus, or mole. Malignant nevus cells (choice B) are a feature of
malignant melanoma.
Microscopic blisters (choice C) at the dermal/epidermal junction are a feature of
dermatitis herpetiformis.
Munro microabscesses (choice D), small collections of neutrophils in the cornified
epidermis, occur in psoriasis.
________________________________________
35The correct answer is B. Sensitivity is defined as the ability of a test to detect the
presence of a disease in those who truly have the disease. It is calculated as the
number of people with a disease who test positive (true positive) divided by the
total number of people who have the disease (true positive + false negative). In this
case, sensitivity equals the number of babies born with trisomy 21 whose mothers
tested positive (100) divided by the total number of babies born with trisomy 21.
This yields 100/200 = 50% (not a very sensitive test).
40% (choice A) corresponds to the prevalence of the disease in the tested
population, which in this case equals the total number of babies with Down's
syndrome (true positive + false negative = 100 + 100= 200) divided by the total
number of people tested (500). This yields 200/500= 40%.
67% (choice C) corresponds to the positive predictive value of the test, which equals
the number of babies with Down's whose mothers test positive (true positives =
100) divided by the total number of mothers testing positive (true positive + false
positive = 150). This yields 100/150 = 67%.
71% (choice D) corresponds to the negative predictive value of the test, which
equals the number of normal babies whose mothers tested negative (250) divided
by the total number of people testing negative (350). This yields 250/350 = 71%.
83% (choice E) corresponds to the specificity of the test, which equals the number
of babies without Down's whose mothers tested negative (250) divided by the total
number of babies without Down's (300). This yields 250/300 = 83%.
An easy way to remember these concepts is:
Sensitivity = true positives/all diseased
Specificity = true negatives/all normal
PPV = true positives/all positives
NPV = true negatives/all negatives
Prevalence = all diseased/total population
________________________________________

36The correct answer is C. This question focuses on neurodegenerative and


dementing illnesses. The correct answer can be deduced by combining clinical and
pathologic features. Dementia with Lewy bodies (or diffuse Lewy body disease) is
characterized by widespread formation of Lewy bodies within neurons of substantia
nigra, cholinergic nuclei (basal nucleus of Meynert), and limbic cortex (especially
cingulate gyrus), for example. Once thought to be infrequent, recent
clinicopathologic investigations have suggested that this might be one of the most
frequent causes of dementia in industrialized countries, perhaps second only to
Alzheimer disease. Peculiar to this dementing disorder is the presence of
extrapyramidal symptoms and visual hallucinations, as well as an undulating clinical
course.
Alzheimer disease (choice A) accounts for at least 60% of all cases of dementia in
industrialized countries and is characterized by high numbers of senile plaques and
neurofibrillary tangles in the hippocampus, neocortex, and cholinergic nuclei of the
basal forebrain. There is no involvement of the substantia nigra and thus no
extrapyramidal symptoms.
ALS (choice B) can be ruled out because of the clinical picture. ALS is caused by
degeneration of motor neurons in the anterior horns of the spinal cord and/or large
pyramidal neurons in the primary motor cortex (area 4). Thus, this condition will
manifest with progressive weakness involving lower and upper extremities. Skeletal
muscles become atrophic (hence the term amyotrophic) and show a denervation
pattern on biopsy. Occasionally, dementia may be associated.
Parkinson disease (choice D) is due to degeneration of dopaminergic neurons in the
substantia nigra and manifests with tremor, rigidity, and akinesia. A subset of
patients with Parkinson disease develops dementia. Thus, Parkinson disease may be
difficult to differentiate from dementia with Lewy bodies. However, Lewy bodies are
absent in cortical regions of patients with classic Parkinson disease.
Pick disease (choice E) is the most frequent of dementing disorders in the category
of frontotemporal dementia. This condition leads to extreme atrophy of frontal and
temporal cortex, with relative sparing of the remaining neocortical regions.
Neocortical neurons bear round intracytoplasmic inclusions, known as Pick bodies,
which contain altered neurofilaments, tau, and ubiquitin. No inclusions are found in
the substantia nigra.
________________________________________
37The correct answer is A. The muscle involved is the serratus anterior, which
draws the scapula forward and rotates it upward. It is also the major muscle used in
all reaching and pushing movements. When this muscle is paralyzed, the medial
border of the scapula is prominent, known as winging of the scapula. This muscle is
innervated by the long thoracic nerve, which originates from C5, 6, and 7.
The lower subscapular nerve (choice B) innervates the subscapularis and the teres
major. The subscapularis is one of the rotator cuff muscles and works with the
supraspinatus, the infraspinatus, and the teres minor to maintain approximation of

the head of the humerus in the glenoid cavity. The subscapularis also assists medial
rotation of the arm while in a dependent position.
The musculocutaneous nerve (choice C) supplies the coracobrachialis. When the
arm is already in an extended position, this muscle will bring it forward and
medially. The biceps brachii and the brachialis are also innervated by this nerve.
The biceps brachii assists with shoulder flexion, elbow flexion, and supination of the
forearm. The brachialis also flexes the elbow joint.
The suprascapular nerve (choice D) innervates the supraspinatus and the
infraspinatus muscles. These muscles, along with the subscapularis and teres minor,
are termed the rotator cuff muscles.
The upper subscapular nerve (choice E), along with the lower subscapular nerve
innervates the subscapularis.
________________________________________
38The correct answer is C. The negative predictive value of a test (PV-) refers to the
percent chance that a normal test result is a true negative (TN; a healthy person
with a normal test result) rather than a false negative (FN; a diseased person with a
normal test result). It is expressed by the following formula:
PV- = TN/(TN + FN)
Similarly, the positive predictive value of a test (PV+) reflects the probability that an
abnormal test result represents a true positive (TP) rather than a FP (a TP is an
abnormal test result in an individual with disease, while a FP is an abnormal test
result in a healthy person). PV+ is calculated using the following formula:
PV+ = TP/(TP + FP)
There is a relationship between the test's sensitivity (the ability of a test to detect
disease in those who truly have the disease), specificity (ability of the test to
correctly identify those without disease) and the PV- and PV+, respectively. Tests
with 100% sensitivity (no FNs) and tests with 100% specificity (no FPs)
automatically have a PV- and PV+ of 100%, respectively.
Changing the reference interval of a test alters its sensitivity, specificity, PV+, and
PV-. In this question, lowering the upper limit of the reference interval of a fasting
glucose from 140 mg/dL to 126 mg/dL increases the test's sensitivity, since a lower
glucose cut-off approaches the normal value for glucose in the normal population
(70-110 mg/dL). Furthermore, increasing the test's sensitivity automatically
increases the test's PV-, since there are fewer FNs.
The test's sensitivity is increased (choice A) rather than decreased by the given
change in the reference interval.
Since the test's sensitivity is increased, the FN rate at the new reference interval is
decreased (choice B).

The test's positive predictive value (choice D) decreases, since the test's specificity,
which ultimately determines its FP rate, decreases as the test's sensitivity
increases. Stated another way, a greater number of normal individuals will have FP
fasting glucose levels when the test's upper limit is decreased to 126 mg/dL.
A test's sensitivity is inversely related to its specificity. For example, changing the
reference interval of a test to increase its sensitivity automatically lowers its
specificity (choice E), since the number of FPs will increase. Similarly, when altering
a reference interval to increase a test's specificity, its sensitivity is reduced because
of an increase in the number of FNs.
In summary, lowering a test's upper limit of normal increases the test's sensitivity,
which decreases the FN rate, and increases the PV-. Increasing the test's sensitivity
also decreases the test's specificity. Decreasing a test's specificity increases the FP
rate and decreases the PV+.
________________________________________
39The correct answer is B. The abnormal depression described is a characteristic
tip-off for a dislocated shoulder joint. This is important to remember, as it may help
you make a rapid diagnosis.
Avulsion of the coronoid process (choice A) of the ulna (at the elbow) can occur
following a severe contracture of brachialis.
Fracture of the mid-shaft of the humerus (choice C) may produce a "bend" in the
upper arm.
Fracture of the surgical neck of the humerus (choice D) can lacerate the axillary
branch of the posterior cord (choice E, also called axillary), causing paralysis of the
deltoid.
________________________________________
40The correct answer is D. This child most likely has b+ thalassemia, an autosomal
recessive disease that, in the homozygous state, produces defects in the
transcription, processing, or translation of b globin mRNA. The predominant
hemoglobin switches from HbF (a2g2) to HbA (a2b2) at about 6 months of age, at
which point these children become markedly anemic because of their inability to
synthesize the b globin chain. The anemia of b thalassemia is microcytic and
hypochromic with anisocytosis.
Amino acid substitutions in b globin (choice A) lead to hemoglobinopathies, of which
sickle cell anemia is the archetype. Homozygotes with these hemoglobin variants do
not produce any HbA, but rather mutant types, such as HbS, HbC and HbM.
Antibodies against fetal blood cells (choice B) are generally produced by a mother
who has been sensitized to fetal blood antigens by transfusion or a previous
pregnancy. Such immune-mediated hemolysis leads to intrauterine anemia and
hydrops fetalis.

Cytoskeletal protein defects (choice C), such as spectrin deficiency causing


hereditary spherocytosis, produce anemia and jaundice that are apparent at birth.
The RBCs contain the normal types of hemoglobin.
Iron deficiency anemia (choice E) is due to inadequate iron stores for the production
of the heme iron in hemoglobin. Although iron deficiency anemia is not uncommon
in infants because human breast milk has only small amounts of iron, the resultant
microcytic anemia is not associated with abnormal types of hemoglo
________________________________________
41The correct answer is A. The lesion described is a hydatidiform mole, a tumor-like
mass that typically forms when an egg that has lost its nucleus, containing only
paternal chromosomes from two sperm, tries to develop. High levels of b-hCG (betahuman chorionic gonadotropin) are produced by these moles. Choriocarcinomas and
gestational trophoblastic tumors can also produce high levels of b-hCG.
Bombesin (choice B) is a useful tumor marker for neuroblastoma, small cell
carcinoma, gastric carcinoma, and pancreatic carcinoma.
CEA (choice C), or carcinoembryonic antigen, is a non-specific marker used to follow
some colorectal, pancreatic, and less commonly gastric and breast carcinomas.
PSA (choice D), or prostate-specific antigen, is a marker for prostatic carcinoma.
S-100 (choice E), is a marker for melanoma, neural tumors, and astrocytoma
________________________________________
42The correct answer is F. This patient has coarctation of the aorta (constriction of
the ascending aorta), which is suggested by a midsystolic murmur over the anterior
part of the chest and back, hypertension in the upper extremities, and absent or
delayed pulsations in the femoral arteries. The upper extremities and thorax may be
more developed than the lower extremities. Patients with coarctation of the aorta
may experience symptoms such as cold extremities as a result of tissue
ischemia.The truncus arteriosus gives rise to the proximal portions of the ascending
aorta and the pulmonary trunk. The third, 4th, and 6th aortic arches and the right
and left dorsal aortae contribute to the remainder of the aorta.
The bulbus cordis (choice A) gives rise to the right ventricle and the aortic outflow
tract.
98% of cases of coarctation of the aorta take place immediately distal to the
offshoot of the left subclavian artery, close to the junction of the ductus arteriosus
(choice B) with the aorta.
The left horn of the sinus venosus (choice C) gives rise to the coronary sinus.
The right common cardinal vein (choice D) gives rise to the superior vena cava.
The right horn of the sinus venosus (choice E) gives rise to the smooth part of the
right atrium.

________________________________________
43The correct answer is A. This woman has a risk profile (female, fat, forties) and
symptomatology consistent with gallstones (cholelithiasis). As would be expected,
contraction of the gallbladder following a fatty meal often exacerbates the pain
caused by gallstones. Cholecystokinin (CCK), the release of which is stimulated by
dietary fat, is the hormone responsible for stimulation of gallbladder contraction. It
is produced in I cells of the duodenum and jejunum. In addition to gallbladder
contraction, CCK also stimulates pancreatic enzyme secretion and decreases the
rate of gastric emptying.
Gastrin (choice B) is produced by the G cells of the antrum and duodenum. Gastrin
stimulates the secretion of HCl from the parietal cells and pepsinogen from the chief
cells of the stomach. Gastrin secretion is stimulated by gastric distention, digestive
products (e.g., amino acids), and vagal discharge.
Pepsin (choice C) is a protease produced by the chief cells of the stomach (as
pepsinogen). It is involved in the digestion of proteins. Pepsinogen release is
stimulated by vagal stimulation, gastrin, local acid production, secretin, CCK, and
histamine.
Secretin (choice D) is produced by the S cells of the duodenum. It is secreted
primarily in response to acidification of the duodenal mucosa. Secretin stimulates
the secretion of bicarbonate-containing fluid from the pancreas and biliary ducts.
This neutralization allows pancreatic enzymes to function. Secretin also inhibits
gastric acid production and gastric emptying.
Somatostatin (choice E) is produced by the D cells of the pancreatic islets and in the
gastric and intestinal mucosa. Somatostatin is an inhibitory hormone-it inhibits most
gastrointestinal hormones, gallbladder contraction, gastric acid and pepsinogen
secretion, pancreatic and small intestinal fluid secretion, and both glucagon and
insulin release.
________________________________________
44The correct answer is E. The bulb of the penis contains the urethra, and is found
in the superficial perineal space. The superficial perineal space is found between the
inferior fascia of the urogenital diaphragm (perineal membrane) and the superficial
perineal fascia (Colles' fascia). The attachments of the superficial perineal fascia to
the posterior edge of the urogenital membrane and to the ischiopubic rami prevent
the urine from escaping posteriorly into the gluteal region or laterally into the thigh.
The urine can escape anteriorly onto the anterior abdominal wall, where it will be
deep to the superficial abdominal fascia (Scarpa's fascia).
The deep perineal space (choice A) is the region within the urogenital diaphragm
that contains the sphincter urethrae muscle and the bulbourethral (Cowper's) gland
in the male.
The ischioanal space (choice B) is the space between the pelvic diaphragm and the
urogenital diaphragm that is continuous with the space between the pelvic
diaphragm and the lateral pelvic wall. This space is mostly filled with fat.

The rectovesical space (choice C) is within the peritoneal cavity of the male and is
the region between the urinary bladder and the rectum.
The retroperitoneal space (choice D) is the region between the parietal peritoneum
and the endoabdominal fascia. This space contains connective tissue, fat, and
several organs, including the kidney and ureter.
________________________________________
45The correct answer is C. Approximately half of all spontaneous abortions are of
fetuses with major chromosomal defects, most commonly trisomy 16, triploidy (due
to fertilization of an egg by two sperm), and 45 X,0 (Turner's syndrome). Trisomy 16
and triploidy do not produce viable offspring, unlike 45 X,0.
Trisomy 8 (choice A) is one of the very rare causes of live birth trisomies.
Trisomy 13 (choice B) is one of the more common live birth trisomy syndromes
(Patau syndrome).
Trisomy 18 (choice D) is one of the more common live birth trisomy syndromes
(Edwards' syndrome)
Trisomy 21 (choice E) causes Down's syndrome.
________________________________________
46The correct answer is B. The atrioventricular (AV) node is in the subendocardium
of the interatrial septum. From the AV node, the Purkinje fibers of the
atrioventricular bundle enter the interventricular septum to carry impulses to the
ventricle. The function of the AV node is to retard the conduction of the cardiac
impulses so that ventricular systole occurs after atrial systole.
The apex of the heart (choice A) is composed of ventricular wall musculature of the
left ventricle. There is no nodal tissue in this region.
The interventricular septum (choice C) contains the common bundle (AV bundle)
and the right and left bundle branches of the cardiac conduction system. These
bundles are composed of Purkinje fibers, which are specialized cardiac muscle cells.
The sinoatrial (SA) node is located in the right atrial wall (choice D), near the
entrance of the superior vena cava. The SA node contains specialized cardiac
muscle cells that depolarize more rapidly than do typical cardiac muscle cells,
thereby serving as the pacemaker of the heart.
The wall of the left atrium (choice E) does not contain any nodal cells.
________________________________________
47The correct answer is D. Poliovirus, which is a single-stranded +RNA virus, is
naked (i.e., non-enveloped) and hence will not be inactivated by lipid solvents such
as ether. The live virus vaccine had colonized the intestinal tract of the infant and
was still being shed 2 weeks after the earlier oral dose. This same virus, the vaccine

strain, is likely to be found in sewage, as all vaccinated infants will shed virus for a
period of time after immunization with OPV.
Adenoviruses (choice A) and parvovirus B19 (choice C) also may cause diarrheal
disease and both are non-enveloped; however, they both have a DNA genome.
Hepatitis C (choice B) is an enveloped, single-stranded +RNA virus; its major target
organ is the liver, not the intestinal tract. It is a fragile agent that does not survive
well outside the body and would not be isolated from raw sewage effluent.
Rotavirus (choice E) is the major cause of diarrheal disease in infants under the age
of 2 years. It is a member of the reovirus family and, as such, is double-stranded.
This virus causes hospitalization of 30% to 40% of the infected infants and kills
hundreds of thousands of infants in developing nations where access to hospitals is
not readily available. Therapy for the watery diarrhea produced by this agent is fluid
and electrolyte replacement.
________________________________________
48The correct answer is C. Lymphocytes and plasma cells aggregate near and
around blood vessels (perivascular accumulation) in this example of chronic
inflammation of the synovium. Lymphocytes have dark nuclei with little visible
cytoplasm. Plasma cells are larger with a distinct cytoplasm and an eccentric
nucleus. The synovial lining is thickened from its normal 1-2 layers. Note that you
did not have to be able to recognize lymphocytes and plasma cells in the
photomicrograph to answer this question. Because RA is associated with a chronic
inflammatory infiltrate, lymphocytes and plasma cells must be the correct answer.
Eosinophils (choice A) are not evident here and do not seem to play a role in RA.
Langhans type giant cells (choice B), or multinucleated histiocytes, are not evident
here but may appear during the later stages of RA.
Neutrophils (choice D) are not evident here; they are instead associated with acute
inflammation.
Type A and B synovial cells (choice E) are the two cell types of the synovial lining.
They increase in number but are not the cells of the perivascular infiltrate.
________________________________________
49The correct answer is C. Sarcoidosis is a multisystem disease characterized by
non-caseating granulomata in a variety of organs. The disease may be symptomatic
(respiratory and constitutional symptoms) or may be discovered incidentally when
chest x-ray or autopsy reveals bilateral hilar adenopathy. Definitive diagnosis is
based on biopsy, which reveals non-caseating granulomata that are negative for
fungi or acid-fast bacilli. Sarcoidosis is more common in individuals of African
descent.
Caroli's disease (choice A) is a congenital malformation of the bile duct system.

Raynaud's disease (choice B) is a vasospasm of vessels that causes temporary


ischemia in the hands.
Scleroderma (choice D), or progressive systemic sclerosis, is characterized by
progressive fibrosis of skin and internal organs.
Systemic lupus erythematosus (choice E) is an autoimmune disease characterized
by vasculitis (which may produce a variety of symptoms depending on the site of
the lesion), rash, renal disease, hemolytic anemia, and neurologic disturbances.
________________________________________
50The correct answer is E. Pseudounipolar cells of spinal and cranial nerve ganglia
derive from the neural crest. Other cell types that derive from neural crest include
Schwann cells, multipolar ganglion cells of autonomic ganglia, chromaffin cells of
the adrenal medulla, odontoblasts, melanocytes, and cells that make the
leptomeninges (pia and arachnoid). All of the other cell types listed derive from the
neural tube.

Answers

________________________________________

________________________________________

1>B

11>E

21>***

31>A

41>A

2>***
33>

12>***
43> C

22>C

32>***

42>***

3>***

13>D

23>***

33>E

43>A

4>***

14>E

24>A

34>***

44>***

5>A

15>***

25>***

35>B

45>C

6>D

16>C

26>B

36>C

46>B

7>D

17>E

27>***

37>A

47>D

8>C

18>B

28>***

38>C

48>C

9>B

19>C

29>D

39>B

49>C

10>D
Answers

20>A

30>***

40>D

50>E

23>

________________________________________

1The correct answer is E. The fibers are elastic fibers. You should
remember three specific sites where these are found: large arteries
(particularly the aorta), vocal cords, and ligamenta flava
(which connect the vertebrae). Small amounts of elastic tissue
are also found in skin, and to a much lesser degree in other
tissues.
Elastic tissue is not found in muscle (choice A) or mesentery (choice C).
Elastic tissue is not found in substantial amounts in the liver (choice B) or spleen
(choice D).
________________________________________
2The correct answer is E. Tularemia is caused by Francisella tularensis. Fifty percent
of the cases have been reported in Missouri, Arkansas, and Oklahoma. Infection can
be transmitted by tick bites or by handling animal carcasses such as rabbits,
squirrels, beavers, muskrats, and deer. The diagnosis is suggested by the history of
exposure, clinical progression, and generalized lymphadenopathy. The
ulceroglandular form of tularemia is the most common clinical presentation.
Anthrax (choice A) is caused by Bacillus anthracis. Typically, infection with B.
anthracis causes a localized skin lesion at the site of inoculation that develops into a
black eschar surrounded by draining lymphadenitis. Woolsorter's disease is the
inhalation form of anthrax.
Leptospirosis (choice B) is an infection caused by contact with urine from infected
animals such as rats and dogs. L. interrogans is the most common isolate. The
disease is biphasic, with the leptospiremic phase characterized by abrupt-onset
headache (98%), fever, chills, conjunctivitis, severe muscle aches, gastrointestinal
symptoms, changes in sensorium, rash, and hypotension. This phase lasts from 3 to
7 days. The immune stage occurs after a relatively asymptomatic period of 1 to 3
days and is characterized by recurrence of fever and generalized symptoms.
Meningeal symptoms often develop during the immune period. In more serious
cases, hepatic dysfunction and renal failure may develop.
Pasteurella multocida (choice C) is associated with dog and cat bites. It causes a
rapidly progressing cellulitis, bacteremia, and, occasionally, infective endocarditis.
Plague (choice D) is caused by Yersinia pestis. It is enzootic in the southwestern
United States. Rats and fleas are the vectors. Clinical presentations include
lymphadenopathy with septicemia or pneumonia (which has the highest casefatality rate).
________________________________________

3The correct answer is D. The rule of thumb is that the levels decrease by half every
half-life. Therefore, 50% will remain after one half-life, 25% will remain after two
half-lives, 12.5% will remain after three half-lives, and 6.25% will remain after four
half-lives. The half-life of this drug is 9 hours, so it will reach approximately 6% of
steady state levels in 36 hours.
________________________________________
4The correct answer is A. Patient's with hyper-IgM syndrome (HIGM) experience very
little, if any, isotype switching. The B cells in these patients cannot undergo the
switch from IgM to IgG, IgA, or IgE that normally occurs during B-cell maturation.
When B cells undergo isotype switching they require two factors: IL-4, which binds
to a specific receptor on the B cell, and the CD40 molecule on the B-cell surface,
which binds to the CD40 ligand (CD40L) on an activated T-cell surface. The
deficiency is due to mutations in the CD40L. This immunodeficiency results in
patients who are IgG- and IgA-deficient, but synthesize large amounts of polyclonal
IgM. Affected individuals are susceptible to pyogenic infections, and often form IgM
autoantibodies to neutrophils, platelets, or tissue antigens. The disease is inherited
as an X-linked recessive in 70% of the cases.
The problem is due to mutations in the CD40L, not the CD40 molecule on the B cell
(choice B).
Gamma-interferon (choice C) is primarily a type I helper T cell (TH1) cytokine,
although it does inhibit the TH2 lineage response to specific antigens.
A decrease in IL-2 (choice D) would inhibit the cell-mediated immune response and
the patient would be susceptible to infection with intracellular microorganisms.
IL-3 (choice E) is considered a growth factor for hematopoietic stem cells and mast
cells. The patient had a normal CBC except for slight neutropenia and
thrombocytopenia.
________________________________________
5The correct answer is C. The findings suggest a 2-4 day old myocardial infarction
(MI). An MI becomes visible grossly after about 12 hours (sometimes earlier) as an
area of pallor and eventually, cyanosis. Neutrophils infiltrate the tissue anywhere
from 12 to 72 hours after infarction, with increasing evidence of coagulative
necrosis.
At 2 hours (choice A), no changes would likely be evident in the infarcted
myocardium.
At about 18 hours (choice B), "waviness" of some myocardial fibers may be
apparent microscopically, and subtle changes in the appearance of the myofibrils
producing alternating densely-colored (contracted) and lightly-colored (expanded)
areas can be observed.
At 5 to 10 days (choice D), both macrophages and neutrophils are present and
granulation tissue with fibroblasts is growing in from the edges.

After 7 weeks (choice E), the area of infarction is typically completely replaced by
scar tissue.
________________________________________
6The correct answer is A. This problem is relatively simple if you know that
removing a parallel resistance from a circuit increases the total resistance of that
circuit. Because the total resistance with all five organs in the circuit is 0.05 mm
Hg/mL/min, removing an organ would produce a total resistance greater than 0.05
mm Hg/mL/min. Choice B can be rejected because the units are incorrect, and
choices C, D, and E can be eliminated because the values of resistance are lower
than the total resistance prior to removal of the organ.
The problem is more difficult when a mathematical solution is required. The
equation for parallel resistances is the following:
Because the total resistance (RT) is 0.05 mm Hg/mL/min, 1/ 0.05 = 20 = 1/R1 +
1/R2 + 1/R3 + 1/R4 + 1/R5. Therefore, each individual resistance must equal 0.25
mm Hg/mL/min since 1/0.25 = 4 and 4 x 5 = 20. Removing one of the resistances
therefore yields the following: 1/RT = 1/0.25 + 1/0.25 + 1/0.25 + 1/0.25 = 16. Thus,
RT = 1/16 = 0.625 mm Hg/mL/min.
________________________________________
7The correct answer is D. This man has G6PD deficiency (as do 10% of AfricanAmerican males). G6PD serves to protect the RBC from oxidative damage by
maintaining high intracellular levels of NADPH. People of Mediterranean descent can
also have G6PD deficiency, but to a much greater degree. Therefore, hemolytic
episodes in this population are more severe (and can be fatal) as compared to those
of the African-American type, which are usually mild and self-limited. Common
oxidative stressors that initiate hemolysis are drug reactions (especially sulfa
drugs), febrile illnesses, and fava bean ingestion.
Acute infectious hepatitis (choice A) would more likely present with abdominal pain,
hepatomegaly, and high elevations of AST and ALT (often into the 1000s).
Cholestatic liver disease (choice B) more often presents with elevation of alkaline
phosphatase along with mild AST and ALT elevations. This patient has elevated
unconjugated bilirubin levels, as in hemolytic disorders. Both hepatocellular
(hepatitis) and cholestatic liver disease cause more conjugated (as opposed to
unconjugated) hyperbilirubinemia.
Fluoxetine's (a selective serotonin reuptake inhibitor; choice C) most common side
effects are anxiety, agitation, and insomnia.
________________________________________
8The correct answer is D. During menopause, there is a loss of functioning follicles
in the ovaries such that GnRH-stimulated LH and FSH secretion do not result in
normal estrogen secretion. The low estrogen levels cannot inhibit gonadotropin
secretion in a negative-feedback fashion, resulting in very high levels of LH and FSH.

Choices A, B, C, and E do not accurately describe normal hormonal levels in


menopause.
________________________________________
9The correct answer is B. This boy's presentation, including the negative
rheumatoid factor, suggests osteoarthritis at a very early age. The urine turning
black on standing is classically associated with alkaptonuria (ochronosis), an
arthritic disease caused by a deficiency of homogentisic acid oxidase
(homogentisate 1,2 dioxygenase). In this condition, homogentisic acid, an
intermediate in the catabolism of tyrosine, accumulates in cells and body fluids,
including synovial fluid and urine.
Childhood polycystic kidney disease (choice A) is a rare autosomal recessive
disorder characterized by cystic enlargement of the kidneys and, usually, the liver.
Death often occurs in infancy.
Paroxysmal nocturnal hemoglobinuria (choice C) is associated with hemolytic
anemia and reddish brown urine.
Phenylketonuria (choice D) is classically associated with a deficiency of
phenylalanine hydroxylase and mental retardation.
Rhabdomyolysis (choice E) may occur with traumatic injury, muscle ischemia,
seizures, excessive exercise, heat stroke, malignant hyperthermia, alcoholism, and
various infectious or metabolic disorders. It would be associated with reddish urine
________________________________________
10The correct answer is A. The muscle is the abductor digiti minimi, the function of
which is to abduct (separate) the toe away from the 4th toe. The abductor digiti
minimi is supplied by the lateral plantar nerve. This muscle also helps act as a
"spring" on the sole of the foot during walking.
Adduction (opposition of the toes, choice B) is supplied by the plantar interossei.
Extension (choice C) is supplied by the extensor digitorum longus.
The flexor digitorum brevis flexes both the middle phalanx on the proximal phalanx
(choice D) and the proximal phalanx on the metatarsal (choice E).
________________________________________
11The correct answer is E. Pick's disease is a condition that is clinically similar to
Alzheimer's disease. It differs from Alzheimer's disease in that the pronounced brain
atrophy characteristically involves the frontal and temporal lobes, with sparing of
the posterior aspects of the cortex. Microscopically, the affected cortex contains
characteristic ballooned neurons (Pick cells) or cytoplasmic inclusions (Pick bodies).
Clinically, there is a slowly progressive dementia with language disturbances and
behavioral changes that may eventually lead to mutism. The progressive nature of
the dementing process has been termed a "descent into a sea of mindlessness."

Alzheimer's disease (choice A) is characterized by dementia, but with diffuse brain


atrophy.
Friedreich's ataxia (choice B) characteristically affects the spinal cord, brainstem,
and cerebellum, rather than cortex.
Huntington's disease (choice C) characteristically affects the caudate nucleus and
putamen. Motor symptoms such as choreoathetosis are prominent.
Parkinson's disease (choice D) characteristically affects the substantia nigra, and
produces prominent motor symptoms, including cogwheel rigidity, a resting tremor,
and a festinating gait.
________________________________________
12The correct answer is A. Glucose is freely filtered by the glomerular capillary
membrane and totally reabsorbed in the proximal tubule under normal conditions.
Therefore, the concentration of glucose is highest in the proximal portion of the
proximal tubule. The concentration of glucose is essentially zero in the thin
descending limb of loop of Henle (choice B), distal convoluted tubule (choice C),
cortical collecting tubule (choice D), and medullary collecting tubule (choice E).
________________________________________
13The correct answer is E. A massive pulmonary embolus, such as this patient
sustained, can interrupt pulmonary blood flow, producing acute cor pulmonale with
abruptly developing right ventricular dilatation. Other parts of the heart are
secondarily affected somewhat later. Acute cor pulmonale is a surgical emergency
requiring immediate correction of the underlying problem, which is usually a
pulmonary embolus lodged early in the pulmonary circulation. Acute cor pulmonale
is less common than chronic cor pulmonale, which is seen as a complication of
many chronic lung diseases.
The left main coronary artery (choice A) and right main coronary artery (choice D)
would be affected secondarily to the reduced blood flow to the left heart and aorta,
from which the coronary arteries arise.
The left ventricle (choice B) would be affected secondarily by reduced blood flow
from the pulmonary veins to the left atrium.
The right atrium (choice C) would be affected after the right ventricle since it is
farther from the circulatory block.
________________________________________
14The correct answer is E. The physical examination is typical for an advanced
alcoholic with hepatic cirrhosis. It is important to recognize these symptoms, as
these patients are notorious for "underestimating" and even denying their alcohol
use.

Bronchogenic carcinoma (choice A) typically presents with cough and/or respiratory


changes, but can present with mass effects in the chest or involvement of
mediastinal nerves or vessels.
Colon cancer (choice B) typically presents with changes in the stool or bowel habits.
Congestive heart failure (choice C) is typically heralded by shortness of breath or
peripheral edema, or both.
Glomerulonephritis (choice D) typically presents with changes in quality or quantity
of urine, and, possibly, fluid retention.
________________________________________
15The correct answer is C. The patient in question has a cerebellar lesion.
Cerebellar dysfunction can lead to a variety of motor dysfunctions, including truncal
ataxia (appearing similar to the gait of an intoxicated individual), intention tremor
(uncontrolled shaking of affected extremity present only with purposeful
movement), dysdiadochokinesia (the inability to perform rapid and regular
alternating movements), dysmetria (inability to stop movements at the desired
point), dysarthria (ataxic speech), hypotonia, and nystagmus.
During the fourth week of embryonic development, the anterior end of the neural
tube develops three vesicles, the prosencephalon (forebrain), the mesencephalon
(midbrain), and the rhombencephalon (hindbrain). By the sixth week, five vesicles
(listed in the answer options) have developed. The rhombencephalon has now
developed into the metencephalon and myelencephalon. The cerebellum and pons
derive from the metencephalon.
The diencephalon (choice A), which is derived from the prosencephalon, develops
into the thalamus, hypothalamus, epithalamus, subthalamus (everything with the
word "thalamus"), posterior lobe of the pituitary, and neural retina.
The mesencephalon (choice B), or midbrain, is the only brain vesicle that does not
produce a secondary vesicle; it remains the mesencephalon.
The myelencephalon (choice D), which is derived from the rhombencephalon,
develops into the medulla oblongata.
The telencephalon (choice E), which is derived from the prosencephalon, develops
into the cerebral hemispheres (cerebral cortex, basal ganglia, and deep white
matter).
________________________________________

16The correct answer is B. The baby has Turner's syndrome, in which only one sex
chromosome, the X chromosome, is present as a result of chromosomal loss early
after fertilization of the egg. The chromosome loss can either occur in all cells, or
only in part of the body, due to random X-chromosome inactivation, producing the
variant known as a Turner mosaic. Affected babies often have a prominent

"webbed" neck related to lymphatic stasis in the neck, sometimes producing a frank
cystic hygroma (large cystic mass composed of dilated lymphatic channels); edema
of the dorsum of the hands and feet caused by similar mechanisms is also seen in
these babies. A baby with these clinical features should have a chromosomal
analysis. Affected babies should also be carefully checked for cardiac anomalies
(notably preductal coarctation of the aorta and aortic stenosis with endocardial
fibroelastosis), since congenital heart disease can cause early death. By puberty,
the neck and extremity edema has resolved, but careful physical examination often
reveals residual redundant skin of the neck and shoulders, producing the mature
form of webbed neck. Mosaic patients and patients with partial deletions of one X
chromosome may present only at puberty with the combination of short stature and
primary amenorrhea. The ovaries (usually not biopsied in obvious cases) lose all of
their oocytes by 2 years of age ("menopause before menarche") and become
atrophic fibrous strands without ova or follicles ("streak ovaries"). Hormonal
replacement in Turner patients should include both estrogens and progestins, since
unopposed estrogens (choice A) can cause atypical adenomatous hyperplasia of the
endometrium.
Insulin (choice C) replacement is not required in these patients.
Replacement of progestin alone (choice D) is not recommended.
Thyroid hormone (choice E) replacement is not required in these patients.
________________________________________

17The correct answer is C. The key to answering this question is knowing the
plasma concentration profile across the 40 weeks of pregnancy for each of the
hormones mentioned. Plasma concentration of human chorionic gonadotropin (hCG)
doubles every 2 days during the first ten weeks of pregnancy and then declines to a
level that is one tenth of the peak for the remainder of the pregnancy. hCG is in the
same hormone family as TSH, FSH, and LH. These are all glycoprotein hormones
with identical a subunits but different b subunits. Hence, there is a similarity in
receptors for these hormones as well. A mutation in the TSH receptor that also
made it responsive to hCG would result in increased thyroid hormone secretion
during pregnancy due to the increased circulating hCG. Under non-pregnant
conditions, thyroid hormone secretion would be normal. This mutation was actually
found in a woman who had experienced several early miscarriages. When she was
treated for hyperthyroidism with propylthiouracil during her pregnancy, her child
was carried to full term.
Plasma concentrations of progesterone and estrogen increase throughout
pregnancy. By 40 weeks, the progesterone concentration may be 200 times greater
than pre-pregnancy levels. Estradiol and estrone increase by about 50 fold, while
estriol increases about 1000 fold. While the T3, estrogen, and progesterone
receptors all originate from the same superfamily, it is unlikely that the woman's
pregnancy-associated hyperthyroidism is due to an action of progesterone or
estrogen on the T3 receptor (choices A and B). If this were the case, plasma levels

of thyroid hormone would be expected to rise throughout pregnancy with highest


concentration occurring just prior to birth. Furthermore, in the non-pregnant state,
increases in estrogen during the follicular phase and increases in progesterone
during the luteal phase would produce problems in thyroid hormone secretion. This
patient had normal thyroid status when not pregnant.
Human chorionic somatomammotropin (hCS) increases throughout pregnancy. It is
related to the anterior pituitary hormones prolactin and growth hormone. A
mutation in either the TRH receptor (choice D) or the TSH receptor (choice E) would
be unlikely to be responsive to hCS because this hormone is unrelated to either TRH
or TSH. Furthermore, if such a mutation could occur, it would produce maximal
thyroid hormone concentration near the end of the pregnancy (not at 10 weeks).
________________________________________
18The correct answer is E. Warfarin is a coumarin anticoagulant used for the
prophylaxis and treatment of thromboembolic complications associated with cardiac
valve replacement and atrial fibrillation, as well as the prophylaxis and treatment of
venous thrombosis and pulmonary embolism. Warfarin may cause necrosis of the
skin (typically on the breasts, thighs, and buttocks) generally between the third and
tenth days of therapy. The lesions are initially sharply demarcated, erythematous,
and purpuric. They may resolve or progress to large, irregular, hemorrhagic bullae
that can eventually lead to necrosis. The mechanism for this reaction is related to
warfarin's ability to deplete protein C, which can lead to a state of
hypercoagulability and thrombosis in the cutaneous microvasculature.
Aspirin (choice A) is commonly used for its antiplatelet effect; however, it would not
be indicated for anticoagulation of a patient with a recent cardiac valve
replacement. Furthermore, aspirin is not associated with the development of this
type of skin necrosis.
Cefazolin (choice B) is a first-generation cephalosporin antibiotic commonly used as
a perioperative prophylactic agent. If the patient was allergic to this antibiotic, an
erythematous rash might have appeared. However, the rash would not lead to the
appearance of large, hemorrhagic bullae.
Heparin (choice C) is an intravenous anticoagulant indicated for the prophylaxis and
treatment of thromboembolic complications associated with cardiac valve
replacement and atrial fibrillation. It is also indicated for the prophylaxis and
treatment of venous thrombosis, pulmonary embolism, and for treatment of some
coagulopathies. Although heparin is associated with the development of
thrombocytopenia, it is not associated with skin necrosis.
Vancomycin (choice D) is an antibiotic typically reserved for treatment of lifethreatening infections caused by gram-positive organisms. If vancomycin is
administered too rapidly via the intravenous route, a maculopapular rash may
appear on the chest and on the extremities. However, once the administration is
complete, the rash usually disappears in a few hours.
________________________________________

19The correct answer is A. Plasma levels of maternal estrogens during pregnancy


are dependent on a functioning fetus. The fetal adrenal cortex and liver produce the
weak androgens, DHEA-S and 16-OH DHEA-S, which are carried to the placenta by
the fetal circulation. The placenta then desulfates the androgens and aromatizes
them to estrogens (16-OH DHEA-S, estriol) prior to delivery to the maternal
circulation. Estradiol and estrone increase approximately 50 fold during pregnancy,
but estriol increases about 1000 fold. When estriol is assayed daily, a significant
drop may be a sensitive early indicator of fetal jeopardy.
Total serum thyroxine concentration may be increased in pregnancy due to an
increase in circulating TBG resulting from increased estrogen. However, free
thyroxine (choice B) remains within the normal range because of feedback
regulatory loops. The decline in estrogen with fetal compromise may gradually
decrease serum thyroxine, but the free thyroxine will remain unchanged.
Human chorionic gonadotropin (choice C) and human chorionic
somatomammotropin (choice D) are both secreted by syncytiotrophoblasts of the
placenta. As long as placental function is intact, blood levels of these two hormones
should not change with fetal compromise.
Placental secretion of progesterone (choice E) during pregnancy is also independent
of any fetal contribution. The placenta relies on maternal cholesterol for
progesterone production. Fetal death has no immediate influence on progesterone
production by the placenta.
________________________________________
20The correct answer is D. There are two main morphologic forms of emphysema,
centriacinar and panacinar. The panacinar variant is related to a1-antitrypsin
deficiency; the entire acinus is enlarged, from the respiratory bronchiole to the
distal alveoli.
Centriacinar emphysema (choice A) is characterized by enlargement of the central
portions of the acinus, i.e., the respiratory bronchiole, and its pathogenesis is
related to exposure to tobacco products and coal dust.
Interstitial emphysema (choice C) is not a true form of emphysema. It results from
penetration of air into the pulmonary interstitium. This may occur when alveolar
tears develop because of a combination of coughing and airway obstruction (e.g.,
children with whooping cough) or a chest wound that injures the underlying lung
parenchyma (e.g., a fractured rib).
Compensatory emphysema (choice B) and paraseptal emphysema (choice E) are
associated with scarring. Both are frequent but usually clinically silent. Paraseptal
emphysema, however, may lead to spontaneous pneumothorax in young patients.
In fact, this form is more severe in areas adjacent to the pleura, where large,
cystlike structures may develop and rupture into the pleural cavity.
________________________________________

21The correct answer is A. The secondary sexual changes seen in alcoholic cirrhosis
appear to be due to both decreased testicular secretion of testosterone and
decreased hepatic extraction of the androgen, androstenedione. Thus,
androstenedione is available for extrasplanchnic aromatization (occurring mostly in
peripheral adipose tissue) to form compounds with estrogenic activity.
Choices B and E do not fully explain the secondary sexual changes.
While tumors of Leydig and/or Sertoli cells may excrete both androgens and
estrogens, the normal Leydig (choice C) and Sertoli cells (choice D) do not usually
secrete estrogens.
________________________________________
22The correct answer is A. The description is typical for basal cell carcinoma. These
skin cancers typically occur on sun-exposed areas of the head, neck, and upper
trunk. Basal cell carcinoma only rarely metastasizes, but can become locally
mutilating if neglected. When located on the face, it may be difficult to adequately
excise without damaging facial structures.
Eczema (choice B) typically involves a larger area of skin and may cause dryness,
discoloration, and thickening of the involved area. Blistering, erythema, or oozing
may also be observed.
Psoriasis (choice C) is characterized by erythematous plaques with a silvery surface.
Urticaria (choice D) causes transient, nonpitting, erythematous wheals.
Verruca vulgaris (choice E), the common wart, causes well-demarcated verrucous
papules, often on the hands.
________________________________________
23The correct answer is E. Spongiosis is the name used for intercellular edema of
the epidermis. Fluid accumulates between the cells, pulling them apart. However,
the intercellular connections (desmosomes) remain largely intact. This produces a
distinctive microscopic appearance in which each cell is surrounded by a broad
white band with tiny dark cross-stripes, somewhat reminiscent of a closed zipper.
The basic terms that describe skin lesions are a favorite target on examinations.
Acantholysis (choice A) is the process of cell separation seen in some blistering
diseases. The intercellular connections are broken in acantholysis, producing
"rounded-up" cells that are dissociated from each other.
Acanthosis (choice B) is a marked thickening of the epidermis due to an increase in
the number and size of the epithelial cells.
Hyperkeratosis (choice C) is an increase in the thickness of the superficial keratin
layer of the epidermis.
The term parakeratosis (choice D) is used when nuclei are present in the normally
anuclear superficial keratin layer.

________________________________________
24The correct answer is A. Epiglottitis is the most common disease of the upper
respiratory tract produced by Haemophilus influenzae type b, a gram-negative
encapsulated rod. It is also a common cause of otitis media in children and may
cause bronchitis, bronchiolitis, and pneumonia in adults. The incidence of serious
disease caused by Haemophilus influenzae type b has decreased greatly with the
introduction of an effective vaccine. The vaccine is composed of the H. influenzae
type b capsular polysaccharides coupled to a carrier molecule, given to children
between 2 and 15 months of age. The patient had not received the Hib conjugate
vaccine and therefore was susceptible to this organism.
Klebsiella pneumoniae (choice B) causes pneumonia and pulmonary abscesses, but
is not considered to be a pathogen in the upper respiratory tract.
Legionella pneumophila (choice C) causes pneumonia in man. The disease may be
mild (an atypical pneumonia) or a fulminating disease with a high mortality (30%).
Mycoplasma pneumoniae (choice D) causes community-acquired atypical
pneumonia. It is the most common cause of pneumonia in young adults.
Streptococcus pyogenes (choice E) is the most common cause of pharyngitis;
however, this patient's presentation strongly suggests epiglottitis.
________________________________________
25The correct answer is E. Verapamil is a "first-generation" calcium channel blocker
that has been associated with an accelerated progression of CHF in certain patients.
This agent has a strong negative inotropic effect, which leads to a decrease in the
force and velocity of myocardial contraction. Hence, this agent would most likely
exacerbate the patient's heart failure. As a general rule, the use of calcium channel
blockers in CHF is reserved for patients who also have hypertension and/or anginal
symptoms.
Amlodipine (choice A) and felodipine (choice C) are the most commonly used
calcium channel blocking agents in patients with CHF. These two medications may
actually produce a small increase in myocardial contractility and cardiac output.
Diltiazem (choice B) is generally avoided in patients with CHF, since it has mild-tomoderate negative inotropic effects leading to a small decrease in myocardial
contractility. However, the negative inotropic effects of verapamil are much greater
than those seen with diltiazem.
Isradipine (choice D) is a calcium channel blocker that can safely be used at lower
doses in patients with CHF; it seems to have no net effect on myocardial
contractility
________________________________________
26The correct answer is E. The opponens pollicis muscle is innervated by the
median nerve. This is the nerve that also supplies sensory innervation to the palmar
aspect of the first and third digits. As the median nerve runs through the carpal

tunnel at the wrist, it may be compressed between the carpal bones and the
transverse ligament, thus producing pain, numbness, tingling, and weakness in the
hand along the median nerve distribution.
The abductor pollicis longus (choice A), the extensor indicis (choice C), and the
extensor pollicis longus (choice D) are innervated by the posterior interosseous
nerve, a branch of the radial nerve.
The adductor pollicis (choice B) is innervated by the ulnar nerve.
________________________________________
27The correct answer is E. The parietal cells of the stomach produce intrinsic factor,
a glycoprotein that binds vitamin B12 in the lumen of the stomach and facilitates its
absorption in the terminal ileum. Patients without a stomach and those with
pernicious anemia (autoimmune destruction of parietal cells) will require B12
replacement therapy. Recall that B12 deficiency will lead to megaloblastic anemia
and the USMLE-favorite picture of a blood smear with hypersegmented neutrophils.
Note that parietal cells also synthesize and secrete HCl.
Chief cells (choice A) are responsible for secreting pepsinogen, the precursor to
pepsin.
G cells (choice B) are gastrin-secreting cells. Gastrin stimulates secretion of acid by
the parietal cells found in the body and fundus of the stomach. Zollinger-Ellison
syndrome is caused by a pancreatic or duodenal tumor that secretes gastrin (a
gastrinoma). It is characterized by the development of severe peptic ulcer disease.
Goblet cells (choice C) are part of the mucosa of the small intestine, not the
stomach. They produce glycoproteins (mucins) that protect and lubricate the lining
of the intestine.
Mucous neck cells (choice D) are mucus-secreting cells located in the necks of the
gastric glands.
________________________________________
28The correct answer is D. The clavicle is the most frequently broken long bone of
the body. However, despite what one might expect, the underlying subclavian
vessels and parts of the brachial plexus are only rarely injured. This is because the
subclavius muscle, which occupies a small groove on the undersurface of the
clavicle, apparently protects the underlying structures. Other functions of this small
muscle are not well understood.
The deltoid (choice A) originates from the lateral aspect of the clavicle and is a
powerful abductor of the arm.
The pectoralis major (choice B) originates from the medial aspect of the clavicle,
and is not as important as the subclavius with regard to protecting underlying
structures.

The sternocleidomastoid (choice C), an important muscle of the anterior neck,


attaches to the medial aspect of the posterosuperior border of the clavicle.
The trapezius (choice E), an important muscle of the posterior neck, attaches to the
lateral aspect of the posterosuperior border of the clavicle.
________________________________________
29The correct answer is C. The disease is Burkitt's lymphoma (the major clue is
"starry sky" pattern), which occurs as a jaw lesion in epidemic form in Africa
(associated with Epstein-Barr virus) and in a sporadic form that usually involves the
pelvic or abdominal organs. Burkitt's lymphoma is associated with c-myc activation
due to a t(8,14) translocation that places the c-myc-containing segment of
chromosome 8 near an actively transcribed gene for immunoglobulin heavy chains.
t(9,22) (choice D) and abl-bcr hybrid (choice A) are associated with chronic myeloid
leukemia (CML).
t(14, 18) (choice E) and bcl-2 (choice B) are associated with follicular lymphomas,
not Burkitt's lymphoma.
________________________________________

30>The correct answer is C. Gonorrhea is on the short list of reportable diseases,


including AIDS (but not HIV positivity), chickenpox, hepatitis A and B, measles,
mumps, rubella, salmonella, shigella, syphilis, and tuberculosis.

________________________________________
31The correct answer is E. The splenic artery passes behind the stomach and gives
off the short gastric artery and the left gastroepiploic artery immediately after
passing the greater curvature. The left gastroepiploic artery has a strong
anastomotic connection to another arterial supply while the short gastric does not,
so the area supplied by branches of the short gastric arteries is more vulnerable to
ischemia in this setting. If the block had occurred proximal to, instead of at the
branch point, the short gastric vessels could be supplied by backflow from the left
gastroepiploic artery.
The left gastric artery (choice A) is not supplied by the splenic artery.
The left gastroepiploic artery (choice B) can be alternatively supplied by its
anastomotic connection to the right gastroepiploic artery.
The right gastric artery (choice C) is not supplied by the splenic artery.
The right gastroepiploic artery (choice D) is normally supplied by the
gastroduodenal artery.
________________________________________

32The correct answer is E. Trichomonas vaginalis (note "vagina" in the name) is the
only organism listed to primarily affect the genital tract rather than the intestinal
tract. The vaginitis it causes is characterized by a frothy, yellow discharge. The
organism can be identified in Pap smears, or, more reliably, by special culture
techniques that are becoming more widely available. Infection in men is usually
asymptomatic, but prostatitis and urethritis can also occur. Treatment with
metronidazole is indicated for both the patient and the sexual partner(s).
The other organisms listed (choices A, B, C, and D) cause gastrointestinal disease.
________________________________________
33The correct answer is E. Pepsin production is a normal physiologic activity of the
stomach that, in conditions of stress, may overwhelm the stomach's weakened
defenses and result in gastric ulceration. Gastric acid production is another
condition that may increase and cause acute ulceration. Furthermore, these two
factors may remain unchanged and still result in gastric ulcers if the gastric
defenses are weakened by stress. All of the other choices represent normal
defensive forces in the stomach.
Increased bicarbonate transport (choice A) would protect the gastric epithelium
from the potentially harmful acidity of the gastric contents. The adherent mucus is
relatively alkaline, providing local protection to the superficial mucosa.
Gastric epithelial cells can normally replicate rapidly, allowing mucosal defects to be
rapidly repaired. Increasing the regenerative capacity of the epithelium (choice B)
would have a protective effect against ulceration.
The gastric mucosa is richly supplied with blood, providing the epithelial cells with
an ample supply of nutrients, oxygen, and bicarbonate to contend with the harsh
gastric microenvironment. Stress ulcers are associated with compromised gastric
blood flow, not increased flow (choice C).
Mucus protects the gastric epithelium by virtue of being water insoluble,
impermeable to pepsin, and slowly permeated by acid (H+). Increasing mucus
production (choice D) has a protective effect for the gastric mucosa.
________________________________________

34The correct answer is E. Follicles in different stages of maturation have different


appearances. The most primitive follicles, primordial follicles (choice D), are inactive
reserve follicles that contain primary oocytes (arrested in prophase of first meiotic
division) surrounded by a single layer of flattened follicular cells. Primary follicles
(choice C), the next stage, are slightly larger and contain a central oocyte
surrounded by one or several cuboidal follicular cells. When several small spaces in
the follicular mass fuse to form the antrum (follicular cavity), the follicle is termed a
secondary follicle (choice E). The secondary follicles continue to enlarge, and
develop a more complex structure that includes cumulus oophorus, corona radiata,
theca interna, theca externa, and zona pellucida. The Graafian follicle (choice B) is

the mature form of the follicle, which extends through the entire cortex and bulges
out at the ovarian surface. After it ruptures and releases the ovum, the corpus
luteum (choice A)develops as the cells of the follicle and the theca interna cells
enlarge, become epithelioid, and secrete estrogen. The granulosa lutein cells
contain yellow pigment and secrete progesterone. If pregnancy does not occur, the
corpus luteum eventually degenerates; if pregnancy occurs, it is maintained
throughout the pregnancy.
________________________________________

35>The correct answer is A. Vitamin B1, or thiamine, is the coenzyme required (as
the pyrophosphate) for the decarboxylation of alpha-ketoacids. An example of this
reaction is pyruvate decarboxylase reaction in alcoholic fermentation. Other
reactions such as that catalyzed by pyruvate dehydrogenase also rely on thiamine
pyrophosphate for decarboxylation, but require other cofactors as well. Thiamine is
also required for the generation of pentose phosphates for nucleotide synthesis in
the pentose phosphate pathway (hexose monophosphate shunt), serving as a
cofactor for transketolase.
Vitamin B2 (choice B), or riboflavin, is a constituent of FMN (flavin mononucleotide)
and FAD (flavin adenine dinucleotide). It functions in hydrogen and electron
transport.
Vitamin B3 (choice C), or niacin (nicotinic acid), is a coenzyme that is also involved
in hydrogen and electron transport. Nicotinic acid functions in the form of NAD and
NADP.
Vitamin B5 (choice D), or pantothenic acid, is conjugated with coenzyme A to act as
a carboxylic acid carrier.
Vitamin B6 (choice E), or pyridoxine, is required as a cofactor for pyridoxal
phosphate and pyridoxamine phosphate. Both of these cofactors are essential to
protein metabolism and energy production.

________________________________________

36>The correct answer is D. This question is simple if you know that tyrosinase is
an enzyme in the biosynthetic pathway for melanin formation from tyrosine. A lack
of tyrosinase causes one form of albinism; a second form is caused by defective
tyrosine uptake. Patients with albinism are vulnerable to developing cancers of the
skin of all types, including basal cell carcinoma, squamous cell carcinoma, and
melanoma. The melanomas are unusual in that they are non-pigmented
(amelanotic) rather than black, since the patients cannot form melanin.

________________________________________

37>The correct answer is A. Class II MHC proteins are expressed on the surfaces of
macrophages, dendritic cells, and B cells; this complex of molecules is recognized
by CD 4+ helper T cells. The T cells of the transplant recipient recognize allogeneic
MHC molecules on the surface of an antigen-presenting cell of the donor. It is
thought that interstitial dendritic cells of the donor are the most important
immunogens because not only do they express class I and II HLA molecules, but
they are also endowed with co-stimulatory molecules. CD 8+ cytotoxic T cells
recognize the class I molecules. CD 4+ cells proliferate as Th1 cells and produce
interleukin 2, which causes differentiation of the CD 8 cells. The CD 8+ cytotoxic
cells of the recipient then cause lysis of the donor cells.
Interleukin-2 (choice B) activates T cells by binding to high-affinity IL-2 receptors (IL2R).
Perforins (choice C) are produced by CD 8+ cytotoxic lymphocytes as they bind to
Class I MHC molecules. The perforins damage the donor cell membranes, resulting
in lysis.
IgE-mediated reactions (choice D) are not associated with graft rejection.
Blocking antibodies (choice E) are employed as a form of immunosuppressive
therapy. Antilymphocyte globulins and monoclonal anti-T cell antibodies
(monoclonal anti-CD3) are used to inhibit rejection of the graft. This process does
not involve class II MHC proteins on donor cells.

________________________________________

38>The correct answer is E. Caffeine inhibits phosphodiesterase and thereby


prevents cAMP from being degraded. Consequently, cAMP continues to activate
protein kinase A, which results in both glycogen and triglyceride breakdown.
Caffeine has no direct or indirect effect on adenylyl cyclase (choice A).
Caffeine does not directly interact with glycogen phosphorylase (choice B) or
hormone-sensitive lipase (choice C) and cannot allosterically activate these
enzymes. However, because cAMP is not degraded due to the action of caffeine on
phosphodiesterase, the cAMP will continue to activate protein kinase A. In turn,
activation of protein kinase A will lead to phosphorylation of glycogen

phosphorylase and hormone-sensitive lipase, resulting in glycogen and triglyceride


breakdown.
Protein kinase A is not phosphorylated under normal physiological circumstances,
and therefore caffeine cannot inhibit its dephosphorylation (choice D).

39>The correct answer is B. Cephalosporins, such as cefuroxime, are believed to


exert their antibacterial effect by binding to one or more of the penicillin-binding
proteins located on the cell walls of susceptible organisms. This action results in the
inhibition of the third, and final, stage of bacterial cell wall synthesis. This is also the
mechanism of action of the penicillins. Cefuroxime is a second generation
cephalosporin used to treat infections in the lower respiratory and urinary tracts, as
well as otitis media. It is also efficacious in the treatment of gonorrhea and is used
for perioperative prophylaxis in various surgical procedures, such as coronary artery
bypass grafting.
Sulfonamide antibiotics, such as sulfamethoxazole-trimethoprim, exert their
antibacterial effect through the competitive inhibition of para-aminobenzoic acid
(PABA) (choice A), thereby inhibiting folic acid biosynthesis required for bacterial
growth.
Quinolone antibiotics, such as ciprofloxacin, inhibit DNA-gyrase (choice C), which is
an enzyme necessary for bacterial DNA replication and repair.
Aminoglycosides, such as gentamicin, irreversibly bind to the 30S subunit of
bacterial ribosomes (choice D), inhibiting bacterial protein synthesis.
Lincosamides, such as clindamycin, irreversibly bind to the 50S subunit of bacterial
ribosomes (choice E), suppressing bacterial protein synthesis. Note that macrolides,
such as erythromycin, reversibly bind to the 50S subunit of bacterial ribosome.

________________________________________

40>C 41>D 42>C

43>D

44>B

46>B 47>C 48>E

49>E

50>B

Answers

45>B

________________________________________

1The correct answer is A. This is a three-step question. First you need to figure out
the diagnosis, then you need to determine the drug of choice for this condition, and
finally, you need to remember the mechanism of action of that drug. The clinical
picture presented suggests malignant hyperthermia. The treatment for this
condition (a USMLE favorite) is dantrolene. Dantrolene prevents the release of Ca2+
from the sarcoplasmic reticulum, thereby reducing skeletal muscle contractions.
Side effects include muscle weakness and hepatotoxicity (if used chronically). Other
uses include spasticity, multiple sclerosis, and cerebral palsy.
Nondepolarizing blockers competitively inhibit the activity of acetylcholine at the
neuromuscular junction (choice B). Examples include curare, atracurium, and
vecuronium.
Baclofen is a GABAB receptor agonist that is inhibitory at synapses in the spinal
cord (choice C).
Uncouplers of oxidative phosphorylation (choice D) include dinitrophenol and
thermogenin (found in brown fat mitochondria and acts to keep blood warm in a
neonate).
________________________________________
2The correct answer is C. Kubler-Ross's death and dying sequence is a step-wise
process with 5 identified stages. The order in which these stages appear is the
following: 1. denial, 2. anger, 3. bargaining, 4. sadness, and 5. acceptance. "Doctor
you must be wrong" is the correct answer since it reflects the patient's inability to
accept the information and indicates the denial of the first stage.
"Can you keep me alive until my daughter graduates from medical school" (choice
A) is a statement from the 3rd, bargaining stage.
"Damn you doctor, you should have caught this earlier" (choice B) is a statement
from the 2nd or anger phase.
"I think it is time that I make a will and say good-bye to everyone" (choice D)
reflects the patient's acceptance of the reality and is a statement from the 5th
phase (acceptance).
"It's no use, I always lose and get the short end of the stick" (choice E) is a
statement from the 4th phase (sadness).
________________________________________
3The correct answer is B. This is a multi-step microbiology question that requires
you to diagnose the illness, identify the microorganism, and remember its key
feature. The first part should be easy: everything about this vignette suggests toxic
shock syndrome. The organism in question is therefore Staphylococcus aureus,

which is coagulase positive. All of the other choices are classic features of other
important pathogenic microorganisms:
Acid-fast organisms (choice A), refers to Mycobacteria. (In addition, Nocardia
species are partially acid fast).
EMB agar (choice C) refers to a selective and differential medium used to isolate
and identify enteric gram-negative bacteria. Gram-positive bacteria will not grow on
EMB agar because the addition of eosin inhibits their growth. Nonlactose fermenters
will have colorless colonies, while fermentation of this sugar will cause the colonies
to appear pink or purple.
Thayer-Martin media (choice D) is a growth medium for pathogenic Neisseria
species. It contains the antibiotic vancomycin, which kills gram-positive organisms
such as Staphylococcus aureus.
The Quellung reaction (choice E) can be used to identify the capsule type of a
microorganism. Encapsulated microorganisms, like the Pneumococci and
Haemophilus, are mixed with specific antisera. If the antiserum is directed against
the microorganism's capsule type, the capsule will be opsonized, absorb water, and
become visible under a light microscope.
________________________________________
4The correct answer is B. The extrinsic pathway of clotting begins with tissue factor
binding to Factor VII or Factor VIIa. All other clotting proteins require proteolytic
cleavage to become active, however Factor VII has a low level of activity in its
inactive form, and it can act with tissue factor and phospholipids to initiate the
clotting cascade. In the extrinsic pathway, Factor VII cleaves Factor X to Xa (choice
C) which acts in concert with Factor V (choice A) to cleave prothrombin to thrombin
(choice E). The final step in the coagulation pathway is the cleavage of fibrinogen to
fibrin by thrombin (choice D). Fibrin polymerizes and crosslinks, thereby forming a
hemostatic net of insoluble protein.
________________________________________
5The correct answer is A. Any perforating wound occurring below the level of the
fourth intercostal space on the right side may damage the liver, which is protected
by the rib cage, although it is an abdominal organ lying inferior to the diaphragm.
At its most lateral aspect, the right atrium (choice B) forms the right border of the
heart, which extends from the 3rd costal cartilage to the 6th costal cartilage just to
the right of the sternum.
The right pulmonary artery (choice C) enters the hilus of the lung at the level of the
T5 vertebra. Since the ribs are angled downward as they pass forward, this entry
occurs above the level of the 5th intercostal space at the midaxillary line.
The superior vena cava (choice D) enters the right atrium at the level of the third
costal cartilage.

At the midaxillary line, the oblique fissure of the right lung (choice E) passes
between the inferior and middle lobes.
________________________________________
6The correct answer is B. This child probably has autosomal dominant von
Willebrand's disease, in which an abnormal von Willebrand's factor (which also
carries factor VIII in the blood) causes a defect in the initial adhesion of normal
platelets to a damaged vessel wall. Since factor VIII levels are also consequently
low, the partial thromboplastin time is also prolonged. Defects in platelet adhesion
are also seen in Bernard-Soulier disease.
Abnormal platelet morphology (choice A) is not seen in von Willebrand's disease,
but may be observed with infiltration of the bone marrow by tumor or fibrosis, or
after splenectomy.
Defects in primary platelet aggregation (choice C) are seen in thrombasthenia,
which is caused by a deficiency or defect in the glycoprotein GpIIb-IIIa complex.
Defects of release of platelet vesicles (choice D) or secondary aggregation (choice
E) are seen in storage pool disease and aspirin use.
________________________________________
7The correct answer is A. The patient's ankle edema, shortness of breath, and
relatively low blood pressure suggest the possibility of congestive heart failure,
which is confirmed by the cardiac enlargement and perihilar infiltrates seen on
chest x-ray. The serum urea nitrogen is elevated while serum creatinine is normal,
suggesting a prerenal cause for the azotemia. Congestive heart failure with its
resulting decreased blood pressure is a known, common cause of decreased renal
perfusion leading to prerenal azotemia.
Postrenal causes of azotemia are typically due to urinary tract obstruction distal to
the kidney (choice B), and usually cause a rise in both urea and creatinine, with the
rise in urea being larger than that in creatinine.
Increased synthesis of urea (choice C) is seen in severe burns and prolonged high
fever.
Renal glomerular disease (choice D) severe enough to cause acute or chronic renal
failure will cause urea and creatinine to rise together.
Renal tubulointerstitial disease (choice E) severe enough to cause renal failure will
cause both urea and creatinine to rise; the creatinine may rise out of proportion to
the urea, particularly in acute tubular necrosis.
________________________________________
8The correct answer is C. The patient has developed bacteremia; the description of
the causative agent is consistent with a staphylococcal organism (catalase positive,
gram-positive cocci that grow on mannitol salt agar). The organism is most likely S.
epidermidis as it was not able to ferment mannitol, and was not hemolytic. Both of

those characteristics tend to rule out S. aureus (choice B). Two other tests that are
commonly used are coagulase production and excretion of DNAse from colonies. S.
aureus is positive in both tests, S. epidermidis is negative.
Enterococcus faecalis (choice A) might grow on the mannitol salt agar as it is
relatively haloduric but these organisms are catalase negative. The enterococci are
extremely variable in hemolytic ability so this characteristic is not useful in species
identification.
Both streptococcal organisms (choices D and E) are catalase negative and betahemolytic on sheep blood agar plates. Also, neither would grow on the mannitol salt
agar. S. pyogenes is sensitive to growth inhibition by bacitracin while S. agalactiae
(group B streptococci) is not.
________________________________________
9The correct answer is C. Severe combined immunodeficiency (SCID) is associated
with deficiencies in both B and T cells due to a defect in differentiation of an early
stem cell. Over 50% of the cases are caused by a gene defect on the X
chromosome, resulting in a defective IL-2 receptor. The disease may exhibit a sexlinked or an autosomal recessive pattern of inheritance. The autosomal recessive
variant is characterized by a deficiency of adenosine deaminase, which results in
accumulation of metabolites that are toxic to both B and T stem cells in the bone
marrow. Children usually die within the first 2 years of life with severe infections
unless they receive bone marrow transplants.
Bloom's syndrome (choice A) is an autosomal recessive disorder included in the
chromosomal instability group of syndromes. It is associated with small body size,
immunodeficiency, light-sensitive facial erythema, and a major predisposition to
cancer.
Chronic granulomatous disease (choice B) is caused by a deficiency of NADPH
oxidase in neutrophils, resulting in loss of the first step of the myeloperoxidase
system, and an absence of the respiratory burst. Patients are susceptible to
staphylococcal infections and granulomatous infections.
Waldenstrm's macroglobulinemia (choice D) is a monoclonal gammopathy
characterized by high serum levels of IgM and hyperviscosity complications.
Wiskott-Aldrich syndrome (choice E) is an immunodeficiency syndrome
characterized by thrombocytopenia, eczema, and recurrent sinopulmonary
infections. The patient has low levels of IgM and increased levels of IgG, IgA, and
IgE.
________________________________________
10The correct answer is A. The lymph nodes of the groin are divided into superficial
and deep groups. The superficial group is further divided into horizontal and vertical
chains. It is worth remembering that tumors from the penis, vagina, and anal canal
can drain to the medial side of the horizontal chain of the superficial group of
inguinal lymph nodes.

Tumors from the ascending colon (choice B) do not usually metastasize early to
easily palpable lymph nodes.
Tumors from the duodenum (choice C) do not usually metastasize early to easily
palpable lymph nodes.
Tumors from the stomach (choice D) can metastasize early to the easily palpable
left supraclavicular nodes (also called Virchow's nodes or sentinel nodes).
Tumors from the transverse colon (choice E) do not usually metastasize early to
easily palpable lymph nodes.
________________________________________
11The correct answer is A. The most important factor in the control of minute-tominute ventilation is arterial PCO2, which influences chemoreceptors located near
the ventral surface of the medulla. As arterial PCO2 rises, CO2 diffuses from
cerebral blood vessels into the cerebrospinal fluid. Carbonic acid is formed and
dissociates into bicarbonate and protons. Protons directly stimulate these central
chemoreceptors, resulting in hyperventilation. Hyperventilating reduces the PCO2 in
the arterial blood and subsequently in the CSF.
Peripheral chemoreceptors located in the carotid and aortic bodies respond to
increases in PCO2 (choice B), but are less important than the central
chemoreceptors. It is estimated that when a normal subject hyperventilates in
response to inhalation of CO2, less than 20% of the response can be attributed to
the peripheral receptors. However, they respond more quickly than their central
counterparts, and are thought to play a role in regulating ventilation after abrupt
changes in PCO2.
There are no known central chemoreceptors that respond to arterial pH (choice C).
Carotid chemoreceptors (choice D) cause hyperventilation in response to decreases
in arterial pH. However, the CO2 acting through central chemoreceptors is the most
important regulator of ventilation under normal conditions.
There are no known central chemoreceptors that respond to arterial PO2 (choice E).
Peripheral chemoreceptors located in the carotid and aortic bodies respond to
decreases in PO2 (choice F) and are solely responsible for the increase in ventilation
due to arterial hypoxemia. However, the CO2 acting through central
chemoreceptors is the most important regulator of ventilation under normal
conditions.
There are two types of Hering-Breuer reflexes (choice G). The Hering-Breuer inflation
reflex increases the duration of expiration after steady lung inflation. The HeringBreuer deflation reflex causes a decrease in the duration of expiration produced by
substantial deflation of the lung. The degree of lung inflation is sensed by
pulmonary stretch receptors thought to reside within the smooth muscle of the
airways. This reflex was once thought to be an extremely important mechanism for
regulation of ventilation. However, it is now known that this reflex does not become

active unless tidal volumes exceed one liter (as in exercise). This reflex may also be
important in newborns.
________________________________________
12The correct answer is C. The infant probably has esophageal atresia, which is
typically caused by posterior deviation of the tracheoesophageal septum. Attempts
at feeding cause fluid to spill into the trachea, and secondarily cause aspiration
pneumonia. Emergent surgical correction is usually required.
Bronchogenic cysts (choice A) are centrally located cysts that are often
asymptomatic and may be associated with cysts of other organs.
Congenital pulmonary cysts (choice B) are often multiple and located in the lung
periphery without connection to the bronchi; they are vulnerable to infection and
rupture complicated by pneumothorax and/or hemoptysis.
Pulmonary immaturity (choice D) produces progressive difficulty in breathing
beginning in the first few hours of life.
Pulmonary sequestration (choice E) represents extrapulmonary lung tissue supplied
by systemic blood vessels rather than by pulmonary arteries.
________________________________________
13The correct answer is C. The subscapularis muscle arises from the anterior
surface of the scapula and inserts onto the lesser tubercle of the humerus. It is one
of the rotator cuff muscles. Its tendon passes on the anterior side of the shoulder
joint capsule, where it reinforces the capsule. Contraction of this muscle causes
internal rotation of the arm at the shoulder.
The infraspinatus muscle (choice A) arises from the posterior surface of the scapula
in the infraspinous fossa and inserts on the greater tubercle of the humerus. It is
one of the rotator cuff muscles. Its tendon passes along the posterior surface of the
shoulder joint capsule, where it reinforces the capsule. Contraction of the
infraspinatus causes external rotation of the arm at the shoulder.
The pectoralis minor muscle (choice B) arises from the chest wall and inserts onto
the coracoid process of the scapula. The pectoralis minor does not attach to the
humerus and therefore does not cause movement of the humerus at the shoulder.
The supraspinatus muscle (choice D) arises from the posterior surface of the
scapula in the supraspinous fossa and inserts onto the greater tubercle. It is one of
the rotator cuff muscles. Its tendon passes along the superior surface of the
shoulder joint capsule, where it reinforces the capsule. Contraction of the
supraspinatus causes abduction of the arm at the shoulder.
The teres minor muscle (choice E) arises from the axillary border of the scapula and
inserts onto the greater tubercle of the humerus. It is one of the rotator cuff
muscles. Its tendon passes along the posterior surface of the shoulder joint capsule,
where it reinforces the capsule. Contraction of the teres minor causes external
rotation of the arm at the shoulder.

________________________________________
14The correct answer is E. The ring described is the zona pellucida, which surrounds
the ovum. The zona pellucida is rich in polysaccharides and glycoproteins and
consequently stains brightly pink or red with PAS stain. Binding of the sperm cell
membrane to the zona pellucida triggers the acrosome reaction, during which
acrosomal enzymes are released that digest the zona pellucida, allowing the
spermatozoon to contact and fuse with the ovum cell membrane.
The follicular cells immediately outside the zona pellucida form the corona radiata
(choice A). The larger cumulus oophorus (choice B) is the hill of follicular cells that
surrounds the ovum.
The theca interna (choice D) and externa (choice C) are formed from the connective
tissue surrounding the follicle.
________________________________________
15The correct answer is D. The paired umbilical arteries arise from the iliac arteries.
They supply unoxygenated fetal blood to the placenta. The single umbilical vein
takes the newly oxygenated fetal blood from the placenta to the liver and then to
the inferior vena cava via the ductus venosus.
________________________________________
16The correct answer is C. Inflammatory breast cancer is a pattern of invasive
breast cancer in which the neoplastic cells infiltrate widely through the breast
tissue. The cancer involves dermal lymphatics and therefore has a high incidence of
systemic metastasis and a poor prognosis. If the lymphatics become blocked, then
the area of skin may develop lymphedema and "peau d'orange," or orange peel
appearance. The overlying skin in inflammatory breast cancer is usually swollen,
red, and tender.
Acute inflammation (choice A) is a rare finding in breast cancer and may be
associated with secondary infection or abscess.
Chronic inflammation in breast cancer (choice B) is a non-specific finding. In
medullary breast cancer, a type of invasive ductal carcinoma, there are a large
number of lymphocytes around the tumor and a desmoplastic reaction is often
absent in the surrounding tissue. This type of cancer carries a somewhat better
prognosis.
Epidermal invasion by cancer cells (choice D) is a poor prognostic indicator.
Intraepidermal malignant cells are called Paget cells. Paget's disease of the nipple is
a type of ductal carcinoma that arises in large ducts and spreads intraepidermally to
the skin of the nipple and areola. There is usually an underlying ductal carcinoma.
Fat necrosis (choice E) is often seen following trauma to the breast, but is not
specifically associated with a particular type of breast cancer, although it may be
confused with breast cancer if areas of calcification are present.
________________________________________

17The correct answer is A. Astrocytes and oligodendrocytes are both derived from
glioblasts, which, in turn, are derived from neuroepithelial cells. Other
neuroepithelial cell derivatives include neuroblasts and ependymal cells.
All the other choices are derived from neural crest cells. Other neural crest
derivatives include the neurons of the parasympathetic and sympathetic ganglia
(including the adrenal medulla), the dorsal root ganglia of the peripheral nervous
system, the sensory ganglia of cranial nerves V, VII, IX, and X, and the
leptomeninges (pia and arachnoid).
________________________________________
18The correct answer is E. The various organs of the body are arranged in parallel,
and therefore contribute a parallel resistance to the peripheral circulation. You
should recall that adding resistances (R1, R2, R3...) in parallel reduces the total
resistance (RT) of a circuit as follows (1/RT = 1/R1 + 1/R2 + 1/R3...) so that
removing a parallel resistance (R1, R2, or R3) increases the total resistance (RT). For
this reason, the total peripheral resistance increases when a kidney is removed.
Another way to think about the problem is the following: each kidney provides a
pathway for blood to flow from the aorta to the vena cava. When a kidney is
removed, there is one less pathway through which blood can flow from the aorta to
the vena cava, which means that the resistance to blood flow from the aorta to the
vena cava (i.e., the total peripheral resistance) must be increased. Similar logic can
be applied to any organ of the body.
Removing a kidney should have not have a lasting effect on arterial pressure
(choice A), assuming that the remaining kidney functions normally.
The cardiac output (choice B) decreases when a kidney is removed.
The pulmonary blood flow (choice C), which is equal to the cardiac output of the
right heart, should decrease when a kidney is removed.
The total renal blood flow (choice D) will decrease when a kidney is removed.
________________________________________

19The correct answer is E. Whenever serum calcium and phosphate are both
decreased, vitamin D deficiency should be considered. In this case, the vitamin D
deficiency is due to fat malabsorption, including the fat-soluble vitamin D,
subsequent to ileal resection. If more than 100 cm of the ileum are removed,
primary bile acid production by the liver cannot keep up with bile salt loss in the
stool. The total bile salt pool decreases and fat absorption, including the fat-soluble
vitamins is poor. Serum calcium is low because of decreased dietary absorption.
Serum parathyroid hormone increases in response to the low calcium. Serum
phosphate is low because of decreased dietary absorption and increased renal
excretion (due to the increased parathyroid hormone). With vitamin D deficiency,
the bones demineralize, producing osteomalacia. Clinical manifestations often go
unnoticed. Vague complaints of weakness and bone pain may be present.

Radiographs of bones in osteomalacia typically reveal the presence of


pseudofractures along the inner aspects of the femur, the pubic rami, and the outer
edges of the scapulas, upper fibula, and metatarsals. These radiolucent bands,
which are perpendicular to the bone surface, may occur because of pulsations of
major arteries that cross the bone.
Osteoporosis (choice A) is characterized by loss of bone mass, both matrix and
mineral. It is usually asymptomatic, and serum levels of calcium, phosphate, and
parathyroid hormone are within the normal range. The first hint of bone loss comes
because of a fracture in the wrist, hip, or vertebra. Dual-energy radiography, or
other similar techniques, can be used to directly quantify the degree of bone loss.
In Paget's disease (choice B), bone mineral turnover is increased compared to
normal. Its cause is unknown, but may be due to a slow virus that infects osteoclast
cells. Both osteoblast and osteoclast activity is increased in focal areas of the bone.
The disease is usually asymptomatic, the chief complaint being bone pain over the
lesions. Laboratory findings include increased serum alkaline phosphatase, but
serum calcium and parathyroid hormone levels are usually normal.
With primary hypoparathyroidism (choice C), serum calcium is decreased and serum
parathyroid hormone is increased, but serum phosphate is also increased (not
decreased). Serum phosphate is increased because urinary excretion is diminished
subsequent to the decrease in parathyroid hormone.
In renal failure (choice D), an increase (not decrease) in serum phosphate
subsequent to decreased urinary excretion is a primary manifestation. Serum
calcium is decreased because the hyperphosphatemia drives the equilibrium
between calcium and phosphate toward hydroxyapatite crystals. This, in turn,
produces an increase in parathyroid hormone secretion with subsequent bone
demineralization (renal osteodystrophy).
________________________________________
20The correct answer is D. During the fourth week of development, the lateral body
folds move ventrally and fuse in the midline to form the anterior body wall.
Incomplete fusion results in a defect that allows abdominal viscera to protrude from
the abdominal cavity, a condition known as gastroschisis.
During development, the midgut normally herniates into the umbilical cord and then
subsequently retracts into the abdominal cavity. Failure of the intestinal loop to
retract from the umbilical cord (choice A) results in omphalocele.
Failure of the yolk stalk to degenerate (choice B) results in an ileal (Meckel's)
diverticulum or a vitelline fistula or cyst. In the early embryo, the gut tube is
connected to the yolk sac by a narrow connection known as the yolk stalk. Normally,
this connection degenerates.
During development, certain peritoneal organs fuse with the posterior abdominal
wall to become secondarily retroperitoneal. Failure of this peritoneal fusion (choice
C) will result in certain organs that are normally immobile being mobile (e.g., mobile
cecum).

Umbilical herniation (choice E) results from abdominal viscera protruding through a


weakness in the abdominal wall after development. Such protrusions are covered by
subcutaneous fascia and skin, distinguishing them from gastroschisis.
________________________________________
21The correct answer is C. The bladder is supplied by the vesicular branches of the
internal iliac arteries. The internal iliacs arise from the common iliac artery. Note
that this is a simple fact question (Which artery supplies the urinary bladder?)
embedded in a clinical scenario.
The external iliac (choice A) also arises from the common iliac artery. It makes no
contribution to the blood supply of the bladder.
The inferior epigastric (choice B) is a branch of the external iliac artery. It serves as
a landmark in the inguinal region. Indirect inguinal hernias lie lateral to the inferior
epigastric arteries, whereas direct inguinal hernias lie medial to these vessels. A
good mnemonic is MD's don't LIe. (Medial-Direct, Lateral-Indirect).
The internal pudendal (choice D) is a branch of the anterior division of the internal
iliac artery. It gives rise to the inferior rectal artery, perineal artery, artery of the
bulb in men, urethral artery, deep artery of the penis or clitoris, and dorsal artery of
the penis or clitoris.
The lateral sacral (choice E) is a branch of the posterior division of the internal iliac
artery. It supplies sacral structures.
________________________________________
22The correct answer is B. Pituitary apoplexy, which is a life-threatening infarction
of the pituitary gland, may result after obstetric hemorrhage (Sheehan's syndrome),
with increased intracranial pressure, or during systemic anticoagulation therapy.
While the patient may have a generalized hypofunction of the pituitary in these
settings, the most important hormones to quickly replace are glucocorticoids
(synthesized by the adrenal cortex under pituitary ACTH control) and thyroid
hormone (under pituitary TSH control), since deficiency of these hormones produces
life-threatening syndromes.
Growth hormone, estrogens, and prolactin (choices A, C, D, and E) do not require
immediate replacement.
Mineralocorticoids (choices D and E) can be replaced if diabetes insipidus develops.
________________________________________
23The correct answer is B. The disease is cretinism, characterized by a profound
lack of thyroid hormone in a developing child, leading to mental retardation and the
physical findings described in the question stem. Cretinism can be due to dietary
deficiency of iodine (now rare in this country because of iodized salt), to
developmental failure of thyroid formation, or to a defect in thyroxine synthesis.
Calcium deficiency (choice A) in children can cause osteoporosis or osteopenia.

Iron deficiency (choice C) can cause a hypochromic, microcytic anemia.


Magnesium deficiency (choice D) is uncommon, but can cause decreased reflexes,
and blunts the parathyroid response to hypocalcemia.
Selenium deficiency (choice E) is rare, but may cause a reversible form of
cardiomyopathy.
________________________________________
24The correct answer is E. Mycobacterium leprae is endemic to parts of Africa, Asia,
and South America. Tuberculoid leprosy is an indolent disease, typically affecting
cooler parts of the body, such as the nose and ears, producing dermal granulomas
with very rare, acid-fast bacilli and damage to peripheral nerves. This is in marked
contrast to lepromatous leprosy, which is progressive and invasive and generally
characterized by the presence of numerous acid-fast bacteria in a histiocytic, but
non-granulomatous tissue response. M. leprae has not been grown in any culture
medium.
Cutaneous leishmaniasis (choice A) is due to Leishmania species that show a
worldwide distribution. The infection is transmitted through a sandfly bite, and the
skin lesion is typically ulcerated. Histology shows intracellular parasites within the
dermis and epidermis. Granulomas are not formed.
Onchocerciasis (choice B), or river blindness, is a roundworm infection transmitted
by black flies of Africa and South America. The microfilaria grow at the site of
inoculation, and cause an inflamed subcutaneous nodule. The organism is seen on
tissue sections.
Rhinoscleroma (choice C) is a destructive granulomatous infection of the
nasopharynx caused by Klebsiella rhinoscleromatis. Gram-negative rods can be
cultured from the lesions.
Smallpox (choice D), the infection caused by the variola virus (a DNA poxvirus),
produces malaise, headaches, and a macular/pustular rash involving the face and
distal extremities. Smallpox has been eradicated through worldwide vaccination
________________________________________
25The correct answer is A. The patient described suffers from myasthenia gravis
(MG), a disorder in which autoantibodies to skeletal muscle nicotinic acetylcholine
receptors cause a reduction in the receptor number, leading to easy fatigability,
weakness of extraocular and facial muscles, and difficulty eating. Symptomatic
improvement in these patients can be obtained with acetylcholinesterase inhibitors.
Two of the most common drugs given are neostigmine and pyridostigmine, both of
which act by carbamylating the acetylcholinesterase enzyme. This temporarily
inhibits the enzyme, thus preventing the degradation of acetylcholine and allowing
greater stimulation of the nicotinic acetylcholine receptors on skeletal muscle.
Edrophonium (Tensilon) is a short-acting competitive inhibitor of
acetylcholinesterase (choice B), used in the diagnosis of MG. The physician selects a
weak muscle and then administers edrophonium to the patient. It the test is

positive, the patient will exhibit increased strength in that muscle for about two
minutes. Edrophonium is not useful for the treatment of MG because of its short
duration of action.
Pralidoxime (2-PAM) is an acetylcholinesterase reactivating agent that
dephosphorylates acetylcholinesterase (choice C). It is used if an individual is
exposed to a phosphorylating acetylcholinesterase inhibitor (e.g., parathion or
nerve gases). 2-PAM has a higher affinity for phosphorus than the enzyme, and thus
can bind the acetylcholinesterase inhibitor and release the enzyme if "aging" of the
phosphate bond has not occurred. This allows the active enzyme to be regenerated.
A direct muscarinic agonist (choice D) such as pilocarpine or bethanechol would
cause enhanced parasympathetic effects such as increased activity in the bowel
and bladder, pupillary miosis and accommodation for near vision. These agents are
used clinically to activate bowel and bladder smooth muscle, and in the treatment
of glaucoma. They have no place in the treatment of MG.
A direct muscarinic antagonist (choice E) such as atropine could be used as an
antidote for an anticholinesterase inhibitor because it blocks the excessive
parasympathetic side effects that would occur with elevated acetylcholine.
Direct-acting nicotinic receptor agonists (choice F) do not have therapeutic
applications except for succinylcholine, a depolarizing skeletal muscle relaxant used
in surgery.
Direct nicotinic receptor antagonists (choice G), such as curare and pancuronium,
are skeletal muscle relaxants. These drugs block the skeletal muscle nicotinic
receptor (NM). Antagonists at the ganglionic nicotinic receptor (NN), such as
hexamethonium or trimethaphan, interrupt sympathetic and parasympathetic
outflow. Neither type of agent would be used in the treatment of MG.
Agents that phosphorylate acetylcholinesterase (choice H), such as echothiophate,
cause irreversible inhibition of acetylcholinesterase and are not useful in the
treatment of MG. Insecticides (e.g., malathion, parathion) and nerve gases are also
phosphorylating agents.
________________________________________
26The correct answer is A. Graves' disease is the most common cause of
hyperthyroidism in a young female and is the only one that causes exophthalmos
("bulging of both eyes"). Grave's disease is an autoimmune disorder in which a
thyroid-stimulating IgG immunoglobulin (TSI) binds to the TSH receptors causing
increased release of thyroid hormone. The exophthalmos is caused by lymphocytic
infiltration of the extraocular muscles.
Hashimoto's thyroiditis (choice B) results in hypothyroidism and is associated with a
diffusely enlarged thyroid gland and antimicrosomal antibodies against the thyroid
parenchyma.
Multinodular toxic goiter (choice C) causes hyperthyroidism, but does not result in
exophthalmos.

Papillary carcinoma (choice D) will only very rarely present as a hypersecreting


nodule. Most cases will be nonsecreting, cold nodules. There is no exophthalmos.
This is the most common thyroid cancer and has the best prognosis of all thyroid
cancers.
Subacute thyroiditis (choice E) is an uncommon form of thyroiditis that lasts
approximately 8 months and is self-limited. Early on, with destruction of thyroid
tissue, there may be release of thyroid hormone and symptoms of hyperthyroidism,
but exophthalmos is generally absent.
________________________________________
27The correct answer is D. The vagina of prepubertal girls and post-menopausal
women is colonized by colonic and skin bacteria, including Staphylococcus
epidermidis, which is normally found on the skin.
The vagina of women of child-bearing age tends to be colonized by Lactobacillus
(choice B) species, yeasts such as Candida (choice A), and Streptococcus species
(choice E).
The presence of Neisseria (choice C), such as N. gonorrhoeae (the cause of
gonorrhea), in the vagina of a 5 year-old strongly suggests sexual abuse.
________________________________________
28The correct answer is E. Sickling crises can be triggered by hypoxia caused by
high altitude or pneumonia. The very high osmolarity of the renal medulla
particularly favors sickling of erythrocytes in the vasa recta. The resultant ischemia
can cause a patchy papillary necrosis, proteinuria, and sometimes, cortical scarring.
Larger blood vessels such as the arcuate arteries (choice A), interlobar arteries
(choice C), and renal arteries (choice D) are usually not occluded by sickled cells.
The glomerular capillaries (choice B) are small enough to be occluded by sickled
cells, but they are well oxygenated and the blood within them is not hypertonic, so
the glomerular capillaries are significantly less likely to become occluded then the
vasa recta.
________________________________________

29The correct answer is C. The patient is presenting with signs and symptoms of
cystic fibrosis (CF). CF is an autosomal recessive disorder of the exocrine glands.
The pulmonary manifestations include acute and chronic bronchitis, bronchiectasis,
chronic bouts of pneumonia, hemoptysis, and cor pulmonale, which can occur late
in the disease. Other common findings include chronic cough, exercise intolerance,
recurrent respiratory infections, digital clubbing, increased anteroposterior
diameter, and basilar crackles. If the pilocarpine sweat test reveals sodium and
chloride levels greater than 80 mEq/L, a diagnosis of CF can be made. The primary
goals of treatment include thinning the mucus secretions, keeping the airways open,
and treating recurrent infections. Thinning of mucus can be achieved with

mucolytics such as N-acetylcysteine. N-Acetylcysteine (Mucomyst) splits the


disulfide linkages between these mucoproteins, resulting in a decrease in mucous
viscosity. It is indicated as adjuvant therapy in the treatment of abnormal viscid or
inspissated mucus secretions in CF, chronic lung disease, post-traumatic chest
complications, and atelectasis secondary to mucus obstruction. Inhaled
bronchodilators are used to open the airways. Furthermore, prednisone has been
shown to increase pulmonary function and increase body weight. The definitive
treatment is lung transplantation.
Dextromethorphan (choice A), a cough suppressant, is contraindicated in patients
with CF since it will prevent the removal of mucus from the lungs.
Ipratropium (choice B) is an anticholinergic that will cause a drying and thickening
of the mucus in this patient; therefore, it is contraindicated.
Pentamidine (aerosolized) (choice D) is an antiprotozoal agent primarily used in the
treatment of Pneumocystis carinii pneumonia in HIV-infected patients.
Vancomycin (choice E) is an anti-infective agent used in the treatment of lifethreatening, gram-positive infections.
________________________________________
30The correct answer is C. Stage 2 has more theta waves than stage 1 and is
associated with sleep spindles (short bursts of 12-16 Hz activity) and K-complexes
(high amplitude slow waves with superposed sleep spindles) on the
electroencephalogram
Transient large amplitude potentials in the occipital areas (ponto-geniculo-occipital
[PGO] spikes) are associated with REM sleep (choice A).
Stage 1 (choice B), or drowsiness, is characterized by the attenuation of alpha
rhythm (8-13 Hz) and the appearance of 4-7-Hz theta waves.
Stages 3 (choice D) and 4 (choice E), or slow wave sleep, are characterized by high
amplitude slow waves, especially in the delta (< 4 Hz) frequency range.
________________________________________
31The correct answer is A. Alpha2-receptor agonists directly inhibit pancreatic
insulin secretion.
Beta2-adrenergic agonists (choice B) stimulate insulin secretion.
Cholecystokinin (choice C) is a hormone that not only causes gallbladder
contraction, but also causes insulin secretion from the pancreas.
Pancreatic glucagon (choice D) release acts as a paracrine stimulus for insulin
secretion.
Ingestion of high-sugar meals (choice E) is a stimulus for the secretion of insulin
from the pancreas.

Muscarinic activity (choice F) in the GI tract enhances secretion of insulin from the
pancreas.
________________________________________
32The correct answer is E. The spleen follows the long axes of ribs 9 to 11 and lies
mostly posterior to the stomach, above the colon, and partly anterior to the kidney.
Therefore, it is the most likely organ of the group to be pierced by a sharp object
penetrating just above rib 10 at the posterior axillary line. Note that the pleural
cavity, and possibly the lower part of the inferior lobe of the lung, would be pierced
before the spleen.
The ascending colon (choice A) is on the wrong side (the right) to be penetrated by
a sharp instrument piercing the left side.
Most of the duodenum (choice B) is positioned too far to the right to be affected by
this injury. Even the third part of the duodenum, which runs from right to left, would
still be out of harm's way. In addition, the duodenum lies at about levels L1 to L3,
placing it too low to be injured in this case.
The superior pole of the left kidney (choice C) is bordered by the lower part of the
spleen. However, it is crossed by rib 12 and usually does not extend above rib 11. It
would probably be too low and medial to be injured in this case, since this
penetration is at the posterior axillary line.
The left lobe of the liver (choice D) is positioned just beneath the diaphragm, just
over and anterior to the stomach. The anterior positioning of this structure makes it
an unlikely candidate for injury in this case. Even with deep penetration at the
correct angle it would not be penetrated before the spleen.
________________________________________
33The correct answer is D. The patient has gout, which is due to precipitation of
monosodium urate crystals in joint spaces (notably the great toe) and soft tissues
(causing tophi, which are often found on the external ears).
Bile pigments (choice A) are found in some gallstones.
Calcium pyrophosphate (choice B) crystals are deposited in pseudogout, which
classically affects the knee or other large joints.
Cystine (choice C) and struvite (choice E) can form kidney stones.
________________________________________
34The correct answer is E. This is a typical case description of chancroid, caused by
Haemophilus ducreyi, a pleomorphic gram-negative rod that displays a
characteristic pattern on Gram's stained slides.
Epithelial cells with intranuclear inclusion bodies (choice A) would be found with
herpes simplex infections, but these lesions would not have the appearance
described in this case history.

Iodine-staining intracellular inclusion bodies (choice B) would be found with genital


lesions of Chlamydia trachomatis, the causative agent of lymphogranuloma
venereum, but this lesion is generally nonpainful.
Koilocytotic squamous epithelial cells (choice C) would be found in infections with
human papilloma virus (HPV), which is associated with venereal warts.
Neutrophils containing gram-negative diplococci (choice D) would be found if this
were gonorrhea, but the case symptoms are not consistent with this disease.
Spirochetes (choice F) would be found on darkfield microscopy if this were a case of
syphilitic chancre, but that chancre would be hard and nontender.
________________________________________

35The correct answer is C. Clues to the correct answer are the underlying disease
(AIDS), which predisposes to opportunistic infections, the typical MRI appearance of
the lesion (ring-enhancing mass), and the histopathologic features (presence of
encysted bradyzoites). In short, this patient has cerebral toxoplasmosis, which
represents one of the most common opportunistic infections in AIDS. Toxoplasma
gondii is a protozoon that infects humans who ingest the oocysts from cat feces or
incompletely cooked lamb or pork. Only immunodepressed patients and fetuses are
vulnerable to this infection. In the fetus, toxoplasmosis causes extensive damage to
brain parenchyma and retina. Toxoplasmosis associated with AIDS manifests with
necrotizing lesions surrounded by chronic inflammation. A ring-enhancing lesion is a
mass that contains a rim of contrast enhancement (bright signal on MRI)
surrounding a dark core corresponding to central necrosis. In AIDS, the most
frequent causes of a ring-enhancing lesion are primary brain lymphoma and
toxoplasmosis.
Anopheles mosquitoes (choice A) transmit malaria parasites. Cerebral malaria is
caused by Plasmodium falciparum, which is able to adhere to endothelial cells of
small cerebral vessels. Vascular occlusion ensues, resulting in numerous small
infarcts.
Bird droppings (choice B) represent the vehicle of infection for two of the most
common opportunistic fungal infections affecting immunocompromised patients:
cryptococcosis and histoplasmosis. Cryptococcus neoformans causes
meningoencephalitis, not intracerebral necrotic lesions. Histoplasma capsulatum
rarely affects the brain. These fungi can be visualized in tissue sections by silver
stains.
Cooling systems (choice D) may harbor Legionella pneumophila, spreading the
bacteria in aerosolized form. L. pneumoniae is a gram-negative bacterium that
causes Legionnaire's disease, a fatal form of pneumonia that first struck participants
at a meeting of the American Legion. It has been reported in immunocompromised
patients as well.

Washbasins (choice E) frequently contain Pseudomonas aeruginosa, which has also


been isolated from respirator devices, cribs, and antiseptic-containing bottles. P.
aeruginosa tends to affect patients with cystic fibrosis, severe burns, or immune
impairment. It may cause bronchopneumonia, osteomyelitis, endocarditis, external
otitis, and keratitis, but not cerebral infection.
________________________________________

36The correct answer is A. During each cardiac cycle, the walls of the ventricle
undergo isometric contraction and relaxation as well as isotonic contraction and
relaxation. Muscle contraction and relaxation is considered to be isometric when the
muscle length does not change, and isotonic when the muscle length does change
with a constant tension on the muscle. Phase 3 corresponds to a period of isometric
contraction, referred to as the period of isovolumetric or isovolumic contraction. The
ventricle is contracting and the pressure is rising, but the volume of the ventricle
remains constant, thus muscle length is relatively constant. The aortic valve opens
when ventricular pressure exceeds about 80 mm Hg, allowing blood to eject from
the heart, which begins a phase of isotonic contraction (phase 2, choice C). This
phase is called the period of ejection. Phase 1 begins when the ventricle relaxes and
the aortic valve closes (period of isovolumetric or isovolumic relaxation). Phase 1 is
a period of isometric relaxation (choice B), referred to as the period of isovolumic or
isovolumetric relaxation. The ventricle relaxes and the pressure falls during phase 1,
but the volume of the ventricle remains constant, thus muscle length is relatively
constant. Phase 4 begins when the mitral valve opens. This is a period of isotonic
relaxation (choice D) in which the relaxed ventricle fills with blood; it is called the
period of filling.
________________________________________

37The correct answer is B. Rheumatoid arthritis (RA) is a chronic, multisystem


inflammatory disorder of unknown etiology. The primary feature of RA is persistent
inflammatory synovitis, symmetrically involving the peripheral joints. The synovial
inflammation leads to cartilage destruction and bone erosions, which can cause
substantial joint deformities. In around 10-20% of patients, the onset of RA is acute,
and accompanied by constitutional symptoms, such as fever, lymphadenopathy,
splenomegaly, and weight loss. Subcutaneous nodules (Heberden's nodes),
pleuritis, pulmonary fibrosis, pericarditis, nerve entrapment syndromes, ocular
changes, and vasculitis may also be seen. Morning stiffness > 1 hr is one of the
distinguishing factors of inflammatory arthritis as compared to non-inflammatory
arthritis. The question is essentially asking for the most appropriate agent for the
alleviation of the patient's acute signs and symptoms. Celecoxib is a selective
cyclooxygenase-2 (COX-2) inhibitor with anti-inflammatory, analgesic, and
antipyretic effects. COX-1 is involved in the production of prostaglandins that
protect the GI lining, while COX-2 is important for the synthesis of prostaglandins
involved in inflammation and pain. Therefore, a selective COX-2 inhibitor can be
used for management of RA in a patient with a past history of ulcers.

Acetaminophen (choice A) has analgesic and antipyretic effects, but not antiinflammatory effects. Therefore, this agent would not be indicated for treatment of
a patient with RA.
Indomethacin (choice C) is a non-steroidal anti-inflammatory drug indicated for the
treatment of acute pain caused by inflammation. Although indomethacin can be
used in the treatment of RA, it would not be recommended in a patient with a past
medical history of ulcers.
Both methotrexate (choice D) and sulfasalazine (choice E) are used in the chronic
management of RA as part of disease-modifying antirheumatic drug (DMARD)
therapy; however, these agents take several weeks to months to elicit their
beneficial effects. Therefore, these agents should not be used in the acute
management of rheumatoid arthritis.
________________________________________
38The correct answer is B. Broca aphasia, caused in this case by an embolus to the
Broca area (inferior frontal gyrus), is often associated with hemiparesis. The aphasia
is characterized by slow, nonfluent speech with deficits in word finding. Because
comprehension is normal, patients are typically aware of the problem and appear
frustrated.
Apraxia (choice A) is a deficit of purposeful movement caused by a central lesion.
Dysarthria (choice C) is a deficit in speech articulation with normal grammar and
word finding.
Global aphasia (choice D) is usually caused by large infarcts in the distribution of
the middle cerebral artery. It is characterized by elements of Broca and Wernicke
aphasia (i.e., an inability to generate or comprehend fluent speech).
Wernicke aphasia (choice E), caused by lesions in Wernicke area, is characterized by
fluent speech that has a normal tempo, but is filled with incorrect words and
neologisms. The patient is unable to comprehend speech.

________________________________________
39The correct answer is B. The disease is systemic lupus erythematosus; the most
useful specific clue to the diagnosis is the presence of autoantibodies to doublestranded DNA (one type of antinuclear antibody or ANA). The characteristic
"butterfly" facial rash of lupus is due to deposition of antibodies and complement at
the dermal/epidermal junction.
Cells similar to fibroblasts growing in a storiform ("pinwheel") pattern (choice A) are
characteristic of dermatofibrosarcoma protuberans, a slow-growing type of
fibrosarcoma.
Horn cysts (choice C) are lamellated collections of keratin that occur in seborrheic
keratoses, a type of benign epithelial tumor of the skin.

Pautrier microabscesses (choice D) are a feature of mycosis fungoides, a cutaneous


T-cell lymphoma.
A sawtooth dermal/epidermal junction (choice E) is a feature of lichen planus, an
inflammatory skin condition.
________________________________________
40The correct answer is B. The colony forming unit-erythroid (CFU-E) is a
unipotential stem cell that develops from a burst forming unit-erythroid (BFU-E),
which develops eventually from the multipotential stem cell. The BFU-E is somewhat
responsive to erythropoietin, but the CFU-E is completely dependent on
erythropoietin. Erythropoietin is normally released from the kidney in response to
hypoxic or anemic conditions. Its half life is about 3-6 hours. Clinically it takes 5
days to see reticulocyte formation in the peripheral blood following erythropoietin
administration.
The basophilic erythroblast (choice A) differentiates from the proerythroblast. It is
recognizable by light microscopy and has a dark basophilic staining due to
hemoglobin synthesis. It is not directly effected by erythropoietin, but is instead
indirectly increased by the increase in precursor cells from the increase in CFU-E
earlier in development.
The multipotential stem cell (choice C) appears earlier in development than CFU-E
and does not increase with erythropoietin. The development of the major
components of blood (RBC, WBC, and platelets) all begin with the multipotential
stem cell (CFU-S). This cell is non-committed and can self-renew. It is located in the
bone marrow and is not recognizable by light microscopy.
The proerythroblast (choice D) is the first recognizable cell in the red cell lineage. It
develops from the CFU-E cell. It is not affected directly by erythropoietin, but
instead increases in number from the increased CFU-E cells.
The reticulocyte (choice E) is the enucleated cell just before the mature red blood
cell. Reticulocytes enter the peripheral circulation, but continue to synthesize
hemoglobin. This cell is not directly stimulated by erythropoietin, but increases in
number as a result of the increase in precursors.
________________________________________

41The correct answer is D. The patient's initial presentation strongly suggests Type
2 diabetes mellitus (NIDDM), which usually begins in middle or late life. Symptoms
often develop gradually, and the diagnosis is frequently made when an
asymptomatic or mildly symptomatic patient is found on routine laboratory
examination to have an elevated blood glucose level. Therapy with an oral
hypoglycemic agent would be appropriate in this case. Since the patient had a
documented anaphylactic reaction to trimethoprim-sulfamethoxazole, he should not
take any "sulfa" drugs, including the sulfonylurea type oral hypoglycemic agents
such as chlorpropamide (choice A) and glipizide (choice B). Metformin is a biguanide

oral hypoglycemic agent, chemically distinct from the sulfonylureas. This


medication is indicated as monotherapy or in conjunction with other oral
hypoglycemic agents in the treatment of NIDDM.
Glucagon (choice C) is the drug of choice for the treatment of severe hypoglycemia;
this agent would worsen the patient's hyperglycemia.
Propranolol (choice E) is a non-selective beta blocking agent used for the treatment
of hypertension and cardiac arrhythmias. Beta blockers are contraindicated since
they "blunt" the appearance of the premonitory signs and symptoms of
hypoglycemia.
________________________________________
42The correct answer is B. An increased number of eosinophils (AKA eosinophilia)
occurs in association with several conditions, the most frequent of which are
immune-mediated diseases (e.g., asthma, hay fever, and pemphigus vulgaris) and
parasitic infestations. This is due to an absolute increase in the number of
circulating eosinophils, brought about by IL-5, which stimulates differentiation of
eosinophilic precursor cells in the bone marrow.
Basophilic leukocytosis (choice A) is a rare event that is sometimes observed in
association with chronic myelogenous leukemia. It is not seen in patients with
asthma.
Lymphocytosis (choice C) may result from a vast array of conditions, but it is not
typical of asthma or other allergic diseases. Lymphocytosis may develop in response
to a number of infections (e.g., brucellosis, whooping cough, hepatitis, infectious
mononucleosis, and tuberculosis) or manifest as part of chronic lymphocytic
leukemia.
Monocytosis (choice D) refers to an increase in number of monocytes, which are
circulating macrophages. Chronic infections (e.g., tuberculosis, rickettsiosis, and
malaria) and chronic inflammatory conditions (e.g., collagen vascular diseases and
inflammatory bowel disease) are the most common underlying causes.
Neutrophilic leukocytosis (choice E) is a typical systemic reaction to acute and
chronic infections, especially those due to bacteria. The increase in neutrophil
number is mediated by IL-1 and TNF, which induce a rapid release of neutrophils
from the bone marrow in acute infections and stimulate proliferation of bone
marrow precursors in chronic infections.
________________________________________
43The correct answer is D. The infant has necrotizing enterocolitis, which can affect
infants any time in the first few months of life, but is most common around the time
an infant is started on oral foods. The functional immaturity of the neonatal gut
apparently leaves it particularly vulnerable to a combination of ischemic injury and
colonization by pathogenic organisms. Formula-fed infants appear to be more
vulnerable than breast-fed infants, possibly due to the absence of maternal
antibodies or other immunoprotective features in formula. Premature and low-birth-

weight infants are also more likely to develop this condition. Affected infants may
have either mild gastrointestinal illness or develop frank, life-threatening gangrene
of the terminal ileum and ascending colon.
Thickening of the pylorus (choice A) is a feature of congenital hypertrophic pyloric
stenosis, which typically presents with vomiting and regurgitation in an older infant.
A massively dilated colon (choice B) suggests Hirschsprung's disease, which may
present in a similar fashion to necrotizing enterocolitis. The two conditions can
usually be distinguished by noting that meconium is usually not passed prior to
feeding of the neonate in Hirschsprung's disease.
Bowel loops in the chest cavity (choice C) are a feature of congenital diaphragmatic
hernia, which causes respiratory distress and, often, death in neonates.
The gastrointestinal tract would be unlikely to appear normal (choice E) in a case
such as this.
________________________________________
44The correct answer is B. The patient described above has Gaucher disease, a
hereditary disease that affects bones and other organs. Glucocerebrosides
accumulate within macrophages in places such as bone marrow. The failure of
proper bone remodeling of the distal femur gives the characteristic "Erlenmeyer
flask" shape on x-ray. These patients suffer Gaucher crises from acute ischemia to
the bone, usually in the pelvis and femoral head. The pain is sudden, severe, and
progressive, lasting 2 or more weeks before fading. The crisis usually follows viral
illness. Other findings include osteosclerotic bone and corticomedullary
osteonecrosis.
Abnormal osteoclasts and mosaic lamellar bone (choice A) describe Paget disease of
bone. This is a disease of disordered bone remodeling that affects people older than
60. Although the etiology is unknown, it may be due to viral effect on osteoclasts.
The osteoclast activity is excessive, and increased absorption occurs. The
osteoclasts appear to have too many nuclei. The lamellar bone slowly acquires a
mosaic pattern because of irregular cement lines. Pain is usually due to fractures of
the misshapen bone.
Benign reactive bone around an osteoid nidus (choice C) describes osteoid osteoma.
This is a common lesion that is found in young people aged 5-25 years. It is usually
seen radiographically in the cortex of the diaphysis as a small, round lesion,
composed of a nidus of osteoid surrounded by woven bone. It is painful, usually at
night; the pain is relieved by aspirin. Surgery is curative.
Malignant sarcoma (choice D) would probably represent an osteosarcoma in this
age and location. Clinically, sarcomas do not present with acute pain, but rather are
associated with steadily progressive pain. Radiographs show a destructive tumor
with elevated periosteum and reactive new bone formation. Treatment includes
amputation.

Well-differentiated cartilage (choice E), if located in the marrow space and not just
caused by a misguided biopsy attempt, would indicate a solitary chondroma or
enchondroma. This is a benign tumor that is probably hamartomatous in nature. It is
asymptomatic and forms during development. It is sometimes found incidentally.
________________________________________

45The correct answer is D. The great majority of ectopic pregnancies (90%) occur in
the fallopian tubes. The other sites are the ovaries, abdominal cavity, and the
intrauterine segment of the fallopian tubes. Any condition that leads to anatomical
abnormalities of the uterus and fallopian tubes may predispose to ectopic
pregnancy. The most frequent of such conditions is pelvic inflammatory disease
(PID), which is usually associated with salpingitis. PID is a common infectious
condition most frequently caused by Neisseria gonorrhoeae and Chlamydia
trachomatis, both sexually transmitted. Other cases are due to a polymicrobial
population, including staphylococci, streptococci, coliform bacteria, and Clostridium
perfringens, acquired during abortion or delivery. PID leads to acute purulent
salpingitis. If this is untreated or inadequately treated, it progresses to salpingooophoritis, tubal abscesses, pyosalpinx, or hydrosalpinx. Even milder cases may
cause adhesions within the tube or between the tube and the ovary that interfere
with implantation of the ovum and result in tubal pregnancy. All the remaining
conditions listed above may also predispose to ectopic pregnancy.
Endometriosis (choice A) refers to the presence of endometrium in abnormal
locations, such as the ovary, uterine ligaments, rectovaginal pouch, and pelvic
peritoneum. It is an important clinical condition manifesting with pain,
dysmenorrhea, and infertility. Scarring at endometriotic sites may cause peritubal
adhesions and ectopic pregnancy.
An intrauterine device (choice B) may also increase the risk of ectopic pregnancy,
but the mechanism of action is not clear.
Leiomyomas of the uterus (choice C) are the most frequent benign tumor in women.
They develop from the smooth muscle as well-circumscribed nodules within the
uterine wall (intramural), in a subserosal or submucosal location. Leiomyomas may
cause significant distortion of the uterine wall and interfere with implantation,
increasing the risk of ectopic pregnancy.
Previous surgery (choice E) of any kind may create scars and peritubal adhesions,
thus predisposing to ectopic pregnancy.
________________________________________
46The correct answer is E. In fully compensated aortic coarctation, blood flow is
normal in the upper and lower body (choices A and B) despite an increased arterial
pressure (about 50% higher) in the upper body (choice C) compared to the pressure
in the lower body. Because resistance = pressure/blood flow, it is clear that
resistance must be lower in the lower portions of the body. The mechanism of this
decrease in resistance below the constriction (and increased resistance above the

constriction) is autoregulation of blood flow. The small arteries and arterioles dilate
(or constrict) in accordance with the metabolic needs of the tissues ensuring that
each tissue receive an adequate amount of blood flow. Thus, the increase in blood
pressure in the upper body leads to constriction of the arterioles, which increases
vascular resistance (choice D), and the lower pressure below the coarctation leads
to dilation of the arterioles, which decreases vascular resistance in the lower body.
________________________________________
47The correct answer is D. Tubular fluid first becomes hypotonic toward the end of
the thick ascending limb of the loop of Henle and will therefore be hypotonic by the
macula densa (which is the border between the thick ascending limb and the distal
convoluted tubule).
Tubular fluid is isotonic at the beginning of the proximal tubule (choice A).
Tubular fluid is isotonic by the end of the cortical collecting duct (choice B) in the
presence of antidiuretic hormone (ADH), since water is reabsorbed until the tubular
fluid osmolarity is the same as the peritubular fluid in the cortex (which has the
same osmolarity as plasma). A person with a urine flow rate of 1 mL/minute is
typically making hypertonic urine, and so has a significant amount of ADH present.
The urine is assumed to be hypertonic since osmolar clearance (Cosm) is usually 2
mL/minute, and urine osmolarity must be greater than plasma osmolarity if Cosm >
urine flow rate.
Tubular fluid at the end of the papillary collecting duct (choice C) will be hypertonic
in the presence of ADH. (See explanation of choice B for why ADH is present.)
Tubular fluid at the tip of the loop of Henle is always hypertonic; essentially no water
or solute is reabsorbed along the thin descending limb (choice E).
________________________________________
48The correct answer is B. The hematologic finding is erythrocytosis, which can be
caused by abnormal erythropoietin secretion by renal cell carcinoma (i.e., a
paraneoplastic syndrome). Absolute erythrocytosis also occurs in a number of other
conditions, such as hypoxia, other types of renal disease, some tumors (e.g.,
hepatocellular carcinoma, meningioma, pheochromocytoma, cerebellar
hemangioblastoma, adrenal adenoma), androgen therapy, Bartter's syndrome, or in
polycythemia vera.
Cancers of the colon (choice A), prostate (choice D), and thyroid (choice E) do not
usually produce inappropriate hormones.
Cancers of the ovary (choice C) can produce male or female sex steroids, but do not
produce erythropoietin.
________________________________________
49The correct answer is B.
________________________________________

50.Correct Answer is C.
Answers

________________________________________

1The correct answer is A. The lesion is a squamous cell carcinoma of the skin.
Actinic keratosis, which is a hyperplastic lesion of sun-damaged skin, predisposes
for squamous cell carcinoma. Another predisposing condition to remember is
xeroderma pigmentosum, which predisposes for both squamous cell and basal cell
carcinomas of skin.
A nevus is a mole, containing characteristic cells called nevocellular cells. If the
nevocellular cells are located at the dermal-epidermal junction (junctional nevus,
choice D), in the dermis (dermal nevus, choice C), or both (compound nevus, choice
B), they do not predispose for squamous cell carcinomas of the skin. Malignant
melanoma (choice E), however, can arise in pre-existing nevi.
________________________________________

2The correct answer is C. In renal failure, the ability of the kidney to secrete
phosphate is impaired. The resultant hyperphosphatemia causes hypocalcemia and
triggers excretion of large amounts of parathyroid hormone. The released
parathyroid hormone is a major contributor to bony changes (e.g., osteitis fibrosa, a
form of localized bone resorption) seen in some patients with chronic renal failure.
Aldosterone (choice A) is a regulator of serum sodium.
Calcitonin (choice B) levels are usually low in chronic renal failure, unless the
parathyroids have so hypertrophied as to cause "tertiary hyperparathyroidism" with
hypercalcemia.
Renin (choice D) is normally secreted by the kidney and may be either decreased or
increased in varying stages and forms of kidney disease. Regardless, renin regulates
blood pressure and aldosterone secretion, rather than bone metabolism.
The active form of Vitamin D (choice E), cholecalciferol, is formed in the kidney from
Vitamin D absorbed from the gut and then processed by the liver. Uremia interrupts
this pathway and consequently causes a functional Vitamin D
________________________________________

3The correct answer is C. Lysozyme is present in tears, saliva, mucus, vaginal


secretions, and several other body fluids. This material lyses the peptidoglycan
layer of the cell wall of bacteria without participation of any of the elements of the
oxidative killing pathway.
Hydrogen peroxide (choice A) is formed in the oxidative killing pathway from the
superoxide ions. This material is also very toxic to bacteria pathogens.
Hypochlorous acid (choice B) is formed from hydrogen peroxide and halide ions in
the presence of myeloperoxidase.
Myeloperoxidase (choice D) is an enzyme present in the oxidative killing pathway
that breaks down hydrogen peroxide.
Superoxide ions (choice E) are formed by NADPH oxidase in the first reaction of the
oxidative killing pathway. These superoxide ions are extremely toxic to bacterial
organisms.
________________________________________

4The correct answer is E. Secobarbital is a short-acting barbiturate with


considerable dependence potential. Withdrawal from short-acting barbiturates can
produce anxiety, delirium, and seizures which may be accompanied by lifethreatening cardiovascular collapse.
Insomnia (choice A) is a complication of barbiturate withdrawal, since barbiturates
are sedative/hypnotic agents, but this complication is not serious enough to be a
contraindication to abrupt cessation of the drug.
Rebound anxiety (choice B) would be quite likely following abrupt cessation of the
barbiturate, but would not constitute a sufficient danger to the patient to preclude
self-detoxification.
Recidivism (choice C) is quite likely in drug abusers, with or without medical
intervention.
Respiratory depression (choice D) is common with acute administration of
barbiturates, but would not be expected with barbiturate abstinence.
________________________________________

5The correct answer is A. The baby has meconium ileus, which is a manifestation of
cystic fibrosis due to the abnormally viscid pancreatic secretions which "get stuck"
in the small bowel. Meconium ileus can cause gut perforation with peritonitis and
intraperitoneal calcifications (that may be visible on plain film). Meconium ileus
complicated by intestinal perforation or formation of fistulas to the bladder or
vagina must be treated surgically. Medical treatments for uncomplicated meconium
ileus are now available, which use enemas, mucolytic agents, or pancreatic
enzymes.

Hirschsprung's disease (choice B) is a cause of congenital constipation related to


absence of ganglion cells in a segment of bowel. The aganglionic bowel segment is
narrowed because the lack of peristalsis keeps stool from moving into the segment.
The distal rectum is always involved, and the lesion may extend proximally as far as
the small intestine. The bowel proximal to the lesion is usually dilated.
A Meckel's diverticulum (choice C) can form due to the persistence of the vitelline
duct, which connects the developing gut to the yolk sac. They are classically located
in the distal ileum within 30 cm of the ileocecal valve, and may contain ectopic
pancreatic tissue or gastric mucosa.
________________________________________

6The correct answer is E. Misoprostol, a methyl analog of prostaglandin E1, is


approved for the prevention of ulcers caused by the administration of nonsteroidal
anti-inflammatory agents. Because this drug is a potential abortifacient, it should
not be given to pregnant women, or to women who are attempting to conceive.
Antinuclear antibodies (choice A) are associated with autoimmune diseases such as
systemic lupus erythematosus, scleroderma, Sjgren's syndrome, and inflammatory
myopathies. The test would be of no value in this case.
A barium swallow (choice B) is not indicated prior to the administration of
misoprostol.
Esophageal manometry (choice C) is used to evaluate the competency of the lower
esophageal sphincter, and to assess esophageal motor activity.
The osmotic fragility test (choice D) is performed by placing erythrocytes into a lowsalt solution. An increased susceptibility to osmotic lysis is found in hereditary
spherocytosis.
________________________________________

7The correct answer is B. Inversion of the heart, so that the morphologic left
ventricle is in the right chest, is called dextrocardia. Isolated dextrocardia is almost
always associated with cardiac defects that may include transposition of the atria
and transposition of the great arteries. However, dextrocardia as part of situs
inversus totalis, with reversal of the thoracic and abdominal organs, is usually
associated with a physiologically normal heart. The cluster of situs inversus,
sinusitis, and bronchiectasis is called Kartagener syndrome, which is caused by
defective ciliary function. Since migration of some embryonic cells during
development appears to be dependent on ciliary function, this is thought to explain
the situs inversus.
You should associate Down syndrome (choice A) with an ostium primum type of
atrial septal defect.

You should associate Kawasaki disease (choice C) with coronary artery aneurysms.
You should associate Marfan syndrome (choice D) with aortic dissection.
You should associate Turner syndrome (choice E) with coarctation of the aorta.
________________________________________

8The correct answer is C. The lesion is commonly called a Baker's cyst, and
anatomically represents an enlarged semimembranous bursa. Baker's cysts are
more commonly seen at the extremes of age.
The deep infrapatellar bursa (choice A) is on the anterior aspect below the knee.
The prepatellar bursa (choice B) is anterior to the patella.
The superficial infrapatellar bursa (choice D) is on the anterior aspect of the leg
below the knee.
The suprapatellar bursa (choice E) is on the anterior aspect of the thigh above the
knee.
________________________________________

9The correct answer is E. The esophagus is covered by non-keratinized, stratified,


squamous epithelium for its entire length. Heartburn is usually a sign of gastric
regurgitation of the acidic contents in the lower esophagus (acid reflux disease).
Acid reflux disease is a serious problem that may be associated with a precancerous
condition (Barrett's esophagus).
________________________________________

10The correct answer is C. This man has received repetitive trauma to his right arm
from operating a jack hammer for long periods of time. The median nerve is
affected, as evidenced by decreased sensation in its distribution (first three-and-ahalf). Thus, the muscles in the hand innervated by the median nerve will likely also
be affected, such as the thenar muscles (abductor pollicis brevis, opponens pollicis,
and flexor pollicis brevis) and the first two lumbricals.
All other intrinsic muscles of the hand are innervated by the ulnar nerve (abductor
digiti minimi, dorsal interossei, choices A and B). The wrist extensors (choice D) are
innervated by the radial nerve and the supinators (choice E) are innervated as
follows: biceps brachii-musculocutaneous nerve, supinator-radial nerve,
brachioradialis-radial nerve.
________________________________________

11The correct answer is C. Nephroblastoma (Wilms tumor) characteristically affects


children between 2 and 4 years of age and can form large spherical masses
composed of variegated tissues including primitive renal epithelial elements, a
sarcomatous-appearing stroma, abortive glomeruli and tubules, and heterologous
tissues such as muscle, cartilage, fat, and bone.
Cortical adenoma (choice A) is a small (under 2 cm) benign tumor that is usually
found incidentally at autopsy.
Hemangiomas (choice B) can occur in the kidney, but would not usually produce a
large mass.
Oncocytomas (choice D) are benign tumors that can be large but do not usually
affect young children.
Renal cell carcinoma (choice E) is usually a disease of older adults.
________________________________________

12The correct answer is E. The facial nerve (CN VII) is intimately related
anatomically to the parotid gland, but controls salivation from both the
submandibular and sublingual glands.
The parotid gland (choices A, B, and C) is innervated by the glossopharyngeal nerve
(CN IX).
________________________________________

13The correct answer is A. Burkitt's lymphoma is actually associated with three


translocations. The common variant t(8;14)(q24;q32), involving the oncogene myc
on chromosome 8, and the heavy immunoglobulin chain on chromosome 14. The
other two variants are: t(8;22)(q24;q11), involving myc and the lambda light chain
immunoglobulin site, and t(2;8)(p12;q24), involving the kappa light chain and myc.
Mantle cell lymphoma (choice B), multiple myeloma (choice C), and small (not
cleaved) cell lymphoma (choice D) are associated with the t(11;14)(q13;q32)
translocation involving bcl -1 and the heavy chain site.
Small cleaved cell lymphoma (choice E) is associated with t(14;18)(q 32;q21),
involving the immunoglobulin chain site and bcl-2.
________________________________________

14The correct answer is C. The defecation reflex that is evoked when the rectum is
distended involves three responses: 1) the internal anal sphincter relaxes, 2) the
external anal sphincter contracts, and 3) a conscious urge to defecate is perceived.
This neural reflex involves the pelvic nerve, which provides the parasympathetic
preganglionic innervation to the internal anal sphincter (composed of smooth

muscle) and also carries the sensory afferent information from the rectum to the
spinal cord, and the pudendal nerve, which carries the somatic efferent inputs to
the external anal sphincter (composed of skeletal muscle). In the patient described
above, the pelvic nerve is most likely damaged since neither the reflex relaxation of
the internal anal sphincter nor the urge to defecate is evoked by rectal distention.
Damage to the internal anal sphincter (choice A) would most likely cause resting
tone to be low and, if anything, lead to fecal incontinence rather than constipation.
Furthermore, damage to the internal anal sphincter could not explain the failure of
the appearance of the urge to defecate.
The external anal sphincter (choice B) appears to function normally since distention
of the rectum evokes the expected contraction. The normal contraction of the
external anal sphincter also suggests that the pudendal nerve (choice D) is intact.
Since distention of the rectum (choice E) evoked the contraction of the external anal
sphincter, it too appears to be functioning normally.
________________________________________

15The correct answer is A. The spots described are called Roth's spots, and
specifically suggest bacterial endocarditis. They represent small fibrin clots that
form on the vegetations and then break off and lodge in small blood vessels,
including those of the retina. Splinter hemorrhages, petechiae, and small, painful
nodules on the finger and toe pads (Osler's nodes) may also be seen. Physical
examination frequently reveals a heart murmur.
Crohn's disease (choice B) can occasionally be associated with uveitis, but not
retinal hemorrhages.
Diabetes mellitus (choice C) can cause a variety of ocular changes including
microaneurysms, flame hemorrhages, and "cotton wool" deposits, but Roth's spots
are not produced.
Glaucoma (choice D) causes increased pressure in the anterior chamber, but does
not produce Roth's spots.
Urosepsis (choice E) does not usually cause any obvious eye findings.
________________________________________

16The correct answer is E. The patient is suffering from mitral regurgitation caused
by rupture of the chordae tendineae. Mitral regurgitation is characterized by a
greatly elevated left atrial pressure toward the end of systole caused by backward
flow of blood from the left ventricle into the left atrium through the leaky mitral
valve. The left atrial pressure is normal at the end of diastole with mitral
regurgitation because blood flows unimpeded from the atrium into the ventricle
when the mitral valve is open. The leak occurs during systole, and is characterized

by a systolic murmur, which was noted in the patient. The chordae tendineae are
tendinous cords that connect each cusp of the mitral valve to the papillary muscles
in the left ventricle. When these rupture, the cusps of the mitral valve cannot be
approximated during systole, which results in backward flow of blood into the left
atrium. The chordae tendineae are often damaged in rheumatic valvular disease,
which increases the probability of rupture.
The aortic pressure and left ventricular pressure tracings are nearly superimposed
during systole in the diagram, which eliminates the possibility of aortic obstruction
(choice A) and aortic regurgitation (choice B).
The rate of rise of left ventricular pressure appears to be normal, which argues
against the possibility of left ventricular infarction (choice C).
In mitral obstruction (choice D), the left atrial pressure is greater than the left
ventricular pressure toward the end of diastole when blood is flowing from the left
atrium into the left ventricle.
________________________________________

17The correct answer is E. This is the classic presentation of Zenker's diverticulum,


which is a false diverticulum formed by herniation of the mucosa at a point of
weakness at the junction of the pharynx and esophagus in the posterior
hypopharyngeal wall. Zenker's diverticulum is also associated with halitosis, and if
the diverticulum fills completely with food, it can cause dysphagia or obstruction of
the esophagus.
Mallory-Weiss tears (choice A) are mucosal tears at the gastroesophageal junction
secondary to repeated, forceful vomiting. They are often seen in alcoholics.
Plummer-Vinson syndrome (choice B) is the triad of dysphagia (due to esophageal
webs in the upper esophagus), atrophic glossitis, and iron-deficiency anemia.
Schatzki rings (choice C) are mucosal rings found in the distal esophagus at the
squamocolumnar junction.
In contrast to a Zenker's diverticulum, the usually asymptomatic traction diverticula
(choice D) are true diverticula involving all of the layers of the esophagus. They are
typically caused by adherence of the esophagus to a scarred mediastinal structure.
________________________________________

18The correct answer is B. In obstructive lung disease such as emphysema, the


MEFV curve begins and ends at abnormally high lung volumes, and the flow rates
are lower than normal at any given lung volume. Note on the diagram that the
patient's total lung capacity is 7 liters and the residual volume is 3.5 liters (dashed
curve). The curve may also have a scooped out appearance as shown on the
diagram. Note that absolute lung volumes cannot be determined from a MEFV curve

alone. An additional method is used to measure residual volume, then the curves
can be placed appropriately on the abscissa.
Lung volumes are expected to be lower than normal in asbestosis (choice A), pleural
effusion (choice C), pneumothorax (choice D), silicosis (choice E), and other types of
restrictive lung diseases.
________________________________________

19The correct answer is B. In the formation of the upper lip, the right and left medial
nasal prominences merge to form the philtrum of the upper lip. The lateral maxillary
prominence then merges with the merged medial nasal prominences. Failure of this
merger to occur results in a unilateral cleft lip.
Failure of a lateral palatine process to fuse with the median palatine process (choice
A), which is synonymous with a failure of the primary palate to fuse with the
secondary palate (choice C), results in a primary cleft palate. Primary clefts of the
palate are found anterior to the incisive foramen of the palate. Primary cleft palates
may occur along with cleft lips but are the result of a separate developmental
defect.
Failure of the right and left medial nasal prominences to merge (choice D) results in
a median cleft lip. This is a rare anomaly. Normally, the right and left medial nasal
prominences merge into a single prominence which forms the philtrum of the lip.
Failure of the right and left palatine processes to fuse (choice E) results in a
secondary cleft palate. Secondary cleft palates are found posterior to the incisive
foramen. Normally, the right and left palatine processes fuse together and fuse to
the primary palate.
________________________________________

20The correct answer is B. Disseminated intravascular coagulation (DIC) is


characterized by consumption of both platelets and clotting factors. The best tests
to order are platelet count (which will be markedly decreased), serum fibrinogen
level (which will be low), and fibrin degradation products (which will be high).
Prothrombin time (PT) measures factors I (fibrinogen), II, V, VIII, and X. Partial
thromboplastin time (PTT) measures VII, prekallikrein, high-molecular weight
kininogen, and factors I, II, V, VII, IX, X, and XI. Both PT and PTT are relatively nonspecific. Thrombin time (TT) is a more specific measure of fibrinogen and would
potentially be a useful test in this setting. However, specific measurement of factor
VIII, kininogen, or prekallikrein levels would not be rational in evaluating DIC.
________________________________________

21The correct answer is A. This is Addison disease, in which severe adrenal disease
produces adrenocortical insufficiency. Causes include autoimmune destruction,
congenital adrenal hyperplasia, hemorrhagic necrosis, and replacement of the
glands by either tumor (usually metastatic) or granulomatous disease (usually
tuberculosis). The symptoms can be subtle and nonspecific (such as those
illustrated), so a high clinical index of suspicion is warranted. Skin
hyperpigmentation is a specific clue that may be present on physical examination,
suggesting excess pituitary ACTH secretion. (The ACTH precursor has an amino acid
sequence similar to MSH, melanocyte stimulating hormone.) Most patients have
symptoms (fatigue, gastrointestinal distress) related principally to glucocorticoid
deficiency. In some cases, however, mineralocorticoid replacement may also be
needed for symptoms of salt wasting with lower circulating volume.
Except in the case of primary pancreatic cancer, complete tumor replacement of the
endocrine pancreas (choice B) would be uncommon. In any event, pancreatic
involvement would be associated with diabetes mellitus.
Involvement of the ovaries (choice C) by metastatic tumor (classically gastric
adenocarcinoma) would produce failure of menstruation.
Involvement of the pituitary gland (choice D) could produce Addisonian symptoms,
but the pigmented skin suggests a primary adrenal problem rather than pituitary
involvement.
Replacement of the thyroid gland (choice E) by tumor would cause hypothyroidism
with lesser degrees of fatigue and cold intolerance; this is an unusual cause of
hypothyroidism and is less likely to be tested than Addison disease caused by
bilateral adrenal metastasis.
________________________________________

22The correct answer is B. When patients present with either a muscle spasm or a
"strained" muscle, a centrally acting skeletal muscle relaxant is typically prescribed.
In addition to these agents, a nonsteroidal anti-inflammatory drug is often added for
additional pain control. Although somnolence can be seen with any of the agents
listed, xerostomia (dry mouth), mydriasis and tachycardia are classic anticholinergic
side effects. Cyclobenzaprine is a centrally acting skeletal muscle relaxant that is
structurally related to tricyclic antidepressants, which are known for their strong
anticholinergic side effects. Additional common anticholinergic side effects seen
with this agent include blurred vision, urinary retention, and constipation. Less
common side effects include agitation, respiratory depression, disorientation,
tachycardia, and widening of the QRS complex.
Baclofen (choice A) is a centrally acting skeletal muscle relaxant that produces
muscle relaxation by inhibition of both monosynaptic and polysynaptic reflexes at
the spinal level. This agent is indicated for the treatment of spasticity resulting from
multiple sclerosis or secondary to spinal cord injuries; it has also been used in the

treatment of trigeminal neuralgia. The most common side effects include transient
drowsiness, fatigue, and hypotension.
Diclofenac (choice C) is a non-steroidal anti-inflammatory drug indicated for the
treatment of a variety of disorders associated with pain and inflammation. The most
common side effects include dyspepsia, nausea, vomiting, abdominal cramps, and
dizziness.
Methocarbamol (choice D) produces muscle relaxation by general CNS depression; it
does not have a direct action on the contractile mechanism of striated muscle or
nerve fibers. This agent is indicated as an adjunct to rest, physical therapy, and
other measures for relief of discomfort in various musculoskeletal conditions.
Reported side-effects include dizziness, vertigo, ataxia, headache, irritability,
bradycardia, hypotension, and syncope.
Rofecoxib (choice E) is a selective cyclooxygenase-2 (COX-2) inhibitor with antiinflammatory, analgesic, and antipyretic effects. This agent is used in adults for
relief of pain and inflammation caused by osteoarthritis and rheumatoid arthritis, as
well other inflammatory conditions. The most common side effects are nausea,
vomiting, diarrhea, abdominal distress, flatulence, and anorexia.
________________________________________

23The correct answer is A. The patient has Dressler's syndrome, which is thought to
be an autoimmune phenomenon resulting in fibrinous pericarditis with fever and
pleuropericardial chest pain several weeks after a myocardial infarction.
Bacterial (choice B), fungal (choice D), and viral (choice E) infections can cause
pericarditis, but would not be suspected in the setting of myocardial infarction.
Chlamydia (choice C) are not a usual cause of pericarditis.
________________________________________

24The correct answer is B. The lesion described is the form of in situ penile
carcinoma known as Bowen's disease. Bowen's disease is thought to carry a 10%
risk of progressing to invasive squamous cell carcinoma, if left untreated.
Bowenoid papulosis (choice A) is a form of in situ penile carcinoma that is
characterized clinically by multiple, reddish-brown, papular lesions.
Condyloma acuminatum (choice C) is a benign lesion that resembles the common
wart. Condylomata are associated with papillomavirus infection.
Erythroplasia of Queyrat (choice D) is a form of in situ penile carcinoma that
produces a soft, red plaque.
Giant condyloma (choice E) is an uncommon variant of condyloma acuminatum that
is locally aggressive, but does not usually metastasize.

________________________________________

25The correct answer is C. Acute epididymitis and orchitis with prominent


neutrophils in a sexually active male are most likely due to infection with Neisseria
gonorrhoeae or Chlamydia trachomatis. N. gonorrhoeae can produce a nonspecific
pattern of acute inflammation (nonspecific epididymitis and orchitis) or can be
sufficiently severe as to cause frank abscesses within the epididymis.
Escherichia coli (choice A) is an important cause of nonspecific epididymitis and
orchitis in children with congenital genitourinary abnormalities and in older men.
Mycobacterium tuberculosis (choice B) can cause tuberculosis of the epididymis and
testes, characterized by granuloma formation.
Pseudomonas sp.(choice D) has been implicated as an important cause of
nonspecific epididymitis and orchitis in older men.
Treponema pallidum (choice E), the causative agent of syphilis, can cause testicular
involvement with gumma formation, endarteritis, and/or a prominent plasma cell
infiltrate.
________________________________________

26The correct answer is C. The cardiac output (CO) is equal to the volume of blood
ejected from the heart during each systole (i.e., the stroke volume; SV) multiplied
by the number of times the heart beats each minute (heart rate; HR). In other
words, CO = SV x HR. Therefore, SV = CO/HR, and since CO = 5000 mL/min, and HR
= 50/min, SV = 5000/50 = 100 mL.
________________________________________

27The correct answer is A. Even if you do not remember the features of the
individual lipid storage diseases, it is worth remembering that Fabry's disease
(described above) is the only one in the group with X-linked genetics. As a general
rule, you should associate all of the lipid storage diseases with severe neurologic
deterioration. The exception is Gaucher's disease (choice B), which tends to affect
the liver and spares the brain.
Gaucher's disease (choice B), Niemann-Pick disease (choice E), and Type I Tay-Sachs
disease are not X-linked; all occur with greater frequency in Ashkenazi Jews.
You should associate Krabbe's disease (choice C) with the presence of "globoid"
cells in degenerating white matter in the brain.
Metachromatic leukodystrophy (choice D) is caused by a deficiency of the enzyme
arylsulfatase A.
________________________________________

28The correct answer is B. This is Bowen's disease, which is one clinical form of
penile carcinoma in situ (the other forms are Bowenoid papulosis and erythroplasia
of Queyrat). Untreated Bowen's disease will, over a period of years, progress to
frank invasive carcinoma in 10% to 20% of patients. Bowen's disease can affect the
genital region of both men and women.
Bowenoid papulosis (choice A) is another form of penile carcinoma in situ,
characterized clinically by multiple reddish-brown papular lesions.
Condyloma acuminatum (choice C) usually produces a papillary lesion.
Erythroplasia of Queyrat (choice D) is another form of penile carcinoma in situ,
characterized by single or multiple shiny red plaques.
Squamous cell carcinoma (choice E) will be frankly invasive on biopsy.
________________________________________

29The correct answer is C. In young women, the most likely causes of cervicitis and
pelvic inflammatory disease (PID) are Neisseria gonorrhoeae and Chlamydia
trachomatis. Gram's staining alone may not be able to distinguish between the two
in women, so culture is warranted. Both organisms induce endocytosis by epithelial
cells.
Neisseria gonorrhoeae is a gram-negative diplococcus. It is endocytosed by mucussecreting epithelia, and is exocytosed into subepithelial tissues leading to
necrotizing acute inflammation and destruction of ciliated and non-ciliated cells.
Through direct extension, it can lead to PID and infertility in women. It can be
detected by visualizing the organism in polymorphonuclear leukocytes (PMNs) in
Gram's stained clinical material more easily in men than in women. Therefore,
culture onto Thayer-Martin agar is usually required for diagnosis in women. Since
the organism can be cultured, it is not an obligate intracellular parasite (compare
with choice D). The organism can be killed by PMNs, antibody, and complement,
however, many of its outer membrane proteins undergo antigenic or phase
variation or cause the production of blocking antibody that interferes with
bactericidal activity. Immunity to the organism does not develop (compare with
choices A and B). Neisseria gonorrhoeae is resistant to penicillin, a b-lactam
antibiotic (compare to choice E), because of a plasmid-mediated penicillinase and a
chromosomally-mediated mutation that decreases the permeability of the outer
membrane and decreases the affinity of penicillin binding proteins. It can be treated
with ceftriaxone, a cephalosporin, which resists the penicillinase, or with quinolones
and azithromycin.
Chlamydia trachomatis is an obligate intracellular parasite (choice D) that induces
endocytosis and resides in the phagosome of infected epithelial cells. It possesses a
gram-negative envelope but lacks peptidoglycan, and hence is intrinsically resistant
to all antibiotics that inhibit peptidoglycan synthesis, such as the b-lactam

antibiotics (compare with choice E). Antibodies are ineffective at controlling


infection. Cell mediated immunity (choice A) is probably the major means of
controlling this infection since a deficiency in cell mediated immunity both increases
susceptibility and severity in animal models. Chronic and clinically latent infections
cause serious morbidity, including pelvic inflammatory disease, fallopian tube
dysfunction and blindness. Chlamydia infection can be treated with macrolides,
quinolones and tetracyclines.
________________________________________

30The correct answer is C. Methotrexate is a type of disease-modifying


antirheumatic drug (DMARD). DMARDs are a varied group of drugs, including
methotrexate, azathioprine, penicillamine, hydroxychloroquine and chloroquine,
organic gold compounds, and sulfasalazine, which are thought to slow the
progression of rheumatoid arthritis by modifying the disease itself. However, these
drugs can take several weeks to several months to produce therapeutic effects. In
rheumatoid arthritis, prednisone is used when persistent synovitis is seen in
multiple joints despite sufficient dosage of NSAIDs. It is also used for "bridge"
therapy when DMARD therapy with methotrexate is first initiated. Because it takes a
long time for the therapeutic effect of DMARDs to become evident, agents like
prednisone are needed to "bridge the gap" between NSAID therapy and DMARD
therapy.
The major disadvantage of using glucocorticoids for an extended period of time is
the severe side effect profile. For example, long term use of prednisone is
associated with hypocalcemia (choice C), fluid retention (not dehydration, choice A),
hypokalemia (not hyperkalemia, choice B), hyperglycemia (not hypoglycemia,
choice D), and hypernatremia (not hyponatremia, choice E). Other adverse reactions
include adrenal suppression, muscle weakness and atrophy, gastritis, nausea,
vomiting, Cushingoid state (moon face, buffalo hump, central obesity),
immunosuppression, hypertension, psychosis, osteoporosis, glaucoma, and
posterior subcapsular cataracts.
________________________________________

31The correct answer is A. The brachial artery is immediately medial to the tendon
of the biceps brachii at the elbow. As the artery enters the forearm, it is covered by
the bicipital aponeurosis, a broadening of the biceps tendon.
The musculocutaneous nerve (choice B) does not cross the elbow. The
musculocutaneous nerve gives off all of its muscular branches to muscles in the
arm. The remainder of the nerve is then renamed the lateral cutaneous nerve of the
forearm, which passes the elbow lateral to the tendon of the biceps.
The profunda brachii artery (choice C) arises from the brachial artery in the proximal
part of the arm. It accompanies the radial artery in the musculospiral groove and

then divides into the radial collateral artery and middle collateral artery, which cross
the elbow lateral to the tendon of the biceps.
The radial nerve (choice D) lies within the musculospiral groove along the back of
the humerus, then passes between the brachioradialis muscle and the brachialis
muscle at the elbow, lateral to the tendon of the biceps.
The ulnar nerve (choice E) crosses the elbow posterior to the medial epicondyle of
the humerus. It then passes between the two heads of the flexor carpi ulnaris and
courses through the forearm deep to this muscle.
________________________________________

32The correct answer is C. A thiazide diuretic would be the drug of choice for this
patient because it is the only class of diuretic which decreases urinary secretion of
calcium. Thiazide diuretics, like hydrochlorothiazide, inhibit the Na+/Clcotransporter in the distal convoluted tubule and promote the reabsorption of
calcium.
Acetazolamide (choice A), a carbonic anhydrase inhibitor, and furosemide (choice
B), a loop diuretic, both induce diuresis at the expense of all three major cationic
electrolytes (Na+, K+, and Ca2+) which are secreted in increased amounts.
Spironolactone (choice D) and triamterene (choice E), so-called potassium sparing
diuretics, block Na+/K+ exchange in the collecting duct. Although they decrease K+
secretion, Na+ and Ca2+ secretion are elevated.
________________________________________

33The correct answer is D. The patient's initial hyperlucent lung fields strongly
suggest the presence of emphysema. The radiologic findings after the increase in
shortness of breath are consistent with free air in the chest, which has collapsed the
left lung and caused a shift in the location of the mediastinum. Such air might have
been introduced by rupture of a bulla. Small pneumothoraces are usually well
tolerated, but larger ones may require decompression (the needle from a syringe is
sometimes used), or even surgical correction if bullae continue to leak air.
Bronchogenic carcinoma (choice A) would be expected to produce a mass lesion.
Pleural effusion (choice B) usually develops slowly, and causes a whitening of lung
fields when fluid is present.
Pulmonary embolism (choice C) can cause a sudden shortness of breath, but would
not cause an increase in the lucency of the lung fields.
Tuberculosis (choice E) would be expected to produce a mass lesion in the lung
________________________________________

34The correct answer is D. Therapy of diabetic ketoacidosis requires more than


insulin. Intravascular volume is often depleted, and initial fluids to restore volume
should include isotonic saline or lactated Ringer's solution. If arterial blood pH is less
than 7.1 or if severe hyperkalemia is present, bicarbonate supplementation should
be used. IV fluids containing 5-10% dextrose (glucose) should be used when the
serum glucose levels fall to 200-300 mg/dL, since high doses of rapidly acting
insulin can cause life-threatening hypoglycemia. Additionally, the serum potassium
concentration should be watched very carefully, since potassium is cotransported
into cells with glucose in the presence of insulin. It is often the case that
hyperkalemia is present initially, secondary to decreased cellular uptake of
potassium with decreased cellular uptake of glucose. However, this can rapidly
change when insulin drives glucose (with potassium) into cells, and a lifethreatening hypokalemia can develop.
Supplementation with calcium (choice A) is not required with insulin administration.
Creatinine (choice B) is a waste product, rather than a nutrient.
Supplementation with magnesium (choice C) is not required with insulin
administration.
Sodium supplementation (choice E) is not required during therapy with insulin.
________________________________________

35The correct answer is D. The amount of glycosylated hemoglobin (HbA1c) is


directly related to the level of glucose in the blood. Since HbA1c is a stable product,
its concentration reflects glucose levels over the past 3-6 months. HbA1c forms as a
result of nonenzymatic glycosylation, a fundamental biochemical abnormality that
accounts for most of the histopathologic alterations in diabetes mellitus. At first,
glucose forms reversible glycosylation products with proteins by formation of Schiff
bases. Rearrangement of Schiff bases leads to more stable, but still reversible,
Amadori products and subsequently to irreversible advanced glycosylation end
products (AGE), of which HbA1c is an example. Blood ketones, blood glucose,
urinary glucose, and blood insulin do not reflect long-standing metabolic
abnormalities of diabetes mellitus and cannot be used to assess long-term glycemic
control.
Blood glucose (choice A) is elevated in both type 1 and type 2 diabetes mellitus.
Hyperglycemia is the diagnostic feature of diabetes mellitus and leads to glycosuria
(choice E) when blood glucose exceeds 160-180 mg/dL.
Blood insulin (choice B) is decreased in untreated type 1 diabetes and normal or
even slightly increased in type 2 diabetes.
Blood ketones (choice C) (acetoacetic acid and b-hydroxybutyric acid) are
synthesized from free fatty acids in response to severe insulin deficiency.
Accumulation of ketone bodies in the blood is a crucial pathogenetic factor in
ketoacidosis, which occurs exclusively in type 1 diabetics.

________________________________________

36The correct answer is D. Haptoglobin (or alpha-2 globulin) is a serum protein that
promptly binds to free hemoglobin in the serum, forming a tight complex. The
haptoglobin-hemoglobin complex is quickly cleared from the blood by the
reticuloendothelial system, which allows the hemoglobin to be salvaged instead of
lost into the urine. Intravascular hemolysis from any cause is rapidly followed by a
dramatic drop in serum haptoglobin.
Bilirubin (choice A) is synthesized in the liver and spleen as an end-product of heme
degradation. Serum bilirubin levels would not change until the haptoglobin in the
blood was completely saturated, and free hemoglobin began to circulate. In this
circumstance, serum bilirubin would increase, not decrease.
C-reactive protein (choice B) is an acute phase reactant produced by the liver in
increased quantity in response to inflammation. This patient's urinary tract infection
would increase C-reactive protein levels, but the hemolysis would not further affect
them.
Serum free iron (choice C) does not change appreciably in intravascular hemolysis.
Ferrous iron in the red blood cell is complexed to hemoglobin, and this complex is
not broken in hemolysis.
Methemoglobin (an oxidized form of hemoglobin; choice E) increases in the blood
along with free hemoglobin in cases of intravascular hemolysis.
________________________________________

37The correct answer is C. This is one of those questions where predicting the
correct answer will facilitate your selecting the appropriate answer choice. Make
sure to read each choice carefully (for example, choices D and E, though both
incorrect in this case, look almost identical and could cost you points if you were not
reading actively).
Choice A refers to the crude mortality rate.
Choice B refers to the postnatal mortality rate.
Choice D is an incorrect rendering of the neonatal mortality rate since it considers
the total number of births instead of the total number of live births.
Choice E refers to the neonatal mortality rate.
________________________________________

38The correct answer is C. This infant suffers from Potter syndrome, caused by an
absence of both kidneys. This is often caused by a failure of the ureteric buds to
develop. The ureteric buds form the ureters, renal pelvis, calyces, and collecting

tubules. The collecting tubules induce the formation of metanephric vesicles, which
differentiate into the tubular components of the nephron.
The paramesonephric ducts (choice A) form the uterine tubes, uterus, and the upper
third of the vagina.
The pronephros (choice B) is a transient collection of cells that disappears during
the fourth week of development. It does not form the permanent kidney.
The urogenital sinus (choice D) forms the urinary bladder (in both sexes). It also
forms the prostatic urethra, prostate, membranous urethra, bulbourethral glands,
penile urethra (proximal part) and Littre's glands in the male. In the female it forms
most of the urethra, urethral glands, paraurethral glands, vestibule of the vagina
and greater vestibular glands.
The urorectal septum (choice E) partitions the cloaca into the urogenital sinus and
rectum (and upper anal canal).
________________________________________

39The correct answer is C. This is a classic description of Fe deficiency anemia (up


to 20% of adult women are Fe deficient!). Fe deficiency anemia is very common in
menstruating women, and is associated with the signs and symptoms described
(fatigue, weakness, pallor). It can also be associated with epithelial changes such as
brittle nails and atrophic tongue. The typical laboratory values are those described
along with a decreased reticulocyte count. On peripheral blood smear, the RBCs are
small (microcytic) and pale in color (hypochromic, due to low Hb levels from
inadequate Fe stores).
Vitamin B12/folate deficiency would give you macrocytosis with hypersegmented
neutrophils (choice A).
Lead poisoning would give you microcytosis with basophilic stippling (choice B).
Schistocytes (choice D) are RBC fragments seen when the cells are destroyed by
shearing forces in the vascular system, such as those present in prosthetic heart
valves and microangiopathic hemolytic anemias.
________________________________________

40The correct answer is C. Salmonella spp., including S. enteritidis and S.


typhimurium, produce a gastroenteritis or enterocolitis. Patients with decreased
gastric acidity, sickle cell disease, defects in immunity, or children younger than 4
years have a more severe course of disease. Salmonella spp. are carried in nature
by animal reservoirs such as poultry, turtles, cattle, pigs, and sheep. The incubation
period is 8-48 hours after ingestion of contaminated food or water.
Chlamydia psittaci (choice A) produces an interstitial pneumonitis accompanied by
headache, backache, and a dry, hacking cough. A pale, macular rash is also found

on the trunk (Horder's spots). Patients at risk include pet shop workers, pigeon
handlers, and poultry workers.
Entamoeba histolytica (choice B) produces a diarrhea (frequently bloody or hemepositive), right lower quadrant crampy abdominal pain, and fever. Patients
frequently have weight loss and anorexia. There is usually a history of travel outside
the U.S. Most cases are chronic. Complications include liver abscesses.
Staphylococcus aureus (choice D) produces a self-limited gastroenteritis due to the
production of preformed, heat-stable enterotoxins. The incubation period is 16
hours. The toxins enhance intestinal peristalsis and induce vomiting by a direct
effect on the CNS.
Yersinia enterocolitica (choice E) usually produces a chronic enteritis in children.
These patients have diarrhea, failure to thrive, hypoalbuminemia, and hypokalemia.
Other findings include acute right lower quadrant abdominal pain, tenderness,
nausea, and vomiting. The infection mimics appendicitis or Crohn's disease.
________________________________________

41The correct answer is B. Meningocele occurs due to a failure of closure of the


caudal portion of the neural tube. In contrast to the more serious
meningomyelocele, in which the spinal cord also herniates through the vertebral
defect, meningocele is typically associated with only minimal neurological damage
and is relatively easy to treat surgically.
Encephalocele (choice A) implies herniation of the brain through a bony defect in
the skull caused by defective closure of the cranial end of the neural tube.
Meningomyelocele (choice C) includes herniation of the spinal cord through a
vertebral defect. Associated neurological deficits are usually severe.
In spina bifida aperta (choice D), there is a complete failure of fusion of the neural
plate associated with malformation of the vertebral arches such that the
underdeveloped neural plate is covered only with skin. No mass lesion is present,
and the patient typically has major neurologic deficits.
Spina bifida occulta (choice E) is the mildest of the neural tube defects, typically
consisting of a small vertebral defect with a normal spinal cord and meninges. The
defect is often associated with a skin dimple that overlies adipose tissue, hair
follicles, or a sinus.
________________________________________

42The correct answer is B. When you hyperventilate, CO2 is blown off. The amount
of CO2 blown off is inversely proportional to alveolar ventilation. This is shown by
the alveolar ventilation equation:
VA = VCO2/PACO2, where

VA = alveolar ventilation
VCO2 = CO2 production
PACO2 = alveolar PCO2
So, if VCO2 remains the same, and VA quadruples, PACO2 must decrease by 4 fold;
40 mm Hg decreases to 10 mm Hg.
________________________________________

43The correct answer is D. The lesion is a mature teratoma. Teratomas located in


the ovary and containing a hair and keratin filled cyst are sometimes called dermoid
cysts. Teratomas contain cells of a variety of types, often including skin, skin
adnexal structures (hair follicles, sweat glands, sebaceous glands), connective
tissues, neural tissue, muscle, and thyroid tissue. If immature tissues such as
primitive neuroepithelial cells or developing skeletal muscle cells are seen, the
lesion is considered potentially malignant and classified as an immature teratoma
(choice A).
Leiomyomas (choice B) are benign tumors of smooth muscle (e.g., uterine
"fibroids"), usually in the female genital tract.
Leiomyosarcomas (choice C) are rare malignant tumors of smooth muscle, usually in
the female genital tract.
Rhabdomyosarcomas (choice E) are malignant skeletal muscle tumors with a
predilection for the head and neck and urogenital regions in children.
________________________________________

44The correct answer is A. The key to this question is understanding that the
effectiveness of each diuretic is dependent on the creatinine clearance (CrCl). For
example, the "loop" diuretics are effective down to a CrCl of around 10 mL/min.
However, the thiazide and potassium-sparing diuretics are effective only down to a
CrCl of around 40 mL/min. Therefore, on the basis of this patient's CrCl of 17
mL/min, only a loop diuretic would be recommended. Furosemide is a loop diuretic
that is indicated for the treatment of edema associated with congestive heart
failure, hepatic cirrhosis, and renal disease.
Hydrochlorothiazide (choice B) and indapamide (choice C) are thiazide diuretics and
therefore would not be recommended for patients with such a low CrCl.
Spironolactone (choice D) and triamterene (choice E) are potassium-sparing
diuretics and therefore would not be recommended for a patient with a CrCl of 17
mL/min.
________________________________________

45The correct answer is C. The juxtaglomerular cells are in the wall of the afferent
arteriole, close to the glomerulus. In response to decreased blood pressure, they
secrete renin, an enzyme that converts angiotensinogen to angiotensin I.
Angiotensin converting enzyme, found in the lungs, converts angiotensin I to
angiotensin II. Angiotensin II increases peripheral vascular resistance directly and
stimulates aldosterone secretion, resulting in increased reabsorption of sodium and
water in the distal convoluted tubules.
The afferent arteriole (choice A) carries blood from the interlobular arteries to the
glomerulus. Filtration of blood occurs in the glomerulus, with the filtrate entering
Bowman's capsule.
The arcuate arteries (choice B) are branches of the interlobar arteries of the kidney.
The arcuate arteries lie in the corticomedullary junction of the kidney and give rise
to interlobular arteries, which enter the cortex of the kidney and supply the
glomeruli.
Kupffer cells (choice D) are found in the liver, along the sinusoids. They are
phagocytic cells that are part of the reticuloendothelial system.
The proximal convoluted tubule (choice E) is directly continuous with Bowman's
capsule. Most of the resorption of the glomerular filtrate occurs in this part of the
nephron.
________________________________________

46The correct answer is E. Any previously healthy person in the Southwestern


United States who develops septic shock or severe pulmonary disease should be
evaluated for plague. Plague is not an extinct disease, but is still encountered in
sporadic cases in various places in the world, including Asia, Africa, parts of Europe,
and the American Southwest. The causative organism is Yersinia pestis, which is
endemic in many wild animal populations, and can be transmitted to humans either
by direct contact or by arthropod bite. Human plague may take many forms,
including pestis minor (mild lymphadenopathy); bubonic plague (prominent
lymphadenopathy); pneumonic plague (as in this patient); and septicemic plague.
The primary pneumonic form typically presents as described. Antibiotics are most
effective if given within the first 24 hours, which can be problematic if medical staff
do not suspect the disease. Since plague is rare in the United States, a high degree
of clinical suspicion is required to make a rapid diagnosis and to institute timely
treatment. If the diagnosis is missed, the mortality rate is quite high.
Brucella abortus (choice A) causes brucellosis, characterized by undulating fever,
lymphadenopathy, and hepatosplenomegaly.
Clostridium perfringens (choice B) causes gas gangrene and gastroenteritis.
Francisella tularensis (choice C) causes tularemia, associated with a spectrum of
manifestations from an influenza-like syndrome to adenopathy with ulceration at
the site of inoculation.

Listeria monocytogenes (choice D) causes listeriosis. Infection during pregnancy


may result in sepsis, abortion or premature delivery. Infection in the neonate may
produce meningitis. In immunocompromised adults, either meningitis or sepsis may
occur.
________________________________________

47The correct answer is D. Although electroconvulsive therapy (ECT) is highly


efficacious in treating major depressions that are refractory to tricyclic
antidepressants and selective serotonin reuptake inhibitors, it produces retrograde
amnesia as its major side effect.
Extrapyramidal symptoms (choice A) are commonly produced by acute
administration of antipsychotic drugs, such as phenothiazines or butyrophenones,
not ECT.
Hearing loss (choice B) is not a common side effect of ECT.
Mania (choice C) is not a recognized side effect of ECT.
Rhabdomyolysis (choice E) does not generally occur with ECT when it is performed
correctly, with the administration of skeletal muscle relaxants.
________________________________________

48The correct answer is B. The total lung capacity is the sum of the functional
residual capacity and inspiratory reserve capacity. The easiest approach to this type
of question is to construct a simple spirogram (see below) and fill in the values
provided in the table. This approach eliminates the need to memorize formulas. The
missing volume or capacity can be easily determined from the spirogram. The total
lung capacity, functional residual capacity, and residual volume are often increased
in the emphysematous lungs of patients with a long-term history of cigarette
smoking.

inspiratory reserve volume

expiratory reserve volume

inspiratory reserve capacity

vital capacity

residual volume

functional residual capacity

tidal volume

total lung capacity

________________________________________

49The correct answer is D. The right ventricle forms most of the anterior wall of the
heart and extends from approximately the right border of the sternum to
approximately 2 inches to the left of the sternum at the level of the fourth
intercostal space.
The left atrium (choice A) forms the posterior wall of the heart. The only portion of
the left atrium seen on the anterior surface of the heart is the left auricular
appendage, which is at the level of the second intercostal space on the left.
The left ventricle (choice B) forms most of the left border of the heart and the
diaphragmatic surface of the heart. It forms the anterior wall of the heart in a region
from approximately 2-3 inches from the left border of the sternum from the third to
the fifth intercostal space.
The right atrium (choice C) forms the right border of the heart. Its anterior surface is
on the right side of the sternum from approximately the third rib to the sixth rib.
The left lung (choice E) is displaced away from the sternum on the left side by the
presence of the heart.
________________________________________

50The correct answer is E. Pemphigus vulgaris is associated with chronic, severe


bullae formation on the skin and oral mucosa. Mucosal lesions are extremely rare in
bullous pemphigoid. This can be used clinically to guide therapy, although a skin
biopsy should be taken to confirm the diagnosis. Both diseases are characterized by
formation of tender bullae that can rupture, leaving red, raw areas. Pemphigus
vulgaris patients eventually become febrile and lose weight, and if untreated, most
will die within one year. Bullous pemphigoid lesions tend to heal and the patients do
very well. This prognostic difference is an important distinction in dermatologic
medicine.
Eosinophils within blisters (choice A) provide an important clue supporting bullous
pemphigoid as the diagnosis which must be ascertained with histologic
examination. The vesicles in pemphigus vulgaris mostly contain rounded
acantholytic keratinocytes "floating" within.
IgA deposits seen with immunofluorescence on the basement membrane (choice B)
is characteristic of a third bullae-forming disease known as dermatitis herpetiformis.
Vesicles form between the dermal-epidermal junction. The disease is associated
with gluten sensitivity and celiac disease.

IgG autoantibody activity (choice C) is common to both pemphigus vulgaris and


bullous pemphigoid.
Nikolsky's sign consists of separation of the epidermis upon manual stroking of the
skin. Bullous pemphigoid is characterized by a negative Nikolsky's sign (choice D),
but Nikolsky's sign is positive in pemphigus vulgaris due to the IgG-mediated
destruction of intercellular bridges between keratinocytes.
Answers

________________________________________

1The correct answer is E. The clinical syndrome is Parinaud's syndrome, caused in


this case by a pineal tumor that is compressing the superior colliculi and pretectal
area of the dorsal midbrain, the region that contains the vertical gaze center. Pineal
tumors also can cause compression of the cerebral aqueduct, resulting in a
noncommunicating hydrocephalus. This has likely already occurred in this patient,
as evidenced by the bilateral papilledema.
The mamillary bodies (choice A) are located on the ventral surface of the
diencephalon. By contrast, the pineal body is a dorsal outpouching of the
diencephalon near the midbrain tectum.
Compression of the optic chiasm (choice B) generally results from a pituitary tumor,
resulting in a bitemporal hemianopia.
The pineal body is at the level of the midbrain. The pons (choice C) is too far caudal
to be affected by a pineal tumor.
The superior cerebellar peduncle (choice D) primarily contains efferent projections
from the cerebellum. Damage to this peduncle would result in cerebellar signs (e.g.,
intention tremor, dysmetria).
________________________________________
2The correct answer is B. Conn's syndrome, or primary hyperaldosteronism, results
from an adrenal tumor that secretes excessive aldosterone. The increased
mineralocorticoid effects of aldosterone lead to renal sodium and water retention
(which explains the hypertension) and increased renal potassium excretion
(hypokalemia). The volume expansion also explains the decrease in hematocrit. The
increased blood volume, increased blood pressure, and hypernatremia will all tend
to suppress renin secretion in an attempt to compensate for the increased
aldosterone.
Addison's disease (choice A), or primary adrenal insufficiency, is characterized by
low plasma concentration of aldosterone, hyponatremia, hypotension, and
hyperkalemia.

In Cushing's syndrome (choice C), blood pressure may be increased because of


crossover mineralocorticoid activity of the increased plasma cortisol. Furthermore,
cortisol makes blood vessels more responsive to catecholamines, which could
increase peripheral resistance. The combination of increased blood pressure and
hypokalemia would, if anything, tend to suppress secretion of aldosterone.
21-Hydroxylase deficiency (choice D) is likely to produce hypotension. In the saltwasting variant of this disorder, the plasma concentration of aldosterone is
decreased and hyponatremia and hyperkalemia result.
Pheochromocytoma (choice E) is another endocrine cause of hypertension. The
increased plasma concentration of catecholamines can cause increased cardiac
output and increased peripheral resistance. Plasma renin activity may be increased
because of increased beta receptor activation on juxtaglomerular cells. This could
produce increased aldosterone secretion and subsequent salt retention.
________________________________________
3The correct answer is E. Antineutrophil cytoplasmic antibodies yielding a
cytoplasmic immunofluorescence pattern (c-ANCA) are found in over 90% of
patients with Wegener's granulomatosis. The classic clinicopathologic complex of
Wegener's granulomatosis usually provides ready differentiation from other
disorders in which c-ANCA is positive. c-ANCA is not considered pathognomonic, but
has a very strong association with Wegener's and is not found in any of the other
diseases listed.
c-ANCA is generally negative in patients with allergic angiitis (choice A), noninfectious granulomatous disease (choice C), and tumors of the upper airway
(choice D).
c-ANCA is typically negative in Goodpasture's syndrome (choice B), but p-ANCA
have been detected in some patients with Goodpasture's syndrome.
________________________________________
4The correct answer is B. The first event that occurs in the pre-B cell (progenitor) is
the gene rearrangement of the heavy chain. The D gene and J gene recombination
event occurs first, followed by V recombination with the D-J region.
The cytoplasmic mu chains (choice A) are the result of immunoglobulin heavy chain
rearrangement, the production of the messenger RNA from this rearrangement, and
the ribosomal synthesis of the protein mu chain. This is the second event that
occurs in the B-cell maturational sequence.
Light chain rearrangement (choice C) occurs when recombination events occur with
the V gene and J gene from the light chain germ line. After this recombination has
occurred, and the messenger RNA for this germ line has produced the light chain
protein, the light chains and heavy chains form.
The complete IgM molecule and IgD molecules (choice D) are present on the surface
of only the mature B cells. This event is the last event to occur during the
differentiation and development of B cells in the bone marrow.

The complete IgM molecule (choice E) is present on the surface of the immature B
cell. This event is one of the last events to occur in the differentiation and
development of B cells.
________________________________________
5The correct answer is B. The process described is commonly called acute idiopathic
thrombocytopenic purpura (ITP), even though the autoimmune basis has been
clearly established (some authors use "immune thrombocytopenic purpura," so that
the initials still work). The thrombocytopenia in this disorder appears to be
secondary to splenic destruction of opsonized platelets, and usually follows a viral
upper respiratory tract infection or exanthem. The acute form of ITP is usually
explosive, but self-limited; a chronic form in adults may respond to steroid therapy
or splenectomy.
In Bernard-Soulier syndrome (choice A), platelets are unable to adhere to collagen.
Thrombasthenia (choice C), is associated with a functional defect in platelets (they
do not aggregate).
In thrombotic thrombocytopenic purpura (choice D), platelets are consumed in
intravascular platelet microthrombi.
In von Willebrand's disease (choice E), deficient von Willebrand factor produces
platelet dysfunction, but thrombocytopenia is not prominent.
________________________________________
6The correct answer is A. Acute gastritis, characterized by patches of erythematous
mucosa, sometimes with petechiae and ulceration, can be seen as a complication of
a variety of other conditions (alcohol use, aspirin and other NSAIDs use, smoking,
shock, steroid use, and uremia), which usually have in common disruption of the
mucosal barrier of the stomach.
Chronic antral (type B) gastritis (choice B) is associated with Helicobacter pylori.
Chronic fundal (type A) gastritis (choice C) is the type associated with pernicious
anemia.
Hypertrophic gastritis (Menetrier's disease; choice D) is an idiopathic condition
characterized by markedly enlarged mucosal folds.
Lymphocytic gastritis (choice E) is thought to be a gastric manifestation of celiac
sprue.
________________________________________
7The correct answer is A. The tumor is a craniopharyngioma, alternatively known as
an adamantinoma or ameloblastoma. Craniopharyngiomas may arise in, or more
commonly above, the sella turcica. The histological pattern recapitulates the
enamel organ of the tooth, with nests or cords of stratified squamous or columnar
epithelium embedded in a loose fibrous stroma. Calcification (and even metaplastic

bone formation) is common in these benign tumors, which are thought to arise from
vestigial remnants of Rathke's pouch.
Glioblastoma multiforme (choice B) characteristically shows at least some enlarged
cells with bizarre nuclei.
Large pituitary adenomas (choice C) contain nests of uniform glandular cells.
Medulloblastoma (choice D) is made of small basophilic cells with relatively large
nuclei for their size.
Pituitary microadenomas (choice E) contain nests of uniform glandular cells.
________________________________________

8The correct answer is C. Spontaneous abortion is a problem associated with


Listeriosis, caused by Listeria monocytogenes. The pattern of abortions was first
recognized in herd animals, notably sheep and cattle, and then listeriosis was later
implicated as a cause of spontaneous abortion in pregnant women. Listeriosis can
occur in either epidemic (food-borne or hospital-acquired) forms or may be sporadic
(noticed in animal or animal product handlers). Soft cheeses like Brie are a
particularly common source of food-borne listeriosis. The disease may range in
severity from asymptomatic carrier cases, to flu-like illness, to spontaneous abortion
or neonatal death, to fatal illness in children or adults secondary to septicemia or
meningoencephalitis. Other localized infections can also occur, primarily in the
immunosuppressed. The treatment of choice is intravenous administration of
ampicillin or penicillin, often in combination with an aminoglycoside. Trimethoprimsulfamethoxazole has been used successfully in patients with penicillin allergy.
Borrelia burgdorferi (choice A) causes Lyme disease.
Leptospira interrogans (choice B) causes leptospirosis.
Spirillium minus (choice D) is a cause of rat-bite fever.
Streptobacillus moniliformis (choice E) is a cause of rat-bite fever.
________________________________________

9The correct answer is B. This individual has an XY karyotype and testes (probably
abdominal), but because of complete androgen resistance, developed a female
phenotype in utero. In the absence of androgen receptors, the indifferent external
genital slit will differentiate into a vagina with clitoris and labia. However, the vagina
will end as a blind sac because Mllerian regression factor secreted by the testes
will prevent the formation of a uterus or uterine tubes. The Wolffian ducts will also
degenerate in the absence of androgen receptors. At puberty, the testes will
respond to the increased LH by increasing testosterone secretion. Masculinization is
not possible because of the absence of androgen receptors, however, significant
gonadal or peripheral aromatization of testosterone to estrogen will produce breast

enlargement and other female secondary sex characteristics. LH will remain high
because of the absence of negative feedback by the testosterone. Pubic and axillary
hair development, which is also androgen-driven, will not occur.
Females with 17-alpha-hydroxylase deficiency (choice A) are born with normal
female internal reproductive tract and external genitalia. This is because the
"default" program is for the female phenotype to develop in utero. With 17-alphahydroxylase deficiency, sex steroids (estrogen in the case of females) cannot be
synthesized and secreted. Affected females will not mature sexually at puberty, but
will remain infantile. Since the patient described above exhibited breast
development, 17-alpha-hydroxylase deficiency is unlikely. Furthermore, the absence
of uterus or uterine tubes is not consistent with 17-alpha-hydroxylase deficiency.
Constitutional delay in onset of menses (choice C) may occur in certain families. It is
thought to be due to slow maturation of the hypothalamic-pituitary-gonadal axis.
However, growth velocity and development of breasts and pubic hair usually occurs
normally. A uterus and uterine tubes would be present since there is no disorder in
embryologic development.
Hyperprolactinemia (choice D) can suppress the hypothalamic-pituitary-gonadal
axis and produce amenorrhea, however, it cannot explain the absence of pubic and
axillary hair or the absence of a uterus and uterine tubes.
Turner's syndrome (choice E) occurs in females with the XO genotype. They are
typically short in stature, rarely reaching 5 feet. The germinal tissue in the ovaries is
replaced with fibrous streaks. The internal reproductive tract is normal. Estrogen
secretion is diminished (primary ovarian disorder) which leads to an increase in LH
and FSH. The height of the above patient makes Turner's syndrome unlikely.
Furthermore, Turner's syndrome could not explain the absence of a uterus and
uterine tubes.
________________________________________
10The correct answer is B. Mycosis fungoides (despite its deceivingly fungal name)
is a cutaneous T cell lymphoma in which the malignant lymphocytes show trophism
for the epidermis. The malignant T cells densely infiltrate the superficial dermis and
epidermis, producing the typical plaques and nodules.
Intracellular inclusions are seen in viral infections, and cytoplasmic inclusions in the
epidermal cells (choice A) are pathognomonic for molluscum contagiosum, a
poxvirus that produces wart-like skin lesions.
Nits (choice C) are the egg cases of lice. Nits, or the lice themselves, can be seen
attached to the hair shafts of people with pediculosis.
Hyphae and yeasts (choice D) are the hallmarks of superficial fungal infections. The
tinea infections are caused by dermatophytic fungi, and can usually be appreciated
in PAS stains of skin biopsies.
Vascular proliferation (choice E) is the characteristic finding in hemangiomas,
Kaposi's sarcoma, and bacillary angiomatosis.

________________________________________
11The correct answer is C. This disease is endocardial fibroelastosis, which is
suspected of being related to intrauterine virus infection, possibly with mumps
virus. The endocardium thickens secondary to a marked increase in fibrous and
elastic tissue, which may extend into the ventricle. The left ventricle is most
commonly involved, but other chambers may also be affected. A variety of other
findings may also be present, including mural thrombi, flattened trabeculae, and
abnormal (usually stenosed) valves. The disease occurs in infantile and adolescent
forms.
Coxsackie virus myocarditis (choice A) would produce a flabby myocardium, patchy
areas of hemorrhage, and four chamber dilatation.
In cardiac amyloidosis (choice B), extracellular amyloid deposits would be seen in
histological sections.
Idiopathic subaortic stenosis (choice D) produces a hypertrophic cardiomyopathy.
Loeffler's endocarditis (choice E) is characterized by a prominent eosinophilic
infiltrate with no increase in elastic tissue.
________________________________________
12The correct answer is A. Cyclobenzaprine is a centrally acting skeletal muscle
relaxant that is structurally and pharmacologically related to tricyclic
antidepressants (eg, amitriptyline). It is used short-term as an adjunct to rest and
physical therapy for relief of muscle spasm associated with acute musculoskeletal
conditions. Like tricyclic antidepressants, the most common side effects are dry
mouth, drowsiness, dizziness, weakness, fatigue, tachycardia, urinary retention, and
abdominal cramping.
Dantrolene (choice B) is also a centrally acting skeletal muscle relaxant; however, it
is structurally and pharmacologically related to hydantoin derivatives, such as
phenytoin. Although dantrolene produces the same CNS effects as cyclobenzaprine,
it does not produce anticholinergic side effects, such as dry mouth and urinary
retention.
Doxycycline (choice C) is an antibiotic used primarily in the treatment of various
sexually transmitted diseases and acne. Its major side effects include diarrhea,
gastrointestinal upset, and phototoxicity.
Ibuprofen (choice D) is a nonsteroidal anti-inflammatory drug used in the treatment
of mild-to-moderate pain caused by inflammation; its most common side effects are
intestinal discomfort and dizziness.
Lorazepam (choice E) is a benzodiazepine used in the management of anxiety
disorders and for the short-term relief of anxiety. Side effects include drowsiness,
sedation, dizziness, and weakness.
________________________________________

13The correct answer is C. The pathologist saw normal chief cells, which are
abundant in the body and fundus of the stomach. Chief cells secrete pepsinogen,
which is stored in apical membrane-bound granules. The body and fundus of the
stomach contain high concentrations of four other types of cells in the epithelium.
The parietal (oxyntic) cells are large, pyramidal, and acidophilic with central nuclei
(look like a "fried egg"). They make and secrete HCl. The mucous neck cells secrete
mucus and appear clear. The enteroendocrine cells have affinity for silver stains and
exhibit a positive chromaffin reaction; these cells synthesize amines, polypeptides,
or proteins.
The cardiac region (choice A) is a narrow, circular band at the transition between
the esophagus and stomach, consisting of shallow gastric pits and mucous glands. It
does not normally contain an abundance of chief cells.
The columns of Morgagni (choice B) are found in the rectum, not in the stomach.
These are mucous membrane infoldings in the submucosa of the proximal anal
canal. They would not contain chief cells.
The greater omentum (choice D) is a four-layered fold of peritoneum that hangs
from the greater curvature of the stomach and attaches to the transverse colon. It
would not contain chief cells.
The pyloric region (choice E) has deep gastric pits into which tubular glands open.
The predominant secretion is mucus. It does not normally contain an abundance of
chief cells.
Note that in this question you could have automatically eliminated choices B and D,
since they are not gastric structures. If nothing else, you would have improved your
guessing odds to 33%.
________________________________________
14The correct answer is B. This patient has lymphogranuloma venereum caused by
Chlamydia trachomatis (type L1, 2, or 3). Chlamydia exhibit distinct infectious and
reproductive forms. The extracellular infectious form is known as the elementary
body (EB), which is incapable of reproduction. It attaches to the host cell and enters
through endocytosis. Once inside the cell, the EB is transformed into the reticulate
body (RB) within the endosome. The RB is capable of binary fission and divides
within the endosome; fusion with other endosomes occurs to form a single large
inclusion. Eventually, the RBs undergo DNA condensation and disulfide bond
bridgings of the major outer membrane protein, forming EBs. The EBs are then
released. Note that C. trachomatis is responsible for several sexually or perinatally
transmitted diseases, including ocular trachoma (types A, B, and C), neonatal
conjunctivitis, nongonococcal urethritis, cervicitis, and pelvic inflammatory disease
(types D-K).
Calymmatobacterium granulomatis (choice A) is a gram-negative rod that causes
superficially ulcerated genital or inguinal papules that coalesce to form substantial
lesions. The appearance of Donovan bodies in histiocytes is diagnostic of this
infection.

Haemophilus ducreyi (choice C) is a gram-negative rod that causes a soft, painful


penile chancre, unlike that of a chlamydial or syphilitic lesion. This infection is
common in the tropics.
Neisseria gonorrhoeae (choice D) is a gram-negative diplococcus responsible for
gonorrhea. Patients typically present with purulent penile discharge, not genital
lesions.
Treponema pallidum (choice E) is the spirochete responsible for syphilis. It may
cause a firm, painless ulcer as a manifestation of primary syphilis, but the organism
does not exist in distinct extracellular and intracellular forms as does Chlamydia.
Secondary syphilis is associated with the appearance of condyloma lata-flat, gray,
wart-like lesions.
________________________________________
15The correct answer is A. The tumor is medullary carcinoma of the thyroid, which
can occur sporadically or as part of multiple endocrine neoplasia (MEN), types IIa
and IIb. The history of pheochromocytoma and the multifocality of this tumor
suggest that this is one of the MEN-type tumors. Other facts to know about
medullary carcinoma of the thyroid include the local production of amyloid (formed
in this case from precipitated calcitonin), the secretion of calcitonin (and also other
biologically active substances), and the link to a genetic defect on chromosome 10.
Parathyroid hormone (choice B) can be produced by parathyroid tumors or can be a
paraneoplastic product of a variety of tumors, including lung cancer.
TSH (choice C) can be produced by pituitary adenomas.
T3 (choice D), or triiodothyronine, and T4 (choice E), or thyroxine, can be produced
by thyroid tumors composed of follicular cells.
________________________________________
16The correct answer is B. The glossopharyngeal nerve (CN IX) and the vagus nerve
(CN X) carry afferent information to the medulla from the carotid sinus and aortic
arch baroreceptors, respectively. The firing rate of these neurons increases with
increasing blood pressure. Therefore, by artificially increasing the firing rate of these
nerves, the medulla receives a false signal that indicates that the blood pressure is
too high. This elicits a baroreceptor reflex, resulting in a decrease in sympathetic
outflow and an increase in parasympathetic outflow, which leads to bradycardia and
hypotension.
________________________________________
17The correct answer is C. Regardless of how long the untranslated regions are, the
number of nucleotides in the coding region of an mRNA is three times the number of
amino acids, since three nucleotides are required to code for each amino acid, and 3
X 64 = 192. In reality, three nucleotides code for the first amino acid (formylmethionine in prokaryotes, methionine in eukaryotes), which may be removed in
posttranslational steps, and three nucleotides at the 3' end (are needed to

terminate the process (i.e., a STOP codon), so the actual number would likely be
slightly higher.
________________________________________
18The correct answer is E. The answer options contain a mix of ligand-gated ion
channel receptors and G protein coupled receptors. A receptor that would reliably
produce excitation, thus increasing in firing rate, would be an excitatory amino acid
receptor. The N-methyl-D-aspartate (NMDA) receptor is an example of this type of
receptor. The NMDA receptor is a ligand-gated ion channel that would permit the
influx of cations (sodium and calcium). The rule of thumb is that cations entering
the cell through ion channels produce depolarization, and anions entering the cell
cause hyperpolarization.
The alpha-2 adrenergic receptor (choice A) is coupled to Gi, and would lead to a
decrease in cAMP levels.
The beta-1 adrenergic receptor (choice B) is coupled to Gs, and would lead to an
increase in cAMP levels.
The gamma-aminobutyric acid (GABA) (choice C) and glycine (choice D) receptors
are inhibitory amino acid receptors. They are ligand-gated ion channel receptors
that allow chloride influx. This could cause hyperpolarization, or at least membrane
potential stabilization, thus preventing excitation of the cell.
________________________________________
19The correct answer is A. Cocaine is a stimulant that causes hypertension and
tachycardia by blocking norepinephrine uptake. This leads to an accumulation of
norepinephrine in the synapse, causing greater stimulation of postsynaptic
receptors. The receptors that mediate the systemic vasoconstriction are alpha-1
adrenergic receptors, and the receptors that mediate the increases in heart rate and
inotropic state are beta-1 adrenergic receptors.
Direct stimulation of alpha-1 receptors (choice B) would increase blood pressure,
but this is not cocaine's mechanism of action.
Direct stimulation of beta-1 receptors (choice C) could increase blood pressure by
increasing inotropic state, but this is not cocaine's mechanism of action.
Direct stimulation of beta-2 receptors (choice D) would cause vasodilation in the
skeletal muscle vasculature, leading to a decrease in blood pressure.
Induction of norepinephrine release (choice E) would increase blood pressure, but
this is the mechanism of action of amphetamine, not cocaine.
Cocaine is not metabolized to a false neurotransmitter (choice F).
________________________________________
20The correct answer is B. The patient is hypoventilating due to the effects of
barbiturates on respiration. This question requires use of the alveolar gas equation:

PAO2 = PIO2 - PACO2/R, where


PAO2 = the partial pressure of alveolar O2
PIO2 = the partial pressure of inspired O2
PACO2 = the partial pressure of alveolar CO2
R = respiratory exchange ratio
We know that PAO2 = 100 mmHg, PACO2 = 80 mmHg, and R = 1, so we can solve
for PIO2:
100 mmHg = PIO2 - 80 mmHg/1
PIO2 = 180 mmHg
PIO2= FO2 (PB - PH2O), where FO2 is the fraction of inspired O2 and PB is the
barometric pressure. PH2O is the water vapor pressure in the airways and always
remains constant at 47 mmHg. Solving for FO2:
180 mmHg = FO2 (500 mmHg - 47 mmHg)
FO2 is approximately .40.
In order for this patient to re-establish a normal value for alveolar PO2, a 40%
oxygen mixture must be inhaled. (The percent of oxygen present in the atmosphere
is 21%.)
________________________________________
21The correct answer is A. The lesion is an example of carcinoma in situ. The
presence of keratin pearls and the distribution of abnormal cells all the way to the
top of the epidermis distinguish this lesion from simple dysplasia (choice B), in
which abnormal cells are seen only in the lowermost layers of the epidermis.
Invasive carcinoma (choice C) would be indicated by a disruption of the basement
membrane or the presence of nests of cells in the dermis.
The term metaplasia (choice D) is used when a normal epithelium is replaced by
another mature, differentiated epithelium.
Tumor cells in lymphatics or blood vessels would suggest progression to metastatic
carcinoma (choice E).
________________________________________

22The correct answer is B. Prostatic hypertrophy in elderly men is very common;


therefore, it should be considered as a primary cause of renal insufficiency until
proven otherwise. The patient's signs and symptoms are consistent with obstructive
uropathy; there is a history of high urine output followed by periods of almost no
urine output. This pattern of urine output leads to the accumulation of urine in the
collecting system, which creates a high pressure system. This high pressure is then

"transmitted" back to the kidney and results in renal insufficiency. Since the patients
obstructive uropathy is most likely caused by prostatic hypertrophy, doxazosin
should be used to treat the cause of these signs and symptoms. Doxazosin is a
peripherally acting alpha-1-adrenergic blocking agent indicated for the treatment of
urinary outflow obstruction secondary to benign prostatic hyperplasia (BPH). It is
also indicated for the treatment of hypertension, especially in men with BPH.
Therefore, the use of this agent will correct the obstructive uropathy and treat his
hypertension.
Benazepril (choice A) is an ACE inhibitor used in the treatment of hypertension;
however, it is known to cause azotemia and oliguria, especially in those with renal
insufficiency. Therefore, this agent would be contraindicated.
Furosemide (choice C) is a "loop" diuretic used to increase urine output in patients
without a urinary tract obstruction.
Hyoscyamine (choice D) is used in the treatment of gastrointestinal disorders
caused by spasm and hypermotility. Since this agent is a potent anticholinergic, it
would not be recommended in a patient with urinary obstruction. Remember,
anticholinergic agents cause urinary retention.
Phenazopyridine (choice E) is a urinary tract analgesic used to decrease the dysuria
associated with urinary tract infections. The use of this agent in patients with renal
insufficiency is not recommended because phenazopyridine can accumulate,
resulting in renal stones and transient renal failure.
________________________________________
23The correct answer is D. The patient probably has acromegaly due to a growth
hormone-secreting adenoma in the anterior pituitary. Hypersecretion of growth
hormone in an adult will not cause an increase in stature, since the epiphyses of
long bones have already fused. However, overgrowth of bone in the face and skull
produces the characteristic protruding jaw and forehead observed in this disorder.
Soft tissue proliferation leads to a coarsening of facial features. The hands and feet
are particularly affected, producing large and thickened spade-like fingers and toes.
Excessive growth hormone decreases the sensitivity of peripheral tissues to insulin
("anti-insulin" effect). This tends to raise blood glucose and produce a compensatory
hyperinsulinemia (not decreased plasma insulin, choice C) that functions to limit the
hyperglycemia. Approximately 50% of patients with acromegaly show impaired
glucose tolerance.
Plasma levels of growth hormone (choice A) and IGF-1 (choice B) are both increased
in acromegaly.
Administration of oral glucose does not suppress growth hormone in acromegaly as
it does in normal individuals (choice E).
________________________________________
24The correct answer is B. The most commonly used method for calculating p
values from a two-by-two contingency table is the chi-squared test. This test is used

for frequency data (such as those above) rather than for comparison of means.
Since the investigator scored the stomachs as either containing ulcers or not
containing ulcers, without attempting to quantify the number of ulcers, the data
represent frequencies rather than means. Chi-squared is calculated as the sum for
all cells of (Observed - Expected)2/Expected. The p value for this value of chisquared is obtained from a table using degrees of freedom equal to (number of rows
- 1) x (number of columns - 1).
ANOVA (choice A) is used to determine if the difference between two or more groups
is significantly different. This method is not applicable to raw frequency data.
Linear regression (choice C) is the process of fitting a straight line to a set of
correlational data points by minimizing the sum of squares of the vertical distances
from the points to the line. This method would not be suitable for the above data.
A paired t-test (choice D) is used to compare the means of two groups. It is not used
to compare raw frequency data.
Correlation is used when two variables are simultaneously measured in a sample.
The Pearson correlation coefficient (choice E) is used when both the independent
and dependent variable are continuous. In this case, the independent variable is not
continuous (since the drug is either given or not given), and the dependent variable
is also not continuous (the ulcers are either present or absent).
________________________________________
25The correct answer is D. The spinal accessory nerve crosses the posterior triangle
of the neck immediately deep to the investing fascia of the neck. This nerve
innervates the trapezius muscle, which is responsible for upward rotation and
elevation of the scapula. A lesion of this nerve in the posterior triangle leads to
paralysis of the trapezius. Without the ability to upwardly rotate the scapula,
abduction and flexion of the arm above the horizontal plane is not possible. Also,
shrugging of the shoulder is impaired with paralysis of the trapezius muscle.
The axillary nerve (choice A)does not pass through the neck. It is a branch of the
brachial plexus and it leaves the axilla to innervate the deltoid and teres minor
muscles.
The great auricular nerve (choice B) and transverse cervical nerve (choice E) are
branches of the cervical plexus, which provide cutaneous innervation to the skin of
the neck. No muscles are innervated by these nerves.
The greater occipital nerve (choice C) is the dorsal ramus of the second cervical
spinal nerve. It provides cutaneous innervation to the skin of the back of the head.
No muscles are innervated by this nerve.
________________________________________
26The correct answer is A. This patient has an aortic dissection (formerly called
dissecting aneurysm), a potentially fatal condition that is too often confused
clinically with myocardial infarction. The most important clinical clue is that the pain
shifts with time. Non-invasive techniques such as transesophageal

echocardiography, computed tomography (CT), and magnetic resonance imaging


(MRI) are increasingly useful in making this diagnosis.
Aortic valve stenosis (choice B) would not be expected to produce severe chest pain
of acute onset.
This patient's clinical history does not suggest either an atherosclerotic (choice C) or
a syphilitic (choice E) aneurysm. Even if he had one of either of these types of
aneurysms and it had begun to rupture, the distinctive feature of severe pain
moving downward would probably not be present.
Myocardial infarction (choice D) is the major diagnosis most often confused with this
patient's condition. The movement of the pain is the major clinical tip-off suggesting
that this is not the correct answer.
________________________________________
27The correct answer is C. The characteristic finding of Berger's disease is IgA
deposition in the glomerular mesangium, which may also contain C3, properdin,
IgG, and IgM. The disease is thought to be due to trapping of large IgA-antigen
complexes in the glomeruli, leading to activation of the alternative complement
pathway. Renal failure in these cases may take decades to develop.
Glomerular crescent formation (choice A) is due to proliferation of epithelial cells
and infiltration by monocyte-macrophages. Crescents are an ominous sign,
suggesting rapidly progressive glomerulonephritis.
Podocyte foot process effacement (choice B), when seen as the only finding or the
principal finding, suggests minimal change disease, which is the major cause of the
nephrotic syndrome in children.
Subepithelial humps (choice D) by electron microscopy suggests poststreptococcal
glomerulonephritis. Immunofluorescence for IgG, IgM, and C3 yields a granular
pattern.
Tram tracking (choice E), referring to a thickening of the walls of glomerular
capillaries, suggests membranoproliferative glomerulonephritis.
________________________________________
28The correct answer is E. The patient has subacute thyroiditis, also known as
granulomatous giant cell or de Quervain's thyroiditis. This condition is thought to be
due to viral infection of the thyroid gland, and may follow an upper respiratory
infection. Early in the process, the follicular destruction causes inappropriate release
of thyroid hormone with resulting hyperthyroidism. Later, the destruction can
proceed to the point that hypothyroidism occurs.
Antibodies that block the TSH receptor (choice A) are a feature of Hashimoto's
disease.
Thyroid carcinoma (choice B) usually does not produce hyperthyroidism.

Lymphoma (choice C) of the thyroid is more likely to occur in the setting of


Hashimoto's disease.
Stimulating TSH receptor antibodies (choice D) are a feature of Graves disease
________________________________________
29The correct answer is C. The 16-year-old probably has type 1 (juvenile onset)
diabetes mellitus, while the 65-year-old probably has type 2 (maturity onset)
diabetes mellitus. These two types of diabetes differ in many respects. Ketoacidosis
is more apt to develop in type 1 diabetes.
type 2 diabetics tend to be obese (choice A), while type 1 diabetics are often thin.
type 1 is usually apparently due to viral or immune destruction of beta cells, while
type 2 is apparently usually due to increased resistance to insulin; consequently the
65-year-old, rather than the 16-year-old, is more likely to have relatively high
endogenous levels of insulin (choice B).
type 2 diabetes can often be controlled with oral hypoglycemic agents (choice C),
while type 1 diabetics generally require insulin. Note that some type 2 diabetics also
may require insulin as the disease evolves.
type 1 diabetes has a strong association with HLA-DR3 and HLA-DR4 (choice E),
while type 2 does not have any strong HLA associations.
________________________________________
30The correct answer is B. The symptoms suggest infection with Neisseria
gonorrhoeae; however, with Gram's stain results alone, it is not possible to
distinguish the gonococcus from normal flora organisms such as Acinetobacter,
unless gram-negative diplococci are found within polymorphonuclear leukocytes.
The fastest and most reliable assay specific for diagnosis of gonorrhea is the use of
commercial DNA probes, with results available in 2-4 hours.
Culture on Thayer-Martin medium (choice A) is indeed the choice for culture of N.
gonorrhoeae, but would yield slower results, and therefore be a secondary choice.
Serology (choice C) is not a good choice because N. gonorrhoeae does not have a
significant capsule (the meningococcus does), and furthermore, serological tests for
the gonococcus have proven insensitive and nonspecific.
The germ tube test (choice D) is one of the diagnostic tests for Candida albicans,
but this case presentation is not characteristic of candidiasis.
The RPR (choice E) is a test for reaginic (heterophilic) antibodies formed early in
infection with Treponema pallidum, but this case presentation is not suggestive of
syphilis.
________________________________________
31The correct answer is D. This rare, but dramatic, condition is called
pseudomyxoma peritonei, and is produced when a malignant or benign (spread by

rupture into the peritoneal cavity rather than true metastasis) mucus producing
tumor (mucinous cystadenoma or mucinous cystadenocarcinoma) produces gel-like
mucus that fills the peritoneal cavity. The usual sites of origin of these tumors are
ovary and appendix. The condition, even when benign, is difficult to treat because
the mucus producing cells are spread all over the peritoneal lining, and cannot be
effectively removed. Home parenteral nutrition may be necessary as the gut
(particularly peristalsis) just does not function well in a sea of Jell-O that slowly
becomes replaced with fibrous tissue bands.
________________________________________
32The correct answer is E. This patient has suffered a cerebrovascular accident as a
consequence of thromboemboli emanating from a mural thrombus formed over the
recent myocardial infarction. Mural thrombus frequently develops over a previously
infarcted segment of myocardium, especially when the infarction is large and a
ventricular aneurysm develops. Ischemic damage to the endocardium, soluble
factors released by the injured myocardium, and altered wall kinetics that produce
sluggish blood flow all favor mural thrombus formation. One important consequence
of a mural thrombus is thromboembolism to the systemic circulation, producing a
stroke, as in this patient's case.
Calcific coronary atherosclerosis (choice A) is very likely responsible for the patient's
original myocardial infarction. Coronary atherosclerosis is not a consequence of
myocardial infarction, and it does not directly predispose to the development of
embolic strokes. Complicated atherosclerotic lesions in the carotid or cerebral
circulation, however, may directly lead to a cerebrovascular accident.
Electromechanical dissociation (EMD; choice B) is a catastrophic event that
frequently leads to sudden death, not stroke. In EMD, although a normal potential is
transmitted through the myocardium, no pulse of arterial blood is sent to the
systemic circulation. EMD may occur as a consequence of pericardial tamponade,
massive pulmonary embolism, or myocardial toxins that prevent normal cardiac
muscle contraction.
Left bundle branch block (choice C) represents failure of electrical transmission
along the Purkinje fibers to the left ventricular myocardium. Bundle branch blocks
are important causes of arrhythmia, but are not responsible for cerebrovascular
accidents.
Myocardial rupture (choice D) is an infrequent consequence of myocardial infarction
that typically occurs approximately 1 week after the infarction. If scarring of the
infarcted segment does not keep pace with necrosis and digestion of the infarcted
tissue, the myocardium can rupture under the high intraventricular pressures, and
pericardial tamponade generally ensues. Myocardial rupture rapidly leads to acute
heart failure, not stroke.
________________________________________
33The correct answer is C. A facial or scalp port-wine nevus (a facial angioma)
should specifically suggest evaluation for Sturge-Weber disease. This disease is one

of the slowly progressive neurocutaneous disorders called phacomatoses. Other


phacomatoses include neurofibromatosis, Von Hippel-Lindau syndrome, and
tuberous sclerosis. Features of Sturge-Weber syndrome include the previously
mentioned "port-wine stain," which may overlie meningeal angiomatosis on the
surface of the ipsilateral hemisphere. Patients may experience focal or generalized
seizures, sometimes with hemiparesis, sensory changes, or hemianopia. Mental
retardation is often present. Most cases are actually sporadic rather than genetic in
origin.
Acoustic neuromas (choice A) are a feature of neurofibromatosis type II.
Cerebellar hemangioblastomas (choice B) are a feature of Von Hippel-Lindau
syndrome.
Neurofibromas (choice D) are a feature of neurofibromatosis type I.
Subependymal and cortical tubers (choice E) are features of tuberous sclerosis.
________________________________________
34The correct answer is E. Vibrio vulnificus septicemia is contracted by consuming
raw oysters. Patients with chronic renal insufficiency, liver disease, hematopoietic
disorders, and a past history of alcoholism are particularly at risk. His past medical
history makes him more susceptible to septicemia than are his family members.
Septicemia cause by Vibrio vulnificus generally begins with chills, fever, and
hypotension, and skin lesions tend to occur 24-48 hours after the onset of the
infection.
Aspergillus fumigatus (choice A) is an opportunistic mold responsible for infections
of wounds and burns in immunocompetent hosts; in immunocompromised hosts, it
can invade visceral organs such as the lungs.
Campylobacter jejuni (choice B) are curved, gram-negative rods that cause
enterocolitis with diarrhea, and, less commonly, chronic gastritis.
Candida albicans (choice C) is a common opportunistic yeast that is found as normal
flora of upper respiratory, gastrointestinal, and vaginal mucosae. It ordinarily causes
thrush and vulvovaginitis; it may disseminate in the immunocompromised patient.
Group A streptococcus (Streptococcus pyogenes; choice D) is an important cause of
bacterial pharyngitis (Strep throat).
________________________________________
35The correct answer is D. Hereditary factor XIII deficiency is an autosomal
recessive condition that is unusual among the factor deficiencies in that the
presentation is often at birth, when the umbilical stump bleeds excessively,
sometimes leading to the neonate's death. Factor XIII is necessary to stabilize clot
formation, and in its absence, clots will rapidly lyse. Cutaneous and muscular
hematomas are common in affected patients. Bleeding after surgery and trauma
can occur, including bleeding into the central nervous system. Spontaneous

abortion in affected women is common. A factor XIII concentrate is available for


treatment.
________________________________________
36The correct answer is B. The patient is presenting with an exacerbation of his
Parkinson's secondary to the administration of metoclopramide; the mechanism of
this side effect is related to metoclopramide's ability to antagonize dopamine
receptors. Metoclopramide is a prokinetic agent indicated for the treatment of
gastroesophageal reflux disease (GERD) and diabetic gastroparesis.
Cisapride (choice A) is a prokinetic agent indicated for the treatment of GERD. This
agent is associated with cardiac arrhythmias, nervousness, diarrhea, and abdominal
cramping.
Nizatidine (choice C) is an H2 receptor antagonist used in the treatment of GERD
and gastric ulcers; this agent is generally well tolerated with very few side effects.
Omeprazole (choice D) is a proton pump inhibitor indicated for the treatment of
gastric ulcerations and GERD; this agent is generally well tolerated with very few
side effects.
Sucralfate (choice E) is a basic aluminum salt that forms an ulcer-adherent complex
at the site of the ulcer and is indicated for the treatment of duodenal ulcer.
Constipation is the most common side effect.
________________________________________
37The correct answer is D. Salivary glands can contain predominantly serous cells,
predominantly mucous cells, or both in their acini. As you proceed from the midline
laterally, acini in the sublingual gland are almost pure mucous cells, while acini in
the submandibular gland (choice E) contain a mixture of serous and mucous cells.
Acini in the parotid gland (choice C) are mostly pure serous cells.
The esophageal mucosa (choice A) and the oral mucosa (choice B) are squamous
epithelia that do not form acini.
________________________________________
38The correct answer is E. The decrease in renal artery diameter causes a reduction
in arterial pressure within the kidney, which results in an initial decrease in
glomerular hydrostatic pressure (choice C) and glomerular filtration rate (choice B).
The fall in glomerular filtration rate decreases the amount of sodium chloride that is
delivered to the macula densa; in turn, the juxtaglomerular cells secrete renin and
angiotensin II is formed. The angiotensin then mainly constricts the efferent
arterioles, which increases glomerular hydrostatic pressure and glomerular filtration
rate. This macula densa feedback mechanism also attempts to return glomerular
hydrostatic pressure (and therefore glomerular filtration rate) to a normal level by
decreasing afferent arteriolar resistance (choice A).
An obstruction of the renal artery would decrease blood pressure in the interlobar
arteries (choice D).

________________________________________
39> Correct Answer is C.
________________________________________
40The correct answer is D. Brunner's glands are located in the submucosa of the
duodenum. These glands are connected to the intestinal lumen by ducts that open
into certain crypts. They secrete an alkaline product that protects the duodenal
mucosa from the acidic chyme and helps achieve optimal pH for pancreatic
enzymes.
Note that if you did not recall the location of Brunner's glands, the question's
description of their function allowed you to deduce it, based on your knowledge of
the anatomy of the small intestine. You should have immediately ruled out choices
C and E because they are too far from the pancreas. Choices B and D would
therefore remain as the best possible answers because of their proximity to the
pancreas. If you remembered the structure and function of the ampulla of Vater, you
were left with the correct answer.
Let's review some other key features of intestinal histology by way of the wrong
answer choices.
The small intestinal villi (choice A) are outgrowths of the mucosa into the lumen.
Their epithelium contains columnar absorptive cells and goblet cells (produce acid
glycoproteins that protect and lubricate the lining of the intestine). Near the base of
each villus, there are tubular glands called crypts whose lining is continuous with
the simple columnar epithelium of the villus. The crypts include Paneth cells, which
produce acidophilic cytoplasmic granules containing bacteriocidal enzymes. The
lamina propria of the small intestine penetrates the core of the villi and is composed
of blood vessels, lymphatics, fibroblasts, and smooth muscle cells.
The ampulla of Vater (choice B) receives bile from the common bile duct and the
main pancreatic duct, delivering it to the duodenum through the major duodenal
papilla.
The mucosa and submucosa of the jejunum (choice C) are both included in the
permanent folds called the plica circulares.
The submucosa of the ileum (choice E) is the home of Peyer's patches, which are
large aggregates of lymphoid nodules.
________________________________________
41The correct answer is E. Bone is formed by type I collagen fibers, ground
substance, and hydroxyapatite crystals. The collagen is oriented in a layered or
lamellar fashion. It can be parallel (trabecular bone and periosteum) or concentric
(Haversian system). When bone is formed quickly, as in a healing fracture site,
metabolic bone disease, or tumor, the collagen is randomly oriented and is called
woven bone.

Compact bone (choice B) is the dense calcified external part of the bone. It is
lamellar bone.
Cancellous (choice A), spongy (choice C) and trabecular bone (choice D) are all
synonymous terms for the thinner network of bone within the cortex. These are also
lamellar bone.
________________________________________
42The correct answer is D. Corticosteroids inhibit the proliferation and function of
osteoblasts, which are modified mesenchymal cells. These agents stimulate
osteoclasts to differentiate from bone marrow macrophages and also stimulate their
activity. The net effect is maximal bone loss with increased resorption and
decreased formation.
________________________________________
43The correct answer is E. There are two ways to arrive at the correct answer to this
question. The first is to simply remember the definition of residual volume (RV): the
amount of air remaining in the lungs after maximal exhalation. The second way is to
recall that the helium dilution technique described above is used to measure
functional residual capacity (FRC) and RV, which narrows the reasonable option
choices to only B and E. All of the other volumes and capacities listed in the
question options can be directly measured with spirometry. Only FRC and RV
represent amounts of air that remains in the lungs.

Expiratory reserve volume (choice A) is the volume expelled by an active expiratory


effort after passive expiration.
Functional residual capacity (choice B) is defined as the amount of air remaining in
the lungs after passive expiration.
Inspiratory capacity (choice C) is the maximal amount of air inspired after a passive
expiration.
Inspiratory reserve volume (choice D) is the amount of air inspired with a maximal
inspiratory effort over and above the tidal volume.
Tidal volume (choice F) is the amount of air that is inspired (or expired) with each
normal breath.
Vital capacity (choice G) is the largest amount of air that can be expired after a
maximal inspiratory effort.
________________________________________
44The correct answer is E. Multicystic kidneys, slowly progressive renal failure, and
a positive family history are characteristics of autosomal dominant (adult) polycystic
kidney disease (APKD). This disease typically presents in the 40s to 60s and is
characterized by marked renal enlargement due to numerous fluid-filled cysts,
which develop between the normally functioning nephrons. APKD is highly

associated with hepatic cysts, and berry aneurysms in the circle of Willis that may
rupture, producing spontaneous subarachnoid hemorrhage. Infrequently, APKD also
produces cysts in the pancreas, spleen, or lungs, but these are not clinically
relevant.
Hepatic cysts in adult polycystic disease do not ordinarily produce symptoms of
hepatic failure (choice A).
Pancreatic cyst formation in APKD is not generally associated with pancreatic
insufficiency (choice B).
Children with autosomal recessive polycystic kidney disease may develop
congenital hepatic fibrosis with hypertension and splenomegaly, but this is not part
of APKD (choice C).
APKD is not considered a risk factor for renal cell carcinoma or any other type of
cancer (choice D).
________________________________________
45The correct answer is D. The electrocardiogram strongly suggests that this
patient is suffering from hyperkalemia. Hyperkalemia is a potentially life-threatening
complication of succinylcholine. Because succinylcholine is a depolarizing skeletal
muscle relaxant, during prolonged muscle depolarization, the muscle can release
substantial amounts of K+. Patients with burns, extensive soft tissue injuries, spinal
cord injury, or muscular dystrophies are at great risk of succinylcholine-induced
hyperkalemia, and succinylcholine should either be avoided or used with extreme
caution.
Atracurium (choice A) and tubocurarine (choice E) are nondepolarizing skeletal
muscle relaxants, and do not increase the risk of hyperkalemia.
Baclofen (choice B) is a spasmolytic, and would not be used in a surgical setting.
Cyclobenzaprine (choice C) is used to relieve acute temporary muscle spasm that is
caused by strain or local trauma. It would never be used as a skeletal muscle
relaxant in a surgical setting.
________________________________________
46The correct answer is B. Diverticula that become impacted with fecaliths
(undigested food residues) can become inflamed and perforate, causing the
equivalent of appendicitis, but typically involve the left rather than right lower
quadrant. This complication of diverticulitis can produce acute peritonitis, evidenced
by the abdominal tenderness, guarding, and leukocytosis, and can kill elderly
debilitated patients. Diagnostic paracentesis may be valuable in determining if
perforation has occurred.
Bladder infection (choice A), usually related to an enlarged prostate, is common in
elderly men, but does not usually cause peritonitis.

Prostatic inflammation (choice C) or infection does not usually spread to the


abdomen.
Acute appendicitis (choice D) is usually a disease of young adults and is
characterized by tenderness in the right lower quadrant.
Ureteral stones (choice E) can cause severe abdominal pain but do not usually
cause peritonitis.
________________________________________
47The correct answer is D. Seborrheic keratoses, as described in the question stem,
are very common lesions of the skin of middle-aged and older individuals. These
benign growths histologically show hyperplasia of the epidermis. While cosmetically
disturbing to some patients, the primary medical concern is that, occasionally,
seborrheic keratoses may mimic the clinical appearance of basal cell carcinomas or
squamous cell carcinomas. Consequently, biopsy should be performed on atypical
appearing or rapidly changing "seborrheic keratoses" to exclude the presence of
cancer.
Chronic eczema (choice A) produces dry, thick, and sometimes discolored skin.
Melanomas (choice B) characteristically look like dark moles with irregular margins
and variations in the degree of pigmentation.
Psoriasis (choice C) produces erythematous plaques with a silvery scale.
Verruca vulgaris (choice E), the common wart, produces verrucous papules that are
most commonly found on the hands. The face and back would be unusual si
________________________________________
48The correct answer is E. The findings described are those of tabes dorsalis, a form
of tertiary syphilis caused by Treponema pallidum. Tabes dorsalis and other forms of
tertiary syphilis are now uncommon in this country, possibly because the common
use of antibiotics may "treat" many unsuspected cases of syphilis.
Haemophilus influenzae (choice A) and Neisseria meningitidis (choice D) can cause
meningitis.
Herpes simplex I (choice B) can cause an encephalitis that typically involves the
frontal and temporal lobes.
Neisseria gonorrhoeae (choice C) causes gonorrhea, which usually does not involve
the CNS.
________________________________________
49The correct answer E. This is something of a trick question, but it is included to
illustrate a specific point of which you should be aware. The biochemistries of
neonates, especially in the first minutes to hours of life, can be strikingly different
from those of adults. Specifically, the normal range of arterial blood pH at birth is

7.11 to 7.36; the normal range of PO2 at birth is the strikingly low 8 to 24 mm Hg;
and the normal range of PCO2 at birth is the low 27 to 40 mm Hg.
PCO2 (choice A) is normally lower for infants than adults.
pH (choices B and D) may vary over the first few hours of life in infants who
subsequently do well from 7.09 to 7.50 (a range broader than reported "normals" in
the question stem, but still not necessarily clinically significant).
PO2 (choice C) can be very low at birth but comes up during the first day of life,
when it may still normally be as low as 54 mm Hg.
________________________________________
50The correct answer is D. You should recognize this as a probable case of multiple
endocrine neoplasia, specifically, MEN III (formerly MEN II b). Features of this
autosomal dominant condition include medullary carcinoma of the thyroid,
pheochromocytoma, and oral and intestinal ganglioneuromatosis (including mucosal
neuromas).
Gastrinomas (choice A) and insulinomas (choice B) are found in MEN I.
Parathyroid adenomas (choice C) are found in MEN I and II.
Pituitary adenomas (choice E) are found in MEN I.
Answers

________________________________________

1The correct answer is E. The patient has pneumococcal pneumonia. In many


bacterial pneumonias, alveoli in large areas of the lungs fill with viscous fluid
containing proteinaceous debris and many neutrophils. This filling limits the rate at
which oxygen can diffuse into the capillary bed, and in many filled alveoli, may even
completely block oxygen diffusion into the bloodstream.
Inadequate ventilation (choice B) is not initially as important as poor diffusion.
Changes in perfusion (choice A), airway resistance (choice C), and lung compliance
(choice D) usually play lesser roles, although a perfusion/ventilation mismatch may
also develop as
blood is shunted through poorly ventilated lung tissue

________________________________________
2 The correct answer is B. Total peripheral resistance (TPR) is equal to the pressure
gradient across the circulation (mean arterial pressure - right atrial pressure)
divided by the cardiac output. Right atrial pressure is assumed to equal 0 mm Hg.

Thus, TPR = 100/4000 = 0.025 mm Hg/mL/min. The "ABC rule" is useful in


remembering the relation between pressure (P), flow (Q), and resistance (R)
because P=QR (in alphabetical order). Note that choices A, C, D, and E can be
eliminated quickly because in each case the units are incorrect.
________________________________________
3The correct answer is B. The baby boy has Hirschsprung's disease (more common
in males), a diagnosis that should spring to mind in an infant who fails to pass
meconium soon after birth and presents with chronic constipation. Diagnosis is
made most easily and most reliably by anal manometry and/or rectal biopsy. On
manometry, internal anal sphincter pressure increases upon rectal balloon
distention in patients with Hirschsprung's disease (normally, pressure decreases
with distention). Rectal biopsy reveals an absence of ganglion cells (which are
derived from neural crest cells) in a portion of the colonic wall. Barium enema would
show a transition zone between the aganglionic area of bowel (narrow) and the
region of normal bowel proximal to it (dilated).
Defective recanalization of the colon (choice A) results in rectal atresia. In this
condition, there is no communication between the rectum and anal canal.
Therefore, neither meconium nor stool can be passed.
Herniation of abdominal contents into the umbilical cord (choice C) describes a
congenital omphalocele. This occurs due to the failure of all or part of the intestines
to return into the abdominal cavity during the tenth week of gestation.
Persistence of the proximal end of the yolk stalk (choice D) results in Meckel's
diverticulum, an ileal outpouching that is more prevalent in males. It usually
contains ectopic gastric mucosa, which can cause ulceration and bleeding.
Inflammation of the diverticulum may produce symptoms and signs similar to
appendicitis.
The presence of a rectourinary fistula (choice E) would result in the passage of
meconium, stool, and gas into the urine, a diagnostic finding.
________________________________________

4The correct answer is C. The 45,XO karyotype results in Turner's syndrome, which
is characterized by ovarian dysgenesis and a variety of somatic abnormalities
including micrognathia, a fish-like mouth, a shield chest, low-set ears, ptosis, and a
webbed neck. Other findings can include coarctation of the aorta, hypertension, and
renal abnormalities. Short stature is invariably present; the cause is not known
because plasma levels of growth hormone (choice A) and thyroid hormone (choice
B) are typically not decreased. Clinical studies have shown, however, that injections
of human growth hormone can increase the final height. The ovaries are usually
streak-like and exhibit only fibrous stroma. The ovarian dysgenesis leads to
decreased secretion of estrogen (not increased, choice E) and inhibin (not
increased, choice D) and persistent infantilism. Plasma levels of FSH are markedly
increased due to the lack of feedback inhibition by ovarian secretions. FSH levels

are high during infancy and again after 9-10 years of age. Combination estrogen
and progesterone replacement therapy can induce the development of secondary
sex characteristics and menses.
________________________________________
5The correct answer is A. Pancreatic calcifications constitute strong radiologic
evidence of chronic pancreatitis. Up to 30% of patients with chronic pancreatitis
eventually develop secondary diabetes mellitus as a consequence of destruction of
islets in addition to pancreatic ducts. This form of diabetes mellitus may also
eventually cause small vessel damage and blindness.
Neither hyperthyroidism (choice B) nor hypothyroidism (choice C) are related to
chronic pancreatitis.
Pheochromocytoma (choice D) is unrelated to chronic pancreatitis.
Chronic pancreatitis does predispose for pancreatic carcinoma, but not for the
gastrinomas that cause Zollinger-Ellison syndrome (choice E).
________________________________________
6The correct answer is C. The patient has Zollinger-Ellison syndrome, in which a
duodenal or pancreatic gastrin-secreting endocrine tumor causes hypersecretion of
gastric acid. Two-thirds of these tumors are malignant. One third are related to MEN
I, which also causes parathyroid hyperplasia/adenomas and pituitary adenomas. If
MEN I is present, multiple duodenal gastrin-secreting microadenomas may be seen.
H2 blockers and proton pump inhibitors may provide symptomatic relief. Definitive
treatment consists of surgical removal of the gastrinoma, when feasible.
Calcitonin (choice A) is usually secreted by medullary carcinoma of the thyroid;
rarely, calcitonin gene-related peptide (CGRP) is secreted by pancreatic endocrine
tumors.
Epinephrine (choice B) is secreted by pheochromocytoma.
Pancreatic endocrine tumors may also secrete vasoactive intestinal peptide (VIP,
choice E), parathyroid hormone (choice D), parathyroid hormone-related peptide,
insulin, glucagon, somatostatin, ACTH, or growth hormone-releasing hormone
(GHRH). Secretion of these hormones is less common than secretion of gastrin, and
is not specifically suggested by the peptic ulcer history.
________________________________________
7The correct answer is D. The sequence is Potter's syndrome, in which bilateral
renal agenesis leads to oligohydramnios (an abnormally low amount of amniotic
fluid) because the kidneys do not produce normal amounts of urine, which
contributes to amniotic fluid. Oligohydramnios causes uterine compression of the
fetus, which in turn causes limb deformities, breech position, and pulmonary
hypoplasia.

Duodenal (choice A) and esophageal (choice B) atresia can cause polyhydramnios,


rather than oligohydramnios, since the amniotic fluid cannot be swallowed, but
continues to be produced by the kidneys.
The heart (choice C) is not directly involved in Potter's syndrome, although major
malformations of the heart may accompany major malformations of the kidneys.
The lungs (choice E) become secondarily hypoplastic as a consequence of
inadequate amniotic fluid, rather than as a primary malformation.
________________________________________
8The correct answer is C. The kidney normally produces erythropoietin, which
stimulates the erythroid cell line in the bone marrow. Renal cell carcinoma can lead
to overproduction of erythropoietin and thereby cause secondary polycythemia.
Other causes of secondary polycythemia are diseases that impair oxygenation,
including pulmonary diseases (including smoking) and congestive heart failure.
Breast cancer (choice A) can present as an axillary mass or as Paget's disease of the
nipple.
Colon cancer (choice B) and stomach cancer (choice D) can present with anemia
secondary to blood loss.
Thyroid cancer (choice E) can present with hypothyroidism.
________________________________________
9The correct answer is A. Diffuse cortical necrosis, as described in this patient, is
usually seen in the setting of disseminated intravascular coagulation, typically in
the context of overwhelming sepsis. It can also be seen following hypotension
combined with vasoconstriction.
Multiple myeloma (choice B) is associated with renal deposition of amyloid protein
and damage to both glomeruli and tubules.
Adult polycystic kidney disease (choice C) would produce enlarged kidneys filled
with cystic masses.
Pyelonephritis (choice D) would produce inflammation, often most severe in the
renal pelvis.
Sickle cell anemia (choice E) usually affects the medulla most severely, and can
cause papillary necrosis.
________________________________________
10The correct answer is A. The patient has Marfan syndrome, an autosomal
dominant disorder caused by a defect in the gene on chromosome 15 encoding
fibrillin, a 350 kD glycoprotein. Fibrillin is a major component of elastin-associated
microfibrils, which are common in large blood vessels and the suspensory ligaments
of the lens. Abnormal fibrillin predisposes for cystic medial necrosis of the aorta,
which may be complicated by aortic dissection. Other features of the syndrome are

subluxated lens of the eye, mitral valve prolapse, and a shortened life span (often
due to aortic rupture).
________________________________________
11The correct answer is D. The classic Reed-Sternberg (RS) cell of Hodgkin's disease
and its variants, known as Hodgkin's cells, must be identified before the diagnosis of
Hodgkin's disease can be made. These cells are the neoplastic cells of the disease,
and although many theories have been postulated, the cell of origin remains
unknown. The RS cell and RS variants express CD30, a lymphoid activation antigen.
CD15, a granulocyte antigen, is also uniformly positive in RS cells.
CD4 (choice A) is expressed by T helper/inducer cells. This antigen is the receptor
for the HIV virus.
CD8 (choice B) is expressed by T cytotoxic/suppressor cells, and by a few NK
(natural killer) cells.
CD20 (choice C) is a B cell antigen expressed on mature B lymphocytes. It is
positive in most B cell leukemias and lymphomas. It is negative in RS cells because
they are neither T nor B lymphocytes.
CD43 (choice E) is expressed on most types of T cells, granulocytes, and plasma
cells, but not on normal B cells. It can help to differentiate certain types of B cell
lymphomas from benign reactive entities.
________________________________________
12The correct answer is D. Pepsin is secreted (in an inactive or zymogen form as
pepsinogen) by the chief cells of the stomach. Pepsinogen is activated by contact
with stomach acid. Although protein digestion usually begins with the actions of
hydrochloric acid and pepsin, pancreatic enzymes complete the job as the food
passes into the small intestine.
Amylases (choice A) hydrolyze 1->4 glycosidic linkages of starches to produce
oligosaccharides, maltose, maltotriose, and limit dextrins. These enzymes are
produced by the pancreas and salivary glands.
Chymotrypsin (choice B) is a proteolytic enzyme released by the pancreas as the
inactive proenzyme, chymotrypsinogen.
Lipases (choice C) are mostly released by the pancreas, and serve to digest various
lipids, including triacylglycerols.
Trypsin (choice E) is a proteolytic enzyme released by the pancreas as the inactive
proenzyme, trypsinogen.
________________________________________
13The correct answer is E. The patient has developed Guillain-Barr syndrome, also
known as inflammatory polyneuropathy. This presentation is classic-rapidly evolving
limb weakness with symmetrical hyporeflexia but normal sensation. The syndrome
frequently follows viral infections and may evolve into complete paralysis with

respiratory failure. Guillain-Barr syndrome is thought to be an autoimmune


disease. The clinical course is correlated with a chronic inflammatory infiltrate and
demyelination of peripheral nerves, especially spinal and cranial motor nerve roots.
Inflammation localized to a small portion of spinal cord (lateral corticospinal tracts;
choice A) or cerebral cortex (precentral gyrus; choice C) may occur in progressive
multifocal leukoencephalopathy (PML) or in multiple sclerosis (MS). PML occurs in
the immunosuppressed, and MS presents with hyperreflexia (upper motor neuron
signs).
The classic autoimmune disease involving the neuromuscular junction (choice B) is
myasthenia gravis. Although the weakness caused by myasthenia gravis may affect
the legs, extraocular muscles are involved in the majority of cases, and isolated
limb weakness is rare.
Diseases primarily affecting the skeletal muscle (choice D) include autoimmune
inflammatory myopathies such as dermatomyositis and polymyositis, which
typically affect the proximal limb muscles more than the distal musculature.
Inclusion body myositis is a slowly developing disease that is asymmetrical and
occurs in older individuals.
________________________________________
14The correct answer is C. The spinal accessory nerve (cranial nerve XI) is a motor
nerve (special visceral efferent) that innervates the sternocleidomastoid and
trapezius muscles. The trapezius, a broad, flat, triangular muscle, has fibers in its
superior part that originate from the external occipital protuberance and the
superior nuchal line, the ligamentum nuchae, and the spinous processes of
vertebrae C-7 through T-4. These upper fibers pass laterally to insert onto the
acromion and the spine of the scapula. Activation of this part of the muscle results
in elevation of the point of the shoulder (acromion moves superiorly), as in
shrugging.
The muscles that promote movement of the arm away from the midline of the body
(abduction, choice A) include the deltoid, subscapularis, supraspinatus, and
infraspinatus muscles. These muscles are innervated by branches of the brachial
plexus, not the spinal accessory nerve (CN XI).
The trapezius adducts the scapula, whereas the major arm adductors (choice B)
include the pectoralis major, teres major, latissimus dorsi, and coracobrachialis
muscles. These muscles are innervated by branches of the brachial plexus.
The muscles that act to laterally rotate the arm (choice D) include the teres minor,
infraspinatus, supraspinatus, and the dorsal portion of the deltoid muscles. These
muscles are innervated by branches of the brachial plexus, not the spinal accessory
nerve (CN XI).
The medial rotators of the arm (choice E) include the subscapularis, the ventral part
of the deltoid, the latissimus dorsi, and the teres major muscles, all of which are
innervated by branches of the brachial plexus.

________________________________________

15The correct answer is C. The nephrotic syndrome describes a group of laboratory


findings associated with glomerular diseases which share the common
characteristic of "leaky glomeruli." Large biochemicals, normally unable to cross out
of glomerular capillaries into Bowman's space, are lost into the urine. Serum
proteins are lost in large quantities and can be detected as both hypoproteinemia
and massive proteinuria. Albumin (choice A), a relatively small plasma protein (MW
66,000) is lost very readily, leading to hypoalbuminemia. Serum concentrations of
small compounds such as potassium (choice E) and glucose (choice D), which are
highly permeable in the normal glomerulus, are unaffected by glomerulonephritides
producing the nephrotic syndrome. Similarly, blood urea nitrogen is unaffected, and
serum ammonia levels (choice B) are unchanged. The final component of the
nephrotic syndrome (besides hypoproteinemia, hypoalbuminemia and massive
proteinuria) is hyperlipidemia. It is apparently a function of bothincreased hepatic
fat synthesis and decreased fat catabolism. Increased cholesterol (choice C),
triglycerides, and lipoproteins are found in serum in membranous
glomerulonephritis, and these lipids leak into the urine, producing lipiduria.
________________________________________
16The correct answer is D. The clues suggesting infection with Pseudomonas
aeruginosa are the green-tinged sputum (due to the formation of blue and green
pigments by P. aeruginosa), and the association with cystic fibrosis. Unfortunately, P.
aeruginosa is very difficult to treat with most antibiotics, and presents a very
difficult therapeutic challenge in the management of cystic fibrosis patients.
Klebsiella pneumoniae (choice A) is most often seen in alcoholics, the elderly, and
diabetics.
Mycoplasma pneumoniae (choice B) is a common cause of community-acquired
atypical pneumonia, and is characterized by a dry, non-productive cough.
Pneumocystic carinii (choice C) usually causes pneumonia in immunocompromised
patients, e.g., AIDS patients.
Streptococcus pneumoniae (choice E) is an important cause of typical communityacquired lobar pneumonia.
________________________________________
17The correct answer is D. The clues suggesting infection with Pseudomonas
aeruginosa are the green-tinged sputum (due to the formation of blue and green
pigments by P. aeruginosa), and the association with cystic fibrosis. Unfortunately, P.
aeruginosa is very difficult to treat with most antibiotics, and presents a very
difficult therapeutic challenge in the management of cystic fibrosis patients.
Klebsiella pneumoniae (choice A) is most often seen in alcoholics, the elderly, and
diabetics.

Mycoplasma pneumoniae (choice B) is a common cause of community-acquired


atypical pneumonia, and is characterized by a dry, non-productive cough.
Pneumocystic carinii (choice C) usually causes pneumonia in immunocompromised
patients, e.g., AIDS patients.
Streptococcus pneumoniae (choice E) is an important cause of typical communityacquired lobar pneumonia.
________________________________________
18The correct answer is A. The patient is suffering from suppurative (bacterial)
cholangitis, which can occur as a complication of infestation by the roundworm,
Ascaris lumbricoides and by the liver flukes, Clonorchis sinensis and Fasciola
hepatica. Biliary tract obstruction produces jaundice and extreme itching, with dark
urine and pale feces. Therapy typically includes emergency endoscopic
sphincterectomy to improve biliary drainage, antibiotics, and anthelminthic agents.
Enterobius vermicularis (choice B) causes pinworm infections.
Plasmodium ovale (choice C) causes malaria.
Taenia solium (choice D) is the pork tapeworm. Adult tapeworms cause taeniasis,
while the larvae are responsible for cysticercosis.
Trypanosoma cruzi (choice E) causes Chagas' disease.
________________________________________
19The correct answer is E. In order to analyze this data, you must look for the
chromosome that matches exactly with the presence of the HexA gene. Hybrid I has
HexA activity and chromosomes 1, 5, 7, 8, 9, 10, 12, 14, 15, 18, 19, 20, 22, and the
X chromosome. Hybrid III also has HexA activity and chromosomes 4, 6, 8, 11, 12,
13, 15, 16, 21, 22, and X. Doing this for all of the hybrids that have HexA activity,
you can determine that only chromosome 15 appears in all of the hybrids that have
the enzyme activity. Therefore, HexA must be found on chromosome 15.
________________________________________
20The correct answer is E. The sequence of the auditory pathway is as follows:
Organ of Corti spiral ganglion in the cochlea vestibulocochlear nerve (CN VIII)
cochlear nuclei (dorsal and ventral) superior olivary nuclei lateral lemniscus
inferior colliculus medial geniculate nucleus of the thalamus (MGN) primary
auditory cortex (Heschl's gyrus).
Each ear projects to both sides of the brainstem and cortex via multiple
commissures, including the trapezoid body (which contains fibers crossing
contralateral to the superior olivary nucleus), the commissure of the inferior
colliculus (connecting the right and left inferior colliculi) and another commissure
that connects the right and left nuclei of the lateral lemniscus. Therefore, a lesion of
any structure up until the superior olivary nuclei will produce an ipsilateral deafness.

The only structure listed that is proximal to the superior olivary nuclei is the organ
of Corti (choice E).
The inferior colliculus (choice A), the lateral lemniscus (choice B), the medial
geniculate body (choice C), and the superior olivary nucleus (choice F) all receive
information from both ears, and unilateral hearing loss could not result from a lesion
of any of these structures.
The medial lemniscus (choice D) is not a part of the auditory system. It is part of the
somatosensory system, which conveys proprioception, discriminative touch, and
vibration information. More specifically, neurons of the gracile and cuneate nuclei
send projections that decussate as the internal arcuate fibers and ascend as the
medial lemniscus to synapse in the ventroposterolateral nucleus (VPL) of the
thalamus.
________________________________________
21The correct answer is B. The lesions are acanthosis nigricans, which looks
somewhat like a mole or wart, but is actually due to epidermal hyperplasia.
Acanthosis nigricans can be seen in obesity, diabetes, and in patients with
underlying cancers, often adenocarcinomas of the chest or abdomen.
The lesions are not characteristic of immunosuppression (choice A) and are not
easily superinfected (choice C). They are also neither malignant (choice D) nor
premalignant (choice E).
________________________________________
22The correct answer is B. Volume = amount/concentration.
The amount of mannitol in the volunteer is equal to the amount injected minus the
amount excreted: 10 g - 1 g = 9 g = 9000 mg. Therefore,

________________________________________
23The correct answer is E. Deletions are routinely detected by two methods:
Southern blotting (using a probe from either a cDNA or genomic library) or
polymerase chain reaction (PCR) using primers specific for the gene. Both of these
techniques will show size changes that will allow the investigator to determine the
size of the deletion. Deletions may sometimes be detected by using restriction
enzymes; for example, the loss of a normally present restriction site may indicate
deletion of the DNA in the vicinity of that site.
A cDNA library (choice A) is synthesized from the mRNA of a cell using the enzyme
reverse transcriptase. It is used to study the DNA and mRNA from which proteins
are transcribed.

FACS (fluorescence-activated cell sorting; choice B) uses antibodies coupled to


fluorescent markers to determine cell surface molecules of whole cells. It is very
useful for determining the stage of development or activation of cells, but does not
tell you anything about the DNA.
A genomic library (choice C) is the chromosomal sequence of a gene, including its
coding and non-coding regions. It is synthesized directly from the DNA of a cell and
is used to study both the coding regions and the areas such as promoters,
enhancers, and introns of a gene.
Plasmids (choice D) are extrachromosomal, independently replicating circular pieces
of DNA that are found in bacteria. They often contain antibiotic resistance genes.
The are used in molecular biology to insert genes into bacteria for expression.
________________________________________
24The correct answer is B. A serious side effect of the antipsychotics is tardive
dyskinesia, which has been seen with virtually every neuroleptic [e.g.,
chlorpromazine (choice A), fluphenazine (choice C), haloperidol (choice D), and
thioridazine (choice F)]. Usually, the symptoms of tardive dyskinesia appear late in
treatment and consist of involuntary, repetitive movements of the lips and tongue
(e.g., tongue thrusting, lip smacking), and, not infrequently, of the extremities and
trunk. Patients over 60 and those with pre-existing CNS pathology are at a higher
risk for this disorder (up to 70%), but other risk factors have not been confirmed.
Clozapine is called an atypical antipsychotic medication because of its lack of
extrapyramidal side effects, including tardive dyskinesia, and would be an
appropriate medication for a patient who is developing tardive dyskinesia.
Metoclopramide (choice E) is a centrally acting antiemetic that has been shown to
cause tardive dyskinesia as well.
________________________________________
25The correct answer is E. Superior vena cava (SVC) syndrome is characterized by
obstruction of venous return from the head, neck, and upper extremities. Over 85%
of cases of SVC syndrome are related to malignancy. Bronchogenic carcinomas
(most commonly small cell cancer and squamous cell cancer) account for over 80%
of these cases. Among bronchogenic carcinomas, the most common causes of SVC
syndrome (in order of frequency) are small-cell carcinoma, epidermoid carcinoma,
adenocarcinoma (choice A), and large-cell carcinoma (choice C). Lymphomas such
as Hodgkin's disease (choice B) and non-Hodgkin's lymphoma are uncommon
causes of SVC syndrome. Rare tumors associated with SVC syndrome include
primary leiomyosarcomas and plasmacytomas. Infectious etiologies include
tuberculosis, syphilis, and histoplasmosis. SVC syndrome can also occur as a result
of an enlarged goiter, and from thrombus formation caused by indwelling
intravenous lines or pacemaker wires.
Non-small cell carcinoma (choice D) is not commonly associated with SVC
syndrome.
________________________________________

26The correct answer is E. Red blood cells (and many other blood cells) contain the
enzyme carbonic anhydrase, which catalyzes the intracellular conversion of CO2 to
bicarbonate and H+ ion. Most of the bicarbonate in the red cell is exchanged across
the plasmalemma for chloride ion. This means that while the bulk of the production
of bicarbonate occurs in the red cell (choice D), the bulk of the actual transport
occurs in serum as bicarbonate. Carbonic anhydrase is not present in serum.
Bicarbonate can also be produced in serum by nonenzymatic means, but the
process is slow.
CO2 is also carried as carbaminohemoglobin (choice A), which forms when CO2
binds to an NH2 side group of the hemoglobin protein, rather than to the heme iron
(Fe2+), as with carbon monoxide and oxygen.
CO2 is not transported in the form of bubbles (choice B), which is a good thing,
because gas bubbles are effectively emboli, which can lead to considerable
morbidity or death.
Some CO2 is carried directly dissolved in blood (choice C). It is 20 times more
soluble in blood than is O2.
________________________________________
27The correct answer is A. The suspected disease is osteogenesis imperfecta, which
is a rare genetic disorder that occurs in both recessive and dominant forms. The
clinical presentation, depending on the specific form, varies from death in utero, to
that described in the question stem, to very mild disease with only a modest
increase in bone fragility. The different types all have defects in the synthesis of
type I collagen, often with insufficient or abnormal pro-a1(1) or pro-a2(1) chains.
These deficits produce an unstable collagen triple helix that is not as strong as
normal collagen.
Defective glycogen (choice B) metabolism is associated with the various glycogen
storage diseases, such as von Gierke disease and Pompe disease. These diseases
tend to present with profound hypoglycemia, hepatomegaly, or muscle weakness.
Defective mucopolysaccharide (choice C) metabolism is associated with the
mucopolysaccharidoses, such as Hurler and Hunter syndromes. These diseases tend
to present with abnormal facies ("gargoylism"), deformed ("gibbus") back, claw
hand, and stiff joints.
Abnormalities of purine metabolism (choice D) are present in gout, which presents
with joint inflammation and often involves the great toe.
Abnormalities of tyrosine metabolism (choice E) are associated with phenylketonuria
(pale hair and skin, mental retardation, musty smelling urine), albinism (pale hair,
skin, increased skin cancer), cretinism (decreased T3 and T4), tyrosinosis (liver and
kidney disease), and alkaptonuria (chronic arthritis and urine that turns black upon
standing).
________________________________________

28The correct answer is D. The patient has SIADH (syndrome of inappropriate


antidiuretic hormone secretion), which can be caused by ectopic ADH secretion by
small cell carcinomas of the lung, CNS disorders, chronic pulmonary disease, and
certain drugs. Features of SIADH include excessive water retention, hyponatremia
(which can lead to seizures when severe), and serum hypo-osmolarity with urine
osmolarity greater than serum osmolarity.
Associate adenocarcinoma (choice A) with peripheral lung cancer and lung cancer
arising in scars.
Associate bronchioloalveolar carcinoma (choice B) with alveolar-like spaces and no
link to smoking.
Large cell carcinoma (choice C) is an aggressive, undifferentiated lung neoplasm.
Associate squamous cell carcinoma (choice E) with bronchogenic origin, a strong
association with smoking, and hypercalcemia.
________________________________________
29The correct answer is E. The patient in the question stem has X-linked (Bruton's)
agammaglobulinemia, which is now thought to be due to a deficiency in a tyrosine
kinase, leading to a B cell maturation arrest at the pre-B cell level.
Selective IgA deficiency has been linked to defective class III MHC genes (choice B).
Severe combined immunodeficiency is apparently a heterogeneous disease, and
different subgroups have been linked to abnormalities of adenosine deaminase
(choice A), the gamma chain of the IL-2 receptor (choice C), and purine nucleotide
phosphorylase (choice D).
________________________________________
30The correct answer is C. This fact is worth remembering: one-fifth of Down's
syndrome patients have congenital cardiovascular disease, most commonly an
ostium primum type of ASD and/or a ventricular septal defect. This is a sufficiently
high incidence to justify at least one echocardiogram in each of these children's
lives. Affected children are also particularly vulnerable to seizures, and as adults
may develop an Alzheimer-like dementia in their mid 40s.
________________________________________
31The correct answer is D. Spongiotic dermatitis is seen in cases of contact
dermatitis, such as poison ivy exposure. The accumulation of inflammatory cells in
the superficial dermis causes marked edema, which splays epidermal keratinocytes
apart and giving a spongy appearance to intercellular bridges. Grossly, the skin has
a weepy appearance with frequent blistering.
Erythema nodosum (choice A) is a form of panniculitis, which is chronic
inflammation in the subcutaneous fat lobules. Erythema nodosum presents as
painful erythematous nodules, often with fever and malaise. It is associated with
infections and drug reactions, and is not a contact dermatitis.

Pemphigus (choice B) is a genetic blistering disorder due to the production of


antibodies to the intercellular cement substances in skin and mucous membranes.
Psoriasis (choice C) is a common chronic inflammatory disease causing plaques and
scales, typically on elbows, knees, and scalp. The pathogenesis of psoriasis is still
unclear; it may be a complement-mediated autoimmune process.
Urticaria (choice E) is an IgE-driven hypersensitivity process, usually occurring in
adults between 20 and 40 years old. Urticaria is characterized by wheals
(edematous pruritic plaques), and typically affects the trunk and distal extremities.
________________________________________
32The correct answer is E. Laron dwarfism is due to a congenital absence of growth
hormone receptors. Growth hormone binding protein (GHBP), which circulates in the
blood, is identical to the extracellular portion of the growth hormone receptor.
Plasma levels of GHBP are directly related to tissue expression of the growth
hormone receptor. The absence of GHBP in the blood of this child confirms the
absence of tissue growth hormone receptor. In Laron dwarfism, plasma
concentrations of IGF-1 are low because there is no growth hormone receptor on
hepatocytes, which secrete most of the blood-borne IGF-1 in response to growth
hormone. Plasma growth hormone concentration is increased because of loss of
negative feedback effects of growth hormone on the hypothalamus to increase the
release of somatostatin into the hypophyseal portal blood. In addition, the negative
feedback effects of IGF-1 at the hypothalamus and pituitary are diminished because
of the low blood levels of this growth factor.
With constitutional short stature (choice A), growth hormone is not decreased.
Rather, this disorder is considered a delay in the normal velocity of growth. Other
family members often have a similar decrease in growth rate. The pubertal growth
spurt is also delayed. Adult height is usually normal to low-normal.
Genetic short stature (choice B) is unlikely in this child because neither of the
parents is short.
Dwarfing also occurs with growth hormone deficiency due to either hypothalamic
(choice C) or pituitary (choice D) dysfunction. However, both of these defects would
be characterized by decreased plasma growth hormone concentration and
decreased IGF-1 concentration.
________________________________________
33The correct answer is D. This is the classic presentation of a fracture of the neck
of the femur. This type of fracture typically occurs in postmenopausal women with
significant bone resorption due to osteoporosis. Dislocation of the head of the femur
can produce a similar effect. The change in the position of the leg is due to the
action of the gluteal muscles, particularly gluteus maximus.
Fracture of the femur at the greater trochanter (choice A) produces a less dramatic
effect, because only the gluteus medius and gluteus minimus attach to it, and since
the overall structure of the femur remains intact.

Fractures occurring lower in the femur, involving the epicondyles (choices B and C)
or shaft (choice E) would not produce outward rotation of the entire leg.
________________________________________
34The correct answer is E. The patient has lobar pneumococcal pneumonia of the
right middle and right lower lobes. Respiratory distress in lobar pneumonia is
predominately due to inequalities of ventilation and perfusion, since the dilated
vessels of the involved lobes transmit a higher-than-usual percentage of the blood
passing through the lungs at the same time that the alveolar fluid prevents normal
ventilation of the affected areas. Inequalities of ventilation and perfusion also can
contribute to hypoxemia in chronic obstructive pulmonary disease, atelectasis,
pulmonary infarction, tumors, and granulomatous diseases.
Decreased surface area of alveolar capillary membranes (choice A) is seen following
lung resection and in diseases such as emphysema.
Decrease of P02 in inspired air (choice B) is seen at high altitude and during
artificial ventilation if the fractional O2 content setting is incorrect.
Hypoventilation of central origin (choice C) is seen with morphine and barbiturate
overdose.
Hypoventilation of peripheral origin (choice D) is seen with acute poliomyelitis,
chest trauma, suffocation, drowning, phrenic nerve paralysis, and Pickwickian
syndrome.
________________________________________

35The correct answer is E. The laboratory values indicate that there is microcytic
anemia. This form of anemia is associated with decreased mean cell volume of
erythrocytes (microcytosis). By itself, microcytic anemia can be due to a number of
factors, including iron deficiency and disorders of globin synthesis. When anemia is
caused by iron deficiency, serum levels of iron and serum ferritin are decreased,
whereas transferrin (plasma protein acting as iron carrier) is increased. Thus, this
case is due to iron deficiency. Which of the above conditions may lead to iron
deficiency? Uterine leiomyomas. These tumors arise from the smooth muscle of the
uterus and often manifest with pain and abundant blood loss during menstruation.
Any chronic blood loss leads to depletion of reserve iron and then to decreased
synthesis of hemoglobin, ie, anemia. Maturation of erythrocytes is relatively
unimpaired, but reduced synthesis of hemoglobin leads to smaller (microcytic) and
paler (hypochromic) red blood cells. Remember that in Western countries, chronic
blood loss is the most common cause of iron deficiency anemia.
b-Thalassemia is due to mutations leading to a complete block or reduction in the
synthesis of the b-globin chain. b-Thalassemia minor (choice A) is the clinical picture
in patients heterozygous for the b+ or bo gene. They are usually asymptomatic, but
their blood smear shows variable degrees of hypochromasia and microcytosis. This
condition must be distinguished from iron deficiency anemia. Accumulation of iron

in b-thalassemia minor leads to normal or increased plasma iron and ferritin, and
reduced transferrin.
Chloramphenicol toxicity (choice B) may result in severe aplastic anemia. In this
case, inability of the bone marrow to produce erythrocytes manifests with a reduced
number of circulating erythrocytes, which are normochromic and normocytic. The
most common causes of aplastic anemia are drugs.
Chronic autoimmune gastritis (choice C) is due to autoimmune destruction of the
gastric mucosa. Autoantibodies blocking the binding between vitamin B12 and
intrinsic factor are present in 75% of these cases. Impaired vitamin B12 absorption
leads to megaloblastic anemia. Since vitamin B12 is an indispensable cofactor for
DNA synthesis, defective cellular maturation and division occur, with abnormally
large erythroid precursors and circulating erythrocytes that have a mean
corpuscular volume up to 120 mm3. Circulating neutrophils are larger than normal
and hypersegmented.
Many chronic diseases, including rheumatoid arthritis (choice D), may cause a form
of anemia due to defective utilization of iron. This is referred to as anemia of chronic
disease, and it may mimic iron deficiency anemia. However, anemia of chronic
disease is associated with increased iron stores, reflected by elevated serum
ferritin, reduced transferrin, and increased iron deposits in the bone marrow.
________________________________________
36The correct answer is A. The condition is progressive multifocal
leukoencephalopathy, which is a rapidly progressive demyelinating disorder in
which the JC virus (a papovavirus) infects oligodendroglial cells in the brain. The
eosinophilic inclusions represent accumulations of JC virus. PML occurs in about 1%
of AIDS patients, and is the AIDS-defining illness in half of the patients who develop
the condition. There is no effective treatment for this disorder.
Shingles and post-infectious encephalitis can follow chickenpox (choice B), but
varicella is not associated specifically with demyelination.
Measles (choice C) can cause an encephalitis, and in some cases, subacute
sclerosing panencephalitis (SSPE) may follow previous measles infection. These
disorders would affect not only white matter, but gray matter as well. Also, SSPE
usually occurs before the age of 18.
Syphilis (choice D) can cause meningitis, encephalitis, and spinal cord damage, but
the disease process would not be limited to the white matter.
Tuberculosis (choice E) can cause a granulomatous meningitis, typically affecting
the base of the brain.
________________________________________
37The correct answer is E. The path of the psoas major lies in the retroperitoneum
and comes close to the appendix. Acute appendicitis can cause either infection or a
sympathetic inflammation of the psoas. This produces clinically a "positive psoas
sign," in which attempts to straighten the patient's flexed (to relieve pain) hip

produce sometimes marked exacerbation of the pain. None of the other muscles
listed pass near the appendix.
________________________________________
38The correct answer is F. The most common cause of dysfunctional uterine
bleeding is anovulation. This is caused by excessive and prolonged estrogen effect
without the postovulatory progesterone effect. This occurs most often around
menarche and menopause, when subtle hormonal imbalances commonly occur. An
estrogenic stimulation causes the endometrial glands to proliferate. Persistent
proliferation without a progesterone phase will eventually break down and bleed
even though there is no secretory change present and the stroma is not menstrual.
That is why this is called "anovulation bleed," because it is a non-ovulatory bleed
that may seem like normal menstruation.
Asynchronous secretory endometrium (choice A) refers to secretory endometrium
that has a mismatch of 2 or more days between the glands and the stroma. For
example, the glands may be at day 17 while the stroma shows more maturity,
corresponding to day 22. This is a type of dysfunctional ovulatory bleeding and
clinically presents with infertility. It is usually due to an inadequate luteal phase
because the corpus luteum is not producing enough progesterone even though
ovulation has occurred.
Decidualized stroma with inactive glands (choice B) is the common histologic
appearance of patients taking oral contraceptives. These patients have
asynchronous glands and stroma. The glands are usually not active and the stroma
appears to be ready for implantation. This reverts to normal with discontinuation of
oral contraceptives.
Early proliferative endometrium (choice C) and late secretory endometrium (choice
D) are physiologic phases of the normal menstrual cycle.
Menstrual endometrium (choice E) histologically shows stromal and glandular
breakdown with bleeding following the full cycle. It follows the secretory phase and
precedes the proliferative phase.
________________________________________
39The correct answer is B. The condition described is dermatitis herpetiformis,
which is strongly associated with gluten-sensitive enteropathy (celiac sprue), and
often has a life-long, intermittent course. Clinically, patients have (excoriated)
groups of papules and vesicles on an erythematous base. The lesions tend to
involve the extensor surfaces of the extremities and the buttocks. Microscopically,
the lesions show subepidermal papillary dermal neutrophilic abscesses, with
granular deposits of IgA and C3 in dermal papillary tips. Patients may respond
dramatically to dapsone therapy.
Bullous pemphigoid (choice A) produces large, tense blisters.
Herpes simplex I (choice C) is characterized by crops of vesicles, but is not
particularly associated with gluten-sensitive enteropathy.

Herpes simplex II (choice D) produces crops of vesicles, but is not particularly


associated with gluten-sensitive enteropathy.
Pemphigus vulgaris (choice E) usually shows prominent oral involvement.
________________________________________
40The correct answer is B. The seventh intercostal nerve (the anterior ramus of the
seventh thoracic spinal nerve) innervates the seventh rib. After passing through the
intervertebral foramen between the seventh and eighth thoracic vertebrae, the
nerve lies in the seventh intercostal space. After passing the angle of the rib, it
occupies a position along the lower border of the rib, in the costal groove. Use of a
local anesthetic at this point will anesthetize the rib.
By the time the intercostal nerve has reached the midclavicular line (choice A), it
has already innervated most of the rib. Use of an anesthetic at this point would not
be effective.
While the intercostal nerve is in the intercostal space medial to the angle of the rib
(choice C), it is not along the lower border of the rib.
The intercostal nerve does not lie along the upper border of the rib (choices D, E,
and F). Thus, injection at these sites would not be effective.
________________________________________

41The correct answer is D. Klinefelter's syndrome is characterized by an XXY


genotype and is associated with seminiferous tubule dysgenesis. In this disorder,
there are usually few symptoms before puberty, which may be delayed. At puberty,
the seminiferous tubules fail to enlarge normally and instead undergo fibrosis and
hyalinization. The result is an inability to produce sperm. The Leydig cells are
hyperplastic and clumped together, and do not function normally (secretion of
testosterone is decreased). As a consequence, plasma levels of LH are increased
(not decreased, choice C) due to the loss of feedback inhibition. The increased LH
stimulates the Leydig cells to increase estrogen (not decreased, choice A) secretion.
The increased estrogen:testosterone ratio is responsible for the gynecomastia, small
penis, sparse body hair, and other feminized features of Klinefelter's syndrome.
Because of dysgenesis of the seminiferous tubules, the secretion of inhibin is
reduced (not increased, choice E). Low plasma levels of inhibin result in increased
plasma FSH (not decreased, choice B) due to loss of feedback inhibition at the
pituitary.
________________________________________
42The correct answer is C. The cardiac output (CO) is equal to the volume of blood
ejected from the heart during each beat of the heart (SV, stroke volume) multiplied
by the number of times the heart beats each minute (HR, heart rate). That is, CO =
SV x HR. Therefore, SV = CO/HR, and since CO = 5000 mL/min, and HR = 50/min,
SV = 5000/50 = 100 mL.

________________________________________
43The correct answer is E. This is a classic example of an infection with herpes
simplex virus (probably type 2). This agent causes lytic infections in mucoepithelial
cells. It is then retrogradely transported into neurons of the sacral ganglia, where it
lays dormant during the asymptomatic phase of the disease.
Fibroblasts (choice A) may be infected by cytomegalovirus, another herpesvirus, but
this produces a distinctive mononucleosis-like syndrome in most normal individuals.
Lymphocytes (choice B) and macrophages (choice C) may be infected by herpes
simplex type 2 but are not the site of residence of the virus during quiescent
periods.
Infection of mucoepithelial cells (choice D) by herpes simplex produces vesicularlike lesions on the mucous membranes during symptomatic periods.
Neurons of the trigeminal ganglia (choice F) may be latently infected with herpes
simplex type 1. This agent is a possible cause of genital ulcers, but is usually
associated with perioral lesions (cold sores).
________________________________________
44The correct answer is C. The suspected disease is temporal arteritis, also known
as giant cell arteritis, because of the focal granulomatous inflammation (with giant
cells) that characterizes the process. The cranial arteries (branches of the carotid
system) are most often involved. Vigorous attempts at diagnosis, which may require
biopsying 2-cm lengths of each temporal artery, are indicated because the process
can be patchy. Correct diagnosis is essential because untreated temporal arteritis
can involve the ophthalmic artery, leading to irreversible blindness.
Atherosclerosis (choice A) is almost never seen in the distal (lower blood pressure)
part of the temporal artery.
Fibrinoid necrosis (choice B) is a feature of polyarteritis nodosa, not temporal
arteritis.
Fungal hyphae (choice D) would suggest a fungal infection rather than temporal
arteritis.
Leukocytoclastic vasculitis (choice E) is a feature of hypersensitivity angiitis.
________________________________________
45The correct answer is B. This patient is experiencing an acute attack of gout. The
association of gout with alcoholism is well documented. Gout is caused by an
overproduction or underexcretion of uric acid. Precipitation of sodium urate (uric
acid is ionized at body pH) in joint fluid causes an acute inflammatory synovitis with
synovial edema and leukocytic infiltrate. It usually affects the joints of the lower
extremity, most commonly, the large toe. Formation of tophi (urate deposits
surrounded by inflammatory cells, including foreign body giant cells) is
pathognomonic.

Colchicine is an anti-gout drug that binds to microtubule proteins and interferes with
microtubule assembly. In this way, it impairs leukocyte chemotaxis, thus preventing
the migration of granulocytes to the inflammatory site. Colchicine is the drug of
choice for attacks of acute gout. Note that NSAIDs can also be used in the treatment
of acute gout. They are as efficacious as colchicine; however, symptomatic
improvement takes longer.
Allopurinol (choice A) is an anti-gout drug that is indicated for the treatment of
chronic gout. Allopurinol functions to inhibit the enzyme xanthine oxidase, which
results in a decreased production of uric acid from its immediate precursor,
xanthine. It lowers both serum and urinary concentrations of uric acid.
Cholestipol (choice C) is a bile acid sequestrant that is useful in lowering cholesterol
levels in familial hyperlipidemias. It is not used for the treatment of gout.
Pravastatin (choice D) is an HMG-CoA reductase inhibitor useful in the treatment of
familial hyperlipidemias. It is not used for the treatment of gout.
Probenecid (choice E) is indicated for the treatment of chronic gout. Probenecid
inhibits uric acid reabsorption and therefore increases the urinary excretion of uric
acid.
________________________________________

46The correct answer is A. The most common agents causing congenital infection in
the United States are cytomegalovirus, herpes simplex virus, and Treponema
pallidum. Congenital cytomegalovirus infection, which occurs in 1 to 2% of all live
births in the U.S., results from transplacental acquisition of a primary or recurrent
maternal infection. Most infections are asymptomatic, but about 5% will present
with hepatosplenomegaly, periventricular brain calcification, petechial
hemorrhages, and hydrops. Sensorineural deafness is a common manifestation in
affected infants. Primary infection of the mother is strongly associated with more
severe symptoms in affected infants. The mortality rate is 30% and most of the
survivors will have neurologic impairment. CMV infections in children and adults are
often asymptomatic; however, the virus is responsible for 10% of infectious
mononucleosis cases (characterized by an absence of sheep RBC agglutinins). CMV
also causes serious disease in immunosuppressed patients, including bone marrow
recipients and AIDS patients. Ganciclovir is the drug of choice, although foscarnet
has also been used with some success.
Infection with hepatitis viruses (choices B, C, and D), if symptomatic, are
characterized by jaundice, lethargy, failure to thrive, abdominal distention, claycolored stools, and elevated transaminase levels. Hepatitis B(choice B) is the only
viral hepatitis agent that is recognized as an important cause of neonatal disease.
The infection is usually acquired during the birth process; therapy includes
hyperimmune human anti-hepatitis B serum followed a few weeks later with active
immunization with rHBsAg injections.

Herpes simplex (choice E) can also infect the neonate. The hallmark of infection is
painful skin vesicles (present in 50% of affected infants); other manifestations
include encephalitis, pneumonia, hepatitis, and disseminated intravascular
coagulopathy. Massive hepatic and adrenal necrosis is common. Therapy is with
acyclovir.
________________________________________
47The correct answer is D. A Meckel's diverticulum is caused by failure of
obliteration of the vitello-intestinal duct. It is classically located in the distal ileum
within 30 cm of the ileocecal valve, and the structure is a true diverticulum with
mucosa, submucosa, and muscularis propria. Many Meckel's diverticula contain
ectopic pancreatic tissue or gastric mucosa, and the acid production from the
gastric mucosa may be sufficient to produce a small peptic ulcer in adjacent
intestinal mucosa. Such small peptic ulcers are occasional sources of mysterious
appendicitis-like pain or intestinal bleeding. Peptic ulceration adjacent to a Meckel's
diverticulum should be suspected in any child who presents with massive, painless
rectal bleeding. Technetium [99mTc] concentrates in gastric mucosa, and the scan
in this patient demonstrated a small amount of ectopic gastric mucosa located in
the diverticulum.
Acute appendicitis (choice A) is usually very painful and does not typically cause
rectal bleeding.
A lesion of the cecum (choice B) would have been revealed by thorough
colonoscopy.
Failure of upper endoscopy to demonstrate a peptic ulcer of the duodenum (choice
C) makes duodenal disease unlikely.
Theoretically, the jejunum (choice E) could have been the source of the problems,
but jejunal bleeding is uncommon and a Meckel's diverticulum is a much more likely
possibility.
________________________________________
48The correct answer is E. The patient is presenting with signs and symptoms of
drug-induced lupus. This complication is associated with procainamide and other
agents, including hydralazine, chlorpromazine, isoniazid, methyldopa, and
quinidine. Procainamide is a class IA agent, similar in action to quinidine, and is
indicated for the treatment of ventricular arrhythmias. This agent is has also been
associated with agranulocytosis, bone marrow depression, neutropenia, hypoplastic
anemia, and thrombocytopenia.
Digoxin (choice A) is a cardiac glycoside used for congestive heart failure (CHF),
atrial fibrillation, and atrial flutter. Signs and symptoms of digoxin toxicity include
nausea, vomiting, anorexia, appearance of yellow-green halos in the visual field,
and the development of cardiac arrhythmias.
Disopyramide (choice B) is a class IA agent indicated for the treatment of
documented ventricular arrhythmias. It possesses strong anticholinergic effects and

is associated with the development of atrial tachyarrhythmias, heart block, and


conduction abnormalities.
Flecainide (choice C) is a class IC agent indicated for the treatment of lifethreatening ventricular arrhythmias. It is associated with paresthesias, ataxia,
flushing, vertigo, tinnitus, depression, and a worsening of cardiac arrhythmias.
Mexiletine (choice D) is a class IB agent indicated for the treatment of lifethreatening ventricular arrhythmias. It is associated with the development of
palpitations, chest pain, CHF, edema, arrhythmias, tremor, nervousness, blurred
vision, CNS stimulation, and convulsions.
________________________________________
49The correct answer is D. The primer molecule required by DNA polymerase is a
short strand of RNA (4-10 bases) complementary to the template strand of the DNA
molecule. The primer is synthesized by a specific RNA polymerase known as
primase. The growing end of the RNA primer is a free 3'-OH group. The primase
does not itself require a primer for initiation of nucleotide synthesis.
DNA ligase (choice A) catalyzes the formation of a phosphodiester bond between
the 3'-OH of one fragment of DNA and the 5'-monophosphate group of an adjacent
DNA fragment.
Polymerase I (choice B) catalyzes the polymerization of nucleotides and also
functions in processing and repair mechanisms.
Polymerase III (choice C) is a part of a multiprotein complex and is the major
replicating enzyme in E. coli.
Topoisomerases (choice E) produce swivel points in the DNA molecule that relieve
the strain induced by the replication fork. These enzymes cut and reseal the DNA.
________________________________________
50The correct answer is E. The most specific lesion of diabetic glomerulosclerosis
for the purposes of the USMLE is the Kimmelstiel-Wilson nodule. These are ovoid,
hyaline, PAS-positive structures found in the mesangial core at the edge of the
glomerulus. Although this lesion is the most distinctive (pathognomonic) for
diabetes, it is not seen in all renal biopsies from diabetic patients. Other changes
that may be present include glomerular capillary basement membrane thickening,
diffuse glomerulosclerosis, hyaline thickening of arteriolar walls, tubular atrophy,
interstitial fibrosis, and PAS-positive capsular drops in the parietal layer of Bowman
capsule.
Mesangial IgA deposits (choice A) are a feature of Berger disease (IgA nephropathy).
Necrotic epithelial cells in tubules (choice B) are a feature of acute tubular necrosis.
Nests of cells with abundant clear cytoplasm (choice C) are a feature of renal cell
carcinoma.
Numerous neutrophils in tubules (choice D) are a feature of acute pyelonephritis

Answers

________________________________________

1>***
________________________________________
2The correct answer is E. This patient has acute hepatitis B. The "window period"
refers to that period in infection when neither hepatitis B surface antigen (HBsAg)
nor its antibody (HBsAb) can be detected in the serum of the patient. It is an
immunologically mediated phenomenon caused by the precipitation of antigenantibody complexes in their zone of equivalent concentrations and, thereby, their
removal from the circulation. Because of this, the first thing that will happen in the
window period is that the serum will become negative for the surface antigen
(HBsAg), as that antigen is precipitated out of the serum by developing levels of its
specific antibody (HBsAb).
HBcAg is not typically measured (choice A) in the serum.
Levels of the c-core antibody HBc IgG (choice B) and HBeAg (choice C) do not have
a relationship to the window period.
Levels of HBsAb (choice D) will not be detectable until there is antibody excess, and
the patient is leaving the window period.
________________________________________
3The correct answer is C. The patient is suffering from multiple myeloma, a
neoplastic proliferation of plasma cells (or their precursors) found within the bone
marrow. These malignant cells are responsible for the production of excessive
amounts of immunoglobulin (usually IgG or IgA), producing an M- protein spike, and
increasing the ESR. The decreased platelet count probably reflects infiltration of the
bone marrow by myeloma cells. It is not unusual for patients with malignant
myeloma to have recurring bacterial infections, particularly pneumococcal
pneumonia, because the overall production of normal immunoglobulins of all
isotypes is decreased. Lytic bone lesions due to infiltration by myeloma cells may
lead to pathological fractures. All of the other disorders listed can be associated
with an M-protein spike, but this is a nonspecific finding.
Chronic lymphocytic leukemia (CLL; choice A) is ruled out by the normal white
count, since this type of leukemia is generally associated with an absolute
lymphocytosis.

Patients with monoclonal gammopathy of undetermined significance (choice B) are


generally asymptomatic, although they have a predisposition for subsequently
developing myeloma, lymphoma, amyloidosis, or Waldenstrm's
macroglobulinemia.
Plasmacytoma (choice D) (solitary myeloma) is a rare, isolated plasma cell
neoplasm in bone or soft tissues. If the primary cancer is in bone, it is likely to
disseminate; extraosseous tumors tend to remain localized. Since this patient had
multiple lytic bone lesions, multiple myeloma is the correct diagnosis.
Waldenstrm's macroglobulinemia (choice E) is a disorder involving neoplasms of
lymphocytoid plasma cells that produce monoclonal IgM. Hypergammaglobulinemia
produces hyperviscosity of the blood which, along with infiltration by tumor produce
the characteristic signs and symptoms. Weakness, weight loss, bone pain,
hepatosplenomegaly, and lymphadenopathy occur commonly.
________________________________________
4The correct answer is B. Cocaine causes contraction band necrosis by blocking the
reuptake of norepinephrine, resulting in excessive vasoconstriction of coronary
vessels, leading to ischemia and infarction of heart tissue. Under these pathological
conditions, myocardial cells switch to anaerobic metabolism and therefore glycolysis
becomes the sole source of ATP via substrate-level phosphorylations by
phosphoglycerate kinase and pyruvate kinase. Phosphofructokinase-1 (PFK-1) is the
rate-limiting enzyme of glycolysis, and its activity would therefore be increased.
Phosphoenolpyruvate carboxykinase (choice A) is a regulatory enzyme in
gluconeogenesis, which is induced by cortisol, epinephrine, and glucagon. It
functions in the hepatic synthesis of glucose when energy levels from betaoxidation of fatty acids are adequate.
Pyruvate dehydrogenase (choice C) produces acetyl-CoA from pyruvate and
coenzyme A, bridging glycolysis and the Krebs cycle. It requires 5 cofactors,
including NAD and FAD, which would no longer be produced by the electron
transport under hypoxic conditions, decreasing its activity.
Succinate dehydrogenase (choice D) is a key enzyme of the Krebs cycle, producing
a reduced equivalent of FAD to feed into the electron transport chain. It is also
known as Complex II. The Krebs cycle only functions if oxygen is in appropriate
concentrations since it is regulated by the levels of NADH, which is only consumed
by the electron transport chain if there is enough oxygen. The absence of oxygen
leads to an accumulation of NADH and a subsequent decrease in the enzyme
activities of the Krebs cycle.
Transketolase (choice E) is a thiamine requiring enzyme of the non-oxidative half of
the hexose monophosphate shunt. The shuffling of sugars in the second half of this
pathway results in the reentry of glyceraldehyde-3-phosphate and fructose-6phosphate into the glycolytic pathway. Transketolase activity in red blood cells is
used as a clinical marker of thiamine deficiency, markedly decreasing in disorders
such as Wernicke-Korsakoff syndrome.

________________________________________
5The correct answer is B. The disease is acute gout, which is characterized by
strongly negatively birefringent needle-like crystals of uric acid in the joint fluid.
Colchicine is able to interrupt these attacks of acute gout.
Tubercular arthritis causes granuloma formation with giant cells and acid-fast
mycobacteria (choice A).
Infectious arthritis due to Salmonella (sickle cell patients) or Haemophilus (children)
would be associated with neutrophils and gram-negative rods (choice C) in joint
fluid.
Infectious arthritis due to N. gonorrhoeae would be associated with neutrophils and
intracellular gram-negative cocci (choice D).
Calcium pyrophosphate (pseudogout) produces roughly cuboidal crystals that are
weakly positively birefringent (choice E).
________________________________________
6The correct answer is B. The three structures in the intercostal space are, from
superior to inferior, the intercostal vein, artery, and nerve. The proper site for
insertion of an intercostal drain is superior to a rib, not directly at the level of the
superior border but slightly higher to avoid the collateral branches of nerve, artery,
and vein. The nerve is the most inferior structure and thus most likely to be
damaged by the drain (producing an anesthetic dermatome). In addition, the nerve
is the least protected by the costal groove.
The intercostal artery (choice A) is the middle structure and thus not the most likely
to be damaged.
The intercostal vein (choice C) is the most superior structure and thus the least
likely to be damaged. In addition, the vein is the structure best protected by the
costal groove (the further superior the structure, the more protected by the costal
groove).
The internal intercostal fibers (choice D) will tend to be separated by the drain. In
addition, the internal intercostal muscle is membranous (internal intercostal
membrane) posteriorly, from the neck of the rib to its angle; therefore, many drains,
when inserted, will not even pass through a layer of internal intercostal muscle.
________________________________________
7The correct answer is C. The point mutation has resulted in the introduction of the
palindrome recognized by NlaIII, GTAC. Enzymatic digestion of the amplified 222 bp
mutated exon 9 should therefore result in two fragments, of combined mass 222.
Digestion of the normal 222 bp amplified exon should not result in fragmentation,
and it should remain a 222 bp fragment. A heterozygote individual has one normal
allele of the LDL receptor gene and one mutated allele. Digestion of the PCRamplified exon 9 should therefore result in 3 different restriction fragments: the 222

bp normal allele, and the fragmented mutant allele. Note that 126 and 96 add up to
222.
One thick band at 222 (choice A) would be expected in an homozygous normal
individual, both alleles remaining intact after NlaIII digestion. The thickness of the
band only shows the abundance of the restriction fragment of that size.
Three bands of 180, 32 and 10 bp (choice B) would indeed indicate digestion by the
restriction enzyme. However, the lack of 222 bp restriction fragment would rule out
the presence of a normal allele. Also note that since the sum of the three fragments
is 222, it would indicate the presence of 2 restriction sites for the enzyme, which is
not compatible with the information given.
Two bands of 126 and 96 bp (choice D) would be the pattern expected from a
homozygous individual for the mutated allele. The thickness of the band reflects the
abundance of restriction fragment of that size.
Two bands of 444 and 222 bp (choice E) would be very difficult to interpret. A
fragment that is larger than the original amplified material could not be created by
enzyme digestion.
________________________________________
8The correct answer is C. This patient has sarcoidosis, a multi-system disease of
unknown etiology characterized by noncaseating granulomas on histological
examination of various organs. Sarcoidosis occurs mostly in blacks in the third to
fifth decades. Approximately 90% of cases involve the lungs, with findings such as
hilar adenopathy and pulmonary infiltration. Other findings include uveitis,
erythema nodosum (the lesions on this patient's lower extremities), arthritis, central
and peripheral neuropathies, cardiomyopathy, and hypercalcemia. Symptoms
include fatigue, exertional dyspnea, and non-productive cough. Diagnosis is made
after excluding other causes by examination of a tissue biopsy. Radiographic staging
is performed by serial chest x-rays.
Pneumocystis carinii pneumonia (PCP) (choice B) is a disease almost exclusively of
HIV-infected patients, usually with a characteristic chest radiograph appearance of
bilateral fluffy infiltrates.
Systemic sclerosis (choice D), or scleroderma, is a systemic collagen-vascular
disease that involves the skin nearly 100% of the time, and the esophagus in
approximately 75% of patients.
Wegener's granulomatosis (choice E) is a disorder characterized by focal necrotizing
vasculitis in the lung and upper airways associated with granuloma formation, and
necrotizing glomerulitis.
________________________________________
9The correct answer is D. Methamphetamine (and amphetamine) acts by gaining
entrance to dopamine and norepinephrine (and serotonin) nerve terminals, causing
the release of these neurotransmitters via the uptake carriers. Dopamine is believed
to play an important role in the reward system of the brain, and is thought to be a

significant factor in the reinforcing effects of stimulants. One area of the brain that
is thought to be involved in this reward system is the dopaminergic projection from
the ventral tegmental area of the midbrain to the nucleus accumbens of the
forebrain.
Although methamphetamine is a weak inhibitor of monoamine oxidase (MAO), and
would therefore weakly block the metabolism of catecholamines (choice A), this is
not the primary mechanism of action of this drug.
Methamphetamine acts as an indirect-acting agonist, via the release of
neurotransmitter, not as a direct agonist (choices B and C).
Methamphetamine does induce norepinephrine release (choice E), but this plays a
role in the production of systemic side effects (e.g., hypertension), rather than in
the central effect of reinforcement.
________________________________________
10The correct answer is A. Acromegaly is a disease most commonly stemming from
the onset of a pituitary adenoma. Acromegalic persons secrete excessively high
levels of growth hormone and insulin growth factor-1 from pituitary gland tissue.
These hormones are not suppressed by glucose, as they would be in a
nonacromegalic patient.
Measurement of resting ACTH levels (choice B) may be indicated in a person with
Cushing's syndrome.
Measurement of resting prolactin levels (choice C) would be indicated in a person
suspected of having a prolactinoma.
Measurement of height (choice D) in a 59-year-old patient would not be indicated.
Acromegaly, or excess growth hormone, in individuals who have already
experienced closure of bone growth centers results in a broadening of the skeletal
bones, not a lengthening. Lengthening of skeletal bones occurs in gigantism, before
the closure of bone growth centers.
Measuring a patient's shoe size and comparing it to last year's size (choice E), while
indicative of a broadening of skeletal bones, is not a clinically used standard for the
diagnosis of acromegaly.
________________________________________
11The correct answer is E. Systemic lupus erythematosus (SLE) predominantly
affects younger women, and so the question of lupus and pregnancy may arise
frequently in clinical practice. Patients with SLE have an increased incidence of
spontaneous abortion, fetal death in utero, and prematurity. The mother may
experience an exacerbation in the activity of her disease in the third trimester or
peripartum period, and it may be difficult to distinguish between active SLE and
preeclampsia. Therapy of pregnant patients with SLE is problematic, and the
generalist should consult the literature or a specialist when such a patient is
encountered.

Congenital malformations (choices B, C, and D) are not a complication of


pregnancies in patients with SLE.
________________________________________
12The correct answer is A. The patient has Sjgren's syndrome, an autoimmune
disease characterized by dry eyes (keratoconjunctivitis) and a dry mouth
(xerostomia) due to destruction of the lacrimal and salivary glands. Sjgren's
syndrome is also characterized by autoantibody production. The most diagnostic
autoantibodies are those against ribonucleoproteins Ro (SS-A) and La (SS-B),
although coexisting rheumatoid factor and lupus antibodies are not uncommon.
Goiters (choice B) are not typical of Sjgren's syndrome. Although autoimmune
thyroiditis is associated with Sjgren's syndrome, ocular involvement is much more
characteristic than thyroid involvement.
Hemolytic anemia (choice C) is not characteristic of Sjgren's syndrome. Primary
autoantibodies, drugs, and systemic lupus erythematosus may be associated with
hemolytic anemia, but the findings of anti-La and inflammation of the salivary
glands indicate that this patient has Sjgren's syndrome.
Proximal muscle weakness (choice D), in association with autoantibodies, is
expected in polymyositis or dermatomyositis. Although polymyositis may occur in
association with Sjgren's syndrome, keratoconjunctivitis would be much more
common than muscle weakness.
Urethritis (choice E) in autoimmune disease is typical of Reiter's syndrome, not
Sjgren's syndrome.
________________________________________
13The correct answer is D. Cysticercosis is a parasitic infection caused by the larval
cysts of the tapeworm Taenia solium. Patients acquire the infection by ingesting the
eggs, which reach a larval stage in various tissues. When the central nervous
system (CNS) is involved, the condition is known as neurocysticercosis. It is the
most common parasitic infection of the CNS. Infection with this organism is most
frequently encountered in individuals from Mexico, South Central America, the
Philippines, and Southeast Asia. In the CNS, the cysts act as space-occupying
lesions and can cause hydrocephalus and/or seizures. Seizures are the most
common initial presentation of patients with neurocysticercosis and may be focal or
generalized. Signs of increased intracranial pressure such as headache, nausea,
vomiting, or visual changes may also be present.
Amebiasis (choice A) is caused by Entamoeba histolytica. Patients typically present
with diarrhea (often bloody), right lower quadrant abdominal pain, and fever.
Amebic abscesses in the liver are a complication due to invasion of the portal
venous system by the amoeba.
Cytomegalovirus (choice B) produces neonatal infections and infections in
immunocompromised patients such as AIDS patients. CD4 counts are usually <100
cells/mm3. The most common clinical presentation is chorioretinitis producing

floaters, visual field deficits, and painless loss of vision. CMV also produces
encephalitis and may produce calcifying lesions in the CNS.
Echinococcosis (choice C) is a parasitic nematode infection caused by either
Echinococcus granulosis or E. multilocularis. The disease is hydatid cyst disease.
The patient ingests the eggs from dogs and becomes an intermediate host. The
cysts are classically in the liver, are calcified, and the patient shows eosinophilia.
Toxoplasmosis (choice E) is a protozoan infection acquired by eating undercooked
meat or by exposure to cat feces. Primary toxoplasmosis is usually asymptomatic. In
patients with normal immunity, the organism can cause a heterophile-negative,
mononucleosis-like syndrome. In patients with AIDS, it causes ring-enhanced focal
brain lesions and pneumonia.
________________________________________
14The correct answer is A. The patient most probably has syringomyelia, in which
softening and cavitation around the central canal of the spinal cord damages
crossing fibers of the spinothalamic tract with resulting bilateral loss of pain and
temperature (but not touch) sensation in the upper extremities. Many of the
patients with syringomyelia have Arnold-Chiari malformation, in which there is a
congenital protrusion of the cerebellum and medulla through the foramen magnum.
Broca's aphasia (choice B) is a nonfluent aphasia due to damage to the inferior
frontal gyrus.
Horner's syndrome (choice C) is due to damage to the sympathetic innervation of
the face, with resulting ptosis, miosis, and anhidrosis.
Tabes dorsalis (choice D) is a degeneration of the dorsal columns and dorsal roots of
the spinal cord due to tertiary syphilis, producing impaired proprioception and
locomotor ataxia.
Wernicke's encephalopathy (choice E) is a form of psychosis seen in alcoholics with
vitamin B1 (thiamine) deficiency.
________________________________________
15The correct answer is E. The patient is suffering from myasthenia gravis, which is
caused by an autoimmune attack on the nicotinic acetylcholine receptor of the
skeletal muscle motor end plate. Symptoms include early muscle fatigue, often
beginning with the small, active muscles around the eyes. Myasthenia gravis can be
associated with thymoma and bronchogenic carcinoma.
Hypercalcemia can be produced as a paraneoplastic effect in lytic bony metastases,
squamous cell lung cancer, breast cancer (choice A), renal cell carcinoma (choices A
and D), multiple myeloma (choice C), and parathyroid adenomas (choice D).
The syndrome of inappropriate antidiuretic hormone secretion (SIADH) can be
produced by bronchogenic carcinoma and intracranial neoplasms (choice B).
________________________________________

16The correct answer is A. Following transection of a nerve with subsequent close


surgical approximation of the two cut ends, the nerve can regrow down the nerve
trunk at the rate of about 1 mm/day under optimal conditions. The process typically
involves formation of several nerve sprouts from which one becomes dominant.
Interestingly, while there is almost always some degree of "miswiring" during the
reconnection, the brain adapts as the newly reinnervated muscles are used, and
eventually usually automatically provides the appropriate signals to perform a
desired task. In the absence of a nerve trunk to grow down, the sprouting process
can cause the formation of painful traumatic neuromas as the nerve sprouts form a
knotted mass of sensitive nerve fibers.
________________________________________
17The correct answer is B. This question is difficult for two reasons-it asks for the
viral family instead of the virus itself, and it includes distracters that all look alike.
You probably realized that the child in question suffers from croup
(laryngotracheobronchitis); the classic clue here is the "barking cough." You might
have remembered that the virus responsible for croup is the parainfluenza virus.
The toughest part was remembering that parainfluenza virus (along with measles
virus, mumps virus, and respiratory syncytial virus) belongs to the paramyxovirus
family. These viruses all have negative-strand RNA and an enveloped helical
nucleocapsid.
The papovaviruses (choice A) are DNA viruses with a naked icosahedral
nucleocapsid. This family includes the human papilloma viruses (which cause warts
and are associated with penile, laryngeal, and especially, cervical cancer), the BK
virus that can affect immunosuppressed patients, the JC virus associated with
progressive multifocal leukoencephalopathy, and the simian SV40 virus.
The parvoviruses (choice C) are small, single-stranded DNA viruses. Only one
serotype (B19) causes diseases in humans. It causes erythema infectiosum in
children (characteristic "slapped cheek" rash), aplastic crises in patients with
hemolytic diseases, and hydrops fetalis or stillbirth in anemic fetuses.
The picornaviruses (choice D) are positive single-stranded RNA viruses with a naked
icosahedral nucleocapsid. This family includes the polioviruses, echoviruses,
coxsackieviruses, enterovirus 72 (HepA virus), and the rhinoviruses (common cold).
The poxviruses (choice E) are double-stranded DNA viruses. This family includes the
viruses responsible for smallpox and molluscum contagiosum.
________________________________________
18The correct answer is A. The patient has weakness of the anterior compartment
muscles (loss of plantar flexion) and lateral compartment muscles (unopposed
inversion) of the leg. These muscles are innervated by the deep peroneal nerve and
superficial peroneal nerve respectively. Both of these nerves are branches of the
common peroneal nerve. Also, the superficial and deep peroneal nerves provide
sensory innervation to the dorsum of the foot. The common peroneal nerve is

superficially located at the neck of the fibula. The patient compressed this nerve
against the mattress of the bed while lying on her right side for long periods of time.
The femoral nerve (choice B) innervates the muscles and the skin of the anterior
thigh. None of the findings relate to the femoral nerve.
The obturator nerve (choice C) innervates the muscles of the medial thigh and the
skin in this region. None of the findings relate to the obturator nerve.
The sciatic nerve (choice D) divides to become the tibial nerve (choice E) and the
common peroneal nerve. The tibial nerve innervates the muscles of the calf and the
skin of the sole of the foot. The calf muscles are responsible for plantar flexion,
which is normal in this patient. Similarly, the patient has retained sensation on the
sole of the foot.
________________________________________
19The correct answer is A. Losartan competitively inhibits angiotensin II at the AT-1
receptor. Angiotensin II is the most powerful sodium-retaining hormone of the body.
The effects of angiotensin II can be attributed to (1) direct stimulation of sodium
reabsorption in the proximal and distal tubules of the kidney; (2) stimulation of
aldosterone secretion, which is another sodium-retaining hormone; and (3)
constriction of the efferent arterioles in the kidney, which increases peritubular
colloid osmotic pressure, thereby enhancing reabsorption of salt and water from the
proximal tubule. Therefore, when the effects of angiotensin II are blocked with
losartan, the ability of the kidneys to excrete sodium increases greatly.
Because angiotensin II normally stimulates aldosterone (choice B) secretion,
losartan decreases plasma aldosterone levels.
The decrease in arterial pressure (choice C) caused by losartan (because of its
effect to increase the loss of salt and water) should also decrease the total
peripheral resistance by an autoregulatory mechanism. That is, when arterial
pressure is lowered, the arterioles dilate, which maintains blood flow at a constant
level in the peripheral tissues.
Overstretching of the two atria, usually as a result of increased blood volume,
causes atrial natriuretic peptide (choice D) to be released into the blood. Because
losartan increases the ability of the kidneys to excrete sodium (and therefore
water), the blood volume should decrease, and the plasma levels of atrial natriuretic
peptide should decrease.
Angiotensin II is a potent vasoconstrictor. Blocking its effects with losartan would be
expected to decrease total peripheral resistance (choice E).
________________________________________
20The correct answer is E. The patient is presenting with signs and symptoms
suggestive of Crohn's disease, which is an idiopathic inflammatory process that can
affect any portion of the alimentary tract. This condition is often characterized by
intermittent bouts of low-grade fever, diarrhea, malaise, and weight loss, as well as
focal tenderness and a palpable tender mass in the lower abdomen. There is

radiographic evidence of ulceration, stricturing, or fistulas of the small intestine and


colon. Nonpharmacologic therapy can be efficacious in some cases, but more severe
cases may require corticosteroids, such as prednisone, which dramatically suppress
the clinical signs and symptoms.
Antidiarrheal agents (eg, diphenoxylate with atropine (choice A) or loperamide)
should be used very cautiously in these patients since there is a very high risk of
toxic megacolon.
Hydrocortisone suppositories (choice B) are indicated for the treatment of distal
ulcerative colitis, not Crohn's disease.
Hyoscyamine (choice C) is an anticholinergic agent that may alleviate the
postprandial abdominal pain of a patient with irritable bowel syndrome when
administered 30-60 minutes before a meal.
Mesalamine (choice D) is a 5-aminosalicylic acid derivative indicated for the
treatment of ulcerative colitis. Although this agent may provide some benefit in the
treatment of Crohn's disease, prednisone is the drug of choice for treatment of
acute "flare-ups" seen in patients with this disease.
________________________________________
21The correct answer is A. Selective IgA deficiency (<5 mg/dL) is the most common
of all the primary immunodeficiency diseases. The incidence reported in the US has
ranged from 1:250 to 1:1000. IgA has two subclasses, IgA1 and IgA2. IgA1
predominates in the serum, while IgA2 predominates in mucosal secretions as a
dimer bound together by a J chain with a secretory piece attached. Recurrent
bacterial and viral infections of the respiratory tract can be attributed to a lack of
secretory IgA (sIgA), the predominant immunoglobulin of the mucosal immune
system.
IgD (choice B) has not been given any particular function other than to act as a
receptor on the B cell. It can be found in very low levels in serum.
IgG (choice C) is the major immunoglobulin found in the humoral immune response.
A patient with a low IgG will experience pyogenic infections.
IgM (choice D) is found in the early response to an antigen. If the patient was
deficient in IgM he would have also been characteristically low in IgG and would
have experienced recurrent pyogenic infection, usually commencing by the age of
5-6 months.
________________________________________
22The correct answer is D. The free edge of the lesser omentum contains three
important structures: the common bile duct, the hepatic artery, and the portal vein.
Neither the cystic duct (choices A, B, and C) nor the hepatic vein (choices B, C, and
E) lies in the free edge of the lesser omentum.
________________________________________

23The correct answer is D. The results of the retrograde cholangiopancreatography


indicate the presence of an obstruction in the common bile duct. This would lead to
an obstructive jaundice. The liver's ability to take up, conjugate, and secrete
bilirubin is not impaired. However, the conjugated bilirubin would "back up" and
regurgitate into the circulation, producing a conjugated hyperbilirubinemia (not
unconjugated hyperbilirubinemia, choice E). Since conjugated bilirubin is water
soluble, it can be filtered by the kidney and show up in the urine. The excessive
filtration of conjugated bilirubin can produce a darkening of the urine (tea-colored).
Excessive hemolysis (choice A) produces an unconjugated hyperbilirubinemia
because the ability of the liver to take up the increased bilirubin from heme
metabolism is exceeded.
An obstruction of the common bile duct would prevent the delivery of bile to the
duodenum. As a consequence, the pigmentation of the stool would be diminished
(not increased, choice B).
Urinary urobilinogen is a reflection of circulating urobilinogen. Urobilinogen enters
the blood as part of the enterohepatic circulation. Bilirubin secreted in the bile is
ordinarily metabolized by gut bacteria to urobilinogen. Most urobilinogen remains in
the gut, and after further metabolism, provides the pigmentation to the stool.
However, some of the urobilinogen is absorbed by the ileum and enters the
enterohepatic circulation. Some of this absorbed urobilinogen ends up in the urine.
With an obstruction of the common bile duct, less urobilinogen will reach the small
intestine, and hence, less will enter the hepatic portal vein. Therefore, urinary
urobilinogen will be decreased (not increased, choice C).
________________________________________
24The correct answer is C. Early in sickle cell anemia, affected children usually have
an enlarged spleen. The spleen enlarges because the children are in a state of
hypersplenism, in which the spleen vigorously phagocytizes any abnormally shaped
erythrocytes. During the first decade of life, sickling within the spleen occludes its
blood flow, causing repeated small infarctions, leading to "autosplenectomy." The
autosplenectomy is actually helpful to the patient hematologically, as it may
ameliorate the chronic anemia. Unfortunately, the lack of a functional spleen
renders the patients susceptible to infections with Salmonella and encapsulated
organisms such as Streptococcus pneumoniae and Haemophilus influenzae.
Patients with sickle cell anemia are not particularly vulnerable to chronic infection
(choice A), although acute infections can trigger painful crises.
Patients with sickle cell anemia do not have an increased incidence of either
Hodgkin's disease (choice B) or non-Hodgkin's lymphoma (choice D).
Portal hypertension (choice E) causes an enlarged, rather than a small, spleen
________________________________________

25The correct answer is E. Creatinine clearance is used clinically to estimate


glomerular filtration rate (GFR). Therefore, the 50% decrease in creatinine clearance
in this patient suggests that GFR has decreased by 50% over the past year. Because
creatinine is freely filtered but not reabsorbed (choice D), the filtration rate and
excretion rate of creatinine are equal during steady state conditions. When GFR
decreases, the rate of creatinine excretion will also decrease, causing the rate of
creatinine excretion to fall below the rate of creatinine production. The result is an
increase in plasma creatinine concentration. When plasma creatinine levels have
increased by 2-fold, normal amounts of creatinine can then again be excreted
(compare to choices B and C) because the excretion rate of creatinine is equal to
the product of GFR and plasma creatinine concentration. In summary, when GFR
decreases, the plasma creatinine concentration continues to increase until the rate
of creatinine filtration (and excretion) becomes equal to the rate of creatinine
production by the body.

The blood urea nitrogen concentration (choice A) increases when GFR is reduced
________________________________________
26The correct answer is B. Muscle cells contain desmin, which is a component of the
intermediate filaments found in these cells.
Epithelial cells contain cytokeratin (choice A).
Neuroglia contain glial fibrillary acidic proteins (choice C).
Neurons contain neurofilaments (choice D).
Connective tissue contains vimentin (choice E).
________________________________________
27The correct answer is B. The neoplasm described here is known as
lymphoepithelioma, an inappropriate designation that emphasizes the histologic
features of a lymphocyte-rich squamous cell carcinoma. This tumor, AKA
nasopharyngeal carcinoma, is the most frequent childhood malignancy in certain
African regions and one of the most frequent adult tumors in southern China.
Epstein-Barr virus genome has been identified in most of these neoplasms.
Cigarette smoking (choice A) is a risk factor for a number of malignant neoplasms,
such as lung cancer. In the upper respiratory tract, cigarette smoking is a strong
predisposing factor for laryngeal carcinoma, but it has not been linked to the
pathogenesis of nasopharyngeal carcinoma.
Radiation injury (choice C) is a well-known risk factor for many human neoplasms,
such as leukemia and thyroid cancer, but not for nasopharyngeal carcinoma.
Although any cell type is susceptible to neoplastic transformation by radiation, there
is a sort of hierarchy of vulnerability. The most frequent radiation-induced cancers
are leukemias and thyroid carcinoma, followed by breast, lung, and salivary gland

tumors. Skin, bone, and the gastrointestinal tract are least vulnerable to radiation
carcinogenesis.
Overexpression of the bcl-2 gene (choice D) is found in most B-cell follicular
lymphomas. bcl-2 is a member of a large family of genes that regulate apoptosis.
bcl-2 inhibits apoptosis, so that its overexpression allows neoplastic lymphocytes to
survive for long periods and accumulate progressively in lymph nodes and bone
marrow.
Trisomy 8 (choice E) has been identified in some cases of olfactory neuroblastoma, a
malignant tumor of small round cells arising from the olfactory mucosa and
resembling adrenal neuroblastoma.
________________________________________
28The correct answer is D. Pituitary adenomas are benign neoplasms of the anterior
lobe associated with excess hormone secretion. Depending on which cell type is
affected, different clinical syndromes may be present. Adenomas may occur at any
age, and affect both sexes, but most commonly affect men between the ages of 20
and 50. Overall, they are rare and may have an association with MEN I (pituitary
adenoma, parathyroid neoplasm, pancreatic islet adenoma). Prolactinomas are the
most common type of pituitary adenomas, resulting from neoplastic growth of the
lactotroph cells in the anterior pituitary. Young women with prolactinomas present
clinically with amenorrhea, galactorrhea, and infertility. Men usually have impotence
and decreased libido.
Acromegaly (choice A) is generally caused by a functional adenoma secreting
excess growth hormone. This disorder occurs after the epiphyses of the long bones
have fused and causes thickening and increased coarseness of the bones and skin.
Systemic manifestations include arthritis, insulin resistance, cardiomegaly, and
sexual dysfunction.
Cushing's syndrome is the result of a corticotroph adenoma producing excess ACTH,
which stimulates adrenal corticosteroid hypersecretion. It is less common than
prolactinoma or null cell (no hormone) adenoma. Cushing's disease (choice B) refers
to ACTH-dependent hyperadrenalism, whereas Cushing's syndrome refers to
hypercortisolism from any cause. Obesity, hypertrichosis, and amenorrhea are the
usual clinical results.
Neurogenic diabetes insipidus (choice C) is due to disease of the posterior pituitary
leading to vasopressin (antidiuretic hormone; ADH) deficiency and is not associated
with pituitary adenoma. Although the etiology is not clear in approximately onethird of cases, others are associated with craniopharyngiomas. The disorder is
characterized by polyuria, thirst, and polydipsia.
Virilization (choice E) is often associated with gonadotroph adenoma (uncommon).
Clinically, it causes acquired hypogonadism in men, as well as headache and visual
disturbances.
________________________________________

29The correct answer is B. Thiazide diuretics increase the urinary excretion of


sodium and water by inhibiting sodium reabsorption in the early distal tubule. This
increases delivery of tubular fluid to the distal tubule, and contraction of plasma
volume causes aldosterone secretion to increase. The net effect is to increase
sodium reabsorption at the distal tubules, concomitantly increasing the loss of
potassium and hydrogen ions.
The increase in potassium excretion has caused hypokalemia (low plasma
potassium), which also stimulates aldosterone secretion. This combination of
sodium and potassium depletion acts to generate a metabolic alkalosis. Arterial pH
is increased and arterial H+ concentration (choice A) is decreased with metabolic
alkalosis.
Sodium depletion tends to decrease plasma sodium (choice C) levels, although the
effect is usually small.
Overuse of the thiazide diuretic has caused depletion of sodium and potassium by
the kidneys, not retention of sodium and potassium (choices D and E).
________________________________________
30The correct answer is C. This is the chromosomal mutation for Ewing sarcoma.
This case demonstrates the typical presentation of Ewing sarcoma, occurring mainly
in the long bones of children and adolescents. It presents with pain and has a
unique onion-skin effect on x-ray. Histologically, it is known as a small, round, bluecell tumor. A number of different tumors look similar, and molecular methods are
increasingly used for diagnosis.
del 1p (choice A) is associated with leiomyosarcoma, which is mainly found in the
extremities, subcutis, and retroperitoneum. It is located outside of bone in the soft
tissues and histologically is composed of bundles of spindle cells.
t(2,13) (choice B) is associated with alveolar rhabdomyosarcoma, a tumor of young
persons that can affect the extremities. It is composed of small round blue cells that
form small nests or alveoli. The radiographic appearance shows a soft tissue mass
rather than a bony lesion. This is an aggressive malignancy.
t(12,22) (choice D) is associated with clear cell sarcoma, also known as malignant
melanoma of soft parts. It appears like a malignant melanoma, which arises in the
soft tissue rather than the skin. It usually is found on the tendons of extremities of
young patients. Histologically, the cells can have pigment or clear cytoplasm.
t(X,18) (choice E) is associated with synovial sarcoma, which is a tumor arising
around a joint space, usually in a young person. The knee and shoulder are the
most common location. Histologically, the cells can be spindled or plump, pink, and
epithelioid.
________________________________________
31The correct answer is D. The diagnosis of myxedema, due to long-standing (e.g.,
often several years duration) hypothyroidism in adults, is warranted. The clinical
manifestations are those listed in the question stem. Myxedema can result from the

many causes of hypothyroidism: Hashimoto's thyroiditis, idiopathic primary


hypothyroidism, iodine deficiency, drugs, pituitary lesions, hypothalamic lesions,
and damage to the thyroid by surgery or radiation. It is not warranted at this stage
in the patient's evaluation to assign a specific cause for the myxedema, as the
appropriate work-up has not yet been done.
Cretinism (choice A) is caused by hypothyroidism in infancy.
Graves disease (choice B) usually produces hyperthyroidism.
Hashimoto's thyroiditis (choice C) is an important cause of hypothyroidism, but this
diagnosis would require a biopsy demonstrating infiltration with lymphocytes,
macrophages and plasma cells, often associated with germinal center formation.
Subacute granulomatous thyroiditis (choice E) usually causes hyperthyroidism but
can cause transient hypothyroidism; this specific diagnosis would necessitate a
biopsy demonstrating disruption of thyroid follicles, a neutrophilic infiltrate, cellular
aggregates, and multinucleated giant cells.
________________________________________
32The correct answer is C. This is the typical clinical presentation of carcinoma of
the prostate, which is probably the most frequent cancer in males (although lung
cancer is the most frequent neoplastic cause of death). Autopsy studies show that
small foci of adenocarcinoma are found in the prostate of up to 70% of men older
than 80. Prostatic specific antigen (PSA) is expressed by both normal prostatic
tissue and adenocarcinoma of the prostate. A rise in PSA plasma levels, therefore,
simply results from any increase in the mass of prostatic tissue and cannot be used
to differentiate between cancer and hyperplasia. Cancer arises more frequently in
the peripheral zone of the gland, producing firm nodules on the posterior surface
that can be detected on rectal examination. Metastasis to the bone can be
osteoblastic (with formation of new bone) or osteolytic (with destruction of bone).
Osteoblastic metastases in the vertebral column are characteristic of prostatic
adenocarcinoma.
Chronic prostatitis (choice A) presents with dysuria and perineal discomfort. The
diagnosis relies on finding at least 10 leukocytes per high-power field on
microscopic examination of expressed prostatic secretions. Chronic prostatitis is
further classified into bacterial prostatitis if cultures are positive for bacteria, or
abacterial prostatitis if cultures are negative. Increased plasma PSA is frequently
seen.
Multiple myeloma (choice B) is a neoplasm of plasma cells that frequently manifests
with osteolytic (not osteoblastic) lesions in the vertebral column as well as other
bones, such as the cranial vault and ribs.
Prostatic hyperplasia (choice D) is an extremely common disorder in middle-aged
and elderly men. It involves the periurethral zone of the gland, leading to
compression of the urethra, difficulty in urination, urinary retention, and recurrent
cystitis. PSA levels are increased in a manner roughly proportional to the degree of
prostatic enlargement.

Prostatic intraepithelial neoplasia (PIN; choice E) is the precursor of invasive


prostatic adenocarcinoma and consists of atypical proliferation of epithelial cells
confined within the basement membrane of prostatic acini. In one third of cases, PIN
progresses to invasive cancer within 10 years. By definition, PIN is not associated
with metastasis. Other examples of cancer precursors are CIN (cervical
intraepithelial neoplasia) and VIN (vulvar intraepithelial neoplasia).
________________________________________
33The correct answer is B. The woman has preeclampsia, the features of which
include proteinuria and increased blood pressure. A wide variety of other features of
preeclampsia can also be seen, including excessive weight gain, generalized
edema, ascites, hyperuricemia, hypocalciuria, increased plasma concentration of
von Willebrand factor and cellular fibronectin, reduced plasma concentration of antithrombin III, thrombocytopenia, increased hematocrit, increased liver enzymes,
intrauterine growth retardation, and intrauterine hypoxia. Modern theories suggest
that the true primary lesion may involve the endothelium, and that medical control
of hypertension (choice A) actually only treats a small part of the syndrome. At
present, the only definitive therapy is delivery of the baby, and obstetricians often
play a delicate game trying to delay delivery for a premature baby's sake as long as
possible while judging the severity of the preeclampsia and its immediate risks to
mother and fetus.
Low dose aspirin (choice C) may have a modest effect in preventing pre-eclampsia,
but this is not yet well established.
Oxygen supplementation (choice D) is used in some cases of cerebral hemorrhage
secondary to preeclampsia, but does not constitute definitive therapy.
Renal dialysis (choice E) is usually not necessary in preeclampsia, although both
renal cortical necrosis and renal tubular necrosis can occasionally occur in this
disorder.
________________________________________
34The correct answer is D. The maximum expiratory flow-volume (MEFV) curve is
often used as a diagnostic tool for identifying obstructive and restrictive lung
diseases. In restrictive lung diseases such as interstitial fibrosis, the MEFV curve
begins and ends at abnormally low lung volumes, and the flow rates are often
higher than normal at any given lung volume. Note on the diagram that the total
lung capacity is ~3.2 liters and the residual volume is ~0.8 liters in the patient
(dashed curve). The residual volume cannot be determined from a MEFV curve
alone, so must be measured using a different technique before the curves can be
placed appropriately on the abscissa.
Lung volumes would expected to be higher than normal in asthma (choice A),
bronchospasm (choice B), emphysema (choice C), old age (choice E), and other
conditions involving narrowing of the airways or reduced radial traction of the
airways, allowing them to close more easily.
________________________________________

35The correct answer is B. There are two forms of aortic coarctation, infantile and
adult. This infant manifests the characteristic signs of the infantile form of aortic
coarctation, which is associated with patent ductus arteriosus (PDA). The stenotic
segment is localized proximal to a PDA. Since blood pressure drops distal to the
PDA, blood will shunt from the pulmonary artery to the aorta through the PDA. Thus,
cyanosis develops in the lower part of the body only.
The adult form of aortic coarctation (choice A), in contrast, is not associated with
PDA. This is more common than the infantile form and leads to hypertension
proximal to the stenosis (ie, in the head and upper limbs) and hypotension in the
lower half of the body. There is no cyanosis.
Isolated PDA (choice C), if large enough, will allow significant left-to-right shunting,
resulting in pulmonary overload, secondary pulmonary hypertension, and right
ventricular hypertrophy (chronic cor pulmonale). Persistence of PDA is promoted by
prostaglandin E, whereas inhibitors of prostaglandin synthesis (indomethacin or
other NSAIDs) facilitate closure of the ductus.
Pulmonary stenosis (choice D) is an infrequent form of congenital heart disease that
presents with chronic cor pulmonale because of increased resistance to blood flow
in the pulmonary artery. Right-sided heart failure develops without cyanosis.
Tetralogy of Fallot (choice E) is one of the most frequent types of congenital heart
disease in general, and the most frequent cause of cyanotic congenital heart
disease. Its features include subpulmonary stenosis, ventricular septal defect, an
overriding aorta, and right ventricular hypertrophy. If the degree of subpulmonary
stenosis is severe, right-to-left shunting ensues and cyanosis is produced. In this
case, cyanosis involves the entire body, not the lower half only.
________________________________________
36The correct answer is A. The disease is chlamydial cervicitis. This venereally
transmitted infection is usually suspected after treatment for gonorrhea fails to
relieve symptoms. However, a few cases are picked up when cytoplasmic inclusions
composed of aggregates of chlamydia are identified on Pap smears. Confirmation
can be made with fluorescent antibodies that pick up both the aggregates and
individual bacteria known as "elementary bodies." The organisms are obligate
intracellular parasites because they have lost the ability to synthesize ATP. Although
some authors consider them to be "bacteria," others do not, since this is such a
fundamental difference between the chlamydia and free living bacteria.
Failure to synthesize cholesterol (choice B) is not usually cited as a problem specific
to any type of organism, although viruses are not able to do so without using host
machinery.
Some viruses cannot synthesize DNA (choice C), but the disease in question is not
caused by a virus.
Protein synthesis (choice D) by viruses requires host ribosomes.

Some viruses cannot synthesize RNA (choice E), but the disease in question is not
caused by a virus.
________________________________________
37The correct answer is D. Polycystic ovary syndrome often results in amenorrhea
because of excessive ovarian androgen production secondary to increased plasma
LH. The initiating event in this disorder is not known, but could be due to excessive
secretion of GnRH by the hypothalamus. The pituitary responds to the abnormal
pattern of GnRH release by increasing LH secretion, but decreasing FSH secretion.
The increased LH causes theca cell hyperplasia (not atrophy, choice A) and
excessive androgen secretion. With low FSH levels, granulosa cells would be
atrophied (not hyperplastic, choice C) and would have insufficient aromatase
activity to convert the androgen to estrogen. The high local androgen concentration
may be responsible for the formation of numerous small cystic follicles. One or both
of the ovaries are often significantly enlarged. The hirsutism associated with
polycystic ovarian syndrome is likely due to the increased ovarian secretion of
androgen.
Ovarian dysgenesis with chromosomal mosaicism (choice B) is typical of Turner's
syndrome. Individuals with the XO genotype typically have hypogonadism, in part,
because of abnormal ovarian development. The ovary may contain multiple cell
lines with varying chromosomal composition. Turner's syndrome is characterized by
a constellation of physical abnormalities (short stature, webbed neck, shield chest)
that are not present in this patient.
The presence of seminiferous tubules in the gonad (choice E) of a phenotypic
female would suggest testicular feminization syndrome. In this syndrome, XY
genotypic males are produced that lack androgen receptors. Although the individual
exhibits female phenotypic external genitalia and secondary sex characteristics, a
uterus is absent and menstrual cycles never begin. The absence of androgen
receptors would also preclude the possibility of hirsutism.
________________________________________
38The correct answer is D. Type 1 diabetes (DM 1), previously known as juvenile
onset or insulin-dependent diabetes, is due to low insulin production as a
consequence of autoimmune destruction of pancreatic beta cells. Severe insulin
deficiency causes marked increases in the use of fats as a source of energy.
Ketones, acetoacetate and beta-hydroxybutyrate are produced in excess, and
diabetic ketoacidosis may develop with potentially dire consequences. Type 2
diabetes (DM 2) is a consequence of insulin resistance by the tissues, despite very
high levels of serum insulin, initially (insulin levels typically fall as the disease
progresses). Ketoacidosis is highly unusual in DM 2, since insulin is present.
In DM 1 there is usually complete loss of beta cells by puberty; thus insulin
dependence begins in childhood. DM 2 has an adult age of onset (choice A).

There is approximately 50% twin concordance in DM 1, suggesting that


environmental factors must also play a "triggering role" in DM 1. The twin
concordance rate is much higher in DM 2 (~90%) (choice B).
Insulin levels are nearly zero in DM 1. Conversely, DM 2 is a disease of insulin
resistance, and is usually associated with increased insulin levels (choice C).
Body weight has no bearing on the pathogenesis of DM 1, whereas DM 2 occurs
predominantly in the obese (choice E).
________________________________________
39The correct answer is C. The disease is osteoporosis, and estrogen replacement in
post-menopausal women appears to play an important role in preventing or limiting
development of osteoporosis in post-menopausal women.
Cortisol (choice A) excess, as in endogenous or exogenous Cushing's syndrome, is a
contributing cause of osteoporosis.
Epinephrine (choice B) levels appear to be unrelated to osteoporosis.
Thyroxine (choice D) excess (e.g., in thyrotoxicosis) may contribute to bone loss in
some cases of osteoporosis.
Vasopressin (choice E) levels appear to be unrelated to osteoporosis.
________________________________________
40The correct answer is B. The floor of the orbit is also the roof of the maxillary
sinus. The bone separating the orbit from the maxillary sinus is frequently quite thin
and may fracture from increased intraorbital pressure caused by a direct blow to the
eye. The subsequent bleeding into the maxillary sinus will result in blood draining
from the sinus into the nasal cavity.
The ethmoid air cells (choice A) are medial to the orbit. This bone is also frequently
quite thin.
The oral cavity (choice C) is separated from the orbit by the maxillary sinus.
The pterygopalatine fossa (choice D) is posteromedial to the orbit. The medial
portion of the inferior orbital fissure communicates between the orbit and the
pterygopalatine fossa.
The sphenoid sinus (choice E) is posteromedial to the orbit. There is no
communication between the sphenoid sinus and the orbit.
________________________________________
41The correct answer is B. Niacin, or vitamin B3, is an agent that results in the
following physiologic changes: LDL reductions tend to occur in 5-7 days with the
maximal effect seen in 3-5 weeks; triglycerides and VLDL are reduced by 20% to
40% in 1-4 days; and HDL levels can increase by 20%. This agent is indicated as
adjunctive therapy in patients with elevated cholesterol and triglycerides when diet
and other nondrug therapies are inadequate. The most common adverse effect of

this agent is generalized flushing with a sensation of warmth, especially in the facial
area. This reaction may be so severe in some patients that they discontinue
therapy. Other common adverse effects include hepatotoxicity, tachycardia
(compare with choice A), hypoalbuminemia (compare with choice C), hyperglycemia
(compare with choice D), nausea, vomiting, hyperuricemia, glucose intolerance,
pruritus, peptic ulcer disease, and dry skin.
________________________________________
42The correct answer is C. The patient has ankylosing spondylitis, which is very
strongly associated with HLA-B27. HLA-B27 is also associated with Reiter's
syndrome, which can be remembered as the syndrome in which you "can't see
(anterior uveitis conjunctivitis), can't pee (urethritis), and can't climb a tree
(arthritis)."
Budd-Chiari syndrome (choice A) is liver disease secondary to occlusion of the
inferior vena cava or hepatic veins.
Goodpasture's syndrome (choice B) is pulmonary hemorrhage and glomerular
disease secondary to antibodies to basement membrane.
Reye's syndrome (choice D) is an often fatal liver disease that can follow viral
infection.
Sjgren's syndrome (choice E) is an autoimmune disease that damages salivary
glands and tear glands.
________________________________________
43The correct answer is B. The volume of a fluid compartment can be measured by
placing a substance into the compartment, allowing it to disperse evenly throughout
the compartment, and then measuring the extent to which the indicator is diluted in
the fluid. The volume of a compartment can be determined using the following
formula:

The extracellular fluid volume can be measured using inulin as the indicator: 0.1 g
inulin was administered intravenously and the concentration of inulin in the
compartment was 1 mg/100 mL an hour later (when the inulin had dispersed evenly
in the extracellular fluid compartment). Therefore,

________________________________________

44The correct answer is C. Transcutaneous electrical nerve stimulation is a method


used to lessen severe, chronic pain by overly stimulating the involved neurons. This
is thought to trigger inhibitory interneurons in lamina II of the spinal cord, thereby

partially blocking the transmission of pain impulses. These interneurons are


considered to be "gate-keepers," that can, to some degree, isolate the peripherally
generated signals from the brain.
Allodynia (choice A) is the term used for the perception of pain following a normally
innocuous stimulation of a mechanoreceptor.
Central pain (choice B), such as that caused by thalamic lesions (choice E), is pain
that originates at the level of the brain rather than in the periphery.
Referred pain (choice D) is the perception of pain initiated in one body site (typically
an internal organ) as being localized to another body site (frequently on the more
superficial parts of the body). An example is that of diaphragmatic pain, which may
be referred to the top of the shoulder. The phenomenon occurs because both
internal organs and more superficial structures may arise from, and consequently
be innervated by, the same dermatome.
________________________________________
45The correct answer is B. Ingestion of exogenous thyroxine (sometimes called
factitious thyrotoxicosis) could explain the patient's symptoms related to
hyperthyroidism, and could explain the increase in serum T4. The decrease in
radioactive iodine uptake is due to decreased serum TSH resulting from the
negative feedback effects of the excess T4. Note that hyperthyroidism is not always
associated with increased radioactive iodine uptake.
Both Graves' disease (choice A), which is characterized by increased production of
thyroid-stimulating immunoglobulins, and a TSH-secreting tumor in the pituitary
(choice D), would produce signs and symptoms of hyperthyroidism, but would be
associated with increased radioactive iodine uptake. Overactivation of the TSH
receptor in the thyroid gland in both cases would increase the iodine trapping
mechanism in the follicle cells.
Thyroid hormone receptor insensitivity (choice C) would produce signs and
symptoms consistent with hypothyroidism (cold intolerance, lethargy, bradycardia,
etc.). Furthermore, the negative feedback effects of thyroid hormone in the
hypothalamus and pituitary would be diminished (since this is also a receptormediated event) and serum levels of TSH would be increased, producing an
increase in radioactive iodine uptake. Note that hypothyroidism is not always
associated with decreased radioactive iodine uptake.
With toxic adenoma (choice E), the thyroid gland autonomously secretes excessive
thyroid hormone; increased iodine uptake would be needed to support this
overproduction. The increased serum levels of thyroid hormone inhibit TSH
secretion from the anterior pituitary and the thyroid tissue undergoes atrophy.
________________________________________
46The correct answer is A. A tumor of the apical segment of the upper lobe may
impinge on the recurrent laryngeal nerve since it ascends from the superior
mediastinum to the root of the neck in a groove between the trachea and

esophagus. The recurrent laryngeal nerve supplies all intrinsic muscles of the larynx
except the cricothyroid.
Mitral valve insufficiency (choice B) causes a backup of blood into the left atrium.
The enlarged left atrium exerts pressure on the esophagus, which lies posterior to it,
causing dysphagia (difficulty swallowing).
Pulmonary embolus (choice C) causes an obstruction to arterial blood flow of the
lung, resulting in infarction of the affected segment. It may also cause pleurisy,
resulting in pain conveyed by intercostal nerves.
When the right subclavian artery (choice D) arises from the arch of the aorta, rather
than from the brachiocephalic trunk, it passes posterior to the esophagus to reach
the right upper extremity. This may cause compression of the esophagus, resulting
in dysphagia.
Thymoma (choice E), or tumor of the thymus gland, may cause dyspnea (difficulty
breathing) due to pressure on the trachea. It may also cause engorgement of deep
and superficial veins of the neck due to pressure on the superior vena cava.
________________________________________
47The correct answer is E. Even though the surface LDL receptors were digested by
the trypsin, the recycling of unoccupied receptors to the cell surface provides a
continual supply of new receptors to bind the labeled LDL. The LDL-receptor
complex is internalized by receptor-mediated endocytosis.
Active transport (choice A) is the energy-dependent movement of molecules across
a membrane and against a concentration gradient.
Facilitated diffusion (choice B) is the transport of low-permeability molecules with
the aid of a carrier protein.
Phagocytosis (choice C) is the process by which cells such as macrophages and
neutrophils engulf large particles.
Pinocytosis (choice D) is the uptake of small molecules in solution, and is receptor
independent.
________________________________________

48The correct answer is C. The patient suffers from multiple myeloma, a neoplastic
proliferation of plasma cells (or their precursors) found within the bone marrow.
These malignant cells are responsible for the production of excessive amounts of
immunoglobulin (usually IgG or IgA). Plasma cells synthesize a greater amount of
light chains than heavy chains. The intact immunoglobulins are not excreted by the
kidney, but light chains are filtered at the glomerulus and found in the urine. In
normal individuals, polyclonal light chains will be found in the urine, reflecting the
polyclonal population of plasma cells producing the light chains. In monoclonal
proliferations such as multiple myeloma, the light chains will be monoclonal. This

patient is making a monoclonal population of kappa light chains and excreting them
in the urine as Bence-Jones proteins. It is not unusual for patients with myeloma to
have recurring bacterial infections, particularly pneumococcal pneumonia.
Remember, this patient is making decreased levels of normal immunoglobulins of all
isotypes, thus making her susceptible to the bacteria to which she is exposed.
Infiltration of bone by the myeloma cells may lead to pathological fractures.
Bence Jones proteins are light chains, not heavy chains (choice A).
The serum protein electrophoresis showed a monoclonal spike of kappa chains, so
the Bence-Jones proteins will be monoclonal kappa chains. Rarely, a biclonal plasma
cell dyscrasia can produce Bence-Jones proteins composed of both kappa and
lambda light chains (choice B), but this is not the case here.
Bence-Jones proteins composed of lambda light chains (choice D) would not be
produced by a monoclonal myeloma producing kappa chains in the serum.
Bence-Jones proteins are light chains of the immunoglobulin molecule and not the
intact monomer IgG (choice E), which is composed of two light chains and two
heavy chains.
________________________________________
49The correct answer is C. About 95% of the salt (sodium chloride) that is
consumed by a person is excreted by the kidneys; the remaining 5% is excreted in
the sweat and feces. The total intake of salt (amount of salt consumed each day)
must equal the total output of salt (amount of salt excreted each day) under normal
steady conditions, ie, salt intake = salt output. Therefore, it is clear that 25 g of salt
must be excreted by the kidneys each day when 25 g of salt is consumed each day.
95% of 25 is around 23 g.
________________________________________
50The correct answer is A. The patient described in this question is suffering from
absence seizures, which typically appear during childhood, between the ages of 5
and 7. In absence seizures, the patient has many episodes of brief disruption of
consciousness throughout the day. These seizures are not accompanied by the
convulsions and complete loss of consciousness often associated with epilepsy, but
rather by the absence of motor or sensory symptoms (hence the blank look on the
patient's face).
Children with attention deficit hyperactivity disorder (choice B) also have a limited
attention span and normal intelligence. However, they also exhibit hyperactivity,
impulsiveness, emotional lability, and irritability, which lead to behavioral problems
in school.
Children with infantile autism (choice C) may present with a short attention span,
but their most striking deficits lie in their difficulty with social interactions and
communication skills. Infantile autism is a developmental disorder that usually
manifests itself before age 3, and most autistic children have an IQ below the
normal range.

Phonological disorders (choice D) are a class of communication disorders in which


the age- and intelligence-appropriate speech sounds are developmentally delayed.
Schizophrenia with childhood onset (choice E) is quite rare. Children with this
disorder demonstrate normal intelligence, and may show a limited attention span.
However, these children also manifest the same psychiatric symptoms seen in
adult-onset schizophrenic
Answer

________________________________________

1The correct answer is C. The interstitial fluid volume cannot be measured directly
because it occupies the spaces between the cells and is part of the extracellular
fluid volume along with the plasma volume. Interstitial fluid volume is calculated by
subtracting the plasma volume from the extracellular fluid volume. Extracellular
fluid volume was estimated in the subject using inulin as the indicator. Therefore,
interstitial fluid volume = 20 L (inulin space) - 4 L (plasma volume) = 16 L.
Inulin is a reasonable indicator (or marker) for the extracellular space because it
disperses relatively evenly throughout the extracellular fluid, but does not enter the
cells to a significant extent. Because the various substances used to estimate
extracellular fluid volume (e.g., inulin, chloride, sodium, and sucrose) provide
different values, especially when these substances enter the cells (e.g., sodium and
chloride), one often speaks of the inulin space, the sodium space, the chloride
space, or the sucrose space instead of the true extracellular fluid volume.
________________________________________
2The correct answer is A. Case-control studies are retrospective and are as
described in the question stem. Case-control studies allow researchers to compute
an odds ratio.
In cohort studies (choice B), subjects are assembled on the basis of some common
experience (such as attending medical school) and are then monitored for a
specified amount of time at regular intervals (e.g., taking USMLE Steps 1, 2, and 3;
see also longitudinal studies below) until they develop the outcome of interest (they
become practicing physicians) or the follow-up time ends. The cohort study
minimizes many of the biases evident in case-control designs and is the definitive
observational clinical study. Cohort studies allow researchers to compute a relative
risk.
Cross-sectional studies (choice C) usually have more modest goals than those of
case-control and cohort studies. A variable or group of variables is measured in a
sample of a larger population to get an idea of the distribution and
interrelationships of those variables in that population.

Longitudinal studies (choice D) identify individual subjects and follow them over a
given period of time. For example, the study of cholesterol-lowering drugs on
cardiovascular events requires that the same subject is observed over a significant
period of time (e.g., 10 years).
A randomized controlled trial (choice E) is considered the most rigorous and
powerful approach to answering a clinical question in which two treatments,
strategies, or therapies are compared or when one therapy is compared to placebo.
In this type of study, subjects are assigned treatments on a randomized basis
________________________________________
3The correct answer is E. For this question, you need not worry about the actual
value of the airway pressure. There is only one listed answer that would produce a
positive airway pressure, and that is total lung capacity. To measure the airway
pressure, a patient inspires or expires from a spirometer, and then relaxes while his
airway pressure is measured. It is easy to determine whether the airway pressure
would be negative or positive by practicing on yourself. Inhale to total lung capacity
and relax. You will feel the sensation of wanting to blow air out--this creates a
positive pressure in your airways. Any volume above functional residual capacity
(FRC) will create a positive airway pressure, and any volume below FRC will create a
negative airway pressure.
The functional residual capacity (choice A) is the volume of air that remains in the
lung after a normal expiration. The FRC is the equilibrium volume when the elastic
recoil of the lungs is balanced by the tendency of the chest to spring out. Because
this is the volume when the patient is "at rest," the airway pressure is zero. Breathe
out normally (to reach FRC) and notice that there is no pressure in your airways.
Minimal volume (choice B) can only be achieved with an excised lung. It is the
volume of air remaining in an excised lung that is maximally deflated. It is smaller
than the residual volume because the chest wall is not there to help draw the lung
open.
The residual volume (choice C) is the volume of air remaining in the lungs after
maximal expiration. At volumes less than FRC, like residual volume, the airway
pressure would be less than 0 cm H2O. Exhale all the way (to residual volume), and
relax--you will feel the sensation of wanting to draw air in--this creates a negative
pressure in your airways (like a vacuum).
Tidal volume (choice D) is the volume of air that is inhaled or exhaled with each
normal breath.
________________________________________
4The correct answer is B. Clozapine is an antipsychotic drug that has been shown to
cause agranulocytosis. Agranulocytosis is an acute condition characterized by
pronounced leukopenia, with great reduction in polymorphonuclear leukocytes (<
500 cells per mm3). Infected ulcers are likely to form in the throat, intestinal tract,
and other mucous membranes as well as on the skin. Other side effects caused by
clozapine include orthostatic hypotension, sinus tachycardia, hypersalivation,

temperature elevation, lowered seizure threshold, and constipation. Clozapine is


generally prescribed only after several other alternative neuroleptic medications
have failed, because of the possibility of agranulocytosis and the drug's prohibitive
cost.
Chlorpromazine (choice A) is an antipsychotic drug that has antimuscarinic side
effects such as dry mouth and constipation. It can also cause orthostatic
hypotension, sedation, and tardive dyskinesia. It does not cause agranulocytosis.
Fluoxetine (choice C) is a selective serotonin reuptake inhibitor (SSRI), an
antidepressant drug that can cause anxiety and insomnia, altered appetite and
weight loss, activation of mania or hypomania, seizures, and cognitive motor
impairment.
Haloperidol (choice D) is an antipsychotic drug that has less antimuscarinic side
effects than does chlorpromazine, but has more extrapyramidal effects, such as
acute dystonia (face, neck, and back spasms-abnormal posture), parkinsonism,
neuroleptic malignant syndrome (catatonia, rigidity, stupor, fever, dysarthria,
fluctuating BP), and akathisia (restlessness). It can also cause tardive dyskinesia. It
does not cause agranulocytosis.
Imipramine (choice E) is a tricyclic antidepressant drug that can cause orthostatic
hypotension, anticholinergic effects, antihistamine effects, and hypomania. It does
not cause agranulocytosis.
Phenelzine (choice F) is an MAO inhibitor that can cause orthostatic hypotension
(once the body adapts to higher basal levels of catecholamines, it is no longer able
to further vasoconstrict in response to stress), hepatotoxicity, and hypomania. It
does not cause agranulocytosis. Remember the risk of developing a hypertensive
crisis when taking an MAO inhibitor with tyramine-containing foods such as wine
and aged cheeses.
________________________________________
5The correct answer is E. For many years, Whipple's disease was suspected of
having a bacterial etiology because bacterial forms could be seen on electron
microscopy. However, the identity of the causative agent remained elusive. It has
recently been found to be a bacterium which is now named Tropheryma whippelii.
Clostridium difficile (choice A) causes pseudomembranous colitis, generally after
antibiotic administration.
Enterotoxigenic E. coli (choice B) is associated with tropical sprue.
Isospora bella (choice C) is a cause of diarrhea in AIDS patients.
Salmonella sp. (choice D) can cause diarrheal illness after ingestion of contaminated
poultry or beef.
________________________________________

6The correct answer is E. Valproic acid is the drug of choice for specific myoclonic
syndromes because it very effective and it is nonsedating.
Carbamazepine (choice A), ethosuximide (choice C), and phenobarbital (choice D)
are not used for the treatment of this disorder.
Clonazepam (choice B), a benzodiazepine, can be effective in this disorder, but high
doses are generally required, causing marked sedation.
________________________________________
7The correct answer is B. The underlying lesion in retinopathy of prematurity
(retrolental fibroplasia) is an inappropriate proliferation of vessels in the inner layers
of the retina. If the process is disrupted early, the retina may heal with little residual
damage. However, persistence of the process predisposes for permanent damage
due to exudation, hemorrhage, and secondary detachment of the retina.
Ganglion cell and optic nerve degeneration (choices A and C) are features of retinal
damage due to glaucoma.
Pigment deposition in the retina (choice D) is unrelated to retinopathy of
prematurity.
Pigmented epithelium degeneration (choice E) is a feature of retinitis pigmentosa
and also of senile macular degeneration.
________________________________________

8The correct answer is D. This patient is having an acute asthma attack. Asthma is
an obstructive lung disease primarily affecting air movement out of the lungs
(exhalation). The airways (especially the large bronchioles) are hypersensitive to
irritating stimuli, such as allergens and smoke. Local mast cell histamine release
causes mucous secretion and smooth muscle contraction leading to
bronchoconstriction. Certain allergic "triggers" (cats, pollen, hay, etc.) can
precipitate attacks. Symptoms include coughing, difficulty breathing, and wheezing.
Physical examination of the lungs often reveals wheezes and coarse lung sounds,
although in severe cases one may hear only decreased breath sounds and no
wheezes due to very poor air flow. On pulmonary function tests asthmatics show an
obstructive pattern. Both the FEV-1 (maximum volume exhaled in one second) and
the FVC (maximum volume of air that can be exhaled in one breath) are decreased.
The hallmark of obstructive lung disease, however, is a decreased FEV-1/FVC ratio.
The total lung capacity (TLC) is often increased in acute asthma attacks as patients
tend to hyperinflate to obtain more oxygen while being unable to exhale efficiently.
Thus, this patient has a low FEV-1, a low FVC, and a high TLC.
________________________________________
9The correct answer is C. Sleep patterns differ from person to person, however
some generalizations can be made regarding age and sleep. Elderly individuals
have more awakening and arousal at night, they tend to awaken earlier, and have

less total sleep. Multiple factors can contribute to sleep disturbances in the elderly,
including primary sleep disorders, sleep disorders secondary to other physical and
psychiatric conditions, as well as medication-induced sleep problems.
With increased age, the needs for sleep decreases (compare with choice A). A
newborn might sleep up to 22 hours daily, but an adult can generally feel rested
with 6 to 8 hours of sleep.
The amount of time spent in the deepest stage of non-REM sleep is decreased with
age, but the amount of REM sleep is not similarly affected (compare with choice B).
The elderly generally achieve less total nighttime sleep (compare with choice D).
Mild sleep disturbances (compare with choice E) can be associated with normal
________________________________________
10The correct answer is A. The presence of mucosal neuromas, particularly when
multiple and when the patient has a marfanoid syndrome, is a marker for multiple
endocrine neoplasia type IIb (MEN IIb). MEN IIb predisposes for medullary carcinoma
of the thyroid and pheochromocytoma. Pentagastrin-stimulated calcitonin studies
may suggest the presence of thyroid C cell hyperplasia, which may precede frank
medullary carcinoma.
High serum gastrin (choice B) suggests pancreatic or duodenal gastrinomas, which
may be a component of MEN I.
High serum insulin (choice C) or vasoactive intestinal peptide (choice E) also
suggest pancreatic endocrine tumors, which may be a component of MEN I.
Parathyroid hyperplasia or adenomas with parathyroid hormone secretion (choice D)
can be seen in MEN I and MEN IIa, but are not part of MEN IIb.
________________________________________
11The correct answer is B. The half-life of a drug can be determined using the
following equation:
________________________________________
12The correct answer is B. The patient has gastric carcinoma, which has been
strongly linked, in at least some studies, to prior gastric infection with Helicobacter
pylori. H. pylori has also been implicated in the etiologies of gastric peptic ulcer,
chronic gastritis, and (questionably) gastric lymphoma.
Epstein-Barr virus (choice A) has been linked to African Burkitt's lymphoma and
nasopharyngeal carcinoma.
Human papilloma virus (choice C) has been linked to a variety of warts, condyloma,
and genital cancers.
Molluscum contagiosum virus (choice D) is a poxvirus that causes small tumor-like
papules of the skin.

Schistosoma haematobium (choice E) has been linked to bladder cancer.

________________________________________

13The correct answer is D. The cephalic vein arises from the elbow along the
anterior lateral surface of the arm. It can be reliably found in the deltopectoral
groove between the deltoid and pectoralis major. This groove lies diagonally (arising
medially) below the clavicle. The cephalic vein is good for cannulation because its
location is particularly constant.
The cephalic vein is unrelated to the triceps, although it does pass in the arm over
the biceps (choice A).
The cephalic vein is not related to either the triceps or the brachialis (choice B).
The cephalic vein is not related to either the coracobrachialis or the brachialis
(choice C).
The cephalic vein is not related to the sternocleidomastoid (choice E).
________________________________________
14The correct answer is D. The graph indicates that blood flow decreases with
hypoxia. Only the lungs exhibit vasoconstriction in response to hypoxia (pulmonary
hypoxic vasoconstriction). This is an adaptive mechanism that causes blood to
shunt away from regions of the lung which are poorly ventilated (e.g., because of
airway obstruction) to areas which are better ventilated. In other organs,
vasodilation generally occurs in response to hypoxia.
________________________________________
15The correct answer is B. DIC or consumptive coagulopathy represents
pathological activation of the coagulation system by another underlying disease,
with consequent consumption and depletion of the cellular and humoral
components of the coagulation cascade. The fibrinolytic mechanisms are also
activated, and an uncontrolled cycle of bleeding and clotting develops. As a
consequence, levels of all clotting proteins (choices A and C) become depleted,
platelet counts drop (choice E), and the fibrinolytic proteins are depleted as well
(choice D). Fibrin degradation products (choice B), which are normally low in the
serum, rise markedly in this disease; identification of these proteins can be an
important indicator
________________________________________
16The correct answer is E. In the rare pure red cell aplasia, the erythroid marrow
elements are absent or nearly absent, while granulopoiesis and thrombopoiesis
remain unaltered. This condition occurs in both primary and secondary forms, both
of which are thought to be related to autoimmune destruction of erythroid
precursors. There is a relatively weak general association between cancers and red

cell aplasia, which is probably due to triggering of autoimmune disease by the


cancers. Additionally, there is a specific association of which you should be aware
between thymic tumors (thymoma) and autoimmune hematologic diseases,
specifically including pure red cell aplasia. This association is strong enough that a
patient with pure red cell aplasia should be specifically evaluated for thymoma.
The other answer choices are distracters with no specific association with red cell
aplasia, although cancer in general can be a predisposing factor.
________________________________________
17The correct answer is C. The stapedius muscle is innervated by the facial nerve.
This muscle is located in the middle ear and attaches to the neck of the stapes.
Contraction of the stapedius reduces the amplitude of oscillation of the stapes and
thus reduces the perceived loudness of a sound. Paralysis of this muscle may result
in hyperacusis.
The anterior belly of the digastric muscle (choice A) is a muscle in the floor of the
mouth that is innervated by the mandibular division of the trigeminal nerve.
The geniohyoid muscle (choice B) is one of the suprahyoid muscles in the neck. This
muscle is innervated by C1 spinal nerve fibers that travel for a short distance with
the hypoglossal nerve.
The stylopharyngeus muscle (choice D) is one of the longitudinal muscles of the
pharynx that acts to elevate the pharynx. It is innervated by the glossopharyngeal
nerve.
The masseter muscle (choice E) is one of the muscles of mastication. All of the
muscles of mastication are innervated by the mandibular division of the trigeminal
nerve.
________________________________________
18The correct answer is B. The patient has amyloidosis, which can be present in the
manner illustrated. Originally, it was thought that amyloid represented a single
protein. However, as the various settings in which amyloidosis occurs were further
explored, it was discovered that a variety of clearly distinct proteins could produce
the material. Eventually, it was observed that what these diverse proteins had in
common was a beta-pleated sheet tertiary structure that apparently is reflected in
the apple-green birefringence. None of the other properties are thought to
contribute to birefringence when viewed with polarized light.
The ability to bind oxygen (choice A) is important to hemoglobin.
Electrophoretic mobility (choice C) is used to separate serum proteins.
Hydroxyproline content (choice D) is important to collagen structure.
Molecular weight (choice E) is a basic protein property.
________________________________________

19The correct answer is B. The symptoms described are typical for a patient with
glucagon excess. Glucagon is secreted by alpha cells of the pancreatic islets of
Langerhans. Increased levels are rare, usually due to carcinoma (70%) or adenoma
(30%) of the islets. Two-thirds of patients with carcinoma have liver metastasis at
the time of diagnosis. Increased glucagon causes a syndrome similar to diabetes
mellitus, due to antagonism of insulin effects. Patients also have migratory skin
rashes, alopecia, hyperpigmentation of the skin, and glossitis. Diagnosis is made by
measuring glucagon.
The acinar cell (choice A), the main exocrine cell of the pancreas, contains digestive
enzymes. Increases in the release of these enzymes (especially amylase and lipase)
occur with pancreatitis. Acute pancreatitis is considered an emergent medical
condition. Chronic pancreatitis is associated with fibrosis and atrophy of the acinar
structures.
Beta cells (choice C) are responsible for insulin release. An adenoma of beta cells
(insulinoma) would cause hyperinsulinism. 70% are solitary and 10% are multiple.
Insulinomas cause hypoglycemia, dizziness, confusion, and excessive sweating.
Glucose needs to be given promptly to avoid coma and death. Diagnosis is made by
finding increased insulin and hypoglycemia.
Delta cells (choice D) are islet cells that secrete somatostatin. Tumors producing
increased somatostatin are usually malignant. Clinically, a diabetes mellitus-like
syndrome occurs along with diarrhea. Diagnosis is made from elevated serum
somatostatin levels, but can be difficult because of the hormone's short half-life.
The G cell (choice E) is the islet cell that secretes gastrin. Gastrin excess is usually
associated with gastrinomas, 70% of which are malignant. Zollinger-Ellison
syndrome ensues, causing low gastric and duodenal pH, mucosal ulceration, and
diarrhea. Diagnosis is made from high serum gastrin levels.
________________________________________
20The correct answer is D. This is one presentation of syphilis, which can involve
the testis and epididymis in both the acquired and congenital types of the disease.
Characteristically, the testis is usually involved before the epididymis.
Microscopically, the testis may show either gumma formation or the findings
described in the question stem.
Gonorrhea (choice A) usually causes an acute epididymitis and orchitis with
prominent neutrophils.
Mumps orchitis (choice B) would usually be accompanied by parotitis.
Nonspecific orchitis (choice C) is characterized by prominent neutrophils.
Tuberculosis orchitis (choice E) is characterized by granulomas with acid-fast
bacteria.
________________________________________

21The correct answer is C. Aspirin-induced asthma is an infrequent form of asthma.


It is related to the direct pharmacologic action of aspirin on the metabolism of
arachidonic acid. Aspirin inhibits the cyclooxygenase pathway without affecting the
lipoxygenase pathway, leading to a decreased ratio of prostaglandins
(bronchodilators) to leukotrienes (bronchoconstrictors). The disrupted balance
between these two groups of arachidonic acid metabolites leads to
bronchoconstriction in predisposed patients.
Direct release of bronchoconstrictor substances (choice A) is one of several
pathogenetic mechanisms that may mediate occupational asthma, triggered by
inhalation of a number of chemicals, including epoxy resins, plastics, cotton fibers,
toluene, formaldehyde, and penicillin products.
Enhanced vagal stimulation (choice B) plays an essential role in non-atopic asthma.
This variety of asthma, AKA nonreaginic asthma, is initiated by viral infections of the
upper respiratory tract (e.g., common cold and flu), which appear to lower the
threshold of respiratory mucosa to parasympathetic (vagal) stimulation. Recall that
vagal stimulation exerts a bronchoconstrictor influence on the lungs.
Type I hypersensitivity reactions (choice D) are crucial in the pathogenesis of atopic
asthma following prior exposure to a number of allergens. T-cell activation instructs
B cells to produce IgE directed against a given allergen. On re-exposure, IgE on the
surface of mast cells binds the allergen and induces mast cell degranulation. The
mediators released from mast cells cause bronchospasm and recruit more
inflammatory cells, including eosinophils, lymphocytes, and basophils.
Type IV hypersensitivity reactions (choice E) do not occur in asthma.
________________________________________
22The correct answer is C. The septum primum (first interatrial septum) develops
by growing from the cranial end of the embryonic atrium toward the endocardial
cushions. The gap that exists between the two atria during this period is the ostium
primum. As the septum primum continues its growth, the ostium primum gets
smaller. When the septum primum completes its growth and completely fuses with
the endocardial cushions, the ostium primum is closed. Failure of the septum
primum to fuse completely with the endocardial cushions leaves a persistent ostium
primum, known as a primum type atrial septal defect.
The ostium primum does not form within the septum primum (choice A). The ostium
primum is the communication between the two atria that exists during the
formation of the septum primum. That is, the ostium primum is the space within the
developing atrium not yet occupied by the septum primum.
The ostium secundum normally forms within the septum primum (choice B) before
the ostium primum closes by fusion of the septum primum with the endocardial
cushions. Failure of the ostium secundum to form would result in embryonic death,
because there would be no pathway for blood to pass from the right atrium to the
left atrium when the ostium primum closes, thus depriving the embryo of
oxygenated blood.

Most of the septum primum normally disappears. The part that remains forms the
valve of the foramen ovale. This part of the septum primum normally does not fuse
with the septum secundum (choice D) during prenatal life. After birth, the valve of
the foramen ovale is pushed against the septum secundum as a result of the
increased pressure in the left atrium. This achieves functional closure of the
foramen ovale. Fusion does not normally occur at this time, but usually occurs later
in life in most people. In some people, however, complete fusion never occurs
(probe patency).
The septum secundum normally does not fuse with the endocardial cushions (choice
E).
________________________________________
23The correct answer is D. The disease is Wilson's disease. This is an autosomal
recessive abnormality of copper metabolism that can present with either neurologic
(psychiatric symptoms, bradykinesia and rigidity, tremors, or chorea) or hepatic
(hepatitis, cirrhosis, or asymptomatic liver function test abnormalities) findings. The
corneal rings described in the question stem are the pathognomic Kayser-Fleischer
rings due to copper deposition. Treatment is with life-long administration of the
copper-chelating agent, penicillamine.
Deferoxamine (choice A) is used parenterally to chelate circulating iron.
EDTA (choice B) is an effective chelator of divalent (and trivalent) cations such as
lead. It is administered as the Na2CaEDTA salt to avoid hypocalcemia.
Lithium (choice C) is used to treat bipolar disorder.
Phenobarbital (choice E) is used to treat epilepsy.
________________________________________
24The correct answer is C. The posterior tibial artery passes from the posterior
compartment of the leg into the sole of the foot by passing posterior to the medial
malleolus. In this position, it is accompanied by the posterior tibial vein and the
tibial nerve. These neurovascular structures lie between the flexor hallucis longus
tendon and the flexor digitorum longus tendon. Upon entering the foot, the vessels
and nerve divide into medial and lateral plantar branches. The order of the
structures passing behind the medial malleolus are, from anterior to posterior,
tibialis anterior tendon, flexor digitorum tendon, neurovascular bundle, and flexor
hallucis longus tendon [mnemonic: Tom (tibialis posterior), Dick (flexor digitorum
longus tendon), and (artery) Harry (flexor hallucis longus tendon)].
No neurovascular structures pass posterior to the lateral malleolus (choices A and
B). The peroneus longus and brevis tendons pass posterior to the lateral malleolus.
The peroneus tertius tendon lies on the dorsum of the foot and is an extension of
the extensor digitorum tendon.
The flexor hallucis brevis tendon (choice D) does not pass behind the medial
malleolus. It lies entirely within the sole of the foot.

The artery passes behind the medial malleolus but not in this location (choice E).
See explanation above.
________________________________________
25The correct answer is D. Cranial nerve VI is the abducens nerve, which supplies
the abductor of the eye, the lateral rectus. A paralysis of the lateral rectus leads to
unopposed adduction, causing the eye to point toward the nose.
Cranial nerve III (choice A) is the oculomotor nerve, which supplies all of the
muscles of the eye except the superior oblique and lateral rectus. Paralysis of III
would impair abduction, not adduction, of the eye. The eye would tend to rotate
downward and outward.
Cranial nerve IV (choice B) is the trochlear nerve, which supplies the superior
oblique muscle. This muscle serves to depress and abduct (down and out) the
eyeball. Paralysis of IV tends to produce double vision, but does not cause an
obvious deficit in conjugate gaze without careful testing.
Cranial nerve V (choice C) is the trigeminal nerve, which is a mixed sensory and
motor nerve that supplies the face. It provides sensory innervation to the face and
innervates the muscles of mastication. It does not innervate the eye muscles.
Cranial nerve VII (choice E) is the facial nerve, which innervates the muscles of
facial expression, but not the muscles of the orbit. CN VII is additionally involved in
salivation, lacrimation, taste, and general sensation from the external ear.
________________________________________
26The correct answer is A. Because creatinine is freely filtered by the glomerulus,
but not secreted or reabsorbed to a significant extent, the renal clearance of
creatinine is approximately equal to the glomerular filtration rate. In fact, creatinine
clearance is commonly used to assess renal function in the clinical setting. When a
kidney is removed, the total glomerular filtration rate decreases because 50% of the
nephrons have been removed, which causes the creatinine clearance to decrease.
In turn, the plasma creatinine concentration (choice D) increases until the rate of
creatinine excretion by the kidneys (choice E) is equal to the rate of creatinine
production by the body. Recall that creatinine excretion = GFR x plasma creatinine
concentration. Therefore, creatinine excretion is normal when GFR is decreased
following removal of a kidney because the plasma concentration of creatinine is
elevated.
Creatinine is a waste product of metabolism. Creatinine production (choice B) is
directly related to the muscle mass of an individual, but is independent of renal
function.
The daily excretion of sodium (choice C) is unaffected by the removal of a kidney.
The amount of sodium excreted each day by the remaining kidney exactly matches
the amount of sodium entering the body in the diet.
________________________________________

27The correct answer is E. This patient is experiencing the very painful passage of a
renal stone, which is often accompanied by hematuria. His history of recurrent
urolithiasis with calcium-containing stones implies a disorder in the regulation of
calcium concentration. Hyperparathyroidism is associated with increased
parathormone (PTH) levels, which can produce hypercalcemia, hypercalciuria, and,
ultimately, renal stones.
Anemia of chronic disease (choice A) does not produce calcium stones. It is an
attractive distracter because the patient presents with a chronic condition and
hematuria. Note that urinary blood loss is not usually significant enough to produce
an anemic state.
Chronic Proteus infection (choice B) would produce struvite (magnesium-ammonium
phosphate), not calcium stones. Staghorn calculi are also seen.
Factor VIII deficiency (choice C) occurs in hemophilia, a hereditary clotting disorder.
It is not associated with calcium stones.
Hyperaldosteronism (choice D) results in potassium depletion, sodium retention,
and hypertension. Primary hyperaldosteronism (Conn's syndrome) is associated
with adrenocortical adenomas in 90% of patients and is characterized by decreased
renin. Secondary hyperaldosteronism results from excessive stimulation by
angiotensin II that is caused by excess renin production (plasma renin-angiotensin
levels are high). Neither condition is associated with renal stones.
________________________________________
28The correct answer is C. The heart has a Taussig-Bing malformation, which is
considered to be a variant of transposition of the great arteries because the aorta
arises from the morphologic right ventricle.
In the "common pattern" of transposition (choice A), the aorta arises from a
morphologic right ventricle on the right side of the heart, and the pulmonary artery
arises from a morphologic left ventricle on the left side of the heart.
In the "corrected pattern" of transposition (choice B), the aorta arises from a
morphologic right ventricle on the left side of the heart, and the pulmonary artery
arises from a morphologic left ventricle on the right side of the heart.
In tetralogy of Fallot (choice D), the aorta overlies a septal defect, and the proximal
pulmonary artery is stenotic.
In truncus arteriosus (choice E), a single large vessel overlies a ventricular septal
defect.
________________________________________
29The correct answer is E. The patient most likely has candidiasis, which can appear
in any area of the oral mucosa. Inhaled corticosteroids, such as beclomethasone,
are associated with the development of candidiasis in asthmatic patients, especially
those who do not "wash out" their mouth with water after each usage. On the basis
of the description, the patient has the pseudomembranous form of oral candidiasis

or "oral thrush." There is also an erythematous form that presents with flat red and
white lesions that cannot be "rubbed off." Oral candidiasis responds very well to
antifungal therapy. Nystatin is an antifungal agent used locally for treatment of
infections caused by many different Candida species. As a side note, the fact that
the patient is penicillin allergic does not impact the treatment decision; it is only a
distracter.
Acyclovir (choice A) is an antiviral agent used in the treatment of infections caused
by herpes simplex virus types 1 and 2 and varicella-zoster virus. This agent would
be indicated in individuals with herpes zoster infections, which typically appear as
vesicular eruptions and/or ulcers on the cheek, tongue, gingiva, or palate.
If the patient presented with lymphoid hyperplasia of the posterior pharynx covered
by a punctuate or coalescent exudate that is gray, yellow, or white, one might
suspect streptococcal infection, which is associated with severe pharyngitis and
fever. The posterior pharynx is diffusely erythematous with a gray, yellow, or white
exudate. If this were the case, erythromycin (choice D), a macrolide antibiotic,
would be the treatment of choice in a penicillin-allergic patient.
Both amoxicillin (choice B) and cefixime (choice C) are b-lactam antibiotics used to
treat a wide range of bacterial infections. These agents are both known to cause
allergic reactions in penicillin-allergic patients.
________________________________________
30The correct answer is B. Zileuton is a recently approved inhibitor of 5lipoxygenase, the first enzyme that converts arachidonic acid to leukotrienes. Thus
zileuton, which is taken orally, interrupts the leukotriene pathway. Leukotrienes are
synthesized in many inflammatory cells in the airways, such as mast cell
macrophages, eosinophils and basophils. LTC4 and LTD4 are thought to be
responsible for many of the symptoms of asthma, including bronchoconstriction,
increased bronchial reactivity, hypersecretion of mucus, and mucosal edema. In
addition, LTB4 is a potent chemotactic agent for neutrophils. Zileuton and similar
agents are efficacious in the treatment of asthma because of their inhibition of
leukotriene production.
Zafirlukast is another drug that is used to interrupt the leukotriene pathway. This
drug acts as an LTD4 antagonist (choice A); it is taken orally.
Methylxanthines, such as theophylline, inhibit phosphodiesterase (choice C), thus
increasing intracellular levels of cAMP. This results in smooth muscle relaxation. At
therapeutic doses, methylxanthines also block adenosine receptors.
Corticosteroids prevent the release of arachidonic acid from cell membranes by
inhibiting phospholipase A2 (choice D). This reduces the production of both
leukotrienes and prostaglandins. Corticosteroids also inhibit the production of
cytokines which are thought to play an important role in initiating the inflammatory
cascade provoked by antigen inhalation and viral infection. Examples of
corticosteroids include beclomethasone, budesonide, flunisolide, fluticasone and
triamcinolone.

________________________________________
31The correct answer is D. Autoimmune destruction of parietal cells would lead to
decreased secretion of gastric acid and intrinsic factor. The diminished availability of
intrinsic factor would result in poor absorption of dietary vitamin B12. Over time, the
vitamin B12 deficiency could lead to pernicious anemia, which is characterized by
increased production of macrocytes (megaloblasts) by the bone marrow.
Because of the decrease in gastric acid secretion, luminal bacteria (choice A) would
most likely exhibit increased (not decreased) growth. One of the functions of HCl
secreted by the parietal cells is to sterilize the gastric lumen.
Patients with Type A gastritis have an increased likelihood of developing gastric
carcinoma (not decreased, choice B).
A decrease in acid secretion leads to increased secretion of gastrin (not decreased,
choice C) by antral G cells. This is because low gastric pH (less than 3) inhibits
gastrin secretion via paracrine release of somatostatin from cells in the gastric
mucosa that can sense the acidity. With decreased parietal cells, the pH of the
gastric lumen would rise and remove this inhibitory component.
Because less acid would be delivered to the duodenum with parietal cell
destruction, less secretin would be released into the blood.. This would result in
decreased pancreatic bicarbonate secretion (not increased, choice E).
________________________________________
32The correct answer is D. The 3' end of a eukaryotic intron contains a splice
acceptor site, with an invariant AG just before the end of the intron. This sequence
is highly conserved, as it is essential for correctly recognizing and splicing out the
intervening sequences, or introns, from the nascent RNA transcript prior to
transport out of the nucleus. At the 5' end of the intron is an equally important GT
(GU in RNA) sequence that is also necessary for splicing (splice donor site).
Capping (choice A) occurs almost immediately after synthesis of the first 30
nucleotides or so. The triphosphate of GTP condenses with the available 5'
diphosphate on the growing RNA chain to form a "cap" recognized during protein
synthesis that also protects the RNA from degradation.
Hybridization (choice B) is the process by which two molecules of nucleic acid
anneal to each other based on nucleotide base-pairing. Alteration of one nucleotide
in the intervening sequence shown will produce only very minor effects on
hybridization.
In the polyadenylation process (choice C), an AAUAAA sequence near the 3' end of
the RNA transcript is recognized, the RNA is cleaved by an endonuclease, then a
poly-A polymerase adds 100 to 200 adenylate residues to the RNA. Failure to
recognize this sequence would result in failure of polyadenylation.
Transcription (choice E), the process by which DNA is "read" to yield RNA via the
actions of RNA polymerase, should not be affected by the mutation.

________________________________________
33The correct answer is D. Primary aldosteronism (Conn's syndrome) is a condition
of hyperaldosteronism originating in the adrenal gland. The causes include an
aldosterone-secreting adrenocortical adenoma, hyperplasia of the zona
glomerulosa, and very rarely, an adrenal carcinoma. It is characterized by
hypertension secondary to sodium retention, hypokalemia, and a decreased serum
renin due to a negative feedback of increased blood pressure on renin secretion.
Cushing's syndrome (choice A) is the result of increased glucocorticoid production,
particularly cortisol. Physical signs typically include "moon facies," truncal obesity,
"buffalo hump," and purple abdominal striae.
Diabetes mellitus (choice B) is a condition of inadequate insulin production that
presents with hyperglycemia and ketoacidosis.
Pheochromocytoma (choice C) is a rare tumor of chromaffin cells occurring most
commonly in the adrenal medulla. The tumor secretes epinephrine and
norepinephrine, resulting in secondary hypertension.
Secondary aldosteronism (choice E) results from an activation of the reninangiotensin system caused by renal ischemia, edema, and renal tumors. In contrast
to primary aldosteronism, secondary aldosteronism is associated with increased
serum renin.
________________________________________
34The correct answer is E. The thyroid lesion is medullary carcinoma of the thyroid.
The coexistence of parathyroid hyperplasia suggests Sipple's syndrome (MEN II).
These patients also tend to develop pheochromocytoma. Both MEN II and MEN III
are associated with the ret oncogene.
Associate bcl-2 (choice A) with follicular and undifferentiated lymphomas.
Associate C-myc (choice B) with Burkitt's lymphoma.
Associate erb-B2 (choice C) with breast, ovarian, and gastric carcinomas.
Associate L-myc (choice D) with small cell carcinoma of the lung.
________________________________________

35The correct answer is E. Diuretic therapy has resulted in the type of metabolic
alkalosis called "contraction alkalosis." Loop diuretics increase salt and water
excretion by inhibiting tubular reabsorption of Na+ and Cl- in the kidney. The salt
and water that are lost from the body contain very little HCO3- so that virtually all of
the HCO3- contained in the ECF is retained in the body. In effect, the HCO3- present
in the ECF (which includes the edema fluid) becomes more and more concentrated
as urine containing relatively little HCO3- is excreted from the body. For example, if
the ECF volume before diuretic therapy were 16 L, the total amount of HCO3- in the
ECF would be 384 mEq (16 L x 24 mEq/L = 384 mEq). The excretion of 5 L of HCO3--

free urine would cause the HCO3- contained in the ECF to be concentrated into 11 L,
thereby increasing the HCO3- concentration in the ECF to 35 mEq/L (384 mEq/11 L
= 35 mEq/L). Therefore, it is the loss of extracellular fluid from the body that
increases the concentration of HCO3- in the plasma.
Because only small amounts of HCO3- are normally excreted in the urine,
decreasing HCO3- excretion (choice A) would have a small effect on the
concentration of HCO3- in the plasma.
Diuretics lead to increased levels of aldosterone (choice B) and potassium (ie,
hypokalemia, not hyperkalemia, choice C) in the plasma by a variety of
mechanisms.
Hyperventilation (choice D) decreases arterial PCO2; note that diuretic therapy has
increased arterial PCO2.
________________________________________
36The correct answer is D. Suspect fracture of the scaphoid (navicular) bone in any
young adult who has "fallen on an outstretched hand" (classic clue). The physician
was concerned about the risk of avascular necrosis because, in some people, the
blood supply of the bone is located distally and a fracture would deprive the
proximal region of the bone of its arterial nourishment. Osteoarthritis is a common
complication of scaphoid fractures that do not heal properly.
The lunate bone (choice B) is commonly dislocated in patients who have fallen on
an outstretched hand. Median nerve injury is a frequently associated occurrence.
The capitate (choice A), pisiform (choice C), and trapezoid (choice E) bones of the
wrist are less commonly associated with falls on an outstretched hand.
________________________________________
37The correct answer is A. Acute mastitis, frequently complicated by breast abscess
formation, typically develops in the postpartum period, when the nipples are first
subjected to the physical stresses of breast feeding. Bacteria enter the breast via
cracks in the nipple, and flourish in the microenvironment of the lactating breast.
Acute mastitis causes redness, pain, and swelling in the affected breast; Staph.
aureus is the most common pathogen.
Although endocarditis (choice B) could send septic emboli to the breast, Staph.
aureus endocarditis is more commonly associated with bacteremia and fevers,
proliferative glomerulonephritis, valvular dysfunction, and emboli to the brain,
kidneys, heart, and gut.
Inflammatory breast carcinoma (choice C) does not invoke an inflammatory
response, and it is not associated with an infection. Inflammatory breast carcinoma
is characterized by tumor spread into the dermal lymphatics, producing diffuse
induration and skin tenderness with the typical peau d'orange appearance.
Mastitis arising in the perimenopausal period (choice D) is usually a chronic mastitis,
caused by obstruction of ducts due to inspissated secretions. Chronic mastitis is

sterile. The breast tissue shows lymphocytes and plasma cells surrounding dilated
ducts filled by cellular debris.
Paget's disease (choice E) presents as skin changes on the breast or nipple, in
association with ductal carcinoma in the underlying breast that is percolating out
into the epidermis. Paget's disease produces a eczematous skin lesion that may be
crusted or weeping, but it is not related to bacterial infection.
________________________________________
38The correct answer is E. About 65% of the water filtered by the glomeruli (180
L/day) is reabsorbed in the proximal tubule, 15% in the loops of Henle (choice C),
10% in the distal tubules (choice B), and less than 10% in the collecting ducts
(choices A and D); about 1 L of water is normally excreted as urine each day. The
amount of water reabsorbed in the proximal tubule and loop of Henle is not affected
by ADH, because ADH does not affect tubular permeability in these segments of the
nephron. However, ADH increases the permeability of the distal tubules and
collecting duct, which increases reabsorption of water. When ADH levels are high,
the urine output can decrease to less than 0.5 L/day; when ADH levels are low, the
output of urine can increase to more than 30 L/day. Even at these extremes,
however, most of the water in the glomerular filtrate is still reabsorbed in the
proximal tubule
________________________________________
39The correct answer is B. The carpal bone that has been dislocated is the lunate,
one of the four proximal carpal bones. This bone has been dislocated into the carpal
tunnel, which is anterior to the carpal bones. The median nerve is the only nerve
that traverses the carpal tunnel.
The anterior interosseous nerve (choice A) is a branch of the median nerve that
arises in the proximal forearm and innervates the deep muscles of the anterior
forearm. This nerve does not traverse the carpal tunnel.
The posterior interosseous nerve (choice C) is a branch of the radial nerve that
arises in the proximal forearm and innervates several muscles of the posterior
compartment of the forearm. This nerve does not traverse the carpal tunnel.
The radial nerve (choice D) innervates posterior muscles of the arm, some posterior
muscles of the forearm, and areas of skin on the posterior surface of the upper limb.
It does not pass through the carpal tunnel.
The ulnar nerve (choice E) innervates some muscles in the anterior compartment of
the forearm and many muscles in the hand. The nerve crosses the wrist superficial
to the flexor retinaculum and therefore is not within the carpal tunnel. It is spared
compression by the dislocation of the lunate.
________________________________________
40The correct answer is A. Neuroblastoma is a childhood tumor of the sympathetic
nerves, which most commonly arises along the paravertebral chain or within the
adrenal medulla. Neuroblastoma has two characteristic genetic markers: a 1p

deletion and an N-myc translocation. N-myc is an oncogene that resides on


chromosome 2p, and is amplified up to 300 times in neuroblastoma. Amplification of
this gene is associated with a poorer prognosis.
Cellular aneuploidy (choice B) and hyperdiploidy reflect a better prognosis for
neuroblastoma, whereas diploid and tetraploid tumors have an intermediate or poor
prognosis.
Tumor presentation at a younger age predicts a better outcome for neuroblastoma.
Children under one year of age (choice C) have an excellent prognosis, regardless of
tumor stage.
Ganglion cells (choice D) within the tumor are a reflection of focal differentiation of
the neuroblastoma into ganglioneuroma. Better differentiated tumors have a more
favorable outcome.
In general, tumor stage is not very predictive of neuroblastoma survival.
Nonetheless, higher grades of tumor (III and IV) extend across the vertebral midline
and into distal viscera or nodes (choice E).
________________________________________
41The correct answer is B. Entamoeba histolytica is an intestinal amoebic parasite
that either can cause relatively mild diarrhea, or can behave more aggressively,
causing dysentery (bloody diarrhea with abdominal pain and dehydration),
peritonitis, or liver abscess formation (such as this patient has). Treatment is with
metronidazole.
Cryptosporidium parvum (choice A), Giardia lamblia (choice C), and Isospora belli
(choice D) can cause chronic diarrhea, but would not be expected to cause a liver
abscess.
Trichomonas vaginalis (choice E) is not a cause of diarrhea, but instead causes
vaginitis.
________________________________________
42The correct answer is D. Nodular glomerulosclerosis (Kimmelstiel-Wilson disease),
as described above, is the most distinctive glomerular lesion associated with
diabetes mellitus. Other glomerular findings in diabetes include widespread
capillary basement membrane thickening and diffuse glomerulosclerosis.
Minimal change disease, the most common cause of nephrotic syndrome in
children, is characterized by the podocyte changes described in choice A.
Hyperplastic arteriolitis (choice B) is associated with hypertensive damage to the
kidneys. Other features include fibrinoid necrosis and necrotizing glomerulitis.
Goodpasture syndrome is associated with linear deposits of IgG along the basement
membrane (choice C).

Membranous glomerulonephritis is characterized by PAS-positive, electron-dense


deposits distributed principally along the epithelial side of the capillary basement
membrane (choice E).
________________________________________
43The correct answer is B. The only nerve root that is common to all the described
symptoms is C6. Wrist flexion is dependent on nerve roots C6, C7, C8 and T1; elbow
flexion is dependent on C5, C6, C7; elbow extension is dependent on C6, C7, C8;
arm abduction is dependent on nerve roots C5, C6; and sensation over the deltoid is
mediated by C5 and C6. The palmar surface of the 1st three digits derives its
sensory innervation from the median nerve, C6, C7, and C8.
C5 (choice A) is incorrect because it is not involved in elbow extension, wrist flexion,
or providing sensation to the hand.
C7 (choice C) is incorrect because it is not involved in abduction of the arm.
C8 (choice D) is incorrect because it is not involved in elbow flexion or abduction of
the arm.
T1 (choice E) is involved in wrist flexion, but not the other functions listed.
________________________________________
44The correct answer is A. The patient is probably suffering from
choledocholithiasis, a condition in which a gallstone becomes lodged in the common
bile duct. She is displaying "Charcot's triad" (fever, jaundice, and right upper
quadrant pain), which is indicative of cholangitis (infection of the biliary tree
proximal to an obstruction such as a gallstone or malignancy). Gallstones are very
common, occurring in as many as 15-20% of the general population. The most
common type of stone contains cholesterol, which precipitates from supersaturated
bile within the gallbladder. Some risk factors for cholesterol stones are increasing
age, rapid weight loss, oral contraceptive use, and either disease of or resection of
the terminal ileum (the site at which bile salts are reabsorbed). Pigmented
gallstones made of calcium bilirubinate are less common and occur in patients with
hemolytic disorders and certain types of biliary tract infections.
The key point in this case is the fact that the patient is jaundiced, eliminating all
choices other than a stone in the common bile duct. Stones within the cystic duct
(choice B) or gallbladder (choice C) do not cause jaundice.
A stone within the small intestine (choices D and E) could cause jaundice only if it
were very large and physically obstructing the biliary tree from within the intestinal
lumen, which would be very unlikely.
________________________________________
45The correct answer is C. Two pieces of information are necessary to answer this
question. First, you must know that the postcentral gyrus of the parietal lobe is the
primary sensory cortex. Second, you must be familiar with the homunculus of the
primary sensory cortex, which is very similar to that of the primary motor cortex.

The foot area is located on the medial aspect of the sensory homunculus in the
interhemispheric fissure. Therefore, stimulation of this area would cause her to feel
her foot.
Stimulation of her visual cortex in the occipital lobe might produce flashes of light
(choice A).
Stimulation of the lateral aspect of the primary sensory cortex (the postcentral
gyrus) could produce tactile sensation in her hand (choice B).
Stimulation of the lateral aspect of the primary motor cortex (the precentral gyrus)
could produce hand movement (choice D).
Stimulation of the medial aspect of the motor cortex (in the interhemispheric
fissure) could cause her foot to move (choice E).
________________________________________
46The correct answer is E. The suboccipital triangle lies in the neck at the base of
the skull. The suboccipital triangle is bounded by the inferior oblique, rectus major,
and superior oblique muscles. The vertebral artery can be found within the
suboccipital triangle, lying on the posterior arch of the atlas, lateral to the midline,
typically at about the level of the lower portion of the ear lobe. Care should be taken
in accessing the vessel, since the first cervical nerve lies between the vertebral
artery and the posterior arch of the atlas.
The anterior triangle of the neck (choice A) is in the anterior neck, and is bounded
by the neck midline, the mandible, and the sternocleidomastoid muscle. The
anterior triangle of the neck is subdivided into the muscular, carotid,
submandibular, and submental triangles.
The small muscular triangle (choice B) lies between the superior belly of the
omohyoid and the sternohyoid muscle.
The posterior triangle of the neck (choice C) is on the side of the neck, and is
bounded by the sternocleidomastoid muscle, trapezius, and the middle third of the
clavicle.
The submental triangle (choice D), also called the suprahyoid triangle, is found
below the lower jaw, and is bounded by the hyoid bone and anterior bellies of the
digastric muscles.
________________________________________
47The correct answer is C. The disease is neurofibromatosis (von Recklinghausen's
disease), which is characterized by caf-au-lait spots (the light brown macules in the
question stem), Lisch nodules (pigmented iris hamartomas), and multiple peripheral
nerve tumors (neurofibromas). The neurofibromas are initially benign, but may
undergo malignant transformation.

Tumors of the bladder (choice A) are usually acquired rather than familial. Cigarette
smoking, Schistosoma infection, exposure to industrial substances, and analgesic
abuse may all predispose to bladder neoplasia.
Gardner's syndrome, a familial form of colon cancer (choice B), can be accompanied
by osteomas, epidermal cysts, fibromatosis, abnormal teeth, and thyroid and
duodenal cancer.
Caf-au-lait spots are not considered to be premalignant lesions of skin (choice D).
Basal cell nevus syndrome and xeroderma pigmentosa are examples of genetic
conditions that predispose for skin cancer.
Stomach tumors (choice E) are usually acquired, although some familial polyposis
syndromes may also be associated with gastric polyps.
________________________________________
48The correct answer is B. The most common breast mass in men, especially under
25 years of age, is gynecomastia-a benign proliferation of ductal and stromal
elements of the breast. It is generally an idiopathic condition, probably related to
pubertal hormonal changes.
Fibrocystic changes (choice A) in the breast, which reflect physiological responses in
the breast to cyclical levels of sex hormones, are not observed in men. Fibrocystic
changes are most common in the late reproductive years and include fibrosis, cyst
formation, and a variety of epithelial changes such as hyperplasia and apocrine
metaplasia.
Intraductal papillomas (choice C) are benign neoplasms commonly evolving in the
major lactiferous ducts beneath the nipple. They most commonly present with a
bloody nipple discharge and are rare in men.
Carcinoma of the male breast (choices D and E) is rare, and almost always develops
in the breasts of elderly men. Male breast carcinomas have a somewhat worse
prognosis than their female counterpart. Grossly and microscopically they resemble
ductal carcinoma in the female.
________________________________________
49The correct answer is A. The increase in muscle blood flow that occurs during
exercise is caused by dilation of the arterioles (i.e., decreased arteriolar resistance).
In normal skeletal muscles, the blood flow can increase as much as 20-fold during
strenuous exercise. Most of this increase in blood flow can be attributed to the
dilatory actions of metabolic factors (e.g., adenosine, lactic acid, carbon dioxide)
produced by the exercising muscles.
Exercise causes the concentration of carbon dioxide (choice B) and lactic acid
(choice C) to increase in the muscles.
Mass discharge of the sympathetic nervous system (choice D) occurs throughout
the body during exercise, causing arterioles to constrict in most tissues. The

arterioles in the exercising muscles, however, are strongly dilated by vasodilator


substances released from the muscles.
A decrease in vascular conductance (choice E) occurs when the vasculature is
constricted. Resistance and conductance are inversely related, so that a decrease in
arteriolar resistance is associated with an increase in arteriolar conductance.
________________________________________
50The correct answer is E. The thyroid disease is Hashimoto's thyroiditis, an
autoimmune disease in which the thyroid parenchyma is destroyed by a
lymphocytic infiltrate. The infiltrate typically contains mature follicles; the remaining
scanty follicular cells often have eosinophilic granular cytoplasm and are called
Hurthle cells or oncocytes. Clinically, patients usually have hypothyroidism,
although brief periods of hyperthyroidism ("Hashitoxicosis") may also be seen. Like
myasthenia gravis, Hashimoto's disease may be associated with thymic disorders
including thymic hyperplasia, benign thymomas, and malignant thymomas.
Colon (choice A) cancer is associated with ulcerative colitis and adenomatous
polyps.
The risk of esophageal cancer (choice B) is increased with Barrett's esophagus and
in Plummer-Vinson syndrome.
You should associate neurofibromas of peripheral nerve (choice C) with caf au lait
spots on the skin.
Skin cancer (choice D) occurs with greater frequency in association with xeroderma
pigmentosa and actinic keratosis.
Answers

________________________________________

________________________________________

1>The correct answer is F. This is a classic presentation for Hodgkin's disease, a


form of lymphoma characterized by neoplastic proliferation of Reed-Sternberg cells
admixed with variable numbers of reactive lymphocytes, neutrophils, and
eosinophils. About half the patients have usually painless adenopathy in the neck or
supraclavicular area at the time of diagnosis. A minority have constitutional
symptoms such as fatigue, weight loss, and night sweats, but these can be
important clues. Staging of Hodgkin's disease is based on the extent of spread and
the presence or absence of constitutional symptoms. The man in question has
involvement of cervical lymph nodes, mediastinal nodes, and the liver at the time of

diagnosis, so his disease would be stage III (involvement of lymph nodes or


extralymphatic organs on both sides of the diaphragm). The presence of
constitutional symptoms makes this stage IIIB (if constitutional symptoms were
absent, it would be IIIA [choice E]).
Stage I disease (choices A and B) is limited to a single lymph node region or a single
extralymphatic organ.
Stage II disease (choices C and D) can involve two or more lymph node regions on
one side of the diaphragm, or can involve contiguous extralymphatic organs or
tissues.
Stage IV disease (choices G and H) is defined by the presence of multiple or
disseminated disease foci in extralymphatic organs or tissues. Lymphatic
involvement may occur, but need not be present for the diagnosis
The correct answer is B. Prostatitis can be acute, chronic bacterial, or chronic
abacterial. Laboratory investigations, which are essential in differentiating among
these forms, rely on demonstration of leukocytes and/or bacteria in the expressed
prostatic secretions. A diagnosis of chronic abacterial prostatitis is made when 1)
more than 10 leukocytes per high-power field are present on microscopic
examination of prostatic secretions and 2) no bacteria are isolated from cultures of
prostatic secretions. This is the most common form of chronic prostatitis. Its
pathogenesis is obscure, although microorganisms such as Mycoplasma hominis,
Chlamydia trachomatis, and Ureaplasma urealyticum have been implicated, but
never proven, as etiologic agents. PSA levels may be elevated in prostatitis.
Acute bacterial prostatitis (choice A) manifests with systemic signs and symptoms
of acute infection (fever, chills, and malaise), perineal pain, and extreme tenderness
of the prostate on rectal examination. Numerous leukocytes are seen on
microscopic examination of expressed prostatic secretions. Cultures lead to isolation
of the offending agent, usually Escherichia coli, providing a target for antibiotic
therapy. Clinical history alone excludes acute prostatitis as the cause of this
patient's complaints.
Chronic bacterial prostatitis (choice C) is clinically indistinguishable from chronic
abacterial prostatitis, except for a history of recurrent urinary tract infections in the
former. Cultures of prostatic secretions are positive in chronic bacterial prostatitis,
allowing identification of infectious agents, usually the same as the ones causing
prior urinary tract infections (most often E. coli).
Prostatic adenocarcinoma (choice D) is a very common malignant tumor of elderly
men. It tends to develop from the peripheral zone of the prostate, manifesting with
firm nodules appreciable on rectal examination and osteoblastic metastases to the
vertebral column. Perineal pain and increased frequency of urination are rare
manifestations.
Prostatic hyperplasia (choice E) manifests with increased frequency and other
urinary symptoms, but pain is generally absent. Leukocytes are not demonstrated in
expressed prostatic secretions (unless there is concomitant prostatitis).

Remember: > 10 leukocytes/high-power field = prostatitis


________________________________________
2The correct answer is A. These are autonomic motor ganglion cells and nerve
fibers surrounded by smooth muscle of the muscularis externus. The close proximity
of the smooth muscle and the orientation of the fibers are keys to deciding that this
is a myenteric plexus. Ganglion cells usually have a spherical cell body, although
they are considered multipolar, with a round nucleus and a prominent "owl's eye"
nucleolus. The nerve fibers typically appear wavy in light microscopic views. The
reduction in number of these cells and associated fibers leads to the lack of
peristaltic control in the small and large bowel. The aganglionic bowel segment is
narrowed because the lack of peristalsis keeps stool from moving into that segment.
The distal rectum is always involved, and the lesion can extend proximally
anywhere from a few centimeters past the rectum all the way up to the small
intestine. The bowel proximal to the lesion is usually dilated.
In kwashiorkor (choice B), a condition of extreme malnutrition, protein-secreting
cells atrophy. The cells in this figure do not contain protein granules. In such cells,
the rough endoplasmic reticulum would also be abundant, and this is not evident
here.
In polio (choice C), motor neurons are lost from the anterior horn of the spinal cord.
Schwann cells produce schwannomas (choice D). Schwann cells are present among
the nerve fibers, but they are not abundant, and they are not specifically affected in
Hirschsprung's disease.
Vitamin C deficiency leads to scurvy (choice E), characterized by abnormal collagen
hydroxylation. The wavy fibers do not have the typical bundle arrangement of
collagen, but rather, have an appearance that is characteristic of nerve
________________________________________
3The correct answer is E. Zollinger-Ellison syndrome is caused by a gastrin-secreting
tumor (gastrinoma). Gastrinomas may occur in the pancreas or duodenum, but may
have other locations as well. Gastrin is normally secreted from the gastric mucosa
and is elevated postprandially, but in patients with Zollinger-Ellison syndrome, the
extra-gastric location of the tumor does not allow food in the gastric lumen to have
an effect on gastrin secretion. Thus, one should expect little to no change in
circulating gastrin levels after eating in a patient with gastrinoma. In G-cell
hyperplasia (which should be considered in the differential diagnosis of gastrinoma),
serum gastrin levels typically increase after a test meal.
Pentagastrin (choices B, D, and F) is a synthetic, 5-amino-acid peptide that is used
in testing gastric acid secretion. It is not found in man.
________________________________________
4The correct answer is B. A persistent truncus arteriosus results from a failure of the
aorticopulmonary septum to form. The aorticopulmonary septum divides the
truncus arteriosus into the ascending aorta and the pulmonary trunk. If this septum

fails to form, the single truncus arteriosus will receive blood from both the right and
left ventricles, allowing the deoxygenated blood to be mixed with the oxygenated
blood. The systemic circulation therefore receives blood that is not fully oxygenated,
causing cyanosis.
A patent ductus arteriosus (choice A) results in blood passing from the aorta to the
pulmonary trunk postnatally. This left-to-right shunt does not cause cyanosis.
Cyanosis is caused by right-to-left shunts. Prenatally, the ductus arteriosus allows
the passage of blood from the pulmonary trunk to the aorta. After birth, when the
pressure gradient reverses and the pressure is higher on the left side of the heart,
the flow in a patent ductus arteriosus reverses and becomes a left-to-right shunt.
Atrial septal defects (choices C and D) allow for left-to-right shunting of blood
postnatally because the pressure is higher in the left atrium than in the right atrium.
Only right-to-left shunts cause cyanosis.
Ventricular septal defects (choice E) also produce a left-to-right shunt because of
the left-to-right pressure gradient. Therefore, these are noncyanotic defects
________________________________________
5The correct answer is E. This patient presents with symptoms suggestive of acute
infection (elevated temperature, greenish-yellow sputum) on a background of
chronic bronchitis, which is common in smokers. Hyperplasia and hypertrophy of
mucous glands in chronic bronchitis causes them to be present at deeper levels in
the bronchial wall than usual. The ratio of the gland depth to the total thickness of
the bronchial wall is termed the Reid index, which would be increased in this
patient.
Apical cavitary lesions (choice A) might be indicative of cavitary tuberculosis. This
condition is not associated with excessive mucus production. Hemoptysis and
weight loss might also be expected as clinical findings.
Curschmann spirals (choice B) are found in asthmatic patients and represent mucus
casts of small airways. This patient does not have the typical episodic history of
acute asthmatic attacks with acute dyspnea as the major clinical problem.
Elevated sodium chloride levels in sweat (choice C) are present in cystic fibrosis.
This condition has an onset in early life and is associated with excessive production
of thick mucus, which predisposes to infection of the airways and permanent
damage.
Enlarged hilar lymph nodes (choice D) might suggest bronchogenic carcinoma or a
granulomatous process, which would be less likely than chronic bronchitis. In
addition, patients with carcinoma often present with hemoptysis and weight loss,
rather than excessive mucus production.
________________________________________
6The correct answer is B. The P50 is defined as the partial pressure of oxygen
necessary to bind 50% of available hemoglobin. CO will decrease the P50 of Hb for
O2. Another way of stating this is that CO left-shifts the oxygen-hemoglobin

dissociation curve. CO is deadly because it not only binds hemoglobin with a greater
affinity than does O2 (240 times better), it also left-shifts the curve, thereby making
it more difficult to unload O2 in peripheral tissues.
CO will not diminish the diffusing capacity of the lung (choice A). CO is routinely
used to measure diffusing capacity.
The concentration of oxygen in arterial blood will be approximately the same for
both cases (choice C). If anything, the concentration will be slightly greater in the
patient with CO poisoning because CO2 left-shifts the curve, and therefore the
available binding sites of Hb for O2 will be slightly more saturated.
CO does not change the rate of O2 binding to Hb (choice D). It does, however, bind
to hemoglobin with a 240 times greater affinity than does oxygen.
Neither of these conditions will cause arterial hypoxemia (choice E).
________________________________________
7The correct answer is C. Trihexyphenidyl is a muscarinic antagonist used as
adjunctive therapy in Parkinson's disease. It can improve tremor and rigidity, but
has little effect on bradykinesia. Trihexyphenidyl is given to block cholinergic tone in
the striatum, and therefore, helps to maintain the dopamine/acetylcholine balance
in this region. However, this drug and similar agents that block muscarinic receptors
also block parasympathetic tone to peripheral end organs, producing a number of
side effects. One such side effect is a reduction of salivation, leading to a dry
mouth.
Trihexyphenidyl would cause decreased sweating, not diaphoresis (choice A), by
blocking sympathetic cholinergic (muscarinic) tone to sweat glands.
Trihexyphenidyl would cause constipation, not diarrhea (choice B), by blocking
parasympathetic tone to the gut.
Trihexyphenidyl would cause mydriasis, not miosis (choice D), by blocking
parasympathetic tone to the pupillary sphincter muscle of the eye.
Trihexyphenidyl would cause urinary retention, not urinary incontinence (choice E),
by blocking parasympathetic tone to the bladder.
________________________________________
8The correct answer is A. The combination of edema and frothy urine suggests
nephrotic syndrome. Urine with high protein content is more able to form stable
bubbles than is normal urine, and may be a clue that a patient (or parent) notices.
Chloride (choice B) in urine cannot be detected grossly, and would not cause frothy
urine.
Some people can detect a sweet smell to urine that contains large amounts of
glucose (choice C), and Greek physicians were known to taste urine to diagnose
diabetes, but the presentation suggests nephrotic syndrome.

Sodium (choice D) in urine cannot be detected grossly, and would not cause frothy
urine.
Very concentrated urine with a high urea (choice E) content looks darker in color,
and is not necessarily frothy.
________________________________________
9The correct choice is D. This patient is most likely suffering from cytomegalovirus
(CMV) retinitis. The best drug treatment for this infection is ganciclovir.
Acyclovir (choice A) is not effective in CMV infections. It is used more for HSV type 1
and 2 infections.
Amantadine (choice B) is used either therapeutically or prophylactically for the
influenza A virus.
Flucytosine (choice C) is an antifungal agent.
Zidovudine (choice E) is a first-line drug for the treatment of AIDS. The drug by itself
is ineffective against CMV retinitis.
________________________________________
10The correct answer is B. The patient has a femoral hernia. This hernia forms by
passage of a loop of bowel into the saphenous opening of the fascia lata and then
through the cribriform fascia, to bulge anteriorly under the skin over the saphenous
opening. Femoral hernias are much more common in women than in men. The
major risk of this type (and most other types) of hernia is strangulation of the
herniated small intestine, which may result in death of the involved tissue with
significant risk of subsequent life-threatening peritonitis and sepsis.
A direct inguinal hernia (choice A) represents intestinal structures passing through a
gap in the abdominal wall between transversus abdominis and the internal oblique
muscles; these structures do not pass through the deep ring.
An incisional hernia (choice C) occurs where a surgical incision has weakened the
abdominal wall.
An indirect inguinal hernia (choice D) represents extrusion of abdominal contents
through the deep ring, through the inguinal canal, to emerge at the superficial
inguinal ring. During physical examination, the hernia can sometimes be reduced by
the patient lying on his/her back; if the physician applies pressure to the site of the
deep ring, reherniation will temporarily be prevented, verifying the diagnosis of
indirect inguinal hernia.
An umbilical hernia (choice E) represents a slight herniation of the midgut at the site
of insertion of the umbilical cord.
________________________________________

11The correct answer is C. Nuchal rigidity suggests meningitis. It is convenient to


classify meningitis based on the cerebrospinal fluid (CSF) findings: 1) acute
pyogenic meningitis if neutrophils are markedly increased; 2) acute lymphocytic
meningitis if lymphocytes (alone) are markedly increased, and 3) chronic meningitis
if lymphocytes, plasma cells, macrophages, and fibroblasts are increased. This
patient has chronic meningitis. The classical cause of chronic meningitis is
tuberculosis, whose etiologic agent is Mycobacterium tuberculosis. Other causes
include other indolent meningeal infections such as syphilis, brucellosis, and chronic
fungal infections. The granulomas that are characteristic findings in other tissues
may or may not be present in the meningeal tissue, and are usually not
recognizable in CSF. Tubercular meningitis is now uncommon in this country. In
immigrants from third world countries, a history of pulmonary tuberculosis may be
distant, undiagnosed, or deliberately concealed.
Herpes virus (choice A) and mumps virus (choice B) are causes of acute lymphocytic
meningitis.
Neisseria menigitidis (choice D) and Streptococcus pneumoniae (choice E) are
causes of acute pyogenic meningitis.
________________________________________
12The correct answer is A. The two principal causes of rapidly progressive
glomerulonephritis are anti-glomerular basement membrane (including both
Goodpasture's syndrome and isolated anti-glomerular basement disease) and
primary systemic vasculitis (including Wegener's granulomatosis, microscopic
polyarteritis, idiopathic rapidly progressive glomerulonephritis, Churg-Strauss
syndrome, polyarteritis nodosa, giant-cell arteritis, and Takayasu's arteritis). A very
large variety of other systemic and primary glomerular disease may occasionally
cause rapidly progressive glomerulonephritis, but this is usually not the typical
presentation for these diseases.
Diabetic nephropathy (choice B) typically begins with microalbuminuria and
hypertension and progresses over a 10 to 20 year period to renal failure.
Hypertensive nephropathy (choice C) due to essential hypertension typically
presents with slowly rising BUN and creatinine; hypertensive nephropathy due to
malignant hypertension presents with more rapidly rising BUN and creatinine.
Lupus nephritis (choice D) can have many presentations, but the most typical is
proteinuria, which may be severe enough to cause nephrotic syndrome. Also, 90%
of cases of systemic lupus erythematosus occur in women, usually of child-bearing
age.
Minimal change disease (choice E) typically presents with nephrotic syndrome and
is not consistently associated with recognizable glomerular changes by light
microscopy. Podocyte foot-process fusion can be seen by electron microscopy.
________________________________________

13The correct answer is E. Breast adenocarcinoma occurs in a variety of


morphologic subtypes, some of which have a significantly worse or better prognosis
than the most common invasive ductal carcinoma. Tubular carcinoma occurs in
women younger than 50 and has an excellent prognosis. It consists of well-formed
tubules, which are so well differentiated that the tumor is sometimes mistaken for a
benign lesion. Axillary metastases are present in fewer than 10% of cases at the
time of diagnosis. Of the histologic types of breast cancer listed here, the tubular
variant is certainly associated with the best chances of survival.
Invasive ductal carcinoma (choice A) and invasive lobular carcinoma (choice C) are
the main morphologic types of breast carcinoma. Together, they constitute 80% to
90% of all cases. The prognosis mainly depends on the staging, related to size of
the primary cancer and evidence of lymphohematogenous spread to regional nodes
and distant organs. Grading (that is degree of differentiation of primary tumor) has
a minor influence on prognosis.
Whenever breast cancer of any histologic type grows very rapidly, infiltrating and
occluding the dermal lymphatics, acute swelling and redness of the breast may
develop, leading to a clinical picture referred to as inflammatory carcinoma (choice
B). Inflammatory carcinoma, therefore, is not a true morphologic subtype but a
clinical definition. Any form of breast cancer manifesting as "inflammatory
carcinoma" is likely to be extremely aggressive.
Medullary carcinoma (choice D) has a slightly better prognosis than ductal or lobular
types. It tends to occur in younger women, especially those with mutations of
BRCA1. Grossly, this tumor is soft and well circumscribed. Histologically, this
carcinoma is surrounded by a marked lymphoplasmacytic reaction, which may
account for its propensity to grow more slowly than other forms of cancer. Medullary
carcinoma, however, has a worse prognosis than tubular carcinoma.
________________________________________
14The correct answer is E. The stomach has a very rich anastomotic arterial supply
and can tolerate occlusion of the gastroduodenal artery.
The right gastroepiploic artery, which supplies the distal greater curvature (choice
A), can be alternatively supplied by the splenic artery via the left gastroepiploic
artery.
The right gastric artery, which supplies the distal lesser curvature (choice B), usually
arises from the hepatic artery before the gastroduodenal artery arises.
The short gastric artery, which supplies the proximal greater curvature (choice C),
arises from the splenic artery.
The left gastric artery, which supplies the proximal lesser curvature (choice D),
arises directly from the celiac artery.
________________________________________
15The correct answer is B. Unsuspected (or denied) myocardial infarction is not
uncommon, and death may occur because of untreated complications. A number of

serious complications can occur between 5 and 10 days following infarction, due to
marked weakening of the necrotic myocardium. These include rupture of the
ventricular wall leading to hemopericardium and cardiac tamponade (as this patient
had), rupture of the interventricular septum, and rupture of the papillary muscle.
Arrhythmias are the most common complication 2 days post-infarction (choice A).
Fibrinous pericarditis secondary to an autoimmune phenomenon (Dressler's
syndrome) can be seen several weeks after infarctions (choices C and D).
By 60 days after infarction (choice E), the contracted scar is usually complete, and
residual complications include left ventricular failure and arrhythmias.
________________________________________
16The correct answer is C. Features of papillary carcinoma of the thyroid on
aspiration include papillary clusters, "Orphan Annie" nuclei, and psammoma bodies.
Psammoma bodies are laminated, concentric, calcific spherules seen most
frequently in papillary adenocarcinoma of the thyroid, serous papillary
cystadenocarcinoma of the ovary, meningioma, and malignant mesothelioma.
Anitschkow cells (choice A) are activated histiocytes found in rheumatic heart
disease.
Auer rods (choice B) are cytoplasmic inclusions found in promyelocytic leukemia
cells.
Reed-Sternberg cells (choice D) are typically binucleate (or contain bilobed nuclei)
with prominent "owl's eyes" nucleoli cells found in Hodgkin's disease.
Roth's spots (choice E) are pale retinal spots surrounded by hemorrhage seen in
endocarditis.
________________________________________
17The correct answer is A. The spleen shows the changes of chronic congestive
splenomegaly, typically associated with hepatic cirrhosis. The described small scars
are called Gandy-Gamma nodules, and are thought to be the result of organization
of old hemorrhages.
Hodgkin's disease (choice B) produces large splenic nodules in which ReedSternberg cells can be found surrounded by mature lymphocytes, eosinophils, and
neutrophils.
Rheumatoid arthritis (choice C) and many other chronic inflammatory disorders
induce reactive hyperplasia of the spleen with formation of many large germinal
centers in the splenic follicles.
Sickle cell anemia (choice D) produces many small (often triangularly shaped)
infarctions in the spleen.
Trauma (choice E) can cause splenic rupture.
________________________________________

18The correct answer is D. The disease is adult polycystic kidney disease, which is
an autosomal dominant condition that typically manifests in middle age. While the
kidneys appear very badly deformed, they function surprisingly well because the
cystic spaces actually only affect 10% or less of the nephrons. Pressure exerted by
the cysts can somewhat compromise blood flow to some glomeruli, which is
probably why hypertension, as a consequence of renin secretion, is such a problem
for these patients.
ACTH (choice A) can stimulate cortisol (choice B) secretion, and the cortisol (in high
amounts) can have enough mineralocorticoid activity to cause hypertension.
However, this would be more likely with a pituitary tumor, adrenal tumor, or with
exogenous corticosteroid use.
Parathormone (choice C) regulates calcium metabolism and does not usually affect
blood pressure.
Thyroxine excess (choice E) can cause hypertension as a consequence of its effects
on the heart and vasculature; this might be seen in the setting of hyperthyroidism
due to thyroid disease.
________________________________________
19The correct answer is B. A forced expiration is the simplest test of lung function.
The individual breathes in as much air as the lungs can hold and then expels the air
as rapidly and as forcefully as possible. The forced vital capacity (FVC) is the vital
capacity measured with a forced expiration (3 L for patient Y). The forced expiratory
volume in one second (FEV1) is the amount of air that can be expelled from the
lungs during the first second of a forced expiration (1.5 L for patient Y). The
FEV1/FVC ratio has diagnostic value for differentiating between normal, obstructive,
and restrictive patterns of a forced expiration. FEV1/FVC is a function of airway
resistance. Airway resistance is often increased in emphysematous lungs, which
causes FEV1/FVC to decrease (note that FEV1/FVC is 50% in patient Y and 80% in
the healthy individual represented by trace X).
FVC is also decreased in restrictive lung diseases such as asbestosis (choice A) and
silicosis (choice E) with a normal or slightly increased FEV1/FVC ratio.
Although FVC is decreased in pleural effusion (choice C) and pneumothorax (choice
D), the airway resistance is usually not affected greatly so that FEV1/FVC may be
normal.
________________________________________
20The correct answer is D. The likelihood of an abnormal test indicating disease, or
the positive predictive value of a test (PV+), is expressed with the following formula:
PV+ = TP/(TP + FP)
where a true positive (TP) is an abnormal test result in an individual with disease,
while a false positive (FP) is an abnormal test result in a healthy person. Note that
as the FP rate decreases, the PV+ approaches 100%. The FP rate reflects the test's
specificity, or the chance that a test is normal in a healthy individual. As the FP rate

of a test decreases, the test's specificity approaches 100%. Therefore, a test with
100% specificity always has a PV+ of 100%. This underscores why tests with high
specificity are used to confirm disease, since an abnormal test is more likely to
represent a TP rather than a FP.
From the table we can see the following: TP = 120, FN = 80, TN = 170, and FP = 30.
The likelihood of an abnormal test indicating an AMI is therefore:
PV+ = 120/(120 + 30) = 0.80
Thus there is an 80% chance that an abnormal test is a TP and a 20% chance that it
is a FP.
Choice A is the prevalence of AMI in this study. Prevalence refers to the number of
subjects with disease (TP + FN) divided by the total number of subjects in the study.
Hence, the prevalence of an AMI in this study is (120 + 80)/(120 + 80 + 170 + 30)
= 0.50, or 50%. As expected, the higher the prevalence of disease, the greater the
PV+ , since the number of individuals with disease (TPs) is increased.
Choice B is the test's sensitivity [TP/(TP + FN)], which is calculated as follows: 120/
(120 + 80) = 0.60, or 60%. The test's sensitivity is too low to be useful as a
screening test for an AMI.
Choice C is the negative predictive value (PV-) = [TN/(TN + FN)], which is calculated
as follows: 170/(170 + 80) = 0.68, or 68%. This percentage indicates that there is a
68% chance that a normal test is a TN and a 32% chance that it is a FN. The higher
the prevalence of disease, the lower the PV-, since a greater number of FNs are
present.
Choice E is the test's specificity [TN/(TN + FP)], which is calculated as follows: 170/
(170 + 30) = 0.85, or 85%. Although this test's specificity is higher than its
sensitivity, its specificity is not high enough for it to be useful as a confirmatory test
for an AMI.
________________________________________
21The correct answer is D. Recall that flow = pressure gradient/resistance. Also
recall that resistance (R) is inversely proportional to the fourth power of the radius
(R a 1/ radius4) and proportional to the first power of the blood viscosity (R a
viscosity). The problem can be confusing because the vessels with the highest
pressure gradient and radius also have the highest viscosity. However, the
discerning student will note that the 6-fold increase in radius will cause a 1296-fold
increase in flow, and that the range of pressure and viscosity given in the table will
have a relatively minor effect on flow as compared to the effect of vessel radius.
________________________________________
22The correct answer is B. Atonic or "drop" seizures are characterized by a sudden
loss of postural muscle tone that lasts only a few seconds. Although consciousness
may be impaired briefly, there is rarely postictal confusion. A very brief seizure may
cause only a drop of the head, but a longer seizure may cause the patient to slump
to the ground. This type of seizure may be quite dangerous because of the risk of

head injury with a sudden fall. Drugs prescribed for this condition include valproic
acid, clonazepam, felbamate, vigabatrin, and lamotrigine.
Absence seizures (choice A), also known as petit mal seizures, are characterized by
blank stares and an absence of any change in position. They typically occur in
children. Drugs used in this disorder include ethosuximide, valproic acid, and
clonazepam.
Myoclonic seizures (choice C) are characterized by sudden, brief muscle jerks that
may involve part of the body or the whole body. A physiologic form of myoclonus
occurring in healthy individuals is the sudden jerking movement that sometimes
occurs while falling asleep. Drugs used for this condition include valproic acid and
clonazepam.
Tonic seizures (choice D) are characterized by the sudden onset of sustained axial
and limb muscle contraction. Pure tonic seizures tend to occur in children.
Tonic-clonic seizures (choice E), also known as grand mal seizures, are characterized
by an initial phase with tonic contraction of muscles throughout the body. After a
short time, the seizure evolves into the clonic phase, in which periods of muscle
relaxation are superimposed on muscle contraction. There is a significant postictal
phase. Drugs used for this disorder include carbamazepine, phenytoin, and valproic
acid.
________________________________________
23The correct answer is A. It is a good idea to know the mechanism of action of
various receptors. Alpha-1 adrenergic receptors are coupled to the G protein, Gq.
This G protein causes breakdown of the membrane phospholipid
phosphatidylinositol bisphosphate (PIP2), forming the products diacylglycerol, which
stimulates protein kinase C, and inositol triphosphate (IP3), which releases calcium
from the endoplasmic reticulum.
Beta-1 adrenergic receptors (choice B), like all beta receptors, are coupled to the Gs
G protein, leading to the activation of adenylate cyclase. This increases intracellular
concentrations of cAMP by converting ATP to cAMP. cAMP, in turn, stimulates protein
kinase A, also known as cAMP-dependent protein kinase.
Dopamine-2 receptors (choice C) are coupled to Gi. Stimulation of these receptors
inhibits adenylate cyclase, thus lowering cAMP levels and protein kinase A activity.
Gamma-aminobutyric acid (GABA) receptors (choice D) are not G-protein-coupled
receptors, but are instead ligand-gated ion channel receptors. Stimulation of these
inhibitory receptors causes a chloride flux, thus stabilizing the membrane potential
of the cell.
Nicotinic cholinergic receptors (choice E) are not G-protein-coupled receptors, but
are instead ligand-gated ion channel receptors. Stimulation of these receptors
causes sodium influx, leading to excitation of the cells.
________________________________________

24The correct answer is A. The pericardial space is located between the epicardium
(also known as the visceral pericardium) and the parietal pericardium. A tear of a
blood vessel immediately outside of the heart will cause bleeding into the
pericardial space. This accumulation of blood in the pericardial space causes
increased pressure on the heart, which restricts filling of the heart during diastole
(cardiac tamponade). This reduced filling results in reduced cardiac output and
reduced blood pressure.
The region between the fibrous pericardium and the parietal pleura (choice B) is
outside of the pericardial space. It is part of the mediastinum and it is in this region
in which structures such as the vagus nerve and the phrenic nerve are found.
The epicardium is fused to the myocardium and is the outer layer of the heart wall.
There is no space between the epicardium and the myocardium (choice C).
The parietal pericardium and the fibrous pericardium are fused into a single layer
that forms the outer wall of the pericardial space. There is no space between the
parietal pericardium and the fibrous pericardium (choice D).
The space between the parietal pleura and the visceral pleura (choice E) is the
pleural space. No blood vessels pass through the pleural space
________________________________________
25The correct answer is D. Rupture of a colonic diverticulum, with resulting fecal
peritonitis, is a catastrophic complication of diverticulosis, particularly in the elderly,
with a mortality rate near 50%. The rupture frequently involves a diverticulum that
is not inflamed (or only minimally inflamed) and consequently the rupture may be
inapparent, at least initially. Shock secondary to septicemia develops rapidly,
however. Vigorous resuscitation may be required to stabilize the patient sufficiently
for emergency surgery to resect the distal colon and form a colostomy (Hartmann's
operation).
Acute appendicitis (choice A), acute cholecystitis with gall bladder rupture (choice
B), gastric rupture (choice C), and rupture of an ovarian cyst (choice E) are all
uncommon in this age group.
________________________________________
26The correct answer is E. The suspected tumor is pheochromocytoma, 10% of
which occur in children. The symptoms (paroxysmal hypertension, palpitations,
anxiety) are produced when the tumor secretes epinephrine, norepinephrine, and
other vasoactive amines into the circulation. The diagnosis can be established with
plasma catecholamine concentrations or concentrations of the norepinephrine
metabolite VMA in a 24-hour urine. The latter offers the advantage of providing a
longer time sample so that the intermittent secretion is more likely to be picked up.
Serum albumin (choice A) can be low in liver and renal disease, but is unaffected in
pheochromocytoma.
beta-hCG (choice B) is a marker for choriocarcinoma and related lesions.

CEA (choice C) is a nonspecific marker that is positive in many cases of colorectal


and pancreatic cancer.
Bence-Jones proteins (choice D) are a marker for multiple myeloma, representing
urinary excretion of myeloma light chains.
The correct answer is C. The most distinctive clue is the presence of amyloid, which
specifically suggests medullary thyroid carcinoma. This carcinoma is a tumor of the
neuroendocrine parafollicular cells of the thyroid, rather than the follicular lining
epithelium. These cells produce calcitonin, the precursor protein of which can
precipitate, forming bands and nodules of amyloid that appear histologically
identical to other forms of amyloid. The other thing to remember about medullary
carcinoma of the thyroid is that it can be a component of multiple endocrine
neoplasia (MEN) syndromes type IIa (parathyroid disease, pheochromocytomas,
medullary carcinoma) and type IIb (parathyroid disease, pheochromocytomas,
medullary carcinoma, mucocutaneous ganglioneuromas, Marfanoid habitus).
Because follicular thyroid carcinoma (choice A) closely resembles normal thyroid
tissue, it usually cannot be reliably diagnosed based on fine needle aspiration alone.
Hashimoto's disease (choice B) would show lymphocytes, plasma cells, and
macrophages on aspiration.
Papillary thyroid carcinoma (choice D) can be diagnosed by aspiration if papillary
clusters are seen.
Thyroid adenoma (choice E) cannot be reliably distinguished from well-differentiated
thyroid carcinoma on aspiration.
________________________________________
27The correct answer is C. The most distinctive clue is the presence of amyloid,
which specifically suggests medullary thyroid carcinoma. This carcinoma is a tumor
of the neuroendocrine parafollicular cells of the thyroid, rather than the follicular
lining epithelium. These cells produce calcitonin, the precursor protein of which can
precipitate, forming bands and nodules of amyloid that appear histologically
identical to other forms of amyloid. The other thing to remember about medullary
carcinoma of the thyroid is that it can be a component of multiple endocrine
neoplasia (MEN) syndromes type IIa (parathyroid disease, pheochromocytomas,
medullary carcinoma) and type IIb (parathyroid disease, pheochromocytomas,
medullary carcinoma, mucocutaneous ganglioneuromas, Marfanoid habitus).
Because follicular thyroid carcinoma (choice A) closely resembles normal thyroid
tissue, it usually cannot be reliably diagnosed based on fine needle aspiration alone.
Hashimoto's disease (choice B) would show lymphocytes, plasma cells, and
macrophages on aspiration.
Papillary thyroid carcinoma (choice D) can be diagnosed by aspiration if papillary
clusters are seen.

Thyroid adenoma (choice E) cannot be reliably distinguished from well-differentiated


thyroid carcinoma on aspiration.
________________________________________

28The correct answer is C. Acute interstitial nephritis is due to a hypersensitivity


reaction usually caused by a drug. Drugs implicated in the pathogenesis of acute
interstitial nephritis include non-steroidal anti-inflammatory drugs (NSAIDs), such as
ibuprofen and indomethacin; beta-lactam antibiotics, such as cephalothin and
methicillin; sulfonamides; diuretics, such as furosemide and thiazides; and drugs
like phenytoin, cimetidine, and methyldopa. The typical presentation for acute
interstitial nephritis is the development of acute renal failure, fever, a
maculopapular rash, and eosinophilia. The patient's periorbital edema and wheezing
are also consistent with acute interstitial nephritis. Ibuprofen is the most likely
causative agent for the development of the patient's signs and symptoms. Although
the exact mechanism by which NSAIDs cause this disorder is not fully understood, it
is believed to be related to their ability to decrease prostaglandin formation, which
leads to a reduction in renal bloodflow. None of the other medications are
associated with the development of acute interstitial nephritis.
Acarbose (choice A) is an antidiabetic agent that delays the absorption of glucose
from the intestinal tract.
Clonidine (choice B) is a centrally-acting antihypertensive agent used in the
treatment of hypertension and prophylaxis of migraines. It has been shown to be
efficacious in ameliorating symptoms of alcohol, tobacco, opiate, and
benzodiazepine withdrawal. It is also used in the treatment of attention deficit
disorder with hyperactivity.
Metformin (choice D) is an antidiabetic agent that increases the binding of insulin to
its receptor. One important side of metformin is the development of lactic acidosis.
Metoprolol (choice E) is a beta-blocker used in the treatment of hypertension.
________________________________________
29The correct answer is E. The postgastrectomy symptoms described above are
collectively called the dumping syndrome. Because all or part of the stomach is
removed, an ingested meal will be delivered to the small intestine more quickly
than normal. The large increase in tonicity in the small intestine causes an osmotic
fluid shift from the extracellular fluid (plasma) into the lumen of the gut. The
increased distention of the small intestine increases motility through reflex
mechanisms and causes diarrhea. The blood volume contraction and concomitant
release of vasoactive substances such as bradykinin and/or vasoactive intestinal
peptide can create hypotension and reflex tachycardia. Patients should be
instructed to eat more frequent, smaller meals to reduce the osmotic and/or
carbohydrate load that is delivered to the small intestine. Furthermore, since fats
are the slowest to be absorbed, a diet that is higher in fat will also reduce the
problem of rapid absorption

________________________________________
30The correct answer is B. The tumor is a Leydig (interstitial) cell tumor, which is a
testicular tumor that usually affects men aged 20-60 years. 10% of these tumors
are malignant. These tumors are interesting because they may elaborate androgens
or estrogens and, rarely, corticosteroids. While most patients present with a
testicular mass (like other testicular tumors), in occasional cases the presenting
symptoms are gynecomastia or precocious puberty. The key features in the question
stem are the gynecomastia (which implies estrogen secretion), golden-brown
testicular tumor, large uniform cells with indistinct cell borders, and rod-shaped
crystals (of Reinke).
Immature teratoma (choice A) is intermediate between mature teratoma and
embryonal carcinoma and contains elements of three germ cell layers.
Sertoli cell tumors (choice C) are gray-white to yellow and microscopically form
cord-like structures resembling seminiferous tubules.
Spermatocytic seminoma (choice D) is characterized by a heterogeneous cell
population with medium cells, smaller cells resembling secondary spermatocytes,
and giant cells.
Teratoma with malignant transformation (choice E) contains elements of three germ
layers, at least one of which is clearly malignant.
________________________________________
31The correct answer is A. The patient is presenting with signs and symptoms of
pneumonia. Since the patients signs and symptoms are appearing 20 hours after
admission, his pneumonia is most likely a "community-acquired" pneumonia; one of
the criteria for diagnosing a nosocomial infection is that the infection must not
occur before 48 hours after admission. The appearance of plump gram-negative
encapsulated rods in an alcoholic is highly suggestive of Klebsiella pneumonia. The
most appropriate treatment of Klebsiella pneumonia in this patient is the
administration of a third-generation cephalosporin, such as cefotaxime, ceftriaxone,
or ceftazidime.
Erythromycin (choice B) is a macrolide antibiotic used in the treatment of a variety
of gram-negative and gram-positive infections. Although it is not the treatment of
choice in this patient, it is the treatment of choice of Legionella pneumonia.
Gentamicin (choice C) is an aminoglycoside antibiotic that is generally added to
improve the efficacy of broad spectrum antibiotics, such as penicillin and
cephalosporins, in the treatment of Pseudomonas and Enterococcus infections.
Vancomycin (choices C and D) is an antibiotic used in the treatment of lifethreatening infections caused by gram-positive infections; the use of this agent in a
gram-negative infection is inappropriate.
Because of the severe nature of this infection, the use of an oral agent, such as
trimethoprim-sulfamethoxazole (choice E), would be inappropriate.

________________________________________
32The correct answer is A. The patient is presenting with signs and symptoms of
pneumonia. Since the patients signs and symptoms are appearing 20 hours after
admission, his pneumonia is most likely a "community-acquired" pneumonia; one of
the criteria for diagnosing a nosocomial infection is that the infection must not
occur before 48 hours after admission. The appearance of plump gram-negative
encapsulated rods in an alcoholic is highly suggestive of Klebsiella pneumonia. The
most appropriate treatment of Klebsiella pneumonia in this patient is the
administration of a third-generation cephalosporin, such as cefotaxime, ceftriaxone,
or ceftazidime.
Erythromycin (choice B) is a macrolide antibiotic used in the treatment of a variety
of gram-negative and gram-positive infections. Although it is not the treatment of
choice in this patient, it is the treatment of choice of Legionella pneumonia.
Gentamicin (choice C) is an aminoglycoside antibiotic that is generally added to
improve the efficacy of broad spectrum antibiotics, such as penicillin and
cephalosporins, in the treatment of Pseudomonas and Enterococcus infections.
Vancomycin (choices C and D) is an antibiotic used in the treatment of lifethreatening infections caused by gram-positive infections; the use of this agent in a
gram-negative infection is inappropriate.
Because of the severe nature of this infection, the use of an oral agent, such as
trimethoprim-sulfamethoxazole (choice E), would be inappropriate.
________________________________________
33The correct answer is C. The arterial supply of the stomach is complex and is
consequently a target on the USMLE. The right gastric artery supplies the distal
lesser curvature.
The left gastric artery (choice A) supplies the proximal lesser curvature.
The left gastroepiploic artery (choice B) supplies the proximal greater curvature
below the splenic artery.
The right gastroepiploic artery (choice D) supplies the distal greater curvature.
The short gastric artery (choice E) supplies the proximal greater curvature above
the splenic artery.
________________________________________
34The correct answer is E. This is a case of immediate (type I) hypersensitivity
mediated by IgE, mast cells, and basophils. The cytokine that mediates the isotype
switch to IgE in B lymphocytes is interleukin 4, produced by T-helper-2 cells.
Interferon-g (choice A) is a cytokine that activates macrophages and would not be
involved in this response.

Interleukin 1 (choice B) is an endogenous pyrogen involved in the production of the


acute phase reaction of systemic inflammation.
Interleukin 2 (choice C) is the cytokine that induces T cell proliferation.
Interleukin 3 (choice D) is a hematopoietic colony stimulating factor.
Interleukin 5 (choice F) is an eosinophil stimulating factor.
Interleukin 6 (choice G) is another endogenous pyrogen and a stimulator of B cells,
but not an inducer of isotype switch to IgE production.
________________________________________
35The correct answer is D. The presence of the Philadelphia chromosome, a
translocation from the long arm of chromosome 22 to chromosome 9 [t(9;22)], is
associated with a more favorable prognosis in patients with chronic myelogenous
leukemia.
Acute lymphoblastic leukemia (ALL; choice A) is the most common cause of
leukemia in children. The presence of the Philadelphia chromosome is associated
with a worse prognosis for the patient. This form of leukemia is also associated with
a B-ALL translocation of the c-myc proto-oncogene of chromosome 8 to
chromosome 14 [t(8;14)(q24;q32)].
Acute myelogenous leukemia (AML; choice B) is the most common acute leukemia
in adults. The M2 subtype is associated with the t(8;21) translocation and the M3
subtype is associated with the t(15;17) translocation.
Over half of patients with chronic lymphocytic leukemia (choice C) display one of
several chromosomal abnormalities. This includes trisomy 12 (involves the h-ras
proto-oncogene), translocation t(11;14) (involves k-ras and bcl-1 proto-oncogenes),
and deletion (14q-) or inversion (14q) (involves immunoglobulin heavy chain gene).
Hairy cell leukemia (choice E) is associated with the expression of tartrate-resistant
acid phosphatase (TRAP) on the surface of B cells.
________________________________________
36The correct answer is A. Trilineage failure of blood cell production, producing
pancytopenia, suggests a generalized decrease in marrow cavity size, which can be
due to bony overgrowth in processes such as osteopetrosis. The anemia observed in
these patients is often very refractory to treatment.
Osteomalacia produces a generalized decrease in bone mass (choice B), and is not
associated with pancytopenia.
A large calcified growth on the tibia (choice C) suggests a benign or malignant
tumor of bone, which would not usually replace enough marrow to cause anemia.
Multiple exostoses (choice D) would affect the outer surface, not the marrow cavity
of the bone.

A single "punched out" bony lesion (choice E) located in the upper femur suggests
monostotic fibrous dysplasia, an asymptomatic, benign, bony lesion
________________________________________
37The correct answer is A. Oxygen delivery to the left ventricle is equal to the
oxygen content of the arterial blood entering the heart multiplied by the coronary
blood flow. Because the oxygen content of blood entering the coronary circulation is
similar during all phases of the cardiac cycle, oxygen delivery to the left ventricle is
dependent entirely upon coronary blood flow. The vast majority of blood flow in the
left coronary artery occurs during phase 1, which corresponds to the filling phase of
ventricular diastole. Phase 2 (choice B) corresponds to the period of isovolumic
contraction. Phase 3 (choice C) corresponds to the period of ejection, and phase 4
(choice D) corresponds to the period of isovolumic relaxation. Although phase 4 has
a peak blood flow similar to that of phase 1, the total amount of oxygen delivered to
the myocardium during phase 4 is relatively low because of its short duration.
________________________________________
38The correct answer is B. The condition described in the question stem is very
common among diabetics, and is due to atrophy of lumbricals and interosseus
muscles secondary to diabetic neuropathy. Another finding that may be seen is the
presence of corns and callosities on the dorsal surface of the feet overlying the
protuberant interphalangeal joints. All of these degenerative changes add to the
diabetic's foot disease, predisposing for sores that heal poorly in the poorly
vascularized diabetic foot, often leading to gangrene, requiring amputation of the
distal foot.
Flexor digitorum longus (choice A) is an extrinsic muscle of the foot that flexes the
distal phalanges of the lateral four toes and assists in plantar flexion of the foot.
Peroneus longus (choice C) is an extrinsic muscle of the foot that plantar flexes and
everts the foot.
Tibialis anterior (choice D) is an extrinsic muscle of the foot that dorsiflexes and
inverts the foot.
Tibialis posterior (choice E) is an extrinsic muscle of the foot that plantar flexes and
inverts the foot.
________________________________________
39The correct answer is B. The Ghon complex is the most frequent pathologic form
of primary pulmonary tuberculosis. Mycobacterium tuberculosis first localizes in the
lung parenchyma, then in the hilar lymph nodes. In both these locations, a
granulomatous reaction takes place. These lesions usually heal by fibrosis, leaving
only small scars at the sites of remote tuberculous infection. In some cases, owing
to immunosuppression (e.g., AIDS, immunosuppressant treatment, and
lymphomas), reactivation of dormant bacilli in old lesions or additional re-exposure
leads to secondary tuberculosis, with progression of lesions. Sometimes, active
lesions of the Ghon complex are discovered by chance at autopsy. More frequently,

scars due to remote healed tuberculosis (choice E) are found postmortem and listed
in autopsy reports as incidental findings
Cavitary tuberculosis (choice A) and miliary tuberculosis (choice D) are expressions
of secondary infection, following reactivation of old, usually clinically silent, lesions.
The cavitary form is characterized by development of large areas of liquefactive
necrosis that empty into the airspaces, leading to cavities within the lung
parenchyma. The miliary form is due to lymphohematogenous dissemination and
subsequent seeding of tubercle bacilli throughout the body, with myriad small
granulomas forming in the lungs, spleen, liver, bone marrow, retina, and adrenals,
for example.
Acid-fast bacilli suggest tuberculosis, rather than an infection with a fungus such as
Histoplasma (choice C).
________________________________________
40The correct answer is A. Pancreatic carcinoma often presents with vague
abdominal, back, and gastrointestinal complaints; and physical examination is
generally unrevealing. The tremendous weight loss is very suspicious for carcinoma,
and the migrating thrombophlebitis (Trousseau's sign) is extremely helpful in
making the diagnosis, which should be confirmed with ultrasonography or CT.
Although any carcinoma can elicit migratory thrombophlebitis, it is mostly
associated with tumors of the pancreas, lung, and colon.
Primary sclerosing cholangitis (choice B) is an inflammatory fibrosing disease of the
biliary tree of unknown etiology, although highly associated with inflammatory
bowel disease. It presents with symptoms of liver failure (jaundice, pruritus) and
progresses to biliary cirrhosis.
Pyelonephritis (choice C) may present with back pain that usually localizes to the
costovertebral angle, and is generally associated with fevers and dysuria. Chronic
pyelonephritis proceeds to hypertension and renal failure.
Reflux esophagitis (choice D) presents with heartburn, regurgitation, and dysphagia
and may occasionally be confused with a heart attack by the patient. Long term
consequences include bleeding, strictures, and Barrett's esophagus.
Splenic infarction (choice E), typically associated with arterial thromboembolic
events, generally is an asymptomatic occurrence that does not produce clinical
symptoms unless the entire spleen is lost.
________________________________________
41The correct answer is D. The muscle involved is the psoas major. The abscess in
the groin is called a "psoas abscess," because it occurs after a spinal infection
(classically tuberculosis) spreads along the psoas sheath. The principal action of the
psoas muscle is to flex the thigh at the hip. It also has minimal action in external
rotation of the hip.
Abductors (choice A) of the hip include the gluteus medius and gluteus maximus.

Adductors (choice B) of the hip include the adductors longus, brevis, and magnus.
Extensors (choice C) of the hip include gluteus maximus.
Internal (medial) rotators (choice E) of the hip include gluteus minimus, pectineus,
and gracilis.
________________________________________
42The correct answer is C. Most cases of Tay-Sachs disease are caused by insertion
of four nucleotides into the coding region of the gene for the alpha-subunit of the
hexosaminidase A gene. This produces a frameshift mutation, resulting in the
creation of a premature stop codon downstream from the inserted nucleotides. The
disease presents as described in the question, and is more common in Ashkenazi
Jews. The carrier rate in this population is 1 in 25. There is presently no effective
treatment and survival beyond four years of age is rare.
Confined placental mosaicism (choice A) is caused by a genetic aberration (e.g., a
trisomy) occurring in the trophoblast or extraembryonic progenitor cells in the inner
cell mass of the embryo. This leads to abnormal placental development and
intrauterine growth retardation.
Expansion of tandem nucleotide repeats (choice B) is thought to be etiologically
involved in Fragile X syndrome and Huntington disease.
Nondisjunction (choice D) during meiosis is a common cause of trisomy, e.g.,
trisomy 21, or Down syndrome.
Translocations (choice E) are frequently involved in leukemias, lymphomas, and
solid tumors. A minority of cases of Down syndrome are actually due to a
translocation, rather than trisomy.
________________________________________
43The correct answer is E. The point of this question is that nonselective betablockers are contraindicated in patients with lung disease. This is because
nonselective beta-blockers will cause bronchoconstriction by blocking the beta2receptors responsible for promoting bronchial smooth muscle relaxation (recall that
beta2 agonists are a mainstay of asthma therapy). Acebutolol (choice A), atenolol
(choice B), esmolol (choice C), and metoprolol (choice D) are all cardioselective
beta1-blockers that could be used in a patient with lung/airway disease. Another
cardioselective blocker that is not listed is betaxolol. As for nadolol (choice E), it is a
nonselective beta-blocker and should NOT be used in a patient with lung disease.
________________________________________
44The correct answer is B. The lymphoma described in the question stem is the
follicular type of small cleaved cell (B cell) lymphoma, and its characteristic
translocation is t(14;18)(q32;q21). The site on chromosome 14 involves the heavy
chain of immunoglobulins (IgH), while that on chromosome 18 involves the bcl-2
oncogene. The translocation produces a hybrid bcl-2/immunoglobulin heavy-chain
transcript.

A t(11,14)(q13;q32) translocation involving bcl-1 and IgH (choice A) is seen in small


non-cleaved lymphocytic lymphoma, myeloma, and some mantle cell lymphomas.
Burkitt's lymphoma is usually associated with t(8;14) (myc and IgH; choice C), but
sometimes with t(2;8) (kappa immunoglobulin light chain and myc; choice D) or
t(8;22) (lambda immunoglobulin light chain and myc; choice E).
________________________________________
45The correct answer is A. This question is about an important drug interaction
occurring between erythromycin and cisapride (used for gastroesophageal reflux
disease- GERD). Serious cardiac arrhythmias, including torsades de pointes, QT
interval prolongation, ventricular tachycardia, and ventricular fibrillation, have been
reported in patients taking cisapride with medications that inhibit cytochrome P450
3A4. Examples of medications other than cisapride that have a similar interaction
with erythromycin include ketoconazole, fluconazole, clarithromycin, nefazodone,
and indinavir. It is important to note that QT prolongation, torsades de pointes,
cardiac arrest, and sudden death have occurred in patients taking only cisapride.
None of the other agents listed interact with erythromycin to produce these
proarrhythmic effects.
Famotidine (choice B) is an H2 receptor antagonist used in the treatment of GERD
and gastric ulcers; this agent is generally well tolerated with very little incidence of
drug interactions.
Lansoprazole (choice C) is a proton pump inhibitor indicated for the treatment of
gastric ulcerations and GERD. When this agent is administered with phenytoin, the
clearance of phenytoin is decreased by 15%, leading to an extension of the
therapeutic effect.
Metoclopramide (choice D) is a prokinetic agent indicated for the treatment of GERD
and diabetic gastroparesis. When given with cyclosporine, the toxic effects of
cyclosporine are more pronounced. Furthermore, when metoclopramide is
administered with levodopa, it decreases the effectiveness of levodopa.
Sucralfate (choice E) is a basic aluminum salt indicated for the treatment of
duodenal ulcer. When sucralfate is administered with digoxin, anticoagulants,
ketoconazole, fluoroquinolones, and tetracyclines, there is a dramatic decrease in
the absorption of these agents from this intestinal tract
________________________________________
46.
The correct answer is D. All three of the nerves listed innervate muscles that supply
the thumb. Extension is provided by the extensors pollicis longus and brevis, which
are innervated by the radial nerve.
The median nerve (choices A, B, and C) supplies the thenar group, which allows the
thumb to oppose, flex, abduct, and rotate.

The ulnar nerve (choices E and B) supplies the adductor pollicis, which adducts the
thumb.
________________________________________
47The correct answer is C. Creatinine is formed from muscle creatine and released
into the plasma at a fairly constant rate. Creatinine passes freely through the
glomerular membrane but is not reabsorbed to a significant extent and is secreted
only in small amounts by the kidney tubules. Consequently, the glomerular filtrate
has about the same concentration of creatinine as the plasma. As the tubular fluid
moves along the tubule, all the filtered creatinine continues on into the urine.
Therefore, the excretion rate of creatinine is approximately equal to the filtration
rate of creatinine. The rate of creatinine excretion and filtration can be determined
from the creatinine concentration in the urine (Ucreatinine) and plasma
(Pcreatinine) and the rate of urine flow (V) as follows:
Excretion rate = Ucreatinine V
Filtration rate = Pcreatinine GFR
Because excretion rate and filtration rate of creatinine are approximately equal, as
discussed above, GFR can be determined as follows:
________________________________________

48>E
________________________________________

49> A
________________________________________

50>E

Answers

________________________________________

1The correct answer is C. Congenital "cafe au lait" spots are present in more than
90% of patients with neurofibromatosis. This autosomal dominant disorder is
characterized by multiple neural tumors, especially neurofibromas, pigmented
hamartomas of the iris (Lisch nodules), and cafe au lait spots, which usually occur
over nerve trunks. Although the majority of neurofibromas in this disease are
benign, the tumors can be quite disfiguring and psychologically damaging.
Basal cell carcinoma (BCC; choice A) is a common, slow-growing tumor of sunexposed skin. It develops in adulthood after years of chronic sun damage. Although
malignant melanoma may arise from large congenital nevi, BCC does not.
Neuroblastoma (choice B) is a common childhood tumor that arises anywhere along
the sympathetic chain, and most commonly in the adrenal medulla. The tumor
usually presents as an abdominal mass and is not associated with skin findings.
The ocular neoplasm associated with neurofibromatosis is the Lisch nodule, not the
retinoblastoma (choice D). Retinoblastoma is a neuroepithelial tumor usually
identified by funduscopic examination of a child with visual changes.
Wilms' tumor (choice E) is a neoplasm of primitive renal blastema that may be
associated with congenital malformations of visceral organs, notably the adrenals
and gonads, but not skin. It also presents as an abdominal mass, and is highly
survivable with modern treatment modalities.
________________________________________

2The correct answer is B. Pheochromocytomas, and their related counterparts in


extra-adrenal sites called paragangliomas, are notorious because the only reliable
indicator of metastatic potential is the presence of distant metastases. Very
malignant-appearing tumors may not metastasize and benign-appearing tumors
may produce metastases. In practice, there is no way for the pathologist to make
the decision on frozen (or permanent) section analysis unless metastasis has
already occurred and can be documented. These tumors should all be considered
"potentially malignant."
________________________________________
3The correct answer is C. A mixed acidosis commonly occurs with cardiopulmonary
arrest. Cardiac arrest victims experience some degree of lactic acidosis (metabolic
acidosis) as a result of poorly perfused tissues. A simultaneous respiratory acidosis
due to ventilatory standstill also occurs. This combination of metabolic acidosis and
respiratory acidosis is referred to as a "mixed acidosis." A metabolic acidosis (choice
A) is present when plasma pH and HCO3- concentration are low, and a respiratory
acidosis (choice E) is present when plasma pH is low and arterial CO2 is high.
The table below shows changes in plasma pH, plasma HCO3-, and arterial CO2 for
the various acid-base disturbances.
Acid Base Status

Plasma pH

Plasma Bicarbonate

Arterial Carbon Dioxide

Metabolic acidosis (choice A)

low

low

low

Metabolic alkalosis (choice B)

high

high

high

Mixed acidosis (choice C) low

low

high

Mixed alkalosis (choice D)

high

high

low

Respiratory acidosis (choice E)

low

high

high

low

low

Respiratory alkalosis (choice F) high

________________________________________

4The correct answer is E. The only childhood malignancy listed is Wilms' tumor,
which commonly presents in a toddler as a large abdominal mass. There is now a
90% survival rate for this tumor with combined therapy with surgery,
chemotherapy, and radiotherapy.
Renal cell carcinoma (choice A), squamous cell carcinoma of the bladder (choice C),
and transitional cell carcinoma of the bladder (choice D) are malignant tumors of
adults.
Renal hamartoma (fibroma; choice B) causes a small, gray, benign module in the
renal pyramids and is usually only identified as an incidental finding at autopsy.
________________________________________
5The correct answer is C. The presence of free hemoglobin in the serum and urine,
and "bite cells" due to splenic removal of Heinz bodies (oxidized hemoglobin) all
point to hemolysis. Hemolysis following oxidant injury by drugs (sulfonamides, for
example) or infection suggests glucose-6-phosphate dehydrogenase deficiency or
the related deficiencies of glutathione synthetase, pyruvate kinase, and hexokinase.
These conditions are typically asymptomatic between episodes of hemolysis.
Deficiencies of the alpha (choice A) and beta (choice B) chains of hemoglobin
produce alpha and beta thalassemia, respectively.
Deficiencies of glycoprotein IIb/IIIa (choice D) produce thrombasthenia, a platelet
aggregation defect.
Deficiencies of spectrin (choice E) produce hereditary spherocytosis, characterized
by hemolytic anemia and splenomegaly.
________________________________________
6The correct answer is B. The most common type of atrial septal defect is the
ostium secundum type. Children with Down syndrome, however, are frequently
afflicted with the ostium primum type of atrial septal defects, which may be
accompanied by tricuspid and mitral valve malformations. More complex

atrioventricular septal defects may also occur in this disorder. Children exhibiting
these lesions should be specifically evaluated for chromosomal abnormalities.
Clinically, the lesions produce left-to-right shunts with late cyanosis (after the right
ventricle hypertrophies in response to developing lung disease from the increased
blood flow in the pulmonary system).
Neither cystic fibrosis (choice A) nor Gaucher disease (choice C) is specifically
associated with cardiovascular defects.
Dissecting aortic aneurysm is associated with Marfan syndrome (choice D).
Turner syndrome (choice E) is associated with coarctation of the aorta.
________________________________________
7The correct answer is A. The left atrium forms most of the posterior wall of the
heart. The esophagus passes immediately posterior to the heart. Enlargement of
the left atrium may compress the esophagus and cause dysphagia.
The left ventricle (choice B) forms most of the left border of the heart and most of
the diaphragmatic surface of the heart. The left ventricle is not related to the
esophagus.
The pulmonary trunk (choice C) emerges from the right ventricle on the anterior
surface of the heart. The pulmonary trunk is not related to the esophagus.
The right atrium (choice D) forms the right border of the heart. It is not related to
the esophagus.
The right ventricle (choice E) forms most of the anterior wall of the heart and a
small portion of the diaphragmatic surface of the heart. It is not related to the
esophagus.
________________________________________
8The correct answer is E. Hepatitis E is an important, and until recently,
unrecognized cause of epidemics of enterically transmitted acute hepatitis.
Hepatitis E is caused by an enterically transmitted virus that occurs primarily in
India, Asia, Africa, and Central America. Infection with the virus is associated with a
very high mortality among pregnant women.
CMV (choice A) can cause acute hepatitis, but the disease is usually mild and often
goes unrecognized, except in profoundly immunosuppressed patients.
Hepatitis A virus (choice B) is the major cause of epidemics of enterically
transmitted viral hepatitis, but is not a significant cause of mortality in pregnant
women.
Hepatitis C virus (choice C) is usually transmitted parenterally, rather than
enterically, and is not a significant cause of mortality in pregnant women.
Herpes simplex (choice D) usually causes significant hepatitis only in profoundly
immunosuppressed patients.

________________________________________
9The correct answer is A. Breast cancer is unusual in that it produces both lytic and
blastic metastases to bone. Breast and prostate cancers are the most common
sources of bone metastases, but prostate metastases are usually blastic.
Colon cancer (choice B) does not usually metastasize to bone.
Kidney cancer (choice C), lung cancer (choice D), and thyroid cancer (choice E)
produce lytic lesions when they metastasize to bone.
________________________________________
10The correct answer is A. Seminomas and dysgerminomas are very similar tumors
but differ in two significant respects: the most common age of presentation in men
is in the fourth decade, while in women, it is in the third decade. Also, seminomas
are relatively common in men (30% of testicular germ cell tumors), while
dysgerminomas are rare in women (1% of ovarian tumors).
Both of these tumors are composed of sheets of uniform polyhedral cells with
intervening fibrous septa of connective tissue, lymphocytes, and multinucleated
giant cells. The number of mitoses (choice B) per high-power field and
ultrastructural appearance (choice E) do not differ greatly between the two tumors.
These tumors in pure form are very radiosensitive (90% 5-year survival; choice D),
but can be much more aggressive (choice C) if foci of other germ cell tumors
(notably embryonal carcinoma, choriocarcinoma, and yolk sac tumors) are present
________________________________________
11The correct answer is D. Cisplatin is an antineoplastic commonly used in the
treatment of metastatic ovarian and testicular cancers, as well as advanced bladder
cancer. This medication is associated with profound nausea and vomiting; in fact, it
is considered to be one of the most emetogenic agents on the pharmaceutical
market. The selective 5-hydroxytryptamine-3 (5-HT3) receptor antagonists are
potent antinauseant and antiemetogenic agents indicated for prevention of nausea
and vomiting associated with cancer chemotherapy. Examples of 5-HT3 receptor
antagonists include ondansetron, granisetron, and dolasetron.
Dimenhydrinate (choice A) is an antihistamine with anticholinergic activity; it is
indicated for the treatment of nausea and vomiting associated with motion
sickness.
Dronabinol (choice B) is the principle psychoactive substance present in Cannabis
sativa (marijuana). The mechanism of its antiemetic action is largely unknown; it is
indicated for the treatment of severe nausea and vomiting when conventional
therapies are ineffective.
Metoclopramide (choice C) is a prokinetic agent indicated for the treatment of
gastroesophageal reflux disease and diabetic gastroparesis. Since this agent blocks
dopaminergic receptors in the chemotrigger zone, it is also effective in the
treatment of severe nausea and vomiting.

Prochlorperazine (choice E) is a phenothiazine antiemetic that exerts its action by


blocking dopaminergic receptors in the chemotrigger zone.
________________________________________
12The correct answer is D. Antibiotic-induced colitis (pseudomembranous colitis) is
characterized by severe persistent greenish, foul-smelling diarrhea and severe
abdominal cramps, as well as fever, leukocytosis, and lethargy. This condition is
caused by the toxin produced by Clostridium difficile. This disorder is generally seen
toward the end of clindamycin therapy; however, it may begin up to several weeks
after discontinuation of therapy. The treatment of this form of colitis is to
discontinue medication, provide fluid and electrolyte replacement, and give
corticosteroids (systemic and/or via enema), as well as metronidazole or
vancomycin. Metronidazole is the treatment of choice for treatment of antibioticinduced colitis since it is less expensive than vancomycin and does not encourage
the emergence of vancomycin-resistant bacteria.
Cisapride (choice A) is a prokinetic agent indicated for the treatment of reflux
esophagitis; the primary side effect of this agent is diarrhea.
Gentamicin (choice B) is an aminoglycoside antibiotic primarily used in the
treatment of life-threatening infections caused by gram-negative infections; this
agent would be ineffective in the treatment of C. difficile infection.
Loperamide (choice C) is an antidiarrheal. The use of this agent in patients with
antibiotic-induced colitis is contraindicated, as it may prolong the disease by
delaying the elimination of the toxin.
Sulfasalazine (choice E) is a locally acting sulfonamide indicated for the treatment
of ulcerative colitis and mild regional enteritis. This agent is ineffective in the
treatment of antibiotic-induced colitis.
________________________________________
13The correct answer is D. Carbohydrates and proteins both contain approximately
4 kcal of energy per gram. Fats are the more energy-efficient storage form,
containing 9 kcal of energy per gram. 100 grams of fat would yield 900 kcal of
energy.
________________________________________
14The correct answer is E. This is a classic presentation of Hodgkin's disease, which
is a form of lymphoma characterized by neoplastic proliferation of Reed-Sternberg
cells admixed with variable numbers of reactive lymphocytes, neutrophils, and
eosinophils. The cell of origin of the Reed-Sternberg cells is still disputed. These
cells have a distinctive appearance with a large double nucleus that contains paired,
large, nucleoli which are often red, producing an "owl's eye" effect.
Abnormal plasma cells (choice A) would be a feature of multiple myeloma or some
B-cell leukemias and lymphomas, which are not as likely in this patient.

Giant platelets (choice B) are a feature seen in several myeloproliferative disorders


(notably essential thrombocytopenia), which do not cause lymphadenopathy.
Immature neutrophil precursors (choice C) would most likely be a feature of a
myeloid leukemia, which would not cause a lymphadenopathy.
Melanin pigment (choice D) would be a feature of malignant melanoma, which
would probably have caused lung or liver metastases if it were at such an advanced
stage as to have caused massive lymphadenopathy.
________________________________________
15The correct answer is A. The retropharyngeal space is the fascial plane between
the buccopharyngeal fascia, which is the posterior lamina of the pretracheal fascia,
and the prevertebral fascia, which covers the prevertebral muscles. This fascial
plane allows movement between the visceral compartment of the neck, which
contains the trachea and esophagus, and the vertebral column and prevertebral
muscles. This fascial plane extends from the neck into the thorax and allows the
spread of infection between these regions.
There is no space between the investing layer of fascia and the carotid sheath
(choice B). The carotid sheath is the fascia that surrounds the common carotid
artery, the internal jugular vein, and the vagus nerve. This fascia is continuous with
the investing layer of fascia, as well as with the pretracheal layer and prevertebral
layers of fascia.
The space between the investing layer of fascia and the pretracheal layer of fascia
(choice C) is the region of the anterior triangle of the neck. The investing layer of
fascia encircles the sternocleidomastoid muscle and the trapezius muscle. The
pretracheal layer of fascia encircles the visceral compartment of the neck.
The space between the investing layer of fascia and the prevertebral fascia (choice
D) is the region of the posterior triangle of the neck. This region contains the spinal
accessory nerve and branches of the cervical plexus.
There is no space between the prevertebral fascia and the prevertebral muscles
(choice E). The prevertebral fascia immediately covers the prevertebral muscles
(eg, longus colli), the paravertebral muscles (eg, scalenes), and the deep back
muscles (splenius).
________________________________________
16The correct answer is A. A barrel chest with increased anterior/posterior diameter
is commonly observed in patients with long-standing, severe emphysema. This
change in chest shape occurs because these patients, who have high compliance of
the lung proper, tend to function with their lungs to some degree "over-inflated"
compared to people with normal lung compliance. This over-inflation limits their
ability to take further deep breaths. (The "balloon" of emphysematous lung remains
compliant, but the "box" of the chest wall is not very compliant and limits the
volume of air that can be inhaled). Patients with moderately severe emphysema are
able to maintain an adequate lung ventilation by taking many short breaths

(compare with choice D); this physiology is sometimes expressed by describing


these patients as "pink puffers" (choice E).
Chronic cough (choice B) in emphysema patients is not directly related to the
change in compliance.
Excessive mucus production (choice C) is more characteristic of chronic bronchitis
than of emphysema.
________________________________________
17The correct answer is E. The various points on the volume-pressure diagram
correspond to specific events of the cardiac cycle as follows:
Choice A: Marks the beginning of systole. The mitral valve closes and S1 can be
heard. The end diastolic pressure (5 mm Hg) and end diastolic volume (125 mL) can
be determined on the Y-axis and X-axis from this point.
Choice B: This is the period of isovolumic contraction. Left ventricular pressure
increases rapidly, but left ventricular volume remains constant. All heart valves are
closed.
Choice C: The aortic valve opens, which marks the beginning of the period of
ejection. The pressure at this point is equal to the diastolic blood pressure, which is
about 80 mm Hg on the diagram.
Choice D: This is the period of ejection. The pressure at the apex of the curve is the
peak systolic pressure of the left ventricle.
Choice E: Marks the beginning of diastole. The aortic valve closes and S2 can be
heard. The end systolic volume (50 mL) can be read from the X-axis at this point.
Choice F: The is the period of isovolumic relaxation. Left ventricular pressure is
falling rapidly, but left ventricular volume remains constant. All heart valves are
closed.
Choice G: The mitral valve opens and the period of filling begins.
Choice H: This is the period of filling.
________________________________________
18The correct answer is B. The odontoid process is the part of the axis (second
cervical vertebrae) that fits into and articulates with the atlas (first cervical
vertebrae). It is susceptible to traumatic fracture, and bony fragments can injure the
spinal cord. The scenario presented is unfortunately realistic, and a similar
mechanism is thought to cause deaths by execution-style hanging.
The atlas (choice A) is the first cervical vertebrae, and receives the odontoid
process of the axis.
Fracture of the lower cervical vertebrae (choices C, D, and E) does not usually cause
sudden death, but may cause paralysis of any or all limbs depending on the amount
of damage done.

________________________________________
19The correct answer is A. You should be aware of a specific association between
focal segmental glomerulosclerosis and both IV drug abuse and HIV nephropathy.
This disorder usually presents as an aggressive form of nephrotic syndrome; the
prognosis is generally poor, with rare cases responding to steroid therapy.
Histologically, there is focal (affecting only some glomeruli) and segmental
(affecting only a portion of each glomerular capillary tuft) sclerosis and hyalinization
with focal deposits of IgM and C3.
In IgA nephropathy (choice B), there are mesangial deposits of IgA and mesangial
proliferation.
Lipoid nephrosis (choice C), or minimal change disease, is the most frequent cause
of nephrotic syndrome in children. Light microscopic changes are generally minimal
or absent.
Membranoproliferative glomerulonephritis (choice D) is characterized by
proliferation of glomerular cells and thickening of capillary loops. It is not
particularly associated with drug abuse.
Membranous glomerulonephritis (choice E) is an important cause of nephrotic
syndrome in adults, characterized by diffuse thickening of the walls of glomerular
capillaries.
________________________________________
20The correct answer is B. All of the agents listed, including choices A, C, D and E,
can cause devastating congenital infections with high mortality and often with
major organ malformation (the TORCH agents: Toxoplasma, other, rubella,
cytomegalovirus, herpes simplex). However, it is herpes simplex type II, typically
acquired during delivery, that causes the devastating neonatal encephalitis
described in the question stem. The mortality rate for neonatal herpes is about
65%, and only 10% of the babies escape without neurologic sequelae. A point worth
remembering is that adult herpes encephalitis (in non-immunosuppressed
individuals) is usually due to herpes simplex I, while neonatal and congenital herpes
are usually due to herpes simplex II.
________________________________________
21The correct answer is A. There are three facts that are necessary to know in order
to answer this question: what disease the patient is suffering from, what
pharmacological properties can trigger an attack, and what drug has these
pharmacological properties. The patient described is suffering from an attack of
acute or narrow angle glaucoma. These attacks can be precipitated by drugs with
anticholinergic actions because muscarinic receptors on the pupillary constrictor
muscle of the iris are blocked. This causes pupillary dilation, which further "narrows"
the angle in the anterior chamber of the eye. Amitriptyline is a tricyclic
antidepressant with significant anticholinergic side effects.

Cimetidine (choice B) is an H2 antagonist that reduces gastric acid release. Its trade
name is Tagamet and is now available over the counter. It has no significant
anticholinergic side effects.
Diazepam (choice C) is a benzodiazepine. Its trade name is Valium and it has no
significant anticholinergic side effects.
Malathion (choice D) is an organophosphorus cholinesterase inhibitor that is used as
an insecticide. This agent would increase levels of ACh, thereby widening the angle.
Propranolol (choice E) is a non-selective, beta-adrenergic antagonist. If anything, it
would help to prevent an attack by blocking beta-receptors on the ciliary body,
thereby diminishing aqueous humor production.
________________________________________
22The correct answer is D. Infectious mononucleosis is a benign infection caused by
the Epstein-Barr Virus (EBV), a herpesvirus. Although B lymphocytes are infected by
the virus, the characteristic atypical cells are activated suppresser T cells - thus the
paracortical location (T cell zone) in the lymph node. Lymph nodes in viral infections
show expansion of germinal centers without loss of normal architecture. All
lymphomas, including Burkitt's, Hodgkin's and non-Hodgkin's lymphomas, destroy
the normal architecture of the lymph node.
Burkitt's lymphoma (choice B) produces a sea of monotonous, mitotically active
cells in a "starry sky" appearance.
Other non-Hodgkin's lymphomas (choice E) show either a nodular appearance or
diffuse sheets of cells which replace the germinal centers.
Hodgkin's lymphoma (choice C) also can show nodular or diffuse patterns, but is
characterized by the presence of Reed-Sternberg cells.
AIDS (choice A) is associated with a number of neoplastic and infectious processes
that may alter the lymph node structure. The characteristic lymph node changes in
AIDS are progressive transformation of the germinal centers, not paracortical
hyperplasia.
________________________________________
23The correct answer is C. The child is suffering from absence (petit mal) seizures.
The age of onset is typically from 3 to 7 years; seizures may continue into
adolescence, but generally subside before adulthood. These seizures have been
known to occur up to 100 times a day. Ethosuximide is indicated for this type (but
no other type) of seizure. Other drugs used in the treatment of absence seizures are
valproic acid, clonazepam, and a new agent, lamotrigine.
Carbamazepine (choice A) is used in the treatment of tonic-clonic (grand mal) and
partial (focal) seizures.
Diazepam (choice B) has long been the drug of choice for status epilepticus.
Recently, lorazepam (a shorter acting benzodiazepine) has also been accepted as a

drug for this condition. Intravenous phenytoin is used if prolonged therapy is


required. Phenobarbital has also been used, especially in children. If the status
epilepticus is very severe and does not respond to these measures, general
anesthesia may be used.
Methylphenidate (Ritalin; choice D) is a stimulant used to treat children with
attention deficit disorder. This child has no history of hyperactivity, and the
underlying cause of his "inattentiveness" is his seizure disorder.
Phenytoin (choice E) is effective in all seizure types except for the one in this
question (absence). Note that phenytoin has some idiosyncratic, test-worthy side
effects, including hirsutism and gingival hyperplasia.
________________________________________

24The correct answer is D. The MEFV curve shown above is created when the
patient inhales as much air as possible (point A, the total lung capacity) and then
expires with maximum effort until no more air can be expired (point E, the residual
volume). The descending portion of the curve (indicated by the downward pointing
arrow) represents the maximum expiratory flow at each lung volume along the
curve. This descending portion of the curve is sometimes referred to as the "effort
independent" portion of the curve because the patient cannot increase expiratory
flow rate further when greater effort is expended. Increased effort alone cannot
produce increases in the rate of air flow because of a very powerful limiting factor
called "dynamic compression" of the airways. A given increase in expiratory effort
produces equal increases in both the alveolar pressure (which tends to increase air
flow from the lungs) and the pressure outside the airways which tends to cause
them to collapse. Note from the curve that the maximum expiratory flow rate is
greater when the lungs are filled with a large volume of air compared to when lung
volume is lower. The main reason for this decrease in maximum expiratory flow rate
at the lower lung volumes is that the diameter of the various airways becomes
smaller and therefore easier to compress at lower lung volumes.
At point B, the patient has just begun to exhale with a maximum effort, and at point
C, the patient is still exhaling with a maximum effort and the air flow rate has nearly
reached its maximum value of ~400 liters/minute.
________________________________________
25The correct answer is C. Histological examination of the emphysematous lung
shows loss of alveolar walls with destruction of associated capillary beds. This loss
of lung tissue reduces the elastic recoil of the lung and increases the pulmonary
compliance, i.e., increases the distensibility of the lungs. [Recall that compliance =
volume/pressure.] Note that the volume-pressure curve of the patient is displaced to
the left and has a steeper slope compared to normal. The increase in compliance
associated with emphysema is not reversible.
Adult respiratory distress syndrome (choice A), asbestosis (choice B), and
sarcoidosis (choice E) all cause decreased pulmonary compliance.

Pulmonary edema (choice D), e.g., from congestive heart failure or valvular disease,
decreases pulmonary compliance.
________________________________________
26The correct answer is C. This number is worth remembering. While there is some
variation from individual to individual, glucosuria usually occurs when the blood
level is more than 180-200 mg/dL. The underlying physiology is that glucose is
freely filtered at the glomerulus, but then reabsorbed as it passes along the
proximal tubule. If the filtered glucose load is too great, then it cannot be
completely resorbed. In patients with long-standing type 1 diabetes mellitus, the
renal threshold may be highly variable secondary to renal disease.
________________________________________
27The correct answer is B. Invasive ductal carcinoma is the most likely candidate.
Of the various types of breast adenocarcinoma, invasive ductal carcinoma is by far
the most common variant, accounting for approximately 75% to 80% of all invasive
breast carcinomas. Invasive ductal carcinoma develops from epithelial cells of the
terminal duct. Histologically, it is composed of small, glandular, ductlike structures,
lined by variably anaplastic cells. The most common mode of presentation is a
palpable mass in the breast. Its prognosis depends mostly on staging (spread of
cancer) rather than grading (degree of differentiation).
The colloid (mucinous) variant (choice A) is relatively rare (about 1% to 2%) and
occurs more frequently in older women. Histologically, this carcinoma is
characterized by abundant mucin secretion. It is associated with a better prognosis
than the ductal type.
Invasive lobular carcinoma (choice C) is the second most frequent histologic type of
breast adenocarcinoma, accounting for approximately 10% of all cases. Its
presumed cell of origin is the lobular cell. The most typical histologic characteristic
is the presence of cancer cells lined up in orderly rows ("single-file").
Medullary carcinoma (choice D) tends to occur in younger women and is associated
with a slightly better prognosis. Although a malignant tumor, medullary carcinoma
is well circumscribed and surrounded by a florid lymphoplasmacytic reaction. The
name is due to its soft consistency.
Metastatic cancer may involve the breast like any other organ. Bronchogenic
carcinoma (choice E) may also spread to the breast by lymphatic route or by
contiguity, but this would be less likely than primary breast cancer.
________________________________________

28The correct answer is D. Patient D is the lucky mother of identical (monozygotic)


twins. The chorion forms before the amnion, so the possible combinations are
monoamnionic and monochorionic; diamnionic and monochorionic; and diamnionic
and dichorionic (either fused or separated). The first two of these possibilities are
seen only in identical twins. However, the latter can be seen either in fraternal or

identical twins. Exactly what happens in the case of identical twins depends on the
precise point at which twinning occurs: very early separation produces completely
separate membranes with duplication of both chorion and amnion; somewhat later
separation produces one chorion and two amnions; and very late separation
produces one chorion and one amnion. A dichorionic, diamnionic placenta develops
if splitting occurs early after fertilization, before the chorion forms. This type of
placenta may occur with either monozygotic or dizygotic twins. Thus, we are unable
to determine whether Patient A (choice A) or Patient B (choice B) had identical twins
from the examination of the placentas.
Neither Patient C (choice C) nor Patient E (choice E) had a twin pregnancy. A
circumvallate placenta has an extrachorial portion outside the insertion of the
amnionic membranes, creating a circumferential groove (vallum in Latin). A
bipartite placenta is composed of two equal segments. Circumvallate and bipartite
placentas are examples of placental abnormalities that usually have no clinical
significance.
________________________________________
29The correct answer is C. The history of nosebleeds and menorrhagia, the
petechiae, thrombocytopenia and increased bleeding time all suggest a platelet
disorder. The decreased platelet count suggests a thrombocytopenic disorder rather
than a platelet function disorder. The absence of antinuclear antibody argues
against systemic lupus erythematosus (a significant cause of thrombocytopenia).
The negative drug history rules out drug-associated thrombocytopenia. After other
causes have been ruled out, a tentative diagnosis of idiopathic thrombocytopenic
purpura can be made. This disorder, also called immune thrombocytopenic purpura,
is an acquired thrombocytopenia caused by formation of autoantibodies directed
against the platelet membrane proteins glycoprotein IIb/IIIa, followed by splenic
destruction of opsonized platelets. The disease typically occurs in women from 2040 years of age. Splenectomy and immunosuppressive therapy can generally
control the thrombocytopenia.
Antibodies to the acetylcholine receptor (choice A) are seen in myasthenia gravis.
Antibodies to erythrocyte membrane proteins (choice B) are seen in autoimmune
hemolytic anemia.
Antibodies to intrinsic factor (choice D) are seen in pernicious anemia.
Antibodies to Type IV collagen (choice E) are seen in Goodpasture's syndrome.
________________________________________
30The correct answer is C. Striated (skeletal) muscle not under voluntary control is
an unusual feature of the upper third of the esophagus. The middle third of the
esophagus contains roughly half striated and half smooth muscle; the lower third
contains only smooth muscle. All the other structures listed in the answer choices
contain smooth muscle.
________________________________________

31The correct answer is C. The muscle that adducts the thumb is the adductor
pollicis, which is innervated by the ulnar nerve. The ulnar nerve is derived from the
medial cord of the brachial plexus.
The lateral cord alone (choice A) gives rise to the musculocutaneous nerve.
The medial and lateral cords together (choice B) give rise to the median nerve.
No nerves arise from the combination of posterior and medial cords (choice D).
The posterior cord (choice E) gives rise to the axillary and radial nerves.
________________________________________
32The correct answer is E. The laboratory studies are typical for primary
hypothyroidism, in which the thyroid fails to produce adequate T4 despite
appropriate TSH signals from the pituitary gland. Clinical features of hypothyroidism
include weight gain, cold intolerance, hypoactivity, fatigue, lethargy, decreased
appetite, constipation, weakness, decreased reflexes, facial and periorbital
myxedema, dry and cool skin, and brittle hair.
In contrast, diarrhea (choice A), heat intolerance (choice B), hyperactivity (choice
C), and palpitations (choice D) are features of hyperthyroidism.
________________________________________
33The correct answer is D. The cell division is mitotic rather than meiotic. The cell is
in metaphase, as evidenced by alignment of chromosomes on the metaphase plate.
Doubling of the DNA occurs much earlier, in prometaphase (late prophase), so the
cell has "4N" DNA (double the normal 2N diploid DNA). The chromosomes at the
metaphase plate consist of 2 chromatids each, but the total number of
chromosomes is still the normal 46.
23 chromosomes with 2N DNA (choice A) is wrong because stages with 23
chromosomes only occur in meiosis.
23 chromosomes with 4N DNA (choice B) is wrong because stages with 23
chromosomes only occur in meiosis. Additionally, there is never a point in either
meiosis or mitosis when 23 chromosomes contain 4N DNA.
46 chromosomes with 2N DNA, each chromosome having one chromatid (choice C),
is wrong because the chromosomes have two chromatids and the DNA is therefore
doubled (producing 4N DNA total) in metaphase. The cell normally has 46
chromosomes with 2N DNA only after mitosis is complete.
92 chromosomes (choice E) is wrong because in anaphase and telophase, at which
points there are 92 chromosomes (in two cells which are not yet cleaved), there is a
total of 4N DNA and each of the newly split chromosomes contains one chromatid.
________________________________________

34The correct answer is C. Delta waves are low-frequency, high-amplitude


waveforms that herald the arrival of the deepest type of non-REM sleep, stage 4
sleep. It is during this stage of sleep that somnambulism (sleepwalking) occurs.
Alpha waves (choice A) are characteristic of relaxed wakefulness.
Beta-like activity (choice B) is characteristic of either alert wakefulness or REM
sleep.
Sleep spindles and K-complexes (choice D) are characteristic of stage 2 sleep, which
is a deeper sleep that occupies roughly 45% of the sleep cycle.
Theta waves (choice E ) are characteristic of light sleep (stage 1).
________________________________________
35The correct answer is C. Because the various resistances (R1-R5) are arranged in
parallel, the total resistance of the circuit (RT) is calculated using the following
formula: 1/RT = 1/R1 + 1/R2 + 1/R3 + 1/R4 + 1/R5. Therefore, the resistance of the
circuit is 1/ RT = 1/5 + 1/5 + 1/5 + 1/5 +1 /5 = 1 mm Hg/mL/min. Because flow = D
pressure/resistance, the total flow through the circuit is (100 - 10 mm Hg)/1 mm
Hg/mL/min = 90 mL/min. Note from the equation that the total resistance (RT)
decreases when additional resistances are added in parallel to the circuit.
Conversely, the total resistance increases when parallel resistances are removed.
Because the various organs of the body are arranged in parallel, the total peripheral
resistance increases when an organ is removed.
________________________________________
36The correct answer is E. Chlamydia trachomatis (serotypes A, B, and C) causes a
conjunctival and corneal infection that is spread in developing countries by eyeseeking flies. The lesions begin with formation of lymphoid follicles in the
conjunctiva. With disease progression, there is tissue necrosis, granulation tissue
deposition, and scar formation, leading to lacrimal duct obstruction and distortion of
the eyelids. With the loss of an adequate tear system, the cornea becomes
vulnerable to dehydration and opacification. Also, the vigorous inflammatory
response can directly involve the cornea, with resulting opacity. In developed
countries, chlamydial eye infections are often transmitted venereally rather than by
flies, and may cause conjunctivitis in the newborn and in sexually active young
adults
________________________________________
37The correct answer is C. Achalasia is an acquired esophageal motility disorder
that is characterized by loss of enteric inhibitory neurons. The lower esophageal
sphincter may exhibit increased tone in between swallows and fail to relax normally
with a swallow. Peristalsis in the esophageal body is also abnormal. A swallow may
not induce any peristalsis in the esophageal body or may produce simultaneous
contractions along its entire length.
The esophageal body distal to the diaphragm (choice A) is relaxed in between
swallows. The intraesophageal pressure at this point reflects the intra-abdominal

pressure, which is slightly positive (5 mm Hg). During inspiration, the pressure


inside the distal esophagus rises along with the intra-abdominal pressure; during
expiration this pressure falls. The pressure in the esophageal body proximal to the
diaphragm (choice B) reflects the intrathoracic pressure. It is slightly negative at the
end of inspiration and slightly positive at the end of expiration.
Since the mouth and pharynx are open to the atmosphere, in between swallows, the
pressure within the pharynx (choice D) is atmospheric (0 mm Hg). The pressure
rises abruptly to a maximum of 100 mm Hg at the start of a swallow and returns to
baseline within 0.5 seconds.
At rest, the pressure in the upper esophageal sphincter (choice E) can be as high as
60 mm Hg. It is maintained by the normal elasticity of the sphincteric structures,
and by active contraction of the cricopharyngeal muscle, which composes most of
the sphincter. Shortly after the pharyngeal muscles contract during a swallow, the
upper esophageal sphincter relaxes as the tonic neural input to the cricopharyngeal
muscle (skeletal muscle) is inhibited as part of the swallowing program. Function of
this sphincter is unaffected by achalasia.
________________________________________
38The correct answer is D. Neonatal respiratory distress syndrome is a disease of
immaturity. The immature lung is not able to produce sufficient surfactant to
prevent collapse of many alveoli. Severe diffuse damage to alveoli causes
precipitation of protein ("hyaline membranes") adjacent to many alveolar walls. In
infants that survive, particularly if oxygen was used for therapy, the lungs
eventually become heavily fibrotic (misnamed bronchopulmonary dysplasia).
Abundant neutrophils (choice A) would not be seen unless the patient had also
developed pneumonia.
Fibrosis (choice B) is a late, not early, feature of respiratory distress syndrome.
The air spaces are collapsed, not enlarged (choice C), in this condition.
The histology in these patients is usually markedly abnormal (not choice E).
________________________________________
39The correct answer is D. The forced vital capacity (FVC) is the vital capacity
measured with a forced expiration (which may be less than that measured without
straining). The forced expiratory volume in one second (FEV1) is the amount of air
that can be expelled from the lungs during the first second of a forced expiration.
The FEV1/FVC ratio has diagnostic value for differentiating between normal,
obstructive, and restrictive patterns of a forced expiration.
The FEV1/FVC for the normal individual (trace A) is 4 L/5 L = 80% and 1.5 L/3.0 L =
50% for the patient (trace B), as determined from the diagram. Trace B is an
example of a trace obtained from a patient with chronic obstructive pulmonary
disease. Notice that the rate of expiration is slower and the total volume exhaled is
reduced. This is typical of obstructive disease. Patients with restrictive disease also

have a reduced FVC, but exhale a large percentage of it in the first second,
increasing their FEV1/FVC ratio.
________________________________________
40The correct answer is A. Endemic typhus is caused by Rickettsia typhi and is
found worldwide. It is spread by the feces of the rat flea, and its reservoir is the rat.
Epidemic typhus (choice B) is cause by Rickettsia prowazekii. It is prominent during
times of war and social upheaval and is spread by the feces of the body louse,
Pediculus humanis. Humans are the chief reservoir, although it is also found in
populations of flying squirrels along the Atlantic coast of the U.S.
Q fever (choice C) is caused by Coxiella burnetii of the family Rickettsiaceae. Unlike
the other rickettsial illnesses, Coxiella burnetii is not usually transmitted to humans
by the bite of an arthropod. Since the organism is resistant to dehydration, it can be
acquired by inhaling dust contaminated by animals with asymptomatic infections,
such as goats, sheep, and cattle.
Rocky Mountain spotted fever (RMSF) (choice D) is caused by Rickettsia rickettsii. It
is transmitted by the bite of an infected tick of the genus Dermacentor. Since the
tick passes the infection transovarially to its progeny, the tick is a reservoir. Rodents
and other mammals are also reservoirs. Despite the name, most cases of RMSF
occur in the central states and southern Atlantic seaboard states. Oklahoma has the
highest incidence of the disease.
Scrub typhus (choice E) is caused by Rickettsia tsutsugamushi. It occurs mainly in
Southeast Asia, where it is transmitted by mites. Mites and rodents are the
reservoirs for the organism.
________________________________________
41The correct answer is C. The patient has suffered a supracondylar fracture of the
femur. The supracondylar surface of the femur forms the floor of the popliteal fossa.
This fossa contains the popliteal artery, in close proximity to the femur. After
crossing the knee this artery divides into the anterior and posterior tibial arteries,
which supply the leg and foot.
The anterior tibial artery (choice A) is a branch of the popliteal artery that arises
distal to the knee and would therefore not be compressed by a femoral fracture. The
anterior tibial artery is renamed the dorsalis pedis artery after it crosses the ankle.
The femoral artery (choice B) is in the anterior compartment of the thigh. After
passing through the adductor hiatus to reach the popliteal fossa, the femoral artery
is renamed the popliteal artery.
The posterior tibial artery (choice D) is a branch of the popliteal artery that arises
distal to the knee and would therefore not be compressed by a femoral fracture. The
posterior tibial artery passes through the posterior compartment of the leg and,
after passing behind the medial malleolus of the ankle, divides into the medial and
lateral plantar arteries.

The profunda femoris artery (choice E) arises from the femoral artery in the
proximal thigh. It gives rise to branches that supply the posterior compartment of
the thigh, not the foot. The profunda femoris artery does not enter the popliteal
fossa and would not be compressed by this fracture.
________________________________________

42The correct answer is E. The presence of Reed-Sternberg (RS) cells is


pathognomonic of Hodgkin disease (HD). After long debate, RS cells have been
recognized as the true neoplastic elements of HD. A classic RS cell is a giant,
usually binucleated cell, with large "inclusion-like" nucleoli. The nuclei are similar to
each other, giving the impression of mirror images. Classic RS cells and lacunar
cells are probably transformed B lymphocytes and are positive for CD15 and CD30.
Variants of RS cells are found in different subtypes of HD. Lacunar cells are
mononucleated and surrounded by a pericellular halo. The lymphohistiocytic (L+H)
cell is a variant of RS cell with a characteristic convoluted nucleus resembling a
popcorn kernel ("popcorn cell"). The nodular sclerosis subtype affects adolescents
and young adults. Histologically, it is characterized by the lacunar cell variant and
the presence of broad bands of fibrous tissue that divide the lymph node into
nodules. Nodular sclerosis HD has an excellent prognosis.
Lymphocyte depletion HD (choice B) is rare and controversial. Most cases are large
cell lymphomas, affecting older males with disseminated disease.
Lymphocyte predominance HD (choice C) accounts for 6% of all cases. This subtype
is characterized by the L+H variant of the RS cell. The cells are CD15- and CD30-,
but CD20+. This lymphoma is of follicular B-cell origin and mostly affects young
males with cervical lymphadenopathy.
The mixed cellularity (choice D) form of HD is characterized by frequent RS cells.
There is a biphasic incidence curve, with peaks affecting young adults and those
older than 55. This lymphoma presents at advanced stages.
Diffuse large cell lymphoma (choice A) is an aggressive B-cell lymphoma that
represents 20% of non-Hodgkin lymphomas. It affects older people. Neoplastic
lymphocytes are large, with prominent nucleoli. The histopathologic and clinical
features of this case clearly rule out this form of lymphoma.
________________________________________
43The correct answer is D. HTLV-1, or human T-cell lymphotrophic virus 1, is an
enveloped, single-stranded, RNA retrovirus endemic to southern Japan and the
Caribbean basin, but sporadically seen in the United States. Infection by HTLV-1 can
lead to T-cell leukemia 20-30 years after the infection. The HTLV-associated T-cell
leukemia generally presents as described above, and is very aggressive,
progressing to death in under 1 year.

AIDS (choice A), acquired immunodeficiency syndrome, is transmitted through the


HIV virus. Although both viruses are RNA retroviruses and both infect CD4+ T cells,
the diseases they produce are distinct.
Autoimmune diseases (choice B) produced after infection are typified by rheumatic
fever after infection with Streptococcus. There is speculation that EBV virus
(Epstein-Barr virus) may produce the autoimmune reactions in rheumatoid arthritis,
but autoimmunity plays no role in leukemia after HTLV-1 infection.
Delayed hypersensitivity reactions (choice C) are T-cell-mediated immune responses
typified by the subcutaneous reaction to tuberculin in sensitized individuals. It
characteristically occurs 2-3 days after an exposure, and is a local lymphocytic
response.
Recurrent infections (choice E) occur when latent viruses are reactivated in the host
at the site of primary infection. Recurrent infections are most common with the
herpesviruses, especially HSV-1, HSV-2, and VZV (varicella-zoster virus).
________________________________________
44The correct answer is D. Streptococcus pneumoniae (pneumococcus) is the likely
organism causing this man's pneumonia, based on the clinical presentation as well
as the Gram's stain results. Given this patient's age and symptoms, intravenous
antibiotics should be given in a hospital setting. Despite the occurrence of
resistance of pneumococcus to penicillin, this is still the antibiotic of choice for
empiric treatment of this organism.
Ceftazidime (choice A), a third generation cephalosporin with antipseudomonal
activity, is less effective than first generation cephalosporins or other third
generation cephalosporins (e.g., ceftriaxone) against gram-positive cocci.
Ciprofloxacin (choice B) is a fluoroquinolone with activity against gram negativeorganisms. It is not reliably effective against the pneumococci.
Erythromycin (choice C) is useful for a variety of organisms, but there is a high level
of macrolide resistance among community-acquired pneumococci.
Trimethoprim-sulfamethoxazole is useful for many urinary tract pathogens,
Pneumocystis carinii, Shigella, Salmonella and Serratia, but pneumococci show a
significant amount of resistance to the combination (choice E).
________________________________________
45The correct answer is A. Two types of free-living amoeba can infect the brain and
meninges: Naegleria fowleri and Acanthamoeba species. The former affects healthy
adolescent or adult divers, while the latter causes infection in patients with
immunosuppression because of diabetes, alcoholism, cancer, or HIV infection. The
brain infection characteristically has a prominent perivascular character, which
causes a multifocal hemorrhagic necrotizing meningoencephalitis. Skin ulcers, nasal
infection, or pneumonia may also be present. It is thought that the organisms may
release a toxin causing host tissue necrosis. Systemic antifungal agents (e.g.,
amphotericin) have some activity against this organism, but most cases are fatal.

Entamoeba histolytica (choice B) causes amoebic dysentery and liver abscess.


Giardia lamblia (choice C) is a flagellate, rather than an amoeba, and causes
diarrhea.
Naegleria fowleri (choice D) is an amoebic cause of meningoencephalitis in
previously healthy swimmers and divers.
Trichomonas vaginalis (choice E) is a flagellate, rather than an amoeba, and causes
vaginitis.
________________________________________
46The correct answer is B. The tumor is a pheochromocytoma that intermittently
secretes epinephrine and other vasoactive amines, producing episodes of elevated
blood pressure accompanied by headache. This is the tumor to associate with the
rule of the 10's: 10% malignant, 10% bilateral, 10% extra-adrenal, 10% calcify, 10%
occur in kids, and 10% are familial. It is also a favorite target on examinations,
although the incidence is quite low.
________________________________________
47The correct answer is A. This woman is suffering from dementia of the
Alzheimer's type. A gradual onset of symptoms, general pervasive memory deficit,
difficulties with language, and inability to plan, leading to severe impairment of
daily functioning are all characteristic of dementia of the Alzheimer's type.
Amnestic disorder (choice B) is limited to memory problems and this woman is
demonstrating cognitive dysfunction, such as alterations in language and the loss of
the ability to plan.
Pseudodementia (choice C) is incorrect since it is a major depressive disorder rather
than a dementing condition. There is no evidence for a depressive syndrome in this
patient's presentation.
The diagnosis of substance-induced persisting dementia (choice D) requires
evidence of a history of substance abuse. However, it is the second most likely
diagnosis and should be carefully explored with the husband and other close
relatives and friends.
Vascular dementia (choice E) is generally characterized by a step-wise deterioration,
not the gradual presentation of this case.
________________________________________
48The correct answer is D. This is lymphogranuloma venereum. The enlarged,
abscessed lymph nodes are termed "buboes" (which can also occur in bubonic
plague). The only other commonly discussed sexually transmitted disease in which
they can occur is chancroid. Histologically, the buboes of lymphogranuloma
venereum are enlarged lymph nodes with stellate abscesses. The primary lesion is
usually a self-healing papule or shallow ulcer. The causative organism is Chlamydia
trachomatis.

Condyloma acuminatum (choice A) causes a papillary lesion and does not cause
buboes.
Granuloma inguinale (choice B) causes a spreading ulcer and does not cause
buboes.
Herpes virus infection (choice C) causes tiny vesicles and ulcers and does not cause
buboes.
Syphilis (choice E) causes a painless nodule and does not cause buboes.
________________________________________
49The correct answer is D. The hepatoduodenal ligament is the portion of the lesser
omentum that connects the liver to the first part of the duodenum. Within the
hepatoduodenal ligament are found the proper hepatic artery and its branches, the
common bile duct and its branches, and the portal vein. The cystic artery is usually
a branch of the right hepatic artery, which is a branch of the proper hepatic artery.
The falciform ligament (choice A) is a mesentery that connects the liver to the
anterior abdominal wall. Within the free edge of the falciform ligament is found the
round ligament of the liver (ligamentum teres hepatis), the adult remnant of the
umbilical vein.
The gastrocolic ligament (choice B) is the portion of the greater omentum between
the greater curvature of the stomach and the transverse colon. The gastroepiploic
arteries lie within the gastrocolic ligament along the greater curvature of the
stomach.
The gastrohepatic ligament (choice C) is the portion of the lesser omentum between
the liver and the lesser curvature of the stomach. The right and left gastric arteries
are within the gastrohepatic ligament along the lesser curvature of the stomach.
The splenorenal ligament (choice E) is the mesentery that connects the spleen to
the posterior abdominal wall. The splenic artery and splenic vein are within the
splenorenal ligament.
________________________________________
50The correct answer is C. The long thoracic nerve innervates the serratus anterior
muscle. Paralysis of this muscle weakens upward rotation of the scapula, which is
needed for full abduction or flexion at the shoulder. Weakness of this muscle results
in "winged scapula" in which the medial border of the scapula is pushed away from
the chest wall. A downward blow on the shoulder may result in stretching of this
nerve.
The axillary nerve (choice A) innervates the teres minor and deltoid muscles.
Paralysis of the deltoid will cause weakness in abduction but will not cause a winged
scapula. The teres minor is an external rotator of the shoulder.
The dorsal scapular nerve (choice B) innervates the rhomboideus major and
rhomboideus minor muscles. These muscles retract the scapula. Their paralysis

might cause a minor winging of the scapula but will not affect abduction or flexion
at the shoulder.
The suprascapular nerve (choice D) innervates the supraspinatus and infraspinatus
muscles. The supraspinatus is used during abduction and the infraspinatus is an
external rotator of the shoulder.
The thoracodorsal nerve (choice E) innervates the latissimus dorsi muscle, which is
an extensor at the shoulder and an internal rotator at the shoulder.

Anda mungkin juga menyukai